0% found this document useful (0 votes)
549 views

Potential & Capacitance

Uploaded by

sathyamurali86
Copyright
© © All Rights Reserved
We take content rights seriously. If you suspect this is your content, claim it here.
Available Formats
Download as DOCX, PDF, TXT or read online on Scribd
0% found this document useful (0 votes)
549 views

Potential & Capacitance

Uploaded by

sathyamurali86
Copyright
© © All Rights Reserved
We take content rights seriously. If you suspect this is your content, claim it here.
Available Formats
Download as DOCX, PDF, TXT or read online on Scribd
You are on page 1/ 200

CHAPTER 2 ELECTROSTATIC POTENTIAL AND

CAPACITANCE

that it does not disturb the distribution of


the source charge. Secondly, we just
2.1 ELECTROSTATIC POTENTIAL AND
apply so much external force on the test
POTENTIAL DIFFERENCE
charge that it just balances the repulsive
Introduction. The electric field around electric force on it and hence does not
a charge can be described in two wavs : produce any acceleration in it.
(i) by electric field (⃗
E ), and 51 unit of potential difference is volt
(V). It has been named after the Italian
(ii) by electrostatic or electric potential (V). scientist Alessandro Volta.
The electric field E is a vector quantity, 1 joule
while electric potential is a scalar quantity. 1 volt =
Both of these quantities are the 1coulomb
characteristic properties of any point in a or 1 V = 1 Nm C-1 = 1 JC-1
field and are inter-related.
Hence the potential difference
1. Develop the concepts of potential between two points in an electric field is
difference and electric potential. State and said to be 1 volt if l joule of work has to be
define their SI units. done in moving a positive charge ofl
Potential difference. As shown in Fig. coulomb from one point to the other
2.1, consider a point charge + q located at against the electrostatic forces.
a point O. Let A and B be two points in its Electric potential. The electric potential
electric field. When a test charge q 0 is at a point located far away from a charge
moved from A to B, a work W AB has to be is taken to be zero.
done in moving against the repulsive force
exerted by the In Fig. 2.1, if the point A lies at infinity,
then VA = 0, so that
W
V = VB =
q0
where W is the amount of work done in
Fig. 2.1 To define potential difference. moving the test charge q0 from infinity to
the point B and VB refers to the potential at
charge + q. We then calculate the
point B.
potential difference between points A and
B by the equation : So the electric potential at a point in an
electric field is the amount of work done in
W AB
V =V B−V A= -...(2.1) % moving a unit positive charge from infinity
q0 to that point against the electrostatic
forces.
50 the potential difference between
two points in an electric field may be Work done
defined as the amount of work done in Electric potential =-
Charge
moving a unit positive charge from one
point to the other against the electrostatic SI unit of electric potential is volt (V). The
forces. electric potential at a point in an electric
field is said to be 1 volt if one joule of work
In the above definition, we have has to be done in moving a positive
assumed that the test charge is so small charge of 1 coulomb from infinity to that

1
point against the electrostatic forces. W 1 q
V= or V = ⋅
2.2 ELECTRIC POTENTIAL DUE TO A q0 4 π ε0 r
POINT CHARGE Clearly, V ∝ 1 / r. Thus the electric
2. Derive an expression for the potential due to a point charge is
electric potential at a distance r from a spherically symmetric as it depends only
point charge q. What is the nature of this on the distance of the observation point
potential ? from the charge and not on the direction of
that point with respect to the point charge.
Electric potential due to a point
Moreover, we note that the potential at
charge. Consider a positive point charge q
infinity is zero.
placed at the origin O. We wish to

electrostatic potential (V ∝ 1 / r) and the


calculate its electric potential at a point P Figure 2.3 shows the variation of

electrostatic field (E ∝ 1 / r2) with distance


at distance r from it, as shown in Fig. 2.2.
By definition, the electric potential at point
P will be equal to the amount of work done r from a charge q.
in bringing a unit positive charge from
infinity to the point P.

Fig. 2.2 Electric potential due to a point


charge.
Suppose a test charge q0 is placed at
point A at distance x from O. By
Coulomb's law, the electrostatic force
acting on charge q0 is Fig. 2.3 Variation of potential V and field E
1 q q0 with r from a point charge q.
F= ⋅
4 π ε0 x2 2.3 ELECTRIC POTENTIAL DUE TO A
DIPOLE
The force ⃗ F acts away from the
charge q. The small work done in moving 3. Derive an expression for the
the test charge q0 from A to B through potential at a point along the axial line of a
short dipole.
small displacement ⃗ dx against the
electrostatic force is Electric potential at an axial point of a
dipole. As shown in Fig. 2.4, consider an
F .⃗
dW = ⃗ dx - Fdx cos 180° = - Fdx electric dipole consisting of two point
The total work done in moving the charges - q and + q and separated by
charge q0 from infinity to the point P will be distance 2 a. Let P be a point on the axis
r r
of the dipole at a distance r from its centre
1 q q0 O.
W = ∫ dW =−∫ Fdx=−∫ ⋅ 2 dx
∞ ∞ 4 π ε0 x

[ ]
r r
−q q 0 −q q 0 −1
= ∫
4 π ε0 ∞
−2
x dx=
4 π ε0 x ∞

= [
q q0 1 1
− =
1
]

4 π ε0 r ∞ 4 π ε 0 r
q q0
.
Fig. 2.4 Potential at an axial point of a
dipole.

Hence the work done in moving a unit Electric potential at point P due to the
test charge from infinity to the point P, or dipole is
the electric potential at point P is

2
1 −q 1 q dipole consisting of two point charges - q
V =V 1+V 2= . + . and + q and separated by distance 2 a, as
4 π ε 0 AP 4 π ε 0 BP
shown in Fig. 2.6. We wish to determine
1 q 1 q the potential at a point P at a distance r
¿− . + .
4 π ε 0 r +a 4 π ε 0 r−a from the centre O, the direction OP
making an angle θ with dipole moment ⃗p.
¿
η
[ 1

1
4 π ε 0 r −a r + a ] Let AP = r1 and BP = r2.
Net potential at point P due to the dipole is
¿
4 π ε0 [
q ( r + a )− ( r−a )
2
: −a
2
=
]
1 q ×2 a
. 2 2
4 π ε 0 r −a
1 p
or V = . 2 2 [∵ p=q ×2 a ¿
4 π ε 0 r −a
2 2
For a short dipole, a <¿r , so
1 p
V= . .
4 π ε0 r 2
4. Show mathematically that the
potential at a point on the equatorial line of
Fig. 2.6
an electric dipole is zero.
V =V 1+V 2
Electric potential at an equatorial point
of a dipole. As shown in Fig. 2.5, consider 1 −q 1 q
an electric dipole consisting of charges - q ¿ + .
4 π ε0 r1 4 π ε0 r 2
and + q and separated by distance 2 a.

[ ]
Let P be a point on the perpendicular q 1 1
bisector of the dipole at distance r from its ¿ −
4 π ε0 r 2 r1
centre O.

¿
[ ]
q r 1−r 2
4 π ε0 r1 r2
If the point P lies far away from the dipole,
then
r1 - r2 ≃ AB cos θ = 2a cos θ and r1r2 ≃ r2

∴ V=
q 2 a cos θ

4 π ε0 r
2

Fig. 2.5 Potential at an equatorial point of


or V =
a dipole. 1 p cos θ

4 π ε0 r
2
Electric potential at point P due to the
dipole is 1 ⃗p . r⃗ 1 ⃗p ⋅ r^
or V = =
1 −q 1 q 4 π ε0 r 3
4 π ε0 r2
V = V 1 +V 2= ⋅ + ⋅
4 π ε 0 AP 4 π ε 0 BP Here p = q × 2a, is the dipole moment
−1 q 1 q and r^ = r⃗ / r, is a unit vector along the
= ⋅ 2 2+ ⋅ 2 2 =0 . position vector ⃗
OP = r⃗ . Special Cases
4 π ε 0 √ r +a 4 π ε 0 √ r +a
(i) When the point P lies on the axial line
5. Derive an expression for the electric of the dipole, θ =0° or 180°, and
potential at any general point at distance r
from the centre of a dipole. 1 p
V=± ⋅
Electric potential at any general point 4 π ε0 r2
due to a dipole. Consider an electric

3
i.e., the potential has greatest positive or system of N point charges.
the greatest negative value.
Electric potential at point P due to charge
(ii) When the point P lies on the equatorial q1 is
line of the dipole, θ = 90°, and V = 0,
i.e., the potential at any point on the 1 q1
V 1= .
equatorial line of the dipole is zero. 4 π ε0 r1
However, the electric field at such
Similarly, electric potentials at point P due
points is non-zero.
to other charges will be
6. Give the contrasting features of
electric potential of a dipole from that due 1 q2 1 q3
V 2= . ' V 3= . , …,
to a single charge. 4 π ε0 r2 4 π ε0 r3
Differences between electric potentials 1 qN
V N= .
of a dipole and a single charge. 4 π ε0 r N
1. The potential due to a dipole depends As electric potential is a scalar quantity, so
not only on distance r but also on the the total potential at point P will be equal
angle between the position vector r⃗ of the to the algebraic sum of all the individual
observation point and the dipole moment potentials, i.e.,
vector ⃗p. The potential due to a single
charge depends only on r.
V =V 1+V 2+V 3 + …+V N

2. The potential due to a dipole is


cylindrically symmetric about the dipole
axis. If we rotate the observation point P
¿
1
[ q1 q 2 q3
+ + +…+
4 π ε0 r1 r2 r3
qN
rN ]
N
about the dipole axis (keeping r and θ 1 qi
fixed), the potential V does not change. or V = ∑
4 π ε 0 i=1 r i
The potential due to a single charge is
spherically symmetric. If r⃗ 1 , r⃗ 2 , r⃗ 3 ,… ,r⃗ N are the position vectors
3. At large distance, the dipole of the N point charges, the electric
potential falls off as 1 / r 2 while the potential at a point whose position vector
potential due to a single charge falls off as is r⃗ , would be
1 / r. N
1
2.4 ELECTRIC POTENTIAL DUE TO A V= ∑ 1r ∨¿ . ¿
4 π ε 0 i=1 ¿ ⃗r −⃗i
SYSTEM OF CHARGES
2.5 ELECTRIC POTENTIAL DUE TO A
7. Derive an expression for the electric CONTINUOUS CHARGE DISTRIBUTION
potential at a point due to a group of N
point charges. 8. Deduce an expression for the
potential at a point due to a continuous
Electric potential due to a group of charge distribution. Hence write the
point charges. As shown in Fig. 2.7, expression for the electric potential due to
suppose N point charges q1, q2, q3, ….., qN a general source.
lie at distances r1, r2, r3, ….., rN from a
point P. Electric potential due to a continuous
charge distribution. We can imagine that a
continuous charge distribution consists of
a number of small charge elements
located at positions ⃗r i . It r⃗ is the position
vector of point P, then the electric
potential at point P due to the continuous
charge distribution can be written as
1 dq
V= ∫
4 π ε 0 |r⃗ −⃗r i|
Fig. 2.7 Potential at a point due to a

4
continuously in a volume V, dq = ρ dV,
When the charge is distributed R and carrying charge q. We wish to
calculate its potential at point P at
where p is volume charge density. The distance r from its centre O, as shown in
potential at point P due to the volume Fig. 2.8.
charge distribution will be
1 ρdV
4 π ε0 ∫
VV=
V |r
⃗ −⃗r i|
When the charge is distributed
continuously over an area S, dq = σ dS
where σ is surface charge density.

∴ V S= 4 π ε
1 σdS
∫ |r⃗ −⃗r |
0 S i
Fig. 2.8 Potential due to a spherical shell.
uniformly along a line L, dq = λ dL, where
When the charge is distributed

λ is line charge density.


1 λdL
∴ V L=
4 π ε0 ∫
L | −⃗

r r i|
The net potential at the point P due to
the continuous charge distribution will be
the algebraic sum of the above potentials.
V cont =V V +V S +V L
Fig. 2.9 Variation of potential due to
or
charged shell with distance r from its
V cont =
1
4 π ε0 [∫ |
V
ρdV
+∫
σdS
+∫
λdL
r⃗ −⃗r i| S |⃗r −⃗r i| L |⃗r −⃗r i| ] centre.
(i) When the point P lies outside the shell.
We know that for a uniformly charged
Electric potential due to a general source.
spherical shell, the electric field outside
The potential due to a general source
the shell is as if the entire charge is
charge distribution, which consists of
concentrated at the centre. Hence electric
continuous as well as discrete point
potential at an outside point is equal to
charges, can be written as
that of a point charge located at the
V =V cont +V discrete centre, which is given by
or 1 q
V= [For r > R]

[ ]
qi ε 0 r
1 ρdV σdS λdL 4 π
V= ∫ +∫ +∫ + ∑
4 π ε 0 V |r⃗ −⃗r i| S |r⃗ −⃗r i| L |⃗r −⃗r i| All point charges(ii)|r⃗When
−⃗r i| point P lies on the surface of the
shell. Here r = R. Hence the potential on
2.6 ELECTRIC POTENTIAL DUE TO A the surface of the shell is
UNIFORMLY CHARGED THIN
SPHERICAL SHELL 1 q
V= [For r = R]
4 π ε0 R
9. Write expression for the electric
potential due to a uniformly charged (iii) When point P lies inside the shell. The
spherical shell at a point (i) outside the electric field at any point inside the shell is
shell, (ii) on the shell and (ii) inside the zero. Hence electric potential due to a
shell. uniformly charged spherical shell is
constant everywhere inside the shell and
Electric potential due to uniformly
its value is equal to that on the surface.
charged thin spherical shell. Consider a
Thus,
uniformly charged spherical shell of radius

5
1 q 1 q
V= [For r < R] V= ⋅
4 π ε0 R 4 π ε0 r
Figure 2.9 shows the variation of the
potential 1/ due to a uniformly charged
3. Electric potential at a point due to N
spherical shell with distance r measured
point charges,
from the centre of the shell. Note that V is

along a horizontal line and thereafter V ∝


N
constant (=q/ 4πε0R) from r = 0 to r = R 1 qi
V= ∑
4 π ε 0 i=1 r i
1 / r for points outside the shell.
4. Electric potential at a point due to a
For Your Knowledge
dipole,
Electric potential is a scalar quantity while
1 p cos θ 1 ⃗p ⋅⃗r
potential gradient is a vector quantity. V= = ⋅ 3
4 π ε0 r 2
4 π ε0 r
The electric potential near an isolated
positive charge is positive because work Units Used
has to be done by an external agent to Charge q is in coulomb, distance r in
push a positive charge in, from infinity. metre, work done W in joule and potential
The electric potential near an isolated difference V in volt.
negative charge is negative because the Example 1. If 100 J of work has to be
positive test charge is attracted by the done in moving an electric charge o/4C
negative charge. from a place, where potential is -10 V to
The electric potential due to a charge q at another place, where potential is V volt,
its own location is not defined - it is find the value of V.
infinite. Solution. Here WAB =100 J, q0 = 4C, VA =
Because of arbitrary choice of the -10 V, VB = V.
reference point, the electric potential at a W AB
point is arbitrary to within an additive As V B - VA =
constant. But it is immaterial because it is q0

∴ V - (-10) =
the potential difference between two 100
points which is physically significant. =25
4
For defining electric potential at any point,
generally a point far away from the source or V = 25 - 10 = 15 V.
charges is taken as the reference point. Example 2. Determine the electric
Such a point is assumed to be at infinity. potential at the surface of a gold nucleus.
As the electrostatic force is a conservative The radius is 6.6 × 10 -15 m and the atomic
force, so the work done in moving a unit number Z = 79. Given charge on a proton
positive charge from one point to another =1.6 × 10-19C.
or the potential difference between two [Himachal 96]
points does not depend on the path along
Solution. As nucleus is spherical, it
which the test charge is moved.
behaves like a point charge for external
Examples based on points.
Electric Potential Here q = ne = 79 × 1.6 × 10-19 C,
Work done r = 6.6 × 10-15 m
1. Potential difference = or V =
∴ V =
Charge 9 −19
1 q 9× 10 ×79 ×1.6 ×10
W ⋅ = −15
4 π ε0 r 6.6 × 10
q
V
2. Electric potential due to a point charge
= 1.7 × 107 V.
q at distance r from it,
Example 3. (i) Calculate the potential at a

6
point P due to a charge of 4 × 10-7 C 9 −4
1 q 9× 10 ×2 ×10 6
located 9 cm away, (ii) Hence obtain the ⋅ = =18 ×10 V .
4 π ε0 r 0.10
work done in bringing a charge of 2 × 10-9
C from infinity to the point P. Does the Example 5. Electric field intensity at point
answer depend on the path along which 'B' live to a point charge 'Q' kept at point
the charge is brought ? [NCERT] ‘A' is 24 NC-1 and the electric potential at
point 'B' due to same charge is 12 JC -1.
Solution, (i) Here q = A× 10-7 C, r = 9 cm
Calculate the distance AB and also the
= 0.09 m
magnitude of charge Q. [CBSE OD
Electric potential at point P is 03C]
−7
1 q 9 4 ×10 Solution. Electric field of a point
V= ⋅ =9 ×10 × = 4 × 104 charge,
4 π ε0 r 0.09
V. 1 Q
E= ⋅ 2 = 24 NC-1
4 π ε0 r
Electric potential of a point charge,
1 Q
Fig. 2.10 V= ⋅ = 12 JC-1
4 π ε0 r
(ii) By definition, electric potential at point
The distance AB is given by
P is equal to the work done in bringing a
unit positive charge from infinity to the V 12
r= = = 0.5 m
point P. Hence the workdone in bringing a E 24
charge of 2 × 10-9C from infinity to the
point P is The magnitude of the charge,

W = q0 V =2 × 10-9 × 4 × 104 = 8 × 10-5 J 1


Q = 4π ε0 Vr = 9 × 12 × 0.5 = 0.667
No, the answer does not depend on
9 ×10
× 10-9 C
the path along which the charge is
brought. Example 6. To What potential we must
charge an insulated sphere of radius 14
Example 4. A metal wire is bent in a circle
cm so that the surface charge density is
of radius 10 cm It is given a charge of 200
equal to 1 μ Cm-2 ?
μC which spreads on it uniformly.
Solution. Here r = 14 cm = 14 × 10-2 m,
Calculate the electric potential at its
centre. σ = 1 μ Cm-2 = 10-6 Cm-2
[CBSE OD 95C] ∴ V =
2
Solution. Here q =200 μC =2 × 10-4 C, 1 q 1 4π r σ 1
⋅ = ⋅ = ⋅4 πrσ
4 π ε0 r 4 π ε0 r 4 π ε0
r = 10 cm = 0.10 m
We can consider the circular wire to be 22
= 9 × 109 × 4 × × 14 × 10-2 × 10-6 V
made of a large number of elementary 7
charges dq. Potential due to one such = 15840 V.
elementary charge dq at the centre,
Example 7. A charge of 24 μC is given to
1 dq a hollow metallic sphere of radius 0.2 m
dV = ⋅
4 π ε0 r Find the potential [CBSE D 95]
Total potential at the centre of the (i) at the surface of the sphere, and
circular wire, (ii) at a distance of 0.1 cm from the centre
1 dq 1 of the sphere.
V =ΣdV =Σ ⋅ = Σdq
4 π ε0 r 4 π ε0 r Solution, (i) q = 24 μC = 24 × 10 -6 C, R =
= 0.2 m

7
Potential at the surface of the sphere is and OP = x. For x < 0 (i.e., to the left of
9 −6 O), the potentials of the two charges
1 q 9 ×10 × 24 ×10 cannot add up to zero. Clearly, x must be
V = ⋅ = V =
4 π ε0 R 0.2 positive. If x lies between O and A then
1.08 × 106 V.
V1 + V 2 = 0
(ii) As potential at any point inside the
sphere = Potential on the surface
∴ Potential at a distance of 0.1 cm from
1 q1
[
+
q2
4 π ε 0 x 0.15−x
=0 ]
[ ]
the centre = 1.08 × 106 V. 3 ×10
−8
2 ×10
−8
or 9 ×10 9 − =0
Example 8. Twenty seven drops of same x 0.15−x
size are charged at 220 V each. They
coalesce to form a bigger drop. Calculate 3 2
or − =0
the potential of the bigger drop. [Punjab x 0.15−x
01] which gives x = 0.09 m = 9 cm
Solution. Let radius of each small drop = r The other possibility is that x may also
Radius of large drop = R lie on OA produced, as shown in Fig. 2.12.
4 4
Then π R3 = 27 × π r3
3 3
or R = 3r
Potential of each small drop, Fig. 2.12
1 q As V1 + V2=0
V= ⋅
4 π ε0 r
∴ Total charge on 27 drops, ∴ [ ]
−8 −8
1 3 × 10 2× 10
− =0
4 π ε0 x x−0.15
Q = 27q = 27 × 4πε0rV
which gives x =0.45 m = 45 cm
Potential of large drop,
Thus the electric potential is zero at 9
cm and 45 cm away from the positive
charge on the side of the negative charge.
1 Q 1 27 × 4 π ε 0 rV
V’ = ⋅ = ⋅ Example 10. Calculate the electric
4 π ε0 R 4 π ε 0 3r
potential at the centre of a square of side
= 9 V = 9 × 220 = 1980 V. √ 2 m, having charges 100 μC, - 50 μC, 20
Example 9. Two charges 3 × 10 -8 C and - μC, and - 60 μC at the four corners of the
2 × 10-8 C are located 15 an apart. At what square.
point on the line joining the two charges is [CBSE OD 06C]
the electric potential zero ? Take the
potential at infinity to be zero.[NCERT] Solution. Diagonal of the square

Solution. As shown in Fig. 2.11, √ 2


= ( √ 2) +( √ 2) =2 m
2

suppose the two point charges are placed


on X-axis with the positive charge located Distance of each charge from the centre
on the origin O. of the square is
r = Half diagonal = 1 m
∴ Potential at the centre of the square is

Fig. 2.11 Zero of electric potential for two


charges.
V=
1
[
q1 q 2 q3 q 4
+ + +
4 π ε0 r r r r ]
V = 9 × 109
Let the potential be zero at the point P

8
[ ]
'
60× 10 Work done in moving the charge ' e from
−6 −6 −6 −6
100 ×10 50 × 10 20 ×10
− + − O to F is
1 1 1 1
= 9 × 109 × 10-6 × 10 = 9 × 104 V.
Example 11. Four charges + q, + q, - q
VV =e[V F−V O ]=e
[ (√ ) ]
q
π ε 0a
1
5
−1 −0

and - q are placed respectively at the


corners A, B, C and D of a square of side
'a' arranged in the given order. Calculate
¿
qe
( 1
π ε0 a √ 5 )
−1 .
the electric potential at the centre O. If E
Example 12. A short electric dipole has
and F are the midpoints of sides BC and
CD respectively, what will be the work dipole moment of 4 ×1 0−9 Cm Determine
done in carrying a charge 'e' from O to E the electric potential due to the dipole at¿
and from O to F ? point distant 0.3 m from the centre of the
dipole situated (a) on the axial line (b) on
equatorial line and (c) 0 f 1 n line making
an angle of 6 0 o with the dipole axis.
Solution. Here p=4 ×1 0−9 Cm, r =0.3 m.
(a) Potential at a point on the axial line is
9 −9
1 p 9 ×1 0 × 4 ×1 0
V= . 2= V.
4 π ε0 r ¿¿
Fig. 2.13 (b) Potential at a point on the equatorial
Solution. Let OA = line ¿ 0.

OB = OC = OD = r. (c) Potential at a point on a line that


makes an angle of 6 0 o with dipole axis is
Then the potential at the centre O is
1 p cosθ
[ ]
1 q q q q V= .
V O= + − − =0 4 π ε0 r
2
4 π ε0 r r r r
9 −9 o
9 ×1 0 ×4 ×1 0 cos 6 0 V.
Again, the potential at point E is ¿
¿¿
V E=
1
[q
+
q

q
4 π ε 0 AE BE CE DE

q
=0
] Example 1 3. Two point charges of +3μC
and -3 μC are placed 2 × 10-3 m apart
from each other. Calculate (i) electric field
¿, BE ¿ CE ¿ and electric potential at a distance of 0.6


m from the dipole in broad-side-on
Now, AF=BF= a2 +
() a 2 √5 a
2
=
2
position (ii) electric field and electric
potential at the same point after rotating
the dipole through 90°.
The potential at point F is
Solution. Dipole moment,
V F=
1
4 2 t ε0 [ q
+
q

q
AP BF CF DF

q
] p = q × 2l = 3 × 10 -6 × 2 × 10-3 = 6 × 10-
9
Cm
¿
2q
[ 1

4 n ε 0 AF CF
1
]
[∵ AF=BF ,CF=DF ] (i) Electric field in broad-side-on position is
E =

[√ ] ( √15 −1)
9 −9
2q 2 2 q 1 p 9 ×10 × 6× 10 −1
¿ − = ⋅ 3= =250 NC
4 π ε0 5 a a π ε0 a 4 π ε0 r ( 0.6)
3

Work done in moving the charge ‘e’ from Electric potential in broad-side-on position,
O to E is V = 0.
W =e [V E −V O ]=e ×0=0 (ii) When the dipole is rotated through

9
90?? the same point is now in end-on- [CBSE D 06] (Ans. 0.5 m, 0.55 × 10-9C)
position with respect to the dipole.
4. Two points A and B are located in
∴E=
1 2p diametrically opposite directions of a
⋅ 3 = 500 NC-1 point charge of + 2 μC at distances 2.0
4 π ε0 r
m and 1.0 m respectively from it.
9 −9
1 p 9 ×10 ×6 × 10 Determine the potential difference V A -
V= ⋅ = = 150 V. VB.
4 π ε0 r2 ( 0.6)
2

(Ans. - 9 × 103 V)
4πε0 r (0.6)2
5. A hollow metal sphere is charged with
Example 14. Two charges -q and +q are
0.4 μC of charge and has a radius of
located at points A(0,0,-a) and B(0,0,+a)
0.1 m. Find the potential (i) at the
respectively. How much work is done in
surface (ii) inside the sphere (iii) at a
moving a test charge from point P(7,0,0)
distance of 0.6 m from the centre. The
to Q(-3,0,0) ? [CBSE D 09]
sphere is placed in air. (Ans. 36 kV,
Solution. Points P and Q are located 36 kV, 6 kV)
on the equatorial line of the electric dipole
6. Two point charges of + 10 μC and +
and potential of the dipole at any
20 μC are placed in free space 2 cm
equatorial point is zero.
∴ Work done in moving a test charge q0
apart. Find the electric potential at the
middle point of the line joining the two
from P to Q, charges. (Ans. 27 MV)
W = q0 (VQ - VP) = q0(0 - 0) = 0. 7. Two point charges q and -2q are kept
'd' distance apart. Find the location of
the point relative to charge 'q’ at which
potential due to this system of charges
is zero. [CBSE OD 14C]
(Ans. At distance d / 3 from charge q)
8. Two point charges, one of + 100 μC
and another of - 400 μC, are kept 30
cm apart. Find the points of zero
potential on the line joining the two
charges (assume the potential at
infinity to be zero).
Fig. 2.14
(Ans. 6 cm from + 100 μC charge)
Problems For Practice
9. A charge q = + 1 μC is held at O
1. The work done in moving a charge between the points A and B such that
of 3 C between two points is 6J. What is AO = 2 m and BO = 1 m, as shown in
the potential difference between the two Fig. 2.15(a). Calculate the potential
points ? (Ans. 2 V) difference (VA - VB). What will be the
2. The electric potential at 0.9 m from value of the potential difference (VA -
a point charge is + 50 V. What is the VB) if position of B is changed as
magnitude and sign of the charge ? shown in Fig. 2.15(b) ? (Ans. - 4500 V,
[CBSE D 95C] - 4500 V)

(Ans. 5 × 10-9 C, positive)


3. The electric field at a point due to a
point charge is 20 NC-1 and the electric
potential at that point is 10 JC-1.
Calculate the distance of the point
from the charge and the magnitude of
the charge.

10
(Ans. 2.7 × 104 V)
14. Charges of + 1.0 × 10 -11C, -2.0 × 10-11
C, + 1.0 × 10-11 C are placed
respectively at the comers B, C and D
of a rectangle ABCD. Determine the
potential at the comer A. Given AB = 4
cm and BC = 3 cm.
(Ans. 1.65 V)
Fig. 2.15 15. ABCD is a square of side 0.2 m.
Charges of 2 × 10-9, 4 × 10-9, 8 × 10-9
10. Two small spheres of radius 'a' each C are placed at the corners A, B and C
carrying charges + q and - q are respectively. Calculate the work
placed at points A and B, distance ‘d‘ required to transfer a charge of 2 × 10 -
apart. Calculate the potential 9
C from D to the centre O of the
difference between points A and B. square. [Karnataka 88]
(Ans. 2q/4πε0d)
(Ans. 6.27 × 10-7 J)
11. The sides of rectangle ABCD are 15
cm and 5 cm, as shown in Fig. 2.16. 16. Positive charges of 6,12 and 24 nC
Point charges of -5μC and + 2μC are are placed at the three vertices of a
placed at the vertices B and D square. What charge must be placed
respectively. Calculate electric at the fourth vertex so that total
potentials at the vertices A and C. Also potential at the centre of the square is
calculate the work done in carrying a zero ? (Ans. - 42 nC)
charge of 3 μC from A to C. (Ans. 17. Two equal charges, 2.0 × 10-7 C each
2.52 J) are held fixed at a separation of 20
cm. A third charge of equal magnitude
is placed midway between the two
charges. It is now moved to a point 20
cm from both the charges. How much
work is done by the electric field during
the process ? (Ans. - 3.6 × 10-3 J)
Fig. 2.16
18. ABC is a right-angled triangle, where
12. Charges of 2.0 × 10-6 C and 1.0 × 10- AB and BC are 25 cm and 60 cm
6
C are placed at the corners A and B respectively; a metal sphere of 2 cm
of a square of side 5.0 cm as shown in radius charged to a potential of 9 × 10 5
Fig. 2.17. How much work will be done V is placed at B. Find the amount of
in moving a charge of 1.0 × 10 -6C from work done in carrying a positive
C to D against the electric field ? (Ans. charge of 1 C from C to A.
0.053 J)
(Ans. 0.042 J)

Fig. 2.17 Fig. 2.18


13. Calculate the potential at the centre of HINTS
a square ABCD of each side √ 2 m due
to charges 2, - 2, - 3 and 6 μC at four W 6J
1. V = = = 2 V.
corners of it. [Haryana 97] E 3C

11
. ∴ 50 = 9 × 109 ×
1 q q 7. Let the point P of zero potential lie at
2. As V = distance x from the charge q.
4 π ε0 r 0.9
50× 0.9 -9
or q = 9 = 5 ×10 C


9 ×10
As the potential is positive, the charge q
must be positive. 1 q 1 (−2 q) 1 2 d
⋅ + ⋅ =0 or = or x=
4 π ε 0 x 4 π ε 0 (d−x) x d−x 3
3. Electric field of a point charge,
1 q 8. Suppose the point of zero potential is
E= ⋅ = 20 NC-1 located at distance x metre from the
4 π ε0 r2 charge of + 100 μC. Then

[ ]
Electric potential of a point charge, −6 −6
1 100 × 10 400 ×10
1 q V= − =0.
V= ⋅ = 10 JC-1 4 π ε0 x 0.30−x
4 π ε0 r
This gives x = 0.06 m = 6 cm i.e., the point
V 10 of zero potential lies at 6 cm from the
Clearly, distance r = = = 0.5 m charge of + 100 μC.
E 20
Magnitude of charge, 9.

[ ]
−6 −6
10× 0.5 9 1.0 ×10 1.0 ×10
9 V A −V B =9 ×10 −
q = 4πε0 . V. r = 9 = 0.55 × 10 C. 2.0 1.0
9 ×10
= - 4500 V
4. Here q = 2 μC = 2 × 10 -6 C, rA = 2 m, rB
=1m As potential is a scalar quantity, so
change in position of the charge does not
V A −V B =
q
[1 1

4 π ε0 r A r B ] affect the value of potential.
1 +q 1
10. V B−V A = . − .
= 2 × 10-6 × 9 × 109 [ ] 1 1
− V
2 1 −q
=
2q
.
4 π ε0 d 4 π ε0

= - 9 × 103 V. d 4 π ε0 d
5. (i) Potential at the surface, 11.

[ ]
−6 −6
−7 9 9 2× 10 5 × 10 5
1 q 4 ×10 ×9 ×10 V A =9 ×1 0 − =−7.8 ×1 0 V
V= ⋅ = 0.15 0.05
4 π ε0 r 0.1

[ ]
−6 −6
= 36000 V = 36 kV. 9 2 ×1 0 5 ×1 0 5
V C =9× 1 0 − =0.6 ×1 0 V
0.05 0.15
(ii) Potential inside a hollow conductor is
the same as on its surface. −6 5 5
W =q (V C −V A )=3.0 × 10 (0.6 × 10 +7.8 × 10 )
(iii) When r = 0.6 m, ¿ 2.52 J.

[ ]
9 −7
9 ×10 × 4 ×10 1 q1 q2
V= = 6000 V = 6 kV. 12. V C = +
0.6 4 π ε 0 AC BC

6. V = V 1 +V 2=
1
[
q1 q2
+
4 π ε 0 r 1 r2 ] ¿ 9 ×1 0
9
[ 2.0 ×1 0
√ 2× 0.05
+
1.0 × 10
0.05
−6 −6

]
[ ] [√ ]
−6 −6
= 9 × 109
10 ×10
+
20 ×10 2+ √ 2
¿ 9000 V
0.01 0.01 2× 0.05
= 27 × 106 V = 27 MV.

12
q1 q2
[ ] [ ]
−9 −9 −9 −9
1 9 6 ×1 0 12 ×1 0 24 ×1 0 q ×1 0
V D= + V O =9 ×1 0 + + + =
4 π ε 0 AD BD x x x x

[ ]
2.0 ×1 0 1.0 × 10
−6 −6 6 12 24 q
¿ 9 ×1 0
9
+ or + + + =0
0.05 √2 × 0.05 x x x x
q −42
¿ 9000 √
[
2 2+.1
√ 2× 005
V
] or
x
=

or q=−42nC .
x

W =q (V D −V C )
17. The situation is shown in Fig. 2.19.

¿ 1.0 ×1 0−6 × 9000


[ √√2 2+1−2− √ 2
2× 0.05 ] V C −V D =
1 2 ×1 0
4 π ε 0 0.20[+
2× 10
0.20
−7 −7

]
¿ 0.053 J.
[ ]
−7 −7
−1 2 ×1 0 2× 1 0
13. Diagonal of the square +
4 π ε 0 0.10 0.10
¿√¿¿ −4
¿−1.8 ×1 0 V
Distance of each charge from the centre,
r =¿ Half diagonal ¿ 1 m
∴ Potential at the centre of the square is

[ ]
−6 −6 −6 −6
9 2 ×1 0 2 ×1 0 3 ×1 0 6 ×1 0
V =9× 1 0 − − +
1 1 1 1
4
¿ 2.7 ×1 0 V.
AC= √ 4 +3 =5 ¿ 0.05 m ,
2 2
14. cm
AD=BC=0.03 m Fig. 2.19

[ ]
−11 −11 −11
1 1.0 ×1 0 2.0 ×1 0 1.0 ×1 0 W =q (V C −V D )
V= − +
4 π ε0 0.04 0.05 0.03 −7 4 −3
¿−2× 1 0 ×1.8 ×1 0 =−3.6 ×1 0 J.
¿ 1.65 V. 18. Potential of the charged sphere is
15. 1 q
[ ]
2 ×1 0 4 × 10 8 ×1 0
−9 −9 −9 V= .
V D=9× 10 +
9
+ 4 π ε0 r
0.2 0.2 √ 2 0.2
∴ 9 ×1 0 =9 ×1 0 ×
5 9 q
¿ 577.26 V 0.02

[ ]
−9 −9 −9
9 2× 10 4 ×1 0 8× 10 0.02 −6
V O =9 ×1 0 + + or q= =2× 1 0 =2 μC
0.1 √ 2 0.1 √ 2 0.1 √ 2 10
4

¿ 890.82 V Potential at A due to charge q is


9 −6
W =q [V O −V D ]=2 ×1 0 ¿
−9 1 q 9 ×1 0 ×2 ×1 0
V A= . = V
−7
4 π ε0 r 0.25
¿ 6.27 ×1 0 J.
Potential at C due to charge q is
16. Suppose a charge of qnC be placed at
9 −6
the fourth vertex. 9 ×1 0 ×52 ×1 0
V C= V
Let length of half diagonal be x metre. 0.60
Potential difference between A and C is

13
V A −V C =1.8 ×1 0−3
[ 1

1
0.25 0.60]V The quantity
dV
dr
is the rate of change of
potential with distance and is called
1.8× 7 potential gradient. Thus the electric field at
¿ V =0.042 V
300 any point is equal to the negative of the
Work done in moving a charge of + 1C potential gradient at that point. The
from C to A negative sign shows that the direction of
the electric field is in the direction of
W =q ( V A −V C ) =1× 0.042=0.042 J . decreasing potential. Moreover, the field is
in the direction where this decrease is
2.7 RELATION BETWEEN ELECTRIC steepest.
FIELD AND POTENTIAL
Properties relating electric field to electric
10. Show that the electric field at any point potential :
is equal to the negative of the potential
gradient at that point. (i) Electric field is in that direction in which
the potential decrease is steepest.
Computing electric field from electric
potential. As shown in Fig. 2.20, consider (ii) The magnitude of electric field is equal
the electric field due to charge + q located to the change in the magnitude of
at the origin O. Let A and B be two potential per unit displacement (called
adjacent points separated by distance dr. potential gradient) normal to the
The two equipotential surface at the given
point.
11. How can we determine electric
potential if electric field is known at
any point ?
Computing electric potential from electric
Fig. 2.20 Relation between potential and
field. The relation between electric field
field.
and potential is
points are so close that electric field ⃗
E dV
between them remains almost constant. ⃗
E=- E.⃗
or dV = - ⃗ dr
d ⃗r
Let V and V + dV be the potentials at the
two points. Integrating the above equation between
The external force required to move the points ⃗
r 1 and ⃗
r 2 we get
test charge q0 (without acceleration)
against the electric field ⃗
E is given by

F = - q0 ⃗
E ⃗
r2

The work done to move the test charge or V2 - V1 = ∫ ⃗


E ⋅ d r⃗
from A to B is ⃗
r2

W = F . dr = -q0 E. dr where V1 and V2 are the potentials at ⃗ r1


Also, the work in moving the test charge and ⃗
r 2 respectively. If we take ⃗
r 1 at infinity,
from A to B is ⃗
r
then V1 = 0 and put 2 = r⃗ , we get
W = Charge × potential difference ⃗r
V ( ⃗r )=−∫ ⃗
E ⋅ d r⃗
= q0 (VB - VA) = q0 dV ∞

Equating the two works done, we get Hence by knowing electric field at any
-q0E.dr = q0.dV point, we can evaluate the electric
potential at that point.
dV
or E = - 12. Show that the units volt/metre and
dr
newton/coulomb are equivalent. To

14
which physical quantity do they refer ? Example 16. Calculate the voltage needed
to balance an oil drop carrying 10
SI emits of electric field. Electric field at
electrons when located between the
any point is equal to the negative of the
plates of a capacitor which are 5 mm apart
potential gradient. It suggests that the SI
(g = 10 ms-2). The mass of oil drop is 3 ×
unit of electric field is volt per metre. But
10-16 kg.
electric field is also defined as the force
experienced by a unit positive charge, so Solution, q = ne = 10 × 1.6 × 10-19 C,
SI unit of electric field is newton per
m = 3 × 10-16 kg, d = 5mm = 5 × 10-3m
coulomb. Both of these units are
equivalent as shown below. V V −1
E= = Vm
volt joule/coulomb d 5 ×1 0−3

=
metre metre For the charged oil drop to remain
newton – metre newton stationary in electric field,
= =
coulomb−metre coulomb qE=mg
or 1 Vm-1 = 1NC-1 ∴
−19 V −16
Examples based on 10 ×1.6 ×1 0 × −3
=3 ×1 0 ×10
5 ×1 0
Relation between Electric Field and
−16 −3
Potential 3 ×1 0 ×.10 ×5 × 10
or V = −19
=9.47 V.
Formulae Used 10 ×16 × 10
1. Electric field in a region can be Example 17. An infinite plane sheet of
determined from the electric potential charge density 1 0−8 C m−2 is held in air. ln
by using relation, this situation how far apart are two
dV equipotential surfaces, whose p . d . is 5 V
E=- ?
dr
Solution. Electric field of an infinite plane
∂V ∂V ∂V sheet of charge,
or Ex = - , Ey = - , Ez = -
∂x ∂y ∂z
σ
2. Electric field between two parallel E=
2ε0
conductors,
V If Δr is the separation between two
E= equipotential surfaces having potential
d difference ΔV , then
3. Electric potential in a region can be
ΔV
determined from the electric field by E=
using the relation, Δr


r σ ΔV
=
V = - −∫ ⃗
E.⃗
dr 2 ε 0 Δr

2 ε 0 ΔV 2 ×8.85 ×1 0−12 ×5
Units Used or Δr= = −8
σ 10
E is in NC-1 or Vm-1, V in volt, r in metre.
−3
Example 15. Find the electric field ¿ 8.85 ×1 0 m=8.85 mm.
between two metal plates 3 mm apart, Example 1 8 . A spark passes in air when
connected to 12 V battery. the potential gradient at the surface of a
Solution. Electric field, charged conductor is 3 ×1 06 V m−1. What
must be the radius of an insulated metal
V 12 V 3 −1
sphere which can be charged to a
E= = =4 × 10 Vm
d 3 ×10−3 m potential of 3 ×1 06 V before sparking into

15
air? So VB = VA
Solution. Potential gradient, ∆VCA = VC - VA = VC - VB = ΔVCB
dV 6 −1 = -2100 V.
=3 ×1 0 V m
dr Example 20. Three points A B and C lie in
or dV = 3 × 10 dr 6 a uniform electric field (E) of 5 × 103 NC-1
as shown in the figure. Find the potential
or V = 3 × 106 r difference between A and C. [CBSE F 09]
But V = 3 × 106 V
∴ 3 × 106 r = 3 × 106
or r = 1 m.
Example 19. A uniform electric field ⃗E of
300 NC-1 is directed along negative X-
axis. A, B and C are three points in the Fig. 2.22
field, having x and y coordinates (in Solution. Points B and C lie on an

potential differences ∆VBA, ∆VCB and ∆VCA.


metre), as shown in Fig. 2.21. Find the equipotential surface, so VC = VB.
∴ P.D. between A and C = P.D. between A
and B
= -E∆x

= -5 × 103 NC-1 × 4 × 10-2 m [ ∵ E=


∆V
∆x ]
= -200 V. [Δx = AB = √ 52 – 32 = 4 cm]
Example 21. If the potential in the region
of space around the point (-1 m, 2 m, 3 m)
is given by V = (10x2 + 5y2 - 3z2)volt,
calculate the three components of electric
field at this point.
Fig. 2.21
Solution. Here x = -1 m, y = 2m, z = 3 m
Solution, (i) No work is done in moving a
As V = 10x2 + 5y2 - 3z2
unit positive charge from A to B because
∴ E x=
the displacement of the charge is −∂ V −∂ 2 2 2
= (10 x +5 y −3 z )
perpendicular to the electric field. Thus the ∂x ∂x
points A and B are at the same potential.
∴ ∆VBA = 0
−1
¿−20 x=−20 ×(−1)=20 V m .
∂V −∂ 2 2 2
(ii) Work is done by the electric field as E y= = (10 x +5 y −3 z )=−10 y
the positive charge moves from B to C ∂ y dy
−1
(i.e., in the direction of ⃗
E ). Thus the ¿−10 ×2=−20 V m
point C is at a lower potential than the −∂ V −∂ 2 2 2
point B. E z= = (10 x +5 y −3 z )=6 z
∂z dz
ΔV = 6 × 3 = 18 Vm-1.
As E = -
Δx
∴ ∆∆VCB = - E ∆x = - 300 NC-1 × 7 m
Problems For Practice
1. A uniform electric field of 20 NC -1
= - 2100 V. exists in the vertically downward
(iii) Points A and B lie on an equipotential direction. Determine the increase in
surface. the electric potential as one goes up

16
through a height of 50 cm. (Ans. ∴ SI unit of A = NC-1 × m3 = Nm3 C-1
10 V) (x , y, z )
Adx A
2. A uniform electric field of 30 NC -1
Potential, V = - ∫ x 3
= 2
2x
exists along the X-axis. Calculate the ∞

potential difference VB - VA between 5. For the equipotential surface of 60 V,


the points A (4 m, 2 m)and B (10 m, 5
m). kq kq
60 V = =
r 0.10 m
(Ans. - 180 V)
or kq = 60 V × 0.10 m = 6 Vm
3. An electric field ⃗E = 20 i^ + 30 ^j NC-1
∴E=
exists in free space. If the potential at kq 6 -1
2 = 2 Vm
the origin is taken zero, determine the r r
potential at point (2 m, 2 m).
Clearly, E decreases with r. The direction
(Ans. - 100 V) of electric field will be radially outward
4. The electric field in a region is given by because V decreases with r.
A 2.8 EQUIPOTENTIAL SURFACES AND

E= ^
3 i.
x THEIR PROPERTIES

Write the SI unit for A. Write an 13. What is an equipotential surface ?


expression for the potential in the region Give an example.
assuming the potential at infinity to be Equipotential surface. Any surface that

(
zero. Ans . Nm C ,
3 A
2x
−1
2 ) has same electric potential at every point
on it is called an equipotential surface.
The surface may be surface of a body or a
5. Figure 2.23 shows some equipotential surface in space. For example, as we
surfaces. What can you say about the shall see later on, the surface of a
magnitude and the direction of the charged conductor is an equipotential
electric field ? surface. By joining points of constant
6 potential, we can draw equipotential
(Ans. E = -1
2 Vm , radially outward) surfaces throughout the region in which an
r electric field exists.
14. State and prove the important
properties of equipotential surfaces.
Properties of equipotential surfaces : 1.
No work is done in moving a test charge
over an equipotential surface.

Fig. 2.23
HINTS

1. ∆V = - E∆r = - 20 × ( −50
100 )
= 10 V.
Fig. 2.24 An equipotential surface.
2. ∆V = - Ex ∆x = -30(10 - 4) = -180 V. Let A and B be two points over an
equipotential surface, as shown in Fig.
3. ∆V = -Ex Δx - Ey Δy = -20 × 2 – 30 × 2 2.24. If the test charge q0 is moved from A
= -100V.
4. ∵ SI unit of electric field = NC-1
to B, the work done will be

17
WAB = Charge × potential difference distance and (iv) a uniform electric
field.
= q0(VB - VA)
Equipotential surfaces of various charge
As the surface is equipotential, so VB - VA
systems. For the various charge systems,
=0
we represent equipotential surfaces by
Hence WAB = 0. dashed curves and lines of force by full
2. Electric field is always normal to the line curves. Between any two adjacent
equipotential surface at every point. If equipotential surfaces, we assume a
the field were not normal to the constant potential difference.
equipotential surface, it would have a (i) Equipotential surfaces of a positive
non-zero component along the point charge. The electric potential due
surface. So to move a test charge to a point charge q at distance r from it
against this component, a work would is given by
have to be done. But there is no
potential difference between any two
1 q
V= .
points on an equipotential surface and 4 π ε0 r
consequently no work is required to This shows that V is constant if r is
move a test charge on the surface. constant. Thus, the equipotential surfaces
Hence the electric field must be of a single point charge are concentric
normal to the equipotential surface at spherical shells with their centres at the
every point. point charge, as shown in Fig. 2.25. As
3. Equipotential surfaces are closer the lines of force point radially outwards,
together in the regions of strong field so they are perpendicular to the
and farther apart in the regions of equipotential surfaces at all points.
weak field. We know that electric field
at any point is equal to the negative of
potential gradient at that point.
dV dV
i.e., E = - or dr = -
dr E
For the same change in the value of dV
i.e., when dV = constant, we have

dr ∝
1
E
Thus the spacing between the Fig. 2.25 Equipotential surface of a +ve
equipotential surfaces will be smaller in point charge.
the regions, where the electric field is
stronger and vice versa. (ii) Equipotential surfaces of two equal and
opposite point charges : Electric dipole.
4. No two equipotential surfaces can Fig. 2.26 shows the equipotential surfaces
intersect each other. If they interesect, of two equal and opposite charges, + q
then there will be two values of electric and - q, separated by a small distance.
potential at the point of intersection, They are close together in the region in
which is impossible. between the two charges.
2.9 EQUIPOTENTIAL SURFACES OF
VARIOUS CHARGE SYSTEMS
15. Sketch and explain the equipotential
surfaces for : (i) a point charge, (ii) two
point charges + q and -q, separated by
a small distance, (iii) two point charges
+ qand + q separated by a small

18
direction and the magnitude of field ⃗E in a
region of space. If we draw equipotential
surfaces at regular intervals of V, we find
that equipotential surfaces are closer
together in the regions of strong field and
farther apart in the regions of weak field.
Moreover, ⃗E is normal to the equipotential
surface at every point.
Fig. 2.26 Equipotential surfaces for two 2.10 ELECTRIC POTENTIAL ENERGY
equal and opposite charges.
17. What is meant by electric potential
(iii) Equipotential surfaces of two equal energy of a charge system ?
positive charges. Fig. 2.27 shows the
Electric potential energy. It is the energy
equipotential surfaces of two equal and
possessed by a system of charges by
positive charges, each equal to + q,
virtue of their positions. When two like
separated by a small distance. The
charges lie infinite distance apart, their
equipotential surfaces are far apart in the
potential energy is zero because no work
regions in between the two charges,
has to be done in moving one charge at
indicating a weak field in such regions.
infinite distance from the other. But when
they are brought closer to one another,
work has to be done against the force of
repulsion. As electrostatic force is a
conservative force, this work gets stored
as the potential energy of the two charges.
The electric potential energy of a system
of point charges may be defined as the
amount of work done in assembling the
charges at their locations by bringing them
Fig. 2.27 in, from infinity.
(iv) Equipotential surfaces for a uniform 18. Deduce expressions for the potential
electric field. Fig. 2.28 shows the energy of a system of two point
equipotential surfaces for a uniform charges and three point charges and
electric field. The lines of force are parallel hence generalise the result for a
straight lines and equipotential surfaces system of N point charges.
are equidistant parallel planes
perpendicular to the lines of force. Potential energy of a system of two point
charges. Suppose a point charge q1 is at
rest at a point P1 in space, as shown in
Fig. 2.29. It takes no work to bring the first
charge q1 because there is no field yet to
work against.

Fig. 2.28 Equipotential surfaces for a


uniform electric field.
16. Give the importance of equipotential
surfaces.
Importance of equipotential surfaces. Like
Fig. 2.29 P.E. of two point charges.
∴ W1 = 0
the lines of force, the equipotential
surfaces give a visual picture of both the

19
Electric potential due to charge q1 at a charges. The expression for the potential
point P2 at distance r12 from P1 will be energy of N point charges can be written
as
1 q1
V1 = .
4 π ε 0 r 12 qiq j 1 N N
qi q j
1 1
If charge q2 is moved in from infinity to
U= ∑ = . ∑ ∑
4 π ε 0 a ≪ p airs r ij 2 4 π ε 0 i =1 j=i r ij

i≠j
point P2, the work required is
W2 = Potential × charge
1 q1 q2 As double summation counts every pair
= V1 × q 2 = . twice, to avoid this the factor 1/2 has been
4 π ε 0 r 12
introduced.
As the work done is stored as the potential
energy U of the system (q1 + q2), so NOTE The potential at jth charge due to
all other charges can be written as
1 q1 q2 N
U = W1 + W 2 = . . qk
4 π ε 0 r 12 V j= ∑
k=1 , r jk
Potential energy of a system of three point k≠ j
charges. As shown in Fig. 2.30, now we The expression for P.E. of N point
bring in the charge q3 from infinity to the charges can be written as
point P3. Work has to be done against the

[ ]
N N
forces exerted by q1 and q2. 1 1 qk 1 N
U= ∑ qj ∑ = ∑ q j V j.
2 j=1 4 π ε 0 k=1 , r jk 2 j=1
k≠ j

For Your Knowledge


Electric potential energy is a scalar
quantity. While finding its value, the value
of various charges must be substituted
with their proper signs.
The potential energy of two like charges
(q1 q2 > 0) is positive. As the electrostatic
force is repulsive, so a positive amount of
Fig. 2.30 P.E. of three point charges. work has to be done against this force to
Therefore bring the charges from infinity to a finite
separation.
W3 = Potential at point P3 due to q1 and q2
× charge q3 The potential energy of two unlike charges
(q1 q2 < 0) is negative. As the electrostatic
or force is attractive, so a positive amount of
W 3=
1
[
q1 q 2
+
4 π ε 0 r 13 r 23 ]
× q 3=
4 π ε 0 r 13
+
[
1 q 1 q3 q 2 q 3
r 23 ] work has to be done against this force to
take the charges from the given locations
to infinity. Conversely, a negative amount
Hence the electrostatic potential energy of of work is needed to bring the charges
the system q 1+ q2 +q 3 is from infinity to the present locations, so
the potential energy is negative.
U = Total work done to assemble the three
charges As electrostatic force is a conservative
force, so the potential energy of a charge
¿ W 1 +W 2+W 3 configuration is independent of the

[ ]
manner in which the charges are
1 q 1 q 2 q 1 q 3 q2 q 3
or U = + + assembled to the present locations. The
4 π ε 0 r 12 r 13 r 23 potential energy is a characteristic of the
present state of configuration, not on how
Potential energy of a system of N point

20
this state is attained. energy of two point charges q1 and q2,
Positive potential energy implies that work separated by distance r in an electric
can be obtained by releasing the charges, field ⃗
E.
while negative potential energy indicates Potential energy of a system of two point
that an external agency will have to do charges in an external field. Let V (⃗ r 1) )
work to separate the charges infinite
distance apart. and V( (⃗ r 2 ¿ be the electric potentials of
the field ⃗ E at the points having position
Electric potential is a characteristic of an
vectors 1 and ⃗

r r 2 as shown in Fig. 2.31.
electric field, it does not matter whether a
charged object is placed in that field or Work done in bringing q1 from ∞ to ⃗
r1
not. It is measured in JC-1 or volt. On the against the external field
other hand, electric potential energy is the
energy of a charged object in an external = q1V (⃗
r 1)
electric field. More precisely, it is the
energy of the system consisting of the
charged object and the external electric
field (or charges producing that field). It is
measured in joule.
2.11 POTENTIAL ENERGY IN AN
EXTERNAL FIELD
19. Write an expression for the potential
energy of a single charge in an
external field. Hence define electric Fig. 2.31 P.E. of two charges in an
potential. external field.
Potential energy of a single charge. We Work done in bringing q2 from ∞ to ⃗
r2
wish to determine the potential energy of a against the external field
charge q in an external electric field ⃗
E at a
point P where the corresponding external = q2V (r⃗2 )
potential is V. By definition, V at a point P Work done on q2 against the force exerted
is the amount of work done in bringing a by q1
unit positive charge from infinity to the
point P. Thus, the work done in bringing a 1 q1 q2
= .
charge q from infinity to the point P will be 4 π ε 0 r 12
qV, i.e., W = qV
where r12 is the distance between q1 and
This work done is stored as the potential q2.
energy of the charge q. If r⃗ is the position
vector of point P relative to some origin, Total potential energy of the system = The
then work done in assembling the two charges

U (r⃗ ) = qV ¿) 1 q1q2
or U = q1V(⃗
r 1 )+ q2 V (⃗
r 2 )+ ⋅
4 π ε 0 r 12
P.E. of a charge in an external field
= Charge × external electric potential 21. Define electron volt. Express it in
joule.
U
As V = Units of electrostatic potential energy.
q Suppose an electron (q = 1.6 × 10-19 C) is
So we can define electric potential at a moved through a potential difference of 1
given point in an external field as the volt, then the change in its P.E. would be
potential energy of a unit positive charge ∆U = q∆V = 1.6 × 10-19 C × 1 V = 1.6 × 10-
at that point. 19
J
20. Write an expression for the potential This is a commonly used unit of energy in

21
atomic physics and we call it electron volt If the dipole is rotated through a small
(eV). angle dθ against the torque acting on it,
then the small work done is
dW = τ dθ = pE sin θ dθ
Thus electron volt is the potential energy
gained or lost by an electron in moving
through a potential difference of 1 volt.
The total work done in rotating the dipole
1 eV = 1.6 × 10-19 J from its orientation making an angle θ 1,
Multiples and submultiples of eV with the direction of the field to θ2 will be
θ2
1 meV (milli electron volt)
W = ∫ dW =∫ pEsinθdθ
= 10-3 eV = 1.6 × 10-22 J θ1

θ2
1 keV (kilo electron volt) = pE [−cosθ ]θ = pE (cos θj - cos θ2)
1

= 103 eV = 1.6 × 10-6 J This work done is stored as the potential


1 MeV (million electron volt) energy U of the dipole.
= 106 eV = 1.6 × 10-3 J U = pE (cos θ1 - cos θ2)
1 GeV (giga electron volt) If initially the dipole is oriented
perpendicular to the direction of the field
= 109 eV = 1.6 × 10-10 J (θ1 = 90°) and then brought to some
1 TeV (tera electron volt) orientation making an angle θ with the
field (θ2 = θ), then potential energy of the
= 1012 eV =1.6 × 10-7 J.
dipole will be
2.12 POTENTIAL ENERGY OF A DIPOLE
U = pE (cos 90° - cos θ) = pE (0 - cos θ)
IN A UNIFORM ELECTRIC FIELD
22. Derive an expression for the potential or U = - pE cos θ = - ⃗p . ⃗
E
energy of a dipole in a uniform electric Special Cases
field. Discuss the conditions of stable
and unstable equilibrium. 1. Position of stable equilibrium. When θ -
0°,
Potential energy of a dipole placed in a
uniform electric field. As shown in Fig. U = -pE cos 0° = - pE
2.32, consider an electric dipole placed in Thus the potential energy of a dipole is
a uniform electric field ⃗
E with its dipole minimum when its dipole moment is
moment ⃗
p making an angle θ with the parallel to the external field. This is the
field. position of stable equilibrium.

Two equal and opposite forces + q ⃗E and 2. Position of zero energy. When θ = 90°,
-q⃗ E act on its two ends. The two forces U = -pE cos 90° = 0.
form a couple. The torque exerted by the Thus the potential energy of a dipole is
couple will be
τ = qE × 2 a sin θ = pE sin θ
zero when it is held perpendicular to the
external field. This can be explained as
follows. If we hold the dipole perpendicular
where q × 2a = p, is the dipole moment. to the electric field and bring it from infinity
into the field, then the work done on
charge + q by the external agent is equal
to the work done on charge - q. The net
work done on the dipole will be zero and
hence its potential energy is zero.
3. Position of unstable equilibrium. When
θ =180°,
Fig. 2.32 Torque on a dipole in a uniform U = - pE cos 180° = + Pe
electric field.

22
Thus the potential energy of a dipole is field E = A(1/r2); A = 9 × 105 Cm-2.
maximum when its dipole moment is What would the electrostatic energy of
antiparallel to the external field. This is the the configuration be ? [NCERT]
position of unstable equilibrium.
Solution, (a) q1 = 7 μC = 7 × 10-6 C, q2 = -2
Examples based on × 10-6 C, r = 18 cm = 0.18 m
Electric Potential Energy Electrostatic potential energy of the two
charges is
Formulae Used
1. Electric potential energy of a system of
1 q q
U= . 1 2
two point charges, 4 π ε0 r
9 −6 −6
1 q1 q2 9 ×10 ×7 ×10 ×(−2)×10
U= . = = -0.7 J.
4 π ε 0 r 12 0.18
2. Electric potential energy of a system of (b) Work required to separate two charges
N point charges, infinitely away from each other,
1 q j qk W = U2 - U1 = 0 – U = -(-0.7) = 0.7 J.
U= Σ
4 π ε0 all pairs r jk (c) Energy of the two charges in the
external electric field = Energy of
3. Potential energy of an electric dipole in interaction of two charges with the
a uniform electric field, external electric field + Mutual
U = - pE (cos θ2 - cos θ1) interaction energy of the two charges
If initially the dipole is perpendicular to the 1 q1 q2
field = q1V (r1) + q2V (r2) +
4 π ε0 r2
E, θ1 = 90° and θ2 = θ (say), then
U = - pEcos θ = - ⃗p.⃗
E
[ V =Er=
A
r ]
If initially the dipole is parallel to the field A A 1 q1 q2
= q1 + q2 +
E, r1 r2 4 π ε 0 r2

[ ]
θ1 = 0° and θ2 = θ (say), then 7 μC −2 μC
= + × 9 × 105 Cm-2 - 0.7 J
U = - pE (cos θ - 1) = pE (1 - cos θ) 0.09 m 0.09 m
Units Used = (70 - 20) -0.7 = 50 - 0.7 = 49.3 J.
Charges are in coulomb, distances in Example 23. Three charges - q, + Q and -
metre, energy in joule or in electron volt q are placed at equal distances on a
(eV) and dipole moment in coulomb metre straight line. If the potential energy of the
(Cm). system of three charges is zero, find the
1eV = 1.6 × 10-19 C, 1 MeV = 1.6 × 10-13 C. ratio Q/q.

Example 22 Solution. As shown in Fig. 2.33, suppose


the three charges are placed at points A,
(q) Determine the electrostatic potential B and C respectively on a straight line,
energy of a system consisting of two such that AB = BC = r.
charges 7 μC and -2 μC (and with no
external field) placed at (-9 cm, 0,0)
and (9 cm, 0,0) respectively.
(b) How much work is required to
separate the two charges infinitely Fig. 2.33
away from each other ? As the total P.E. of the system is zero, so
(c) Suppose the same system of charges
is now placed in an external electric

23
[ ]
1 −qQ (−q )(−q ) Q (−q ) q = ±4 × 10-5C, r1 = 1.0 m, r2 = 50 cm =
+ + =0 0.50 m
4 π ε0 r 2r r
Let v = speed of each particle at the
q q Q 1 separation of 50 cm.
or -Q + - Q =0 or 2Q = or = =
2 2 d 4
1:4. From energy conservation principle,

Example 24. Two positive point charges of K.E. of the two particles at 50 cm
0.2 μC and 0.01 μC are placed 10 cm separation + P.E. of the two particles at 50
apart. Calculate the work done in reducing cm separation
the distance to 5 cm = P.E. of the two particles at 1.0 m
Solution. Here q1 =0.2 × 10-6C, q2 =0.01 × separation
10-6C 1 1 1 q 1 q2 1 q1q2
mv2 mv2 + ⋅ = ⋅
Initial separation (ri) =10 cm = 0.10 m 2 2 4 π ε0 r2 4 π ε0 r 1

[ ]
Final separation (rf) = 5 cm = 0.05 m q1 q2 1 1
mv2 = − or v2 =
Work done = Change in potential energy 4 π ε0 r1 r2
= Final P.E. - Initial P.E. q1 q2 r 2−r 1
[ ]
= 4 π ε0 m r1 r 2
∴ v2 =
1

q 1 q2
− =
[
1 q 1 q 2 q 1 q2 1 1

]
[ ]
4 π ε0 rf 4 π ε0 ri 4 π ε0 r f r i 4 × 10−5 × (−4 ×10−5 ) × 9 ×109 0.50−1.0
−3
= 0.2 × 10-6 × 0.01 × 10-6 × 9 × 109 5× 10 1.0 × 0.50

[ 1

1
0.05 0.10 ] = 2880 or v = 53.67 ms-1.
Example 27. Four charges are arranged at
= 1.8 × 10 J. -4
the corners of a square ABCD of side d as
shown in Fig. 2.34. (i) Find the work
Example 25. Two electrons, each moving
required to put together this arrangement,
with a velocity of 106 ms-1, are released
(ii) A charge q0 is brought to the centre E
towards each other. What will be the
of the square, the four charges being held
closest distance of approach between
fixed at its corners. How much extra work
them ?
is needed to do this ? [NCERT; CBSE F
Solution. Let r0 be the distance of closest 15]
approach of the two electrons. At this
distance, the entire K.E. of the electrons
changes into their P.E. Therefore,
1 1 1 ee
mv2 + mv2 =
2 2 4 π ε0 r0
9 −19 2
1 e 2 9 ×10 ×(1.6 ×10 )
r0 = . =
4 π ε0 m v 2 9.1×10−31 ×(106 )2
Fig. 2.34
= 2.53 × 10-10 m.
Solution. (i) Given AB = BC = CD = AD =
Example 26. Two particles have equal d
∴ AC = BD = √ d 2 +d 2 = √ 2d
masses of 5.0 g each and opposite
charges of + 4 × 10-5C and -4.0 × 10-5 C.
They are released from rest with a
Work required to put the four charges
separation of 1.0 m between them. Find
together
the speeds of the particles when the
separation is reduced to 50 cm = Total electrostatic P.E. of the four
charges
Solution. Here m = 5.0 g = 5 × 10-3 kg,

24
[ ]
= 1 Q ×2 Q 2 Q × (−3 Q ) Q × (−3 Q )

[ ]
q A q B q A q C q A q D qB q C q B q D qC q D + +
1 4 π ε0 l/2 l/2 l/2
+ + + + +
4 π ε 0 AB AC AD BC BD CD 2
1 14 Q

[ ]
2 2 2 2 2 2 =-
1 −q q q q q q 4 π ε0 l
= + − − + −
4 π ε0 d √ 2 d d d √ 2 d d Work done = Uf - Ui
2
q = -
=- (4− √ 2). 2 2 2
4 π ε0 1 14 Q 1 7Q −1 7 Q
⋅ + = ⋅
(ii) Extra work needed to bring charge q0 4 π ε0 l 4 π ε0 l 4 π ε0 l
to centre E .
W = q0 × Electrostatic potential at E due to Example 29. An electric dipole of length 2
the four charges cm is placed with its axis making an angle
of 60° to a uniform electric field of 10 5 NC-
= 1
. If it experiences a torque of 8 √ 3 Nm,

[ ]
q −q q −q calculate the
+ + +
4 π ε0
( ) d
√2
4 π ε0
( )
d
√2
4 π ε0
( )
d
√2
4 π ε0
( )
d (i) magnitude of the charge on the dipole,
√2 and
=0 (ii) potential energy of the dipole.[CBSE
Example 28. Three point charges, +Q,+2Q OD 2000]
and -3Q are placed at the vertices of an Solution. Here 2a = 2 cm = 0.02 m, θ =
equilateral triangle ABC of side l (Fig. 60°,
E = 105 NC-1, τ = 8√ 3 Nm
2.35). If these charges are displaced to
the midpoints A1 , B1 and C1 respectively,
find the amount of the work done in (i) τ = pE sin θ = q × 2a × E sin θ
∴ 8√ 3 = q × 0.02 × 105 × sin 60°
shifting the charges to the new locations.
[CBSE OD 2015]
8 √ 3 ×2
or q = = 8 × 10-3 C.
0.02× 105 × √ 3
(ii) P.E. of the dipole is
U = - pE cos 0 = -q × 2a × E cos θ
= - 8 × 10-3 × 0.02 × 105 × cos 60° = - 8 J.
Fig. 2.35 Example 30. An electric dipole of length 4
AB 1 an, when placed with its axis making an
Solution. A1B1 = B1C1 = A1C1 = = angle of 60° with a uniform electric field
2 2
experiences a torque of 4 √ 3 Nm
Initial P.E. of the system is Calculate the (i) magnitude of the electric
1 field, (ii) potential energy of the dipole, if
Ui = the dipole has charges of ±8 nC. [CBSE
4 π ε0
OD 04 ; D 06C, 14]

[ Q ×2 Q 2 Q × (−3 Q ) Q × (−3 Q )
l
+
l
+
l ] Solution. Here 2a = 4 cm = 0.04 m, 0 =
60°, x = 4 √ 3 Nm, q = 8 nC = 8 × 10-9 C
1 7Q
2
Dipole moment,
=- .
4 π ε0 l p = q × 2a = 8 × 10-9 × 0.04 = 0.32 × 10-9
Final P.E. of the system is Cm.
Uf = (i) As τ = pE sin θ

25
∴E=
τ 4 √3 3. Two point charges 20 × 10-6C and -4×
= 10-6C are separated by a distance of
p sin θ 0.32 ×10−9 × sin 60°
50 cm in air. (i) Find the point on the
4 √3 ×10 9 ×2 line joining the charges, where the
= = 2.5 × 1010 NC-1. electric potential is zero, (ii) Also find
0.32× √3
the electrostatic potential energy of the
(ii) U = - pE cos θ system. [CBSE OD 08]
= - 0.32 × 10-9 × 2.5 × 1010 × cos 60° = - 4 [Ans. (i) 41 cm from the charge of 20 × 10 -
J. 6
C (ii) - 144 J]
Example 31. A molecule of a substance 4. Two charges, of magnitude 5 nC and -
has permanent electric dipole moment 2 nC, are placed at points (2 cm, 0, 0)
equal to 10-29 Cm. A mole of this and (x cm, 0, 0) in a region of space,
substance is polarized (at low where there is no other external field.
temperature) by applying a strong If the electrostatic potential energy of
electrostatic field of magnitude (106 Vm-1). the system is - 0.5 μj, what is the value
The direction of the field is suddenly of x ?
changed by an angle of 60°.
[CBSE D 08C] (Ans. x-4cm)
Estimate the heat released by the
substance in aligning its dipoles along the 5. Three point charges are arranged as
new direction of the field. For simplicity shown in Fig. 2.36. What is their
assume 100% polarization of the sample. mutual potential energy ? Take q = 1.0
[NCERT] × 10-4 C and a = 10 cm. (Ans. 0.27 J)

Solution. Here p = 10-29Cm, E = 106Vm-1,


θ = 60°, N = 6 × 1023
Work required to bring one dipole from
position θ = 0° to position θ is
W = pE - pE cos θ = pE( 1 - cos θ) Fig. 2.36

= 10-29 × 106(1 - cos 60°) J = 0.5 × 10-23 J


Work required for one mole of dipoles
= W × N =0.5 × 10-23 × 6 × 1023 = 3.0 J
Heat released = Loss in P.E.= Work done Fig. 2.37
= 3.0 J.
6. Determine potential energy of the
Problems For Practice charge configuration shown in Fig.
1. Two point charges +10μC and -lOμC 2.37.
are separated by a distance of 2.0 cm
( )
2
q
in air. (i) Calculate the potential energy Ans. (− √ 2 )
of the system, assuming the zero of 4 π ε0 a
the potential energy to be at infinity. (ii)
Draw an equipotential surface of the 7. Find the amount of work done in
system. arranging the three point charges, on
the vertices of an equilateral triangle
[CBSE D 04] (Ans. - 45 J) ABC, of side 10 cm, as shown in the
2. Two point charges A and B of values + adjacent figure. [CBSE Sample Paper
15 μC and + 9 μC are kept 18 cm 20111
apart in air. Calculate the work done
when charge B is moved by 3 cm
towards A. [CBSE OD 2000] (Ans.
1.35 J)

26
12. Two identical particles, each having a
charge of 2.0 × 10-4 C and mass of 10
g, are kept at a separation of 10 cm
and then released. What would be the
speeds of the particles when the
separation becomes large ? (Ans.
600 ms-1)
(Ans. - 3.24 J ) 13. Find the amount of work done in
rotating an electric dipole, of dipole
8. Calculate the work done to dissociate moment 3.2 × 10-8 Cm, from its
the system of three charges placed on position of stable equilibrium, to the
the vertices of a triangle as shown in position of unstable equilibrium, in a
Fig. 2.38. Here q - 1.6 × 10-10 C. uniform electric field of intensity 10 4 N /
[CBSE D 08 ; OD 13] (Ans. 2.304 × 10-8 J) C. [CBSE Sample Paper 2011]
(Ans. 6.4 × 10-4J)
14. An electric dipole consists of two
opposite charges each of magnitude 1
μC separated by 2 cm. The dipole is
placed in an external electric field of
105NC-1. Find (i) the maximum torque
exerted by the field on the dipole (ii)
the work which the external agent will
Fig. 2.38 have to do in turning the dipole
through 180° starting from the position
θ = 0°.
[Ans. (i) 2 × 10-3 Nm (ii) 4 × 10-3 J]
HINTS
1 q1 q2
1. U = .
4 π ε0 r
Fig. 2.39
−6 −6
9. What is the electrostatic potential 10 ×1 0 . ×(−10)×1 0
¿ 9 ×1 09 × =−45
energy of the charge configuration 20 ×1 0−2
shown in Fig. 2.39 ? Take J.
-8 -8
q1 = + 1.0 × 10 C, q2 = - 2.0 × 10 C, For equipotential surface, see Fig. 2.26 on
-8
q3=+ 3.0 × 10 C, q4 = + 2.0 x 10 C-8 page 2.15.

and a - 1.0 metre. (Ans. - 6.36 × 10-7 J) 2. W =¿ Final P.E. ‐ Initial P.E.
10. Three point charges + q, + 2q and
Qare placed at the three vertices of an
equilateral triangle. Find the value of
¿
[
q1 q2 1 1

4 π ε0 r 2 r1 ]
charge Q (in terms of q), so that
electric potential energy of the system
is zero. (Ans. Q = - 2q / 3)
¿ 9 ×1 09 ×15 ×1 0−6 × 9 ×1 0−6
[ 100 100
15

18 ]
¿ 1.35 J.
11. An electron (charge = -e) is placed at
each of the eight corners of a cube of 3. (i) Suppose the point of zero potential is
side a and an α-particle (charge = + located distance x metre from the charge
2e) at the centre of the cube. Calculate of 20 ×1 0−6 C.
the potential energy of the system.

[ ]
−6 −6
1 20 ×1 0 4 ×1 0
(Ans. 3.89 × 1010 e2 / a joule) Then, V = − =0
4 π ε0 x 0.50−x

27
This gives x=0.41 m=41 cm.

(ii) U =
1
.
q1 q2
¿
9 ×1 09
1.0 [
(1 )(−2 ) +
( 1 )( 3 )
√2
+ ( 1 ) ( 2 ) + (−2 ) ( 3 ) +
(−2 ) × ( 2 )
√2
+ ( 3 )( 2

4 π ε0 r 9× 10 × 10
9 −16
¿− J ¿−6.36 ×1 0−7 J.
9
9 ×1 0 ×20 ×1 0 ×(−4 )× 10
−6 −6
√2
¿ =−1.44
0.50 10. Suppose the charges +q , +2 q and Q
J. are placed al the comers A , B and C of
1 q1 q 2 an equilateral ΔABC of sidε a . Then
4. U =


4 π ε0 r
‐0.5×10-6
1
4 π ε0 [
q ×2 q q ×Q 2q × Q
r
+
r
+
r
=0
]
9 −9 −9
9 ×1 0 ×5 ×1 0 × (−2 ) ×1 0 or 2 q+Q+2 Q=0 or Q=−2q / 3.
¿
(x −2)×1 0−2 11.
On solving, x = 4 cm
3 q
2
U =9 ×1 09
[ 12 (−e ) (−e )
a
+12
(−e ) (−e )
√2 a
+4
(−e )(−e ) (−e
√3 a
+8
√3
5. U =
[ ]
9 2
4 π ε0 a 9 ×1 0 ×4 ×e 3 1 8
¿ 3+ + −
¿ 3 ×9 × 10 × ¿ ¿ J.
9 a √ 2 √ 3 √3
9 2

[ ]
q A q B q A q C q B qC 36 ×1 0 e
7. W ¿
1 ¿ [3+2.12−4.04]
+ + a
4 π ε 0 AB AC BC 2
10 e
¿ 3.89 ×1 0 joule
¿
1
4 π ε0 r[
q ⋅q q (−q ) q (−q )
+
r
+
r ] 12.
a
Here
q=2.0 ×1 0 C ,
−4

−2
1 η
2
9 m=10 g=10 kg , r =10 cm ¿ 0.10 m
¿− =−9 ×1 0 ׿ ¿ J.
4 π ε0 r Let v be the speed of each particle at
infinite separation. By conservation of
8. Initial P.E. of the three charges,
energy, P.E. of two particles at the

U i=
1
4 π ε0 r [
q 1 q 2 q2 q 3 q1 q3
+
r
+
r ] separation of 10 cm
= K.E. of the two particles at infinite
separation
¿
1
4 π ε0 [
q (−4 q ) (−4 q ) × 2 q q ×2 q
r
+
r
+
r ] 1
.
4 π ε0 r
q1q2 1 2 1
= mv + m v
2 2
2

2
1 10 q 9
¿− ⋅ =−9× 10 × 10× ¿ ¿ 1 q1 q2
4 π ε0 r or v =
2
.
4 π ε 0 rm
−8
¿−2.304 ×1 0 J 9 −4 −4
9 ×1 0 ×2.0 ×1 0 × 2.0 ×1 0
Final P.E., U f =0 ¿ −2
0.10 ×1 0
Work required to dissociate the system of 4
¿ 36 ×1 0
∴ v=600 m s−1.
three charges,
W =U f −U i ¿ 2.304 × 10−8 J.
9. 13. Here θ1 = 0°, θ2 = 180°, p = 3.2 ×10-8
Cm,
U=
4 π ε0 a
+
[
1 q 1 q 2 q 1 q 3 q1 q 4 q 2 q3 q 2 q 4 q3 q 4
√2a a
+ +
a
+ + 4
√2 a a E = 10 N/C ] W = pE(cosθ1 - cosθ2)

28
= 3.2 × 10-8 × 104 (cos 0°- cos 180°) outer shells of the atoms are loosely
bound to the nucleus. They get detached
= 3.2 × 10-4 × (1 + 1) = 6.4 × 10-4 J.
from the atoms and move almost freely
14. p = q × 2a = 10-6 ×0.02 = 2 × 10-8Cm inside the metal. In an external electric
(i) τmax = pEsin 90° = 2 × 10-8 × 105 × 1
field, these free electrons drift in the
opposite direction of the electric field. The
= 2 × 10-3 Nm. positive ions which consist of nuclei and
(ii) W = pE (cos θ1 - cos θ2) electrons of inner shells remain held in
their fixed positions. These immobile
= 2 × 10-8 × 105(cos 0°- cos 180°) charges constitute the bound charges.
= 2 × 10-3(1+ 1) = 4 × 10-3 J. In electrolytic conductors, both positive
2.13 CONDUCTORS AND and negative ions act as charge carriers.
INSULATORS However, their movements are restricted
by the external electric field and the
23. What are conductors and insulators ? electrostatic forces between them.
Why were insulators called dielectrics
and conductors non-electrics ? In insulators, the electrons are tightly
bound to the nuclei and cannot be
Conductors and insulators. On the basis detached from the atoms, i.e., charges in
of their behaviour in an external electric insulators are bound charges. Due to the
field, most of the materials can be broadly absence of free charges, insulators are
classified into two categories : poor conductors of electricity.
1. Conductors. These are the substances For Your Knowledge
which allow large scale physical
movement of electric charges through A third important category of materials is
them when an external electric field is the semiconductors which we shall
applied. For example, silver, copper, discuss in chapter 14.
aluminium, graphite, human body, In metallic conductors, electrons of outer
acids, alkalies, etc. shells of the atoms are the free charges
2. Insulators. These are the substances while the immobile positive ions are the
which do not allow physical movement bound charges.
of electric charges through them when In electrolytic conductors, both positive
an external electric field is applied. For and negative ions are the free charges.
example, diamond, glass, wood, mica,
wax, distilled water, ebonite, etc. In insulators, both electrons and the
positive ions are the bound charges.
The rubbed insulators were able to retain
charges placed on them, so they were There is no clear cut distinction between
called dielectrics. The rubbed conductors conductors and insulators - their
(metals) could not retain charges placed electrical properties vary continuously
on them but immediately drained away the within a very large range. For
charges, so they were called non- example, the ratio of the electrical
electrics. properties between a metal and glass
may be as high as 1020.
2.14 FREE AND BOUND CHARGES
2.15 BEHAVIOUR OF CONDUCTORS
24. Discuss the various free and bound IN ELECTROSTATIC FIELDS
charges present in conductors and
insulators. 25. State and prove the various
electrostatic properties shown by
Free and bound charges. The difference conductors placed in electrostatic fields.
between the electrical behaviour of
conductors and insulators can be Electrostatic properties of a conductor.
understood on the basis of free and bound When placed in electrostatic fields, the
charges. conductors show the following properties :

In metallic conductors, the electrons of the 1. Net electrostatic field is zero in the

29
interior of a conductor. As shown in As ϕE = 0, so q = 0
Fig. 2.40, when a conductor is ⃗ Eext ,
placed in an electric field E t, its free
electrons begin to move in the
opposite direction of ⃗ Eext . Negative
charges are induced on the left end
and positive

Fig. 2.41
Hence there can be no charge in the
interior of the conductor because the
Gaussian surface lies just near the outer
boundary. The entire excess charge q
must reside at the surface of the
conductor.
Fig. 2.40 Electric field inside a conductor
4. Potential is constant within and on the
is zero.
surface of a conductor. Electric field at
charges are induced on the right end of any point is equal to the negative of
the conductor. The process continues till the potential gradient,
the electric field ⃗
Eind set up by the induced dV
charges becomes equal and opposite to i.e., E = -
dr
the field ⃗ E (= ⃗
Eext . The net field ⃗ Eext - ⃗
Eind )
inside the conductor will be zero. But inside a conductor E = 0 and
moreover, E has no tangential component
2. Just outside the surface of a charged on the surface, so
conductor, electric field is normal to
the surface. If the electric field is not dV
= 0 or V = constant
normal to the surface, it will have a dr
component tangential to the surface Hence electric potential is constant
which will immediately cause the flow throughout the volume of a conductor and
of charges, producing surface has the same value (as inside) on its
currents. But no such currents can surface. Thus the surface of a conductor
exist under static conditions. Hence is an equipotential surface.
electric field is normal to the surface of
the conductor at every point. If a conductor is charged, there exists an
electric field normal to its surface. This
3. The net charge in the interior of a indicates that the potential on the surface
conductor is zero and any excess will be different from the potential at a
charge resides at its surface. As point just outside the surface.
shown in Fig. 2.41, consider a
conductor carrying an excess charge q 5. Electric field at the surface of a
with no currents flowing in it. Choose a charged conductor is proportional to
Gaussian surface inside the conductor the surface charge density. Consider a
just near its outer boundary. As the charged conductor of irregular shape.
field ⃗ Let σ be the surface charge density at
conductor, the flux ϕE through the
E = 0 at all points inside the
any point
Gaussian surface must be zero.
According to Gauss's theorem,
q
ϕ E=∮ ⃗
E.⃗
dS=
ε0

30
excess charge + q lies on its surface.

Fig. 2.42 A small pill box as a Gaussian


surface of a charged conductor.
of its surface. To determine E at this point,
we choose a short cylinder (pill box) as Fig. 2.43 Electric field vanishes in the
the Gaussian surface about this point. The cavity of a conductor.
pill box lies partly inside and partly outside 2.16 ELECTROSTATIC SHIELDING
the con-ductor. It has a cross-sectional
area ΔS and negligible height. 26. What is electrostatic shielding ?
Mention its few applications.
Electric field is zero inside the conductor
and just outside,, it is normal to the Electrostatic shielding. Consider a
surface. The contribution to the total flux conductor with a cavity, with no charges
through the pill box comes only from its placed inside the cavity. Whatever be the
outer cross-section. size and shape of the cavity and whatever

∴ ϕE = E ΔS
be the charge on the conductor and the
external fields in which it might be placed,
Charge enclosed by pill box, q = σ ∆S the electric field inside the cavity is zero,
i.e., the cavity inside the conductor
By Gauss's theorem, remains shielded from outside electric

ϕE =
q influence. This is known as electrostatic
ε0 shielding. Such a field free region is called
a Faraday cage.

∴ E ΔS =
σ∆S σ The phenomenon of making a region free
or E =
ε0 ε0 from any electric field is called
electrostatic shielding. It is based on the
As ⃗
E points normally outward, so we write fact that electric field vanishes inside the
σ cavity of a hollow conductor.

E = ε n^
0 Applications of electrostatic shielding
where n^ is a unit vector normal to the 1. In a thunderstorm accompanied by
surface in the outer direction. lightning, it is safest to sit inside a car,
rather than near a tree or on the open
6. Electric field is zero in the cavity of a ground. The metallic body of the car
hollow charged conductor. As shown becomes an electrostatic shielding
in Fig. 2.43, consider a charged from lightning.
conductor having a cavity, with no
charges inside the cavity. Imagine a 2. Sensitive components of electronic
Gaussian surface inside the conductor devices are protected or shielded from
quite close to the cavity. Everywhere external electric disturbances by
inside the conductor, E = 0. By placing metal shields around them.

this Gaussian surface is zero (E = 0 ⇒


Gauss's theorem, charge enclosed by 3. In a coaxial cable, the outer conductor
connected to ground provides an
q = 0). Consequently, the electric field electrical shield to the signals carried

cavity (q = 0 ⇒ E = 0). The entire


must be zero at every point inside the by the central conductor.
For Your Knowledge

31
In the interior of a conductor, the electric measure of its capacity to hold a large
field and the volume charge density both amount of charge without running a high
vanish. Therefore, charges in a conductor potential. It depends upon the following
can only be at the surface. Electric field at factors:
the surface of a charged conductor must
1. Size and shape of the conductor.
be normal to the surface at every point.
2. Nature (permittivity) of the surrounding
For a conductor without any surface
medium.
charge, electric field is zero even at the
surface. 3. Presence of the other conductors in its
neigh-bourhood.
The entire body of each conductor,
including its surface, is at a constant It is worth-noting that the capacitance of a
potential. conductor does not depend on the nature
of its material and the amount of charge
If we have conductors of arbitrary size,
existing on the conductor.
shape and charge configuration, then
each conductor will have a characteristic 28. Define the unit of capacitance for a
value of constant potential which may conductor. Give its dimensions.
differ from one conductor to another. Units of capacitance. The SI unit of
A cavity inside a conductor is shielded capacitance is farad (F), named in the
from outside electrical disturbances. honour of Michael Faraday.
However, the electrostatic shielding does The capacitance of conductor is 1 farad if
not work the other way round. That is, if the addition of a charge of 1 coulomb to it,
we place charges inside the cavity, the increases its potential by 1 volt.
exterior of the conductor cannot be
∴ 1 farad =
shielded from the electric fields of the 1coulomb 1C
or 1 F = =1
inside charges. 1 volt 1V
2.17 ELECTRICAL CAPACITANCE OF A CV-1
CONDUCTOR One farad is a very large unit of
27. Define electrical capacitance of a capacitance. For practical purposes, we
conductor. On which factors does it use its following submultiples :
depend ? 1 millifarad = 1 mF = 10-3 F
Electrical capacitance of a conductor. The 1 microfarad = 1 μF = 10-6 F
electrical capacitance of a conductor is the
measure of its ability to hold electric 1 picofarad = 1 pF = 10-12 F
charge. When an insulated conductor is Dimensions of capacitance. The unit of
given some charge, it acquires a certain capacitance is
potential. If we increase the charge on a 2
conductor, its potential also increases. If a 1C 1C 1C2 1(As)
1F= = = =
charge Q put on an insulated conductor 1 V 1 J /C 1 J 1 Nm
∴ Dimensions of capacitance
increases its potential by V, then
Q ∝ V or Q = CV 2 2
A T
=[ M L T A ]
−1 −2 4 2
The proportionality constant C is called = −2
MLT . L
the capacitance of the conductor. Thus
2.18 CAPACITANCE OF AN ISOLATED
Charge
Capacitance = SPHERICAL CAPACITOR
Potential
29. Obtain an expression for the
Hence the capacitance of a conductor capacitance of an isolated spherical
may be defined as the charge required to conductor of radius R.
increase the potential of the conductor by
unit amount.
The capacitance of a conductor is the

32
C
As ε0 =
4π R
So the SI unit of ε0 can be written as farad
per metre (Fm-1). From Coulomb's law, the
SI unit of ε0 comes out to be C2N-1m-2.
Both of these units are equivalent.
The farad (1F = 1 CV-1) is an enormously
large unit of capacitance because the
Fig. 2.44 Capacitance of a spherical coulomb is a very big unit of charge while
conductor. the volt is the unit of potential having
Capacitance of an isolated spherical reasonable size.
conductor. Consider an isolated spherical Examples Based on
conductor of radius R. The charge Q is
uniformly distributed over its entire Capacitance of Spherical Conductors
surface. It can be assumed to be Formulae Used
concentrated at the centre of the sphere.
1. Capacitance of a spherical conductor of
The potential at any point on the surface
radius R,
of the spherical conductor will be
C = 4πε0 R
1 Q
V= . Charge q
4 π ε0 R
∴ Capacitance of the spherical conductor
2. Capacitance = or C = .
Potential V
situated in vacuum is Units Used
Q Charge is in coulomb, potential in volt and
Q capacitance in farad (F).
C= = 1 Q
V ⋅
4 π ε0 R Example 32. An isolated sphere has a
capacitance 50 pF. (i) Calculate its radius,
or C = 4π ε0 R (ii) How much charge should be placed on
Clearly, the capacitance of a spherical it to raise its potential to 104 V ?
conductor is proportional to its radius. Solution. Here C = 50 pF = 50 × 10 -12 F, V
Let us calculate the radius of the spherical =104 V
conductor of capacitance 1 F. 1
(i) R = . C = 9 × 109 mF-1 × 50 × 10-
1 4 π ε0
R= . C =9 × 109 mF-1.1 F 12
F
4 π ε0
= 9 × 109m = 9 × 106km = 45 × 10 2 m = 45 cm.
This radius is about 1500 times the radius (ii) q = CV = 50 × 10 -12 × 104 = 5 × 10-7C =
of the earth (~6 × 10 3 km). So we 0.5 μC.
conclude : Example 33. Twenty seven spherical
1. One farad is a very large unit of drops of radius 3 mm and carrying 10-12 C
capacitance. of charge are combined to form a single
drop. Find the capacitance and the
2. It is not possible to have a single potential of the bigger drop. [Haryana 01]
isolated conductor of very large
capacitance. Solution. Let r and R be the radii of the
small and bigger drops, respectively.
For Your Knowledge
Volume of the bigger drop
The formula : C = 4π ε0 R is valid for both
hollow and solid spherical conductors. = 27 × Volume of a small drop

33
4 4 Solution. Let q be the charge on the
i.e., π R3 = 27 × πr3 spherical conductor and r its radius. Its
3 3
surface charge density is
or R = 3r = 3×3 mm = 9 × 10 3 m
∴ Capacitance of the bigger drop is
q -2
2 = 0.07 C cm ...(i)
4πr
1
C = 4πε0 R = -3
9 .9 × 10 F
When the charge is increased by 4.4 C,
9 ×10 the surface charge density becomes
= 10-12 F = 1 pF q+ 4.4 -2
2 = 0.084 C cm ...(ii)
Charge on bigger drop 4πr
q = 27 × Charge on a small drop Dividing equation (ii) by (i), we get
-12
= 27 × 10 C q+ 4.4 0.084
∴ Potential of bigger drop is
= or q = 22 C.
q 0.07
q 27 ×1012 From equation (i), we get
V= = = 27 V.

√ √
C 10
12
q 22 ×7
r= = = 5 cm =
Example 34. Eight identical spherical 4 π ×0.07 4 × 22× 0.07
drops, each carrying a charge 1 nC are at 0.05 m
a potential of 900 V each. All these drops Capacitance,
combine together to form a single large
drop. Calculate the potential of this large 1 -12
C = 4π ε0 r = 9 × 0.05 = 5.56 × 10
drop. (Assume no wastage of any kind 9 ×10
and take the capacitance of a sphere of F.
radius r as proportional to r). [CBSE
Sample Paper 15] Problems For Practice

Solution. 1. Find the capacitance of a conducting

Capacitance of each small drop, C ∝ r ⇒


sphere of radius 10 cm situated in air.
How much charge is required to raise
C = kr it to a potential of 1000 volt ?
Charge on each small drop, q = CV = (kr × (Ans. 11 pF, 1.1 × 10-8 C)
900)C
2. Assuming the earth to be a spherical
Charge on large drop, q = 8q = 7200 kr C conductor of radius 6400 km, calculate
Volume of a large drop = Volume of 8 its capacitance.
small drops [Himachal 98C ; Haryana 98C]

πR3 = 8 × πr3 ⇒ R = 81/3r = 2r


4 4 (Ans. 711μF)
3 3 3. N drops of mercury of equal radii and
Capacitance of large drop, C’ = kR = 2kr possessing equal charges combine to
form a big drop. Compare the charge,
Hence, the potential of the large drop is capacitance and potential of bigger
q' 7200 kr drop with the corresponding quantities
V' = = = 3600 V. of individual drops.
C’ 2 kr
Example 35. A charged spherical [Punjab 01] (Ans. N, N1/3, N2/3)
conductor has a surface charge density of HINTS
0.07 C cm-2. When the charge is increased
by 4.4 C, the surface charge density 1 -12
1. C = 4πε0R = 9 × 0.10 = 11 × 10
changes by 0.084 C cm~ 2 Find the initial 9 ×10
charge and capacitance of the spherical F = 11 pF.
conductor. q = CV = 11 × 10-12F × 1000V = 1.1 × 10-8

34
C. order of 3 × 106 Vm-1. This puts the limit on
the capacitance of a conductor. Moreover,
1
2. C = 4π ε0 R - 9 × 6.4 × 10
6 if we tend to have a single conductor of
9 ×10 large capacitance, it will have practically
= 0.711 × 10-3F = 711 μF. inconvenient large size.

3. Let q be the charge on each small 31. Why does the capacitance of a
drop and r its radius. conductor increase, when an earthed
connected conductor is placed near
Capacitance of each small drop, C = 4π ε 0 it ? Briefly explain.
r
Principle of a capacitor. Consider a
1 positively charged metal plate A and place
Potential of each small drop, V = .
4 π ε0 an uncharged plate B close to it, as shown
q in Fig. 2.45. Due to induction, the closer
r face of plate B acquires negative charge
and its farther face acquires a positive
If R is the radius of the big drop, then charge. The negative charge on plate B
4 3 4 tends to reduce the potential on plate Λ
πr × N - πR3 or R = N1/3 r while the positive charge on plate B tends
3 3 to increase the potential on A. As the
q' negative charge of plate B is closer to
Charge on the big drop, q' = Nq or = N. plate A than its positive charge, so the net
q
effect is that the potential of A decreases
Capacitance of the big drop. by a small amount and hence its
C’ = 4π ε0 R = 4π ε0 N1/3 r = N1/3 C capacitance increases by a small amount.

C’
or = N1/3.
C
Potential of the big drop,
'
1 q 1 Nq
V' = ⋅ = ⋅ 1/ 3
4 π ε0 R 4 π ε0 N r
2 /3 1 q 2/ 3
=N ⋅ ⋅ =N V
4 π ε0 r
Fig. 2.45 Principle of a capacitor.
V’ Now if the positive face of plate B is
or = N2/3.
V earthed, its positive charge gets
2.19 CONCEPT OF A CAPACITOR AND neutralised due to the flow of electrons
ITS PRINCIPLE from the earth to the plate B. The negative
charge on B is held in position due to the
30. An isolated conductor cannot have a positive charge on A The negative charge
large capacitance, why ? on B reduces the potential of A
The capacitance of an isolated conductor considerably and hence increases its
is small. When a conductor holds a large capacitance by a large amount.
amount of charge, its potential is also Hence we see that the capacitance of an
high. If the associated electric field (E = insulated conductor is considerably
σ / ε0) becomes high enough, the atoms or increased when we place an earthed
molecules of the surrounding air get connected conductor near it. Such a
ionised. A breakdown occurs in the system of two conductors is called a
insulation of the surrounding medium and capacitor.
the charge put on the conductor gets
neutralised or leaks away. For air, the 32. What is a capacitor ? Define
breakdown point occurs at fields of the capacitance of a capacitor. On what
factors does it depend ?

35
Capacitor. A capacitor is an arrangement The capacitance of a capacitor may be
of two conductors separated by an defined as the charge required to be
insulating medium that is used to store supplied to either of the conductors of the
electric charge and electric energy. capacitor so as to increase the potential
difference between them by unit amount.
A capacitor, in general, consists of two
conductors of any size and shape carrying The capacitance of a given capacitor is a
different potentials and charges, and constant and depends on the geometric
placed closed together in some definite factors, such as the shapes, sizes and
positions relative to one another. relative positions of the two conductors,
and the nature of the medium between
Pictorial representation of a capacitor. The
them.
pictorial symbol for a capacitor with fixed
capacitance is as shown in Fig. 2.46(a) SI unit of capacitance is farad (F). A
and for that with a variable capacitance is capacitor has a capacitance of 1 farad if 1
as shown in Fig. 2.46(b). coulomb of charge is transferred from its
one conductor to another on applying a
potential difference of 1 volt across the
two conductors.
Fig. 2.46 Symbols for a capacitor with 2.20 PARALLEL PLATE CAPACITOR
(a) fixed, (b) variable capacitance. 33. What is a parallel plate capacitor ?
Drive an expression for its capacitance.
Capacitance of a capacitor. As shown in On what factors does the capacitance of a
Fig. 2.47, usually a capacitor consists of parallel plate capacitor depend ?
two conductors having charges + Q and -
Q. The potential difference between them Parallel plate capacitor. The simplest and
is V = V+ - V- . Here Q is called the charge the most widely used capacitor is the
on the capacitor. Note that the charge on parallel plate capacitor. It consists of two
capacitor does not mean the total charge large plane parallel conducting plates,
given to the capacitor which is + Q - Q = separated by a small distance.
0. Let A = area of each plate,
d = distance between the two plates
± σ = uniform surface charge densities on
the two plates
± Q = ± σ A = total charge on each plate.

Fig. 2.47 Two conductors separated by an


insulator form a capacitor.
For a given capacitor, the charge Q on the
capacitor is proportional to the potential
difference V between the two conductors.
Fig. 2.48 Parallel plate capacitor.
Thus,
Q ∝ V or Q = CV
In the outer regions above the upper plate
and below the lower plate, the electric
The proportionality constant C is called fields due to the two charged plates
the capacitance of the capacitor. Clearly, cancel out. The net field is zero.
Q σ σ
C= E= - =0
V 2 ε0 2 ε0
or Capacitance = In the inner region between the two
Charge on either conductor capacitor plates, the electric fields due to
P . D . between the two conductors the two charged plates add up. The net

36
field is theorem,
σ σ σ 2 Q Q
E= + = ϕ E=E .4 π r = or E =
2 ε0 2 ε0 ε0 ε0 4 π ε0 r
2

The direction of the electric field is from


the positive to the negative plate and the
field is uniform throughout. For plates with
finite area, the field lines bend at the
edges. This effect is called fringing of the
field. But for large plates separated by
small distance (A >> d2), the field is almost
uniform in the regions far from the edges.
For a uniform electric field,
P.D. between the plates
= Electric field × distance between the
plates Fig. 2.49 Spherical capacitor.
σd The potential difference (caused by the
or V = Ed = inner sphere alone) between the two
ε0
shells will be
Capacitance of the parallel plate capacitor b b b
Q
is V =−∫ ⃗ dr=∫ Edr=∫
E⋅⃗ dr
σA a a a 4 π ε0 r 2
Q ε0 A
C= = σd or C = =
V d
[ ] [ ]
b b
ε0 Q Q −1 Q 1 1

4 π ε0 a
−2
r dr= = −
4 π ε 0 r a 4 π ε0 a b
Factors on which the capacitance of a
parallel plate capacitor depends
1. Area of the plates (C ∝ A).
E points radially inward and ⃗
[∵ ⃗ dr points
E.⃗
outward so ⃗ dr = Edr 180° = - Edr]
2. Distance between the plates (C ∝ 1 / The capacitance of the spherical capacitor
d). is

plates (C ∝ ε).
3. Permittivity of the medium between the Q
Q 4 π ε 0 ab
2.21 SPHERICAL CAPACITOR*
C=
V
=
−[ ]
Q 1 1 or C =
4 π ε0 a b b−a
.

34. What is a spherical capacitor ? Derive 2.22 CYLINDRICAL CAPACITOR*


an expression for its capacitance.
35. What is a cylindrical capacitor ? Derive
Spherical capacitor. A spherical capacitor an expression for its capacitance.
consists of two concentric spherical shells
of inner and outer radii a and b. The two Cylindrical capacitor. A cylindrical
shells carry charges - Q and + Q capacitor consists of two coaxial
respectively. Since the electric field inside conducting cylinders of inner and outer
a hollow conductor is zero, so ⃗ E = 0 for r radii a and b. Let the two cylinders have
< a. Also the field is zero outside the outer uniform linear charge densities of ± λ Cm -
1
. The length L of the capacitor is so large
shell, i.e., ⃗
E = 0 for r > b. A radial field ⃗
E
(L >> radii a or b) that the edge effect can
exists in the region between the two shells
be neglected. The electric field in the
due to the charge on the inner shell only.
region between the two cylinders comes
To determine the electric field at any point only from the inner cylinder, the outer
P at distance r from the centre, consider a cylinder does not contribute due to
concentric sphere of radius r as the shielding. To calculate the electric field E
Gaussian surface. Using Gauss's at any point P in between the two

37
cylinders at a distance r from the central Examples based on
axis, we consider a coaxial Gaussian
Capacitance of Air-Filled Capacitors
cylinder of radius r. Using Gauss's
theorem, the flux through Gaussian Formulae Used
surface must be
q
q 1. Capacitance, C =
ϕ E= V
ε0 2. Capacitance of a parallel plate
ε0 A
capacitor, C =
d
3. P.D. between the two plates of a
capacitor having charges q1 and q2,
q1 – q 2
V=
2C
4. Capacitance of a spherical capacitor,
ab
C = 4π ε0
b−a
Here a and b are the radii of inner and
outer shells of the spherical capacitor.
5. Capacitance of a cylindrical capacitor,
Fig. 2.50 L L
λL C = 2π ε0 b = 2πε0 b
or E.2 πr L = log e 2303 log 10
ε0 a a
Here α and b are the radii of inner and
λ
E= outer coaxial cylinders and L is the length
2 π ε0 r of the capacitor.
∴ Potential difference between the two Units Used
cylinders is
Capacitance C is in farad, charge q in
b b
coulomb, potential difference V in volt,
V =−∫ ⃗ dr=∫ Edr
E⋅⃗ thicknesses d and t in metre.
a a
Constant Used
[∵ ⃗
E and d r⃗ are in opposite directions]
b b
Permittivity constant, ε0 = 8.85 × 10-12 C2N-
λ λ 1 1 -2
m
=∫ dr= ∫ dr
a 2 π ε0 r 2 π ε0 a r Example 36. When 1.0 × 1012 electrons
λ λ are transferred from one conductor to
=
b
[ln r ]a = [ lnb−ln a ] another of a capacitor, a potential
2 π ε0 2 π ε0 difference of 10 V develops between the
λ b two conductors. Calculate the capacitance
or V = ln of the capacitor.
2 π ε0 a
Solution. Here q = ne = 1.0 × 1012 × 1.6 ×
Total charge on each cylinder is Q = Lλ
∴ Capacitance of cylindrical capacitor is
10-19
= 1.6 × 10-7 C
Q Lλ 2 π ε0 L V = 10 V
=
C= V λ b or C =
∴C=
ln b
ln q 1.6 ×10−7
2π ε0 a a = = 1.6 × 10-8 F.
V 10

38
∴ C' =
Example 37. A capacitor of unknown −12 −4
8.85 ×10 ×25 ×10
capacitance is connected across a battery −3 = 2.2 × 10-
1× 10
of V volts. The charge stored in it is 360 11
F
μC. When potential across the capacitor is
reduced by 120 V, the charge stored in it q' = C’ V =2.2 × 10-11 × 12 = 2.64 × 10-10 C
becomes 120 μC. Calculate :
Extra charge given by the battery to the
(i) The potential V and the unknown positive plate is
capacitance C.
q' - q = (2.64 - 1.32) × 10-10 =1.32 × 10-10
(ii) What will be the charge stored in the C.
capacitor, if the voltage applied had
Example 39. Two parallel plate air
increased by 120 V ?
capacitors have their plate areas 100 and
[CBSE D 13] 500 cm2 respectively. If they have the
same charge and potential and the
Solution. (i) Let C be the capacitance of
distance between the plates of the first
the capacitor and V the potential drop
capacitor is 0.5 mm, what is the distance
across the plates. Then
between the plates of the second
q = CV = 360 μC capacitor ? [Punjab 97C]
When the potential difference is reduced Solution. As capacitance, C = q/V and the
by 120 V, two capacitors have the same charge q
and potential V, so they have the equal
q' = C(V - 120) = 120 μC
capacitances, i.e.,
∴ = 3 ⇒ 7 = 180 V
V 360
= C1 = C 2
V −120 120
ε0 A 1 ε0 A 2
q 360 μC or =
C= = = 2μF. d1 d2
V 180 V
(ii) When the voltage is increased by 120 A2
or d2 = d1
V, A1
q" = C(V + 120) = 2 μF × (180 +120) = But A1 =100 cm2, A2 = 500 cm2,
∴ d2 = 0.5 mm = 0.05 cm
600 μC

∴ d2 =
Example 38. A parallel plate capacitor has
plate area of 25.0 cm2 and a separation of 500× 0.05
= 0.25 cm = 2.5 mm.
2.0 mm between its plates. The capacitor 100
is connected to 12 V battery, (i) Find the Example 40. A sphere of radius 0.03 m is
charge on the capacitor, (ii) If the plate suspended within a hollow sphere of
separation is decreased by 1.0 mm what radius 0.05 m. If the inner sphere is
extra charge is given by the battery to the charged to a potential of1500 volt and
positive plate ? outer sphere is earthed, find the
Solution. A = 25.0 cm2 = 25 × 10-4 m2, capacitance and the charge on the inner
sphere.
d = 2.0 mm = 2 × 10-3 m, V = 12 V
Solution. Here a = 0.03 m, b =0.05 m, V
ε 0 A 8.85 ×10−12 ×25 ×10−4 =1500 V
C= = −3 = 1.1
d 2× 10 The capacitance of the air-filled spherical
×10-11 F capacitor is
(i) q = CV = 1.1 × 10 -11 × 12 = 1.32 × 10 -10 4 π ε 0 ab 0.03 × 0.05
C C= = 9
b−a 9 ×10 × ( 0.05−0.03 )
(ii) Here d' = 2.0 -1.0 = 1.0 mm = 1 × 10 -3
m = 8.33 × 10-12 F = 8.33 pF.
Charge, q = CV = 8.33 × 10-12 × 1500

39
= 1.25 × 10-8 C. Q
Field due to surface 1 = , towards
Example 41. The thickness of air layer 2 ε0 A
between the two coatings of a spherical left
capacitor is 2 cm The capacitor has the Q
same capacitance as the sphere of 1.2 m Field due to surface 2 = , towards
diameter. Find the radii of its surfaces. 2 ε0 A
right
4 π ε 0 ab
Solution. Here = 4πε0 R Q
b–a Field due to surface 3 = , towards
2 ε0 A
ab left
or =R
b−a −8
20× 10 C
1.2 Field due to surface 4 = ,
Now b - a = 2 cm and R = m = 60 cm 2 ε0 A
2 towards left


ab As the point P lies inside the conductor,
= 60
2 the field here must be zero.
or ab = 120 Q Q Q −8
20× 10 C
- + + =0
(b + a)2 = (b - a)2 + 4ab 2 ε0 A 2 ε0 A 2 ε0 A 2 ε0 A
= 22 + 4 × 120 = 484 or 2Q – 20 × 10-8 = 0
or b + a = 22 Q = + 10 × 10-8C
or 2 + a + q = 22 [∵ b – a = 2 cm] ∴ Charge on surface 1 = - 10 × 10-8 C
∴ a = 10 cm and b = 12 cm. Charge on surface 2 = + 10 × 10-8 C
Example 42. The negative plate of a Charge on surface 3 = -10 × 10-8 C
parallel plate capacitor is given a charge
Charge on surface 4 = -10 × 10-8 C.
of -20 × 10-8 C. Find the charges
appearing on the four surfaces of the Problems For Practice
capacitor plates.
1. A capacitor of 20 μF is charged to a
Solution. As shown in Fig. 2.51, let the potential of 10 kV. Find the charge
charge appearing on the inner surface of accumulated on each plate of the
the negative plate be - Q. Then the charge capacitor. (Ans. 0.2 C)
on its outer surface will be Q - 20 × 10-8 C
2. A parallel plate air capacitor consists
of two circular plates of diameter 8 cm.
At what distance should the plates be
held so as to have the same
capacitance as that of a sphere of
diameter 20 cm ?
(Ans. 4 mm)
3. A capacitor of unknown capacitance is
connected across a battery of V volt. A
Fig. 2.51 charge of 120 μC is stored in it. When
the potential across the capacitor is
The induced charge on the inner surface reduced by 40 V, the charge stored in
of the positive plate will be + Q and that on the capacitor becomes 40 μC.
the outer surface will be - Q, as the Calculate V and the unknown
positive plate is electrically neutral. To find capacitance. What would have been
Q, we consider the electric field at a point charge in the capacitor if the voltage is
P inside the negative plate. increased by 40 V ? [CBSE D
17C]

40
4. A parallel-plate capacitor has plates of q1 120 μC
area 200 cm2 and separation between C= = =2 μΓ
V1 60 V
the plates 1.0 mm. (i) What potential
difference will be developed if a When the voltage is increased by 40 V,
charge of 1.0 nC is given to the
q = C(T + 40) = 2μF × (60 + 40)V = 200
capacitor ? (ii) If the plate separation is
μC.
now increased to 2.0 mm, what will be
the new potential difference ? (Ans. ε 0 A 8.85 ×10−12 ×200 ×10−4
5.65 V, 11.3 V) 4. C= = −3
d 1× 10
5. Two metallic conductors have net
= 0.177 × 10-9 F = 0.177 nF
charges of + 70 pC and - 70 pC, which
result in a potential difference of 20 V q 1 nC
(i) V = = = 5.65 V.
between them. What is the C 0.177 nF
capacitance of the system ? (Ans.
3.5 pF) (ii) When the plate separation increases
from 1.0 mm to 2.0 mm, the
6. A spherical capacitor has an inner capacitance decreases by a factor of
sphere of radius 9 cm and an outer 2. For the same charge, the potential
sphere of radius 10 cm. The outer difference will increase by a factor of
sphere is earthed and the inner sphere 2.
∴ V = 2 V = 2 × 5.65 = 11.3 V.
is charged. What is the capacitance of
the capacitor ? (Ans. 0.1 nF)
7. The stratosphere acts as a conducting 5. Charge on the capacitor,
layer for the earth. If the stratosphere q = 70 pC = 70 × 10-12 C
extends beyond 50 km from the −12
surface of the earth, then calculate the q 70 ×10 C
C= = = 3.5 pF.
capacitance of the spherical capacitor V 20 V
formed between stratosphere and
6. Here a = 9 cm = 0.09 m, b = 10 cm =
earth's surface. Take radius of the
0.10 m
earth as 6400 km. (Ans. 0.092 F)
4 π ε 0 ab 1 0.09 ×0.10
8. A charge of + 2.0 × 10 -8 C is placed on C= = ⋅ F
9 × 10 (0.10−0.09)
9
the positive plate and a charge of -1.0 b−a
× 10-8 C on the negative plate of a
0.01× 0.10
parallel plate capacitor of capacitance = × 10-9F = 0.1 × 10-9F = 0.1
1.2× 10-3μF. Calculate the potential 0.01
difference developed between the nF.
plates. (Ans. 12.5 V) 7. Here
HINTS a = radius of the earth = 6.4 × 106 m
1. C = 20 μF = 20 × 10-6F, V = 10 kV = 104 b = distance of the stratosphere layer from
V the centre of the earth
Charge, q = CV = 20 × 10-6 × 104C = 0.2 = 6400 + 50 = 6450 km = 6.45 × 106 m
C.
C = 4π ε0
2
ε0 A ε πD ab 1
6
6.4 ×10 ×6.45 ×10
6
2. =4 π ε 0 R or 0 =4 π ε 0 R = ×
d 4d a−b 9 ×109
(6.45−6.4)×10
6

D
2 q1 q2 = 0.092 F.
or d= =¿ ¿ mm. 3. C= =
16 R V1 V2 8. V =


120 μC 40 μC q1−q2 20× 10−8 +1.0 ×10−8
= or 3 V −120=V = −9
=12.5 V .
V V −40 2C 2× 1.2×10
∴ V =60V 2.23 COMBINATION OF CAPACITORS

41
IN SERIES AND IN PARALLEL Q
Cs =
36. A number of capacitors are connected V
in series. Derive an expression for the 1 V
equivalent capacitance of the series or = ...(2)
Cs Q
combination.
Capacitors in series. When the negative From equations (1) and (2), we get
plate of one capacitor is connected to the 1 1 1 1
positive plate of the second, and the = + +
C s C 1 C2 C 3
negative of the second to the positive of
third and so on, the capacitors are said to For a series combination of n capacitors,
be connected in series. we can write
Figure 2.52 shows three capacitors of 1 1 1 1
= + + ….+
capacitances C1, C2 and C3 connected in Cs C1 C 2 Cn
series. A potential difference V is applied
across the combination. This sets up For series combination of capacitors
charges ± Q on the two plates of each 1. The reciprocal of equivalent
capacitor. What actually happens is, a capacitance is equal to the sum of the
charge + Q is given to the left plate of reciprocals of the individual
capacitor C1 during the charging process. capacitances.
The charge + Q induces a charge - Q on
the right plate of C1 and a charge - Q on 2. The equivalent capacitance is smaller
the left plate of C2, etc. than the smallest individual
capacitance.
3. The charge on each capacitors is
same.
4. The potential difference across any
capacitor is inversely proportional to its
capacitance.
37. A number of capacitors are connected
in parallel. Derive an expression for
the equivalent capacitance of the
Fig. 2.52 Capacitors in series. parallel combination.

The potential differences across the Capacitors in parallel. When the positive
various capacitors are plates of all capacitors are connected to
one common point and the negative plates
Q Q Q to another common point, the capacitors
V 1= ,V 2= , V 3=
C1 C2 C3 are said to be connected in parallel.
For the series circuit, the sum of these Figure 2.53 shows three capacitors of
potential differences must be equal to the capacitances C1, C2 and C3 connected in
applied potential difference. parallel. A potential difference V is applied
across the combination. All the capacitors
∴V =V 1+V 2+V 3 =
Q Q Q have a common potential difference V but
+ +
C1 C2 C3 different charges given by
V 1 1 1 Q1 = C1V, Q2 = C2V, Q3 = C3V
or = + + ...(1)
Q C 1 C2 C 3
Clearly, the combination can be regarded
as an effective capacitor with charge Q
and potential difference V. If Cs is the
equivalent capacitance of the series
combination, then

42
supplied by battery) but potential
differences across the capacitors may
be different.
4. In parallel combination, potential
difference on each capacitor is same
but the charges on the capacitors may
be different.
Units Used
Capacitances are in farad, potential
differences in volt and charges in
coulomb.
Example 43. Two capacitors of
Fig. 2.53 Capacitors in parallel. capacitance of 6 μF and 12 μF are
connected in series with a battery. The
Total charge stored in the combination is voltage across the 6 μF capacitor is 2 V.
Q = Q1 + Q2 + Q3 = (C1 + C2 + C3) … (1) Compute the total battery voltage.

If Cp is the equivalent capacitance of the [CBSE OD 06]


parallel combination, then Solution. As the two capacitors are
Q = Cp V … (2) connected in series, the charge on each
capacitor must be same.
∴ Charge on 6 μF capacitor = Charge on
From equations (1) and (2), we get
Cp V = (C1 + C2 + C3) V 12 μF capacitor
or Cp = C1 + C2 + C3 or 6 μF × 2 volt =12 μF × V volt

∴ P.D. across 12 μF capacitor = ¿


For a parallel combination of n capacitors, 6 ×2
we can write =
12
Cp = C1 + C2 + ….. + Cn 1 volt
For parallel combination of capacitors Battery voltage = V1 + V2=2V + lV=3V.
1. The equivalent capacitance is equal to Example 44. Two capacitors of
the sum of the individual capacitances. capacitances 3 μF and 6 μF, are charged
2. The equivalent capacitance is larger to potentials of 2 V and 5 V respectively.
than the largest individual capacitance. These two charged capacitors are
connected in series. Find the potential
3. The potential difference across each across each of the two capacitors now.
capacitor is same.
[CBSE Sample Paper 04]
4. The charge on each capacitor is
proportional to its capacitance. Solution. Total charge on the two
capacitors
Examples based on
= C1V1 + C2V2 = (3 × 2 + 6 × 5) μC = 36
Grouping of capactors μC
Formulae Used In series combination, charge is
conserved.
∴ Charge on either capacitor, q- 36 μC
1. In series combination,
1 1 1 1
= + + +…
C s G1 C 2 C3 Potential on 3 μF capacitor =
2. In parallel combination, Cp = + C1 + q 36 μ C
¿ = =12 V
C2 + C3 +... C1 3 μ F
3. In series combination, charge on each Potential on 6 μF capacitor =
capacitor is same (equal to the charge

43
q 36 μ C Cp = C + C + C=3C=3×25 μF = 75 μF 1
= =6 V
C2 6 μ F V = 4200 V
Example 45. Two capacitors have a ∴ Charge, q = CpV = 75 × 10-6 × 4200
capacitance of 5μF when connected in
parallel and 1.2 μF when connected in = 315 × 10-3 C = 315 mC
series. Calculate their capacitances. Example 48. Calculate the charge
Solution. Let the two capacitances be C2 supplied by the battery in the arrangement
μF and C2 μF. shown in Fig. 2.55.

In parallel, Cp = C1 + C2 = 5 μF
C1 C2
In series, C s= =1.2 μ F
C1 +C 2
C1 (5−C1 )
Or =1.2
5
or
2
C 1−5 C1 +6=0 Fig. 2.55
Solution. The given arrangement is
Hence, C1 = 2 or 3 μF
∴ The capacitances are of 2 μF and 3 μF.
equivalent to the arrangement shown in
Fig. 2.56.
Example 46. Three capacitors of equal
capacitance, when connected in series
have net capacitance C1 and when
connected in parallel have net
capacitance C2. What is the value of
C1 /C2?
Solution. Let C = capacitance of each
capacitor.
For series combination, Fig. 2.56
1 1 1 1 3 C Clearly, the two capacitors are connected
= + + = or C 1= in parallel.
C1 C C C C 3
For parallel combination, Their equivalent capacitance is

C2 = C+C+C=3C ∴ = ⋅
C1 C 1 1 2 C=C1 + C2 =5 + 6=11 μF
= Charge supplied by the battery is
C2 3 3 C 9
q = CV = 11 μF × 10 V = 110 μC.
Example 47. In Fig. 2.54, each of the
uncharged capacitors has a capacitance Example 49. Three capacitors C1, C2 and
of 25 μF. What charge will flow through C3 are connected to a 6 V battery, as
the meter M when the switch S is closed? shown in Fig. 2.57. Find the charges on
the three capacitors.

Fig. 2.54
Solution. As the three capacitors are
Fig. 2.57
connected in parallel, their equivalent
capacitance is Solution. The given arrangement is

44
equivalent to the arrangement shown in
Fig. 2.58(a).

Fig. 2.59
Solution. Here three capacitors of 60
μF each are connected in series. Their
equivalent capacitance C1 is given by
1 1 1 1 3 1
= + + = =
C1 60 60 60 60 20
or C = 20 μF
The given arrangement now reduces
to the equivalent circuit shown in Fig.
Fig. 2.58 2.60(a)

Clearly, C2 and C3 are in parallel. Their


equivalent capacitance is
C = C2 + C3 = 5 + 5 = 10 μF
Now C1 and C form a series
combination, as shown in Fig. 2.58(b).
Their equivalent capacitance is
'
C1C 10 ×10
C= '
= =5 μ F Fig. 2.60
C1 +C 10+10
Clearly, the three capacitors of 10 μF,
Charge drawn from the battery, 10 μF and 20 μF are in parallel. Their
q=CV = 5μF×6 V=30 μC equivalent capacitance is
Charge on the capacitor C1=q = 30 μC C2 = 10 + 10 + 20 = 40 μF
Charge on the parallel combination of Now the circuit reduces to the
C2 and C3 = q = 30 μC equivalent circuit shown in Fig. 2.60(b).
We have two capacitors of 40 μF each
As C2 and C3 are equal, so the charge
connected in series. The equivalent
is shared equally by the two capacitors.
capacitance between A and B is
30 40 × 40
Charge on C2 = charge on C3 = = C= =20 μ F
2 40+40
15 μC
Given V = 100 V
∴ Charge, q = CV =20 μF × 100 V = 2000
Example 50. Find the equivalent
capacitance of the combination of
capacitors between the points A and B as μC = 2 mC
shown in Fig. 2.59. Also calculate the total
Example 51. If Cx =3 pF and C2 =2 pF,
charge that flows in the circuit when a 100
calculate the equivalent capacitance of the
V battery is connected between the points
given network between points A and B.
A and B. [CBSE D 02]

45
C8 = C2 + C3 =2 +2 =4 μF
Capacitors C4 and C5 form a series
combination of capacitance C9 given by
1 1 1 1 1 3 1
= + = + = =
C9 C 4 C 5 12 6 12 4
∴ C 9=4 μ F
Fig. 2.61 The equivalent circuit can be shown as
Solution. Clearly, capacitors 2, 3 and 4 in Fig. 2.63(a)
form a series combination. Their total
capacitance C is given by
1 1 1 1 1 1 1 7
= + + = + + =
C C 1 C 2 C1 3 2 3 6
'

∴ C = pF
' 6
7
The capacitance C forms a parallel
combination with capacitor 5, so their Fig. 2.63 (a)
equivalent capacitance is Capacitors C1 and C8 form a series
'' ' 6 20 combination of capacitance C10 given by
C =C + C2= +2= pF
7 7 C 1 C 8 8 × 4 32 8
C 10= = = = μF
The capacitance C" forms a series C 1 +C8 8+4 12 3
combination with capacitors 1 and 6. The
equivalent capacitance C of the entire Capacitors C6 and C9 form a parallel
network is given by combination of capacitance.

1 1 1 1 7 1 1 61 C11 = C6+ C9 =4 + 4=8 μF


= + + = + + =
C C' ' C1 C 1 20 3 3 60 The given network reduces to the
equivalent circuit Fig. 2.63(b).
∴ C=
60
pF
61
Example 52. From the network shown in
Fig. 2.62, find the value of the capacitance
C if the equivalent capacitance between
points A and B is to be 1 μF. All the
capacitances are in μF.

Fig. 2.63 (b)


Again, capacitors C, and Cn form a
series combination of capacitance C12
given by
C 7 × C11 1 ×8 8
C 12= = = μF
C7 +C 11 1+ 8 9
Now C10 and C12 form a parallel
Fig. 2.62 combination of capacitance C13 as shown
Solution. Capacitors C2 and C3 form a in Fig. 2.63(c).
parallel combination of equivalent
8 8 32
capacitance, C 13=C10 +C 12= + = μF
3 9 9

46
Capacitance of parallel combination,
C1 =2n μF
Capacitance of series combination,
2
C 2= μF
7−n
Fig. 2.63 (c) As these two combinations are in
Finally, the capacitors C and C13 form 10
series, so C s= μF
a series combination of capacitance 1 μF 11


as shown in Fig. 2.63(d). 1 1 1
= + 11 1 7−n
But = +
Cs C1 C2 10 2 n 2
Multiplying both sides by 10 n, we get
11 n = 5 +35n - 5n2
Fig. 2.63 (d) or 5n2-24N-5 = 0

∴ =
1 1 9 32 (n - 5) (5n + 1) = 0
+ or C= μF
1 C 32 23 Or n = 5 [Rejecting -ve value]
Example 53. Connect three capacitors Hence parallel combination of 5
of3μF, 3μF and 6 μF such that their capacitors must be connected in series
equivalent capacitance is 5 μF. with the other 2 capacitors.
Solution. Capacitors connected in Example 55. Find the equivalent
parallel have maximum equivalent capacitance between the points P and Q
capacitance. as shown in Fig. 2.64. Given C = 18 μF
Cmax =3 + 3 + 6= 12 μF and C = 12 μF. [REC 97]
Capacitors connected in series have
minimum equivalent capacitance.
1 1 1 1 5
= + + =
Cmin 3 3 6 6
6
Or C min= =1.2 μ F
5
Fig. 2.64
The required equivalent capacitance of
Equivalent capacitance between
5μF lies between Cmax and Cmjn. So
points F and B is
3 ×6
5 μ F=3 μ F+2 μ F=3 μ F+ μF 18× 18
3+6 + 18=27 μ F
18+18
So we should connect the series
Equivalent capacitance between points A
combination of 3 μF and 6 μF capacitors
and B is 18 × 27
in parallel with the third capacitor of 3 μF.
18× 27
Example 54. Seven capacitors, each of 12+ =12+10.8=22.8 ≈ 23 μ F
capacitance μF are to be connected in a 18+27
configuration to obtain an effective Equivalent capacitance between points A
capacitance of 10 /11 μF. Suggest a and E is 23 × 18
suitable combination to achieve the
desired result. [IIT 90] 23× 18
+ 18=28 μ F
23+18
Solution. Suppose a parallel
combination of n capacitors is connected Equivalent capacitance between
in series with a series combination of (7 - points D and E is
n) capacitors.

47
28× 18
+ 12=23 μ F
28+18
Equivalent capacitance between points D Fig. 2.67
and Q is 23 × 18
Equivalent capacitance between X and
23× 18 Y
+ 18=28 μ F
23+18 10× 10
¿ =5 μ F
Equivalent capacitance between points P 10+10
and Q is 28×18
Example 57. Five capacitors of
28× 18 capacitance 10 μF each are connected
=11 μ F
28+18 with each other, as shown in Fig. 2.68.
Calculate the total capacitance between
Example 56. Four capacitors are the points A and C.
connected as shown in the Fig. 2.65.
Calcidate the equivalent capacitance
between the points X and Y. [CBSE D
2000]

Fig. 2.68
Fig. 2.65 Solution. The given circuit can be
redrawn in the form of a Wheatstone
Solution. Clearly, the first plate of 2 μF bridge as shown in Fig. 2.69.
capacitor,
the second plate of 3 μF capacitor and the
first plate of 5 μF capacitor are connected
to the point A On the other hand, the
second plate of 2 μF capacitor, the first
plate of 3 μF capacitor and the second
plate of 5 μF capacitor are connected to
the point B. Thus the capacitors of 2 μF, 3
μF and 5 μF are connected in parallel Fig. 2.69
between points A and B, as shown in the As C1 = C2 = C4 = C5,
equivalent circuit diagram of Fig. 2.66.
C1 C4
Therefore, =
C2 C5
Thus the given circuit is a balanced
wheatstone bridge. So the potential
difference across the ends of capacitor C3
is zero. Capacitance C3 is ineffective. The
given circuit reduces to the equivalent
Fig. 2.66
circuit shown in Fig. 2.70(a).
Total capacitance of the parallel
combination of capacitances 2 μF, 3 μF
and 5 μF is
C=2+3 + 5=10μF
As shown in Fig. 2.67, this parallel
combination is in series with capacitance
of 10 μF.

48
Fig. 2.70 (a) first row, second row, third row, fourth row,
… respectively. Then
Capacitors C1 and C2 form a series
combination of equivalent capacitance C6 C 1=1 μ F
given by
1 ×1 1
C1 ×C 2 10 ×10 C 2= = μF
C 6= = =5 μF 1+1 2
C 1+C 2 10+ 10
1 1 1 1 1
= + + + =4
Similarly, C4 and C5 form a series C3 1 1 1 1
combination of equivalent capacitance C7
∴C 3=
given by 1
μF
4
C4 × C5 10× 10
C 7= = =5 μ F 1
C 4 +C 5 10+10 Similarly, C 4= μF , and so on.
8
As shown in Fig. 2.70(b), C6 and C7
form a parallel combination. Hence the As these rows are connected in
equivalent capacitance of the network is parallel between points A and B, so the
given by equivalent capacitance between points A
and B is
C = C6 + C7 = 5 + 5 = 10 μF.
1 1 1
C=C 1+C 2 +C3 +C 4 + … …=1+ + + +…
2 4 8
This is an infinite geometric
progression with first term a- I and
common ratio r = 1/2. Hence
a 1
Fig. 2.70 (b) C= = =2 μ F
1−r 1−1/2
Example 58. There are infinite number of
Example 59. Find the equivalent capacitor
capacitors, each of capacitance 1 μF.
of the ladder (Fig. 2.72) between points A
They are connected in rows, such that the
and B.
number of capacitors in the first row,
second row, third row, fourth row, are
respectively 1, 2, 4, 8, ….. The rozos of
these capacitors are then connected
between points A and B, as shown in Fig.
2.71. Determine the equivalent
capacitance of the network between the
points A and B.
Fig. 2.72
Solution. Let C be the equivalent
capacitance of the infinite network. It
consists of repeating units of two
capacitors of 1 μF and 2 μF. The addition
of one such more unit will not affect the
equivalent capacitance. But then the
network would appear as shown in Fig.
2.73.

Fig. 2.71
Solution. Let C1, C2, C3, C4,. . be the
effective capacitances of the capacitors of

49
Fig. 2.73
The equivalent capacitance of the new
arrangement must be equal to C.
2× C
C=1+
2+C
or C2 - C - 2 = 0 Fig. 2.75

or C = 2 μF or -1 μF Solution. Capacitors C2 and C3 form a


parallel combination. Their equivalent
As the capacitance cannot be capacitance is
negative, so the equivalent capacitance of
the ladder is 2 μF. C = C2 + C3= (4+2) μF=6μF

Example 60. If q =20 μF, G, =30 μF and Now C1 and C' form a series
C3 =15 μF and the insulated plate of C, be combination, therefore, the equivalent
at a potential of 90 V, one plate of C, capacitance of the entire network is
being earthed. What is the potential CC
'
3 ×6
difference between the plates of Clr three C= '
= =2 μ F
capacitors being connected in series? C+C 3+6
[CBSE OD 15] The charge on the equivalent capacitor is
Solution. Here C1 = 20 μF, C, =30 μF, q=CV=2 × 10-6 × 1200 C =2.4×10-3 C
C3 =15 μF, V =90 V This must be equal to the charge on C :
and also the sum of the charges on C9
and C3. Thus
−3
q 2.4 ×10
V A −V B = = =800 V
C 1 3× 10−6
Fig. 2.74 V A =1200 V
The equivalent capacitance C of the =1200 V
∴ VB =1200 - 800 = 400 V
series combination is given by
1 1 1 1 1 1 1 3
= + + = + + = Hence VC – VB = 400 - 0 = 400 V
C C1 C 2 C 3 20 30 15 20
q2 = C2 (VC – VB) = 4 × 10-6 × 400 C = 1.6 ×
20 10-3 C
C= μ F
3
q3 = C3 (VC – VB) =2 × 10-6 × 400 C = 0.8 ×
Total potential difference = 90 -0 = 90 V 10-3 C
∴ Total charge, q1 = q = 2.4 × 10-3C.
−6
20 ×10 −6 Example 62. A network of four 10 μF
q=CV = .90=600 × 10 C capacitors is connected to a 500 V supply
3
as shown in Fig. 2.76. Determine (a) the
P.D. between the plates of capacitor equivalent capacitance of the network, (b)
C, is the charge on each capacitor. [NCERT]
−6
q 600 ×10 C
V 2= = =20 V
C 2 30× 10−6 F
Example 61 In the circuit shozvn in Fig.
2.75, if the point C is earthed and point A
is given a potential of + 1200 V,find the
charge on each capacitor and the
potential at the point B.

50
' 10 5000 −6
q=500 ×C =500 × μC= ×10 C
3 3
= 1.7 × 10-3 C
'
q
Also, P.D. across AD = = 500 V
C4
∴ q'= 500 × C4 = 500 × 10 μC
= 5000 × 10-6 C = 5 × 10-6 C.
Example 63. Four capacitors Cv C2, C3
and C4 are connected to a battery of 12 V,
Fig. 2.76 as shozvn in Fig. 2.77. Find the potential
difference betzveen the points A and B.
Solution. (a) In the given network, Cv
C2 and C3 are connected in series. Their
equivalent capacitance C is given by
1 1 1 1 1 1 1 3
= + + = + + =
C C 1 C 2 C3 10 10 10 10
'

' 10
Or C = μF
3
Now C and C4 form a parallel
combination. Therefore, the equivalent Fig. 2.77
capacitance of the whole network is Solution. Let VA be the potential at
' 10 40 point A and VB that at B. Then
C=C +C 4 = +10= μ F=13.3 μ F
3 3 P.D. across C1 = 12 - VA
(b) It is clear from Fig. 2.76 that the P.D. across C2 = VA - 0 = VA
charge on each of the capacitors Cj C2
and C3 is same. Let it be q. Let the charge P.D. across C3 = 12 – VB
on C4 be q'. P.D. across C4 = VB - 0 = VB

∴ P.D. across AB, V 1=


q As the capacitors C1 and C2 are
C1 connected in series, so

q q1= q2
P.D. across BC, V 2=
C2 or C1(12-VA) = C2VA
q or 8 (12 - VA) = 4VA
P.D. across CD, V 3=
C3 or VA = 8a
But V1 + V2 + V3 = V Again, the capacitors C3 and C4 are
connected in series, so

q q q
+ + =500 Q3= q4
C1 C 2 C3
Or C3 (12- VB)=C4 VB
q ( 1 1 1
+ +
C1 C 2 C3
=500) Or 3(12 - VB) = 6 VB
Or VB = 4 V
∴ q ⋅ ' =500
1
The potential difference between the
C

points A and B is
VA - VB =8-4 = 4V.

51
Example 64. Five identical capacitor
plates, each of area A are arranged such
that the adjacent plates are at distance d
apart. The plates are connected to a
source of emf V, as shown in Fig. 2.78.
Find the charges on the various plates.
[IIT84]

Fig. 2.80
Solution. The two capacitors of 3 μF
and 3 μF on the left side of the network
are in parallel, their equivalent
capacitance = 6 μF
The two capacitors of 1 μF and 1 μF
Fig. 2.78 on the other side of the network are also
Solution. As shown in Fig. 2.79, the in parallel, their equivalent capacitance =
given network is equivalent to three 2 μF. So the given network reduces to the
parallel-plate capacitors connected in equivalent circuit shown in Fig. 2.81.
parallel.

Fig. 2.79 Fig. 2.81


Their capacitances are In the steady state, when all the
ε0 A 2 ε0 A ε0 A capacitors are charged, there is no current
, and in the circuit. So there is no potential drop
d d d
across any resistance. Hence
The p.d. across each capacitor is v.
p.d. across 1 μF capacitor
As
= p.d. between points A and C = 100 V
Charge = Capacitance × p. d.
As 6 μF and 2 μF capacitances are in
So charges on various plates are series, the p.d. of 100 V is divided
between them in the inverse ratio of their
+ ε 0 AV −2 ε 0 AV
q 1= , q2 = capacitances i.e., in the ratio 2 : 6 or 1 : 3.
d d
∴ VAB = p.d. across 6 μF=
1
+ 2 ε 0 AV −2 ε 0 AV +ε 0 AV × 100 = 25 V
q 3= , q4 = ,q 5= 4
d d d
3
Example 65. For the network shown in VBC = p.d. across 2 μF = × 100 = 75 V.
4
Fig. 2.80, find the potential difference
between points A and B, and that between Problems For Practice
B and C in the steady state. 1. Two capacitors have a capacitance of
5μF when connected in parallel and
1.2 μF when connected in series.

52
Calculate their capacitances. capacitor in Fig. 2.83, if the equivalent
capacitance of the combination
(Ans. 2 μF, 3 μF)
between A and B is 15 μF.
2. Two capacitors of equal capacitance [CBSE D 94]
when connected in series have net
(Ans. 60 μF)
capacitance C1 and when connected in
parallel have net capacitance C2. What
is the value of C1 / C2?
[CBSE D 93C]
(Ans. C1 / C2 = 1/ 4)
3. Three capacitors of capacity 1, 2 and
3μF are connected such that second Fig. 2.83
and third are in series and the first one 8. In the combination of four identical
in parallel. Calculate the resultant capacitors shown in Fig. 2.84, the
capacity. (Ans. 2.2 μF) equivalent capacitance between points
4. The capacities of three capacitors are P and Q is 1 μF. Find the value of
in the ratio 1:2:3. Their equivalent each separate capacitance.
capacity in parallel is greater than the (Ans. 4 μF)
equivalent capacity in series by 60/11
pF. Calculate the individual
capacitances.
(Ans. 1 pF, 2 pF, 3 pF)
5. The equivalent capacitance of the
combination between A and B in Fig.
2.82 is 4 μF. Fig. 2.84
9. Find the equivalent capacitance of the
combination shown in Fig. 2.85 between
the points A and B.
Fig. 2.82
(i) Calculate capacitance of the capacitor
C.
(ii) Calculate charge on each capacitor if a
12 V battery is connected across terminals
A and B. Fig. 2.85
(iii) What will be the potential drop across
each capacitor? [CBSE D 09]
[Ans. (0.5 μF (ii) 48 μC (iii) 2.4 V, 9.6 V]
( Ans. C 3 +
2G C 2
C1 +C 2 )
6. Flow would you connect 8, 12 and 24 10. For the network shown in Fig. 2.86,
μF capacitors to obtain (i) minimum calculate the equivalent capacitance
capacitance (ii) maximum between points A and B.
capacitance? If a potential difference (Ans. 6 μF)
of 100 volt is applied across the
system, what would be the charges on
the capacitors in each case?
[Ans. (i) In series, Cmin = 4 μF, q = 4 μC,
(ii) In parallel, Cmax = 44 μF, q1 = 800 μC,
q2 = 1200 μC, q3 = 2400 μC]
7. Calculate the capacitance of the
Fig. 2.86

53
11. Calculate the capacitance of the
capacitor C in Fig. 2.87. The
equivalent capacitance of the
combination between P and Q is 30
μF. [CBSE OD 95]
(Ans. 60 μF)

Fig. 2.90
15. A network of six identical capacitors,
each of value Cis made, as shown in
Fig. 2.91. Find the equivalent
capacitance between the points A and
Fig. 2.87 B.

12. Calculate the equivalent capacitance (Ans. 4C/3)


between points A and B of the
combination shown in Fig. 2.88.
(Ans. 0.5 μF)

Fig. 2.91
16. Find the equivalent capacitance
between the points A and B of the
Fig. 2.88 network of capacitors shown in Fig.
2.92.
13. Find the equivalent capacitance (Ans. lμF)
between points A and B for the
network shown in Fig. 2.89.
8
( Ans. μ F)
3

Fig. 2.92
17. Find the capacitance between the
points A and B of the assembly shown
in Fig. 2.93.
Fig. 2.89 (Ans. 2.25 μF)
14. Calculate the equivalent capacitance
between the points A and B of the
circuit given below.
[CBSE F 95]
28
( Ans. μ F)
19
Fig. 2.93
18. Find the resultant capacitance
between the points X and Y of the

54
combination of capacitors shown in
Fig. 2.94.
[Haryana 01]
(Ans. 2.5 μF)

Fig. 2.97
22. Determine the potential difference
across the plates of each capacitor of
the network shown in Fig. 2.98. Take
E2 > E 1.
Fig. 2.94
(E2−E 1)C2 ( E −E1 )C 1
19. The outer cylinders of two cylindrical ( Ans. V 1 = , V 2= 2 )
capacitors of capacitance 2.2 μF each C1 +C 2 C 1 +C 2
are kept in contact and the inner
cylinders are connected through a
wire. A battery of emf 10 V is
connected, as shown in Fig. 2.95. Find
the charge supplied by the battery to
the inner cylinders.
(Ans. 44 μC)
Fig. 2.98
23. Find the potential difference between
the points A and B of the arrangement
shown in Fig. 2.99.
(Ans. - 8 V)

Fig. 2.95
20. In Fig. 2.96, q = 1 μF, q = 2 μF and q =
3 μF. Find the equivalent capacitance
between points A and B.
(Ans. 6μF) Fig. 2.99
24. Determine the potential difference VA –
VB between points A and B of the
circuit shown in Fig. 2.100. Under what
condition is it equal to zero?
C 2 C 3−C 1 C 4 C C
[ Ans . V ( ), 1 = 3 ]
Fig. 2.96 (C 1 +C2 )(C 3+C 4 ) C 2 C 4
21. Four capacitors of equal capacitances
are connected in series with a battery
of 10 V, as shown in Fig. 2.97. The
middle point B is connected to the
earth. What will be the potentials of the
points A and C? (Ans. VA = + 5 V, VC
= - 5 V)
Fig. 2.100

55
25. A variable capacitor has n plates and Then
the distance between two successive
Cp =C+2C+3C=6C
plates is d. Determine its capacitance.

( )
( n−1 ) ε 0 A 1 1 1 1 11 6C
Ans. C= = + + = or C s=
d C s C 2C 3 C 6 C 11

26. A network of four capacitors each of 60 6 C 60


Given C p−C s= pF or 6 C− = pF
12 μF capacitance is connected to a 11 11 11
500 V supply as shown in Fig. 2.101. or C = 1 pF
Determine
So the individual capacitances are 1 pF, 2
(a) equivalent capacitance of the network, pF and 3 pF.
and
5. (i) As 20 μF capacitor and capacitor C
(b) charge on each capacitor. are in series, their equivalent
[CBSE OD 10] capacitance is
[Ans. (a) 16 μF (b) q1 = q2 = q3 =2000 μC, C ×20
q4 = 6000 μC] C AB=
C+ 20
20 C
or 4 μF =
C +20
or 4C+ 80 = 20C
or C = 5 μF.
Fig. 2.101 (ii) Charge on each capacitor,
27. For the network shown in Fig. 2.102, q = CABV = 4 μF × 12 V = 48 μC.
compute (ii) P.D. on 20 μF capacitor =
q 48 μC
= =2.4 V
20 μF 20 μF
q 48 μC
P.D. on capacitor C= = =9.6 V
C 5 μF
6. (j) For minimum capacitance, the three
capacitors must be connected in
Fig. 2.102 series. Then C max=8+12+24=44 μ F
(i) the equivalent capacitance between (ii) For maximum capacitance, the three
points a and b. capacitors must be connected in
(ii) the charge on each of the capacitors parallel. Then Qnax = 8 + 12 + 24 = 44
nearest to a and b when Vab = 900 V. μF.
(iii) Vcd , when Vab = 900 V. (iii) In series combination, charge is same
on all capacitors.
[Ans. (0 1 μF (ii) 900 μC (iii) 100 V]
q = CV = 4μF × 100 V = 4 μC.
HINTS
In parallel combination, charges on the
1. Proceed as in Example 45 on page capacitors are
2.35.
Q1 = q V = 8 μF × 100 V = 800 μC
2. Proceed as in Example 46 on page
2.35. q2 = C2V = 12 μF × 100 V = 1200 μC
2× 3 q3 = C2V = 24 μF × 100 V = 2400 μC.
3. C=1+ =2.2 μF
2+3 7. The combined capacitance of the
4. Let the capacitances be C, 2 C and 3 C. parallel combination of two 10 μF

56
capacitors is 20 μF. This combination 1 1 1 1 1 1 1 1
is connected in series with
= + + = + + =
C7 C2 C 3 C 4 4 2 4 1
∴C 7=1 μ F
capacitance C.


1 1 1
+ =
20 C 15 Also, C5 and C6 are in series, the
equivalent capacitance is
Or
1 1 1 4−3 1
= − = = C5 ×C 6 2 × 4 4
C 15 20 60 60 C 8= = = μF
C 5+ C6 2+4 3
8. All capacitors are in series.


4 1 C7 and C8 form a parallel combination of
= or C=4 μF capacitance,
C 1 μF
4 7
C1 C 2 C1C2 2 G C2 C 9=C 7+ C8 =1+ = μF
9. C= +C 3 + =C 3+ 3 3
C+C 2 C 1 +C2 C 1+ C 2
Now C1 and C9 form a series combination.
1 1 1 1 1 ' The equivalent capacitance C between A
10. = + + = , C =3 μ F
C 9 9 9 3 and B is given by
C=3μF+ C' = 3μF + 3μF = 6μF. 1 1 1 1 3 19 28
= + = + = or C= μ F
+ = ∴ C=60 μ F
1 1 1 C C1 C 9 4 7 28 19
11.
C 60 30 15. The equivalent network is shown in
Fig. 2.104.
1 1 1 1 2
= + + =
C 1+1 1 1+1 1
∴ C = 0.5 μF.
13. C1 and C2 are in parallel between
points A and D. So the equivalent
capacitance between A and D is c'= q + q
= 1+ 1=2 μF Fig. 2.104
The given network now reduces to the Clearly, the equivalent capacitance
equivalent circuit shown in Fig. 2.103.
= [2 C and C in series] 11 [C and 2 C in
Between points A and B, now C and q are
series]
in series and C4 in parallel. Hence the
equivalent capacitance between A and B 2C × C C ×2 C 4 C
¿ + =
is 2 C+C C +2C 3
16. Two 2 μF capacitors at the left side
of the network are in series. Their
equivalent capacitance is 2×2
2 ×2
C s= =1 μ F
2+2
Fig. 2.103 The capacitance Cs and the next capacitor
' of 1 μF are in parallel. Their equivalent
C C2 2 ×1 8 capacitance is Cp = 1+ 1 = 2 μF
C= '
+C 4= + 2= μF
C +C 2 2+1 3
Proceeding in this way, we finally get two
2μF capacitors connected in series.
∴ Equivalent capacitance between A and
14. Capacitors q, q and C4 are connected
in series, their equivalent capacitance
q is given by B

57
2× 2 q (E2−E 1)C 2
¿ =1 μ F P.D. across plates of G= =
2+2 C G1 +C 2
17. The given arrangement is a balanced
q (E2−E 1)C
wheatstone bridge. Proceed as in P.D. across plates of C 2= =
Example 57 on page 2.39. C2 C 1 +C 2
18. The arrangement between the points 23. The given arrangement is equivalent
A and B is a balanced wheatstone bridge. to the circuit shown in Fig. 2.105.
Proceeding as in Example 57, we find that
the equivalent capacitance between A and
B is
C’ = 5 μF
Now the capacitor C and the left out
capacitor of 5 μF are in series. The
equivalent capacitance between points X
and Y will be Fig. 2.105
'
C × 5 5× 5 Proceeding as in the above problem 22,
C= '
= =2.5 μF we get
C +5 5+5
19. The two capacitors are connected in (E 2−E1 )C 1 C 2
q=
parallel G+C 2
∴ C = 2.2 + 2.2 = 44 μF P.D. across the plates of C1,
Charge, q = CV = 4.4 μF × 10 V = 44 μC. q (E2−E 1) C2
V 1= =
20. The three capacitors are connected in G C 1+C 2
parallel between points A and B.
∴ C=C 1+C 2 +C3 =1+2+3=6 μ F
(12 V−24 V)4 μ F
¿ =−8 V
2 μ F+4 μ F
21. Flere VB = 0. As the capacitances are 24. Suppose the charge q1 flows in the
equal on the two sides of point B,
∴ VA - VB = VB –VC
upper branch and q2 in the lower
branch. Then
or VA + VC=2VB=0 1 1 V C C2
V =q1 ( + )or q1=
But VA – VC = 10 V C C2 C 1+C 2
VA = + 5V and VC = -5V.
22. Let charge q flow across the capacitor
Also, V =q2 ( 1 1
+
C3 C4 )
plates until the current stops. In a
V C3C4
closed circuit, or q 2=
C3 +C 4

Σ ΔV = 0
q q
or E1 + −E2 + =0 q2 q 1
C1 C2 V A −V B =(V Q−V B)−(V Q −V A )= −
C4 C2
¿q
( G C2)
G+ C2
=E2−E1 Putting the values of q1 and q2, we get
V C3 VG
(E 2−E1 )G2 C 2 V A −V B = −
C 3 +C 4 C 1 +C 2
Or q=
C 1 +C2
¿V
[ C2 C3−GC 4
( C1 +C 2 )( C 3 +C 4 ) ]
58
For VA – VB = 0, we have This energy is supplied by the battery at
the expense of its stored chemical energy
C1 C3
C2 C3−C4 C4 =0 or = and can be recovered by allowing the
C2 C4 capacitor to discharge.
25. The given arrangement is equivalent Expression for the energy stored in a
to (n- 1) capacitors joined in parallel. capacitor. Consider a capacitor of
capacitance C. Initially, its two plates are
(n−1)ε 0 A
∴ C= uncharged. Suppose the positive charge
d is transferred from plate 2 to plate 1 bit by
12 μ F bit. In this process, external work has to
26. (a) C 123= =4 μ F be done because at any stage plate 1 is at
3 higher potential than the plate 2. Suppose
C eq=C 123 + C4 =4 +12=16 μF at any instant the plates 1 and 2 have
charges O' and - O' respectively, as
(b) shown in Fig. 2.106(a). Then the potential
q 1=q 2=q 3=C123 V =4 μF ×500 V=2000 μ C difference between the two plates will be
q 4=C 4 V =12 μ F ×500 V=6000 μ C ' Q
V=
C
27. (i) Three 3 μF capacitors in series
have equivalent capacitance = 1 μF. The
combination is in parallel with 2 μF
capacitor.
∴ Equivalent capacitance between c and d
= 1 + 2 = 3 μF
The situation is repeated for points e and
f. Hence there are three 3 μF capacitors in
series between points a and b. Equivalent
capacitance between a and b = 1 μF.
(ii) Potential drop of 900 V across a and b
is equally shared by three 3 μF Fig. 2.106 (a) Work done in transferring
capacitors. charge dQ' from plate 2 to plate 1. (b)
Total work done in charging the capacitor
Hence charge on each capacitor nearest may be considered as the energy stored
to a and b in the electric field between the plates.
= 300×3 = 900 μC Suppose now a small additional
(iii) Potential drop of 300 V across e andf charge dQ be transferred from plate 2 to
is equally shared by 3 μF capacitors. plate 1. The work done will be

Hence Vcd = 100 V. ' ' Q '


dW .=V ⋅ d Q = ⋅d Q
C
2.24 ENERGY STORED IN A
CAPACITOR The total work done in transferring a
charge Q from plate 2 to plate 1 [Fig.
38. How does a capacitor store
2.105(b)] will be
energy? Derive an expression for the
Q
energy stored in a capacitor. Q
'
Q 1 Q
'2 Q 2
W =∫ dW =∫ ⋅ d Q =[
'
] = ⋅
Energy stored in a capacitor. A 0 C 2C 0 2 C
capacitor is a device to store energy. The
process of charging up a capacitor This work done is stored as electrical
involves the transferring of electric potential energy U of the capacitor.
charges from its one plate to another. The 1 Q 1
2
2 1
work done in charging the capacitor is U = ⋅ = ⋅C V = QV [∵ Q=CV ]
2 C 2 2
stored as its electrical potential energy.

59
39. If several capacitors are connected If σ is the surface charge density on
in series or parallel, show that the energy the capacitor plates, then electric field
stored would be additive in either case. between the capacitor plates will be
Energy stored in a series combination σ
of capacitors.
E= or σ =ε 0 E
ε0
For a series combination, Q = constant Charge on either plate of capacitor is Q=
Total energy, σ A = ε0EA
∴ Energy stored in the capacitor is
[ ]
2 2
Q 1 Q 1 1 1
U= ⋅ = ⋅ + + +…
2 C 2 C 1 C2 C 3 Q (ε 0 EA) 1
2 2
U= = = ε E 2 Ad
Q
2
Q
2
Q
2 2C ε0 A 2 0
¿ + + +… 2⋅
2C 1 2C 2 2 C3 d
or U=U1 + U2 + U3 +... But Ad = volume of the capacitor
between its two plates. Therefore, the
Energy stored in a parallel energy stored per unit volume or the
combination of capacitors. For a parallel energy density of the electric field is given
combination, V = constant by
Total energy, U 1 2
u= = ε0 E
1 2 1 2 Ad 2
U = C V = [C 1+C 2 +C 3+ …]V
2 2 Although we have derived the above
1 2 1 2 1 2
equation for a parallel plate capacitor, it is
¿ C1 V + C2 V + C 3 V + … true for electric field due to any charge
2 2 2
configuration. In general, we can say that
or U = U1 + U2 + U3 +... an electric field E can be regarded as a
seat of energy with energy density equal
Hence total energy is additive both in
series and parallel combinations of
1 2
to ε 0 E . Similarly, energy is also
capacitors. 2
associated with a magnetic field.
2.25 ENERGY DENSITY OF AN
ELECTRIC FIELD 2.26 REDISTRIBUTION OF CHARGES
40. Where is the energy stored in a 41. If two charged conductors are
capacitor? Derive an expression for the touched mutually and then separated,
energy density of an electric field. prove that the charges on them will be
divided in the ratio of their capacitances.
Energy density of an electric field.
When a capacitor is charged, an electric Redistribution of charges. Consider
field is set up in the region between its two two insulated conductors A and B of
plates. We can say that the work done in capacitances C1 and C2, and carrying
the charging process has been used in charges Q: and Q2 respectively. Let V1 and
creating the electric field. Thus the V2 be their respective potentials. Then Q1
presence of an electric field implies stored = CJVJ and Q2 = CfV2
energy or the energy is stored in the
electric field.
Consider a parallel plate capacitor,
having plate area A and plate separation
d. Capacitance of the parallel plate
capacitor is given by
ε0 A Fig. 2.107 Redistribution of charges.
C=
d Now, if the two conductors are joined

60
by a thin conducting wire, then the positive 2
1 C 1 V 1 +C 2 V 2
charge will flow from the conductor at ¿ (C 1 +C2 )[ ]
2 C1 +C 2
higher potential to that at lowe potential till
their potentials become equal. Thus th 2
1 (C 1 V 1 +C 2 V 2 )
charges are redistributed. But the total ¿ ⋅
charge sti remains Q1 + Q2 2 (C 1+C 2)
If the capacitance of the thin Loss in energy,
connecting wire i negligible and the Ui – Uf
conductors are a sufficient distanc apart
2
so that do not exert mutual electric forces, 1 1 1 (C V +C V )
the: their combined capacitance will be C1 ¿ C1 V 21 + C2 V 22− ⋅ 1 1 2 2
2 2 2 (C 1+C 2 )
+ C2.
1
Total charge ¿ ¿
Common potential = 2 ( C 1 +C2 )
Total capacitance
2 2 2 2 2 2
Q1 +Q2 C 1 V 1+C 2 V 2 +C2 V 2−C1 V 1−C2 V 2−2 C1 C2 V 1 V 2 ¿
Or V = =
C1 +C 2 C 1+C 2 1 C1C2
¿
2 ( C1 +C 2)
[ V 1 +V 2−2 V 1 V 2 ]
2 2

If after redistribution charges on A and


B are Q1 am Q2 respectively, then 2
1 C 1 C 2 (V 1−V 2)
'
Q1=C 1 V and Q =C2 V ¿ ⋅
2 2 C 1+ C 2


Q1 C 1 This is always positive whether V1 > V2
=
Q2 C 2 or W <V2. So when two charged
conductors are connected, charges flow
Thus, after redistribution, the charges
from higher potential side to lower
on the two conductors are in the ratio of
potential side till the potentials of the two
their capacitances.
conductors get equalised. In doing so,
42. When two charged conductors there is always some loss of potential
having different capacities and different energy in the form of heat due to the flow
potentials are joined together show that of charges in connecting wires.
there is always a loss of energy.
Examples based on
Loss of energy in redistribution of
Energy stored in capacitors
charges. Let C1 and C2 be the
capacitances and V1 and V2 be the poteri Formulae Used
tials of the two conductors before they are
1. Energy stored in a capacitor,
connecte< together. Potential energy
2
before connection is 1 2 1 q 1
U = C V = ⋅ = qV
1 2 1 2 2 2 C 2
U i= C 1 V 1 + C 2 V 2
2 2 2. Energy stored per unit volume or the
After connection, let V be their energy density of the electric field of a
common potential then capacitor,

Q1 +Q2 C1 V 1+ C2 V 2 1 2
Total charge u= ε 0 E
V= = = 2
Total capacitance C1 +C 2 C 1+ C 2
3. Electric field between capacitor
Potential energy after connection is σ
plates, E=
1 2 1 2 1 2 ε0
U f = C 1 V + C 2 V = ( C 1+ C 2 ) V
2 2 2 Units Used
Capacitance is in farad, charge in
coulomb, and electric field in NC-1 or Vm-1,

61
energy in joule and energy density in Jm-1.
Example 66. Two parallel plates,
separated by 2 mm of air, have a
capacitance of 3 × 10-14 F and are charged
to a potential of 200 V. Then without
touching the plates, they are moved apart
till the separation is 6 mm. (i) what is the
1 2
potential difference between the plates? U s= C S V
(ii) What is the change in energy? 2
Solution. Charge, q = CV = 3 × 10-14 × 200 1 −12 2
¿ ×6 × 10 ×(50) J
=6 × 10-12 C 2
−10
When the separation increases from 2 ¿ 75 ×10 J
mm to 6 mm, the capacitance becomes
Charge drawn from the battery by series
'd 2 −14 −14 combination,
C = ' ⋅C= ×3 ×10 =10 F
d 6 −12 −10
q s=C s V =6 ×10 ×50 C=3 ×10 C
(i) P.D. between the plates becomes
Equivalent capacitance of the parallel
−12
' q 6 ×10 combination,
V = '= −14
=600 V
C 10 C p=C 1+C 2=12+12=24 pF
(ii) Initial energy stored in the capacitor, Energy stored in the parallel
1 2 1 −14 2 −10
combination,
U = C V = ×3 ×10 ×(200) =6 × 10 J
2 2
Final energy stored in the capacitor
' 1 ' '2 1 −14 2 −10
U = C V = ×10 ×(600) =18 ×10 J
2 2
Increase in energy = W -U = 12× 10-10 J.
Example 67. Two identical capacitors of
12 pF each are connected in series across 1 2
a battery of 50 V. How much electrostatic U p= C p V
2
energy is stored in the combination? If
these were connected in parallel across 1 −12 2
¿ ×24 × 10 × ( 50 ) J
the same battery, how much energy will 2
be stored in the combination now? Also −8
find the charge drawn from the battery in ¿ 3 ×10 J
each case. Charge drawn from the battery by the
[CBSE D 17] parallel combination,
−12 −9
Solution. Equivalent capacitance of series q p=C p V =24 ×10 ×50 C=1.2 ×10 C
combination,
Example 68. A 12 pF capacitor is
C 1 C 2 12× 12 connected to a 50 V battery. How much
C s= = =6 pF electrostatic energy is stored in the
C1 +C 2 12+12
capacitor? If another capacitor of 6 pF is
Energy stored in the series combination, connected in series with it with the same
battery connected across the combination,
find the charge stored and potential
difference across each capacitor.

[CBSE D 17]

62
Solution. Energy stored in 12 pF 6
capacitor,
C eq= μF
7
1 2 1
U = C 1 V = ×12 ×10 ×(50) J=1.5 ×10 J (ii) Charge drawn from the 7 V source,
−12 2 −8
2 2
6
Equivalent capacitance of 12 pF and 6
Q=C eq V = μ F ×7 V=6 μ C
7
pF capacitors, in series, will be
Energy stored,
C 1 C 2 12 ×6
C= = pF=4 pF Q2
−6 2
( 6 ×10 ) × 7
C1 +C 2 12+ 6 U= =
−6
J=21× 10 J=21 μ J
2 C eq 2 ×6 × 10−6
Charge stored in each capacitor of the
series combination, Example 70. Three identical capacitors
−12 −10 C1, C2 and C3 of capacitance 6 μF each
q=CV =4 ×10 ×50 C=2 ×10 C are connected to a 12 V battery as shown.
P.D. across 12 pF capacitor, Find:

q 2× 10
−10
50 (i) charge on each capacitor.
V 1= = V= V
C 1 12× 10 −12
3 (ii) equivalent capacitance of the network.
P.D. across 6 pF capacitor, (iii) energy stored in the network of
capacitors.
−10
q 2 ×10 100
V 2= = −12
V= V [CBSE D09]
C 2 6 ×10 3
Example 69. (i) Find equivalent
capacitance between A and B in the
combination given beloio. Each capacitor
is of 2 μF capacitance.

Fig. 2.108
Solution. (i) q and C2 are connected in
series across 12 battery while C3 is in
(ii) If a dc source of 7V is connected parallel with this combination. Equivalent
across AB, how much charge is drawn capacitance of C1 and C2 is
from the source and what is the energy C1 C 2 6 ×6
stored in the network? C 12= = =3 μ F
[CBSE D 17] C 1+ C2 6+ 6
Solution. (i) the equivalent network of Charge on either of the capacitors q and
the capacitors is shown in the figure. C2 is same.
Q1 = q2 = C12V = 3 μF × 12 V = 36 μC
Charge on C3, q3 =6μF× 12 V = 72 μC
(ii) Equivalent capacitance of the
network,
C = C12 + C3 = 3 μF + 6 μF = 9 μF.
Capacitors C2, C3 and C4 are in parallel.

(iii) Energy stored in the network,
C234 = C2 + C3 + C4 =2 +2 +2 =6 μF
1 2 1 −6 2 −4
Now, C1, C234 and C5 form a series U = C V = ×9 × 10 ×(12) =6.48 × 10 J
2 2
combination.
Example 71 .In Fig. 2.109, the energy
1 1 1 1 1 1 1 7
= + + = + + = stored in C4 is 27 /. Calculate the total
Ceq C 1 C234 C 5 2 6 2 6 energy stored in the system.

63
Fig. 2.109 Solution. Here C =2000 μF = 2 × 10-3 F, V
=1.5 V
Solution. Energy stored in C4 is
Energy stored in the capacitor,
1 2
U 4= C 4 V =27 J 1 1
2 2 −3 2 −3
U = C V = ×2 ×10 ×(1.5) =2.25 ×10 J
2 2
1 −6 2
Or ×6 × 10 ×V =27 Time during which capacitor is
2
discharged for producing flash,
2 27 × 2 6
or V = −6
=9 ×10 t = 0.1 millisecond = 0.1 × 10-3 s = 10-4 s
6 ×10
Power of flash,
Energy stored in C2, U 2.25 ×10
−3
P= = =22.5 W
1 −6 6 t 10
−4
U 2= × 2× 10 ×9 × 10 =9 J
2
Example 73. A 800 pF capacitor is
Energy stored in C3, charged by a 100 V battery. After some
time the battery is disconnected. The
1 −6 6
U 3= × 3 ×10 × 9× 10 =13.5 J capacitor is then connected to another
2 800 pF capacitor. What is the electrostatic
Energy stored in C2, C3 and C4 energy stored? [CBSE F 09]
= U2 + U3 + U4=9 + 13.5 +27 = 49.5 J Solution. Here C1=C2 = 800 pF =8× 10-10F,
Equivalent capacitance of C2, C3 and V1 = 100 V, V2 = 0
C4 connected in parallel Common potential,
= 2+ 3 + 5= 11 μF C 1 V 1 +C 2 V 2 8 ×10−10 ×100+ 0
V= = =50 V

2 2 −10 −10
q q C1 +C 2 8 ×10 +8 ×10
−6
=49.5 J [U= ]
2× 11×10 2C 1 2
U f = ( C 1+ C 2 ) V
Energy stored in C1 2
2 −6
q 49.5 ×2 ×11× 10 1
¿ ( 8 ×10 + 8× 10 ) × ( 50 ) =2× 10 J
−10 −10 2 −6
U 1= = =544.5 J
2 C1 2× 1× 10
−6 2
Total energy stored in the arrangement = Example 74
544.5 + 49.5 = 594.0 J. (i) A 900 pF capacitor is charged by a
Example 72. In a camera-flash circuit 100 V battery. Flow much electrostatic
(Fig. 2.110), a 2000 μF capacitor is energy is stored by the capacitor?
charged by a 1.5 V cell. When a flash is (ii) The capacitor is disconnected from the
required, the energy stored in the battery and connected to another 900
capacitor is discharged by means of a pF capacitor. What is the electrostatic
trigger T through a discharge tube in 0.1 energy stored by the system?
millisecond. Find the energy stored in the
capacitor and the power of the flash. (Hi) Where has the remainder of the
energy gone?
[ISCE 97]
[NCERT: CBSE OD 90]
Solution. (i) The charge on the capacitor is

64
q = CV =900 x 10-12 F × 100 V =9 × 10-8 C are connected. [CBSE OD 15]
The energy stored by the capacitor is Solution. (i) Let q and C, be the
capacitances of the two capacitors and V
1 2 1 1 −8
U = C V = qV = × 9 ×10 C ×100 V be their common potential. Then
2 2 2
Total charge q 1 +q2 C1 V 1+ C2 V 2
= 4.5 × 10-6 J. V= = =
Total capacitance C 1 +C2 C 1+ C 2
C1 V 1
Or V = [∵V 2 =0]
C 1+C 2
(ii) Energy stored in the capacitors
before connection,
Fig. 2.111 1 2
U i= C 1 V 1
(ii) In the steady situation, the two
2
capacitors have their positive plates at the Total energy after connection,
same potential, and their negative plates 2 2
at the same potential. Let the common 1 2 1 C1V 1
u f = ( C 1+ C 2 ) V = ( C 1+ C 2 ) 2
potential difference be V. The charge on 2 2 ( C1 +C 2 )
each capacitor is then cf = CV. By charge

( )
2 2
conservation, q = q/2. 1 C1 V 1 C1
∴ Total energy of the system
¿ = U
2 C 1 +C2 C 1 +C 2 i
'
1 ' ' ' ' ' q Clearly, uf <ui
¿ 2 × q V =q V =q ⋅
2 C Hence the total energy of the
1 q 1
2
1 1 combination is less than the sum of the
¿ ⋅ = ⋅qV = × qV energy stored in the capacitors before
4 C 4 2 2
they are connected.

[ ∵ q' =
q
2
q
and =V
C ] Example 76. Two capacitors of unknown
capacitances C1 and C2 are connected first
1 in series and then in parallel across a
−6 −6
¿ × 4.5× 10 J=2.25× 10 J battery of 100 V. If the energy stored in
2 the two combinations is 0.045 J and 0.25
(iii) There is a transient period before J respectively, determine the values of C1
the system settles to the situation (ii). and C2. Also calculate the charge on each
During this period, a transient current capacitor in parallel combination.
flows from the first capacitor to the [CBSE D 15]
second. Energy is lost during this time in
the form of heat and electromagnetic Solution. For series combination, we
radiation. have
Example 75. A capacitor is charged to 1 C1 C2 2
potential V1 the power supply is U= V
2 C 1+C 2
disconnected and the capacitor is
∴ 0.045= ×(100) …(i)
connected in parallel to another 1 C1C2 2
uncharged capacitor. 2 C 1+ C 2
(i) Derive the expression for the common For parallel combination, we have U =
potential of the combination of (C1+C2)
capacitors.
1 2
(ii) Show that total energy of the U = (C 1+C 2 )V
combination is less than the sum of 2
the energy stored in them before they

65
∴0.25= (C 1+ C2 )×(100)
1 2 Final energy stored,
2 1 2 1 −6 2
U f = ( C 1+ C2 ) V = ( 6+ 4 ) × 10 × ( 90 )
Or C 1+C 2=0.5 × 10 … (ii)
−4 2 2
From (i), = 4.05 × 10-2 J
1 C1C2 2 The loss of energy on connecting the two
0.045= × −4
×(100) capacitors,
2 0.5 ×10
−2
Or C 1 C 2=0.045 ×10
−8 ΔU =U i−U f =( 6.75−4.05 ) ×10
2 2 = 2.7 × 10-2 J = 0.027 J.
Now (C 1−C 2) =(C1 +C 2) −4 C 1 C 2
2 2
Example 78. In the following arrangement
(C 1−C 2) =(C1 +C 2) −4 C 1 C 2 of capacitors, the energy stored in the 6μF
−8 −8 capacitor is E. Find the value of the
¿( 0.25−0.180) ×10 =0.07 ×10 following:
∴ C 1−C2 =√ 0.07 ×10 =026 × 10
−4 −4
... (iii) (i) Energy stored in 12 μF capacitor.
On solving (ii) and (iii), we get (ii) Energy stored in 3μF capacitor.
q = 0.38 × 10-4 F and C2 = 0.12 × 10-4 (iii) Total energy drawn from the battery.
F Charges on capacitors C1 and C2 in
parallel combination are :
Q1 = C1V = 0.38 × 10-4× 100 C = 0.38 × 10-
2
C
Q2 = C2V =0.12 ×10-4× 100 C = 0.12 ×10 -2
C.
Example 77. A capacitor of capacitance 6
μF is charged to a potential of 150 V. Its
potential falls to 90 V, when another Solution.
capacitor is connected to it. Find the 1 2 1 −6 2 −6 2
capacitance of the second capacitor and E= C V = ×6 × 10 V =3 ×10 V
2 2
the amount of energy lost due to the
∴V =
connection. 2 E
−6
Solution. Here C1 = 6 μF, V1 = 150 V, V2 = 3 ×10
0, V=90V, C2=? Energy stored in 12 μF capacitor
Common potential, 1 −6 E
¿ ×12 ×10 × =2 E
C 1 V 1 +C 2 V 2 2 3 ×10
−6

V=
C1 +C 2 (ii) Capacitance of the parallel
−6 combination
6 ×10 × 150+0
or 90 V= −6 = 6 + 12 =18μF
6 × 10 +C2
−6
Charge on parallel combination,
6 × 10 ×150
−6 −6
or C 2+ 6× 10 = =10 ×10 Q = CV = 18 × 10-6 V
90
−6
Charge on3μF capacitor
or C 2=4 × 10 F=4 μ F
= Q = 3 × 10-6 V1
∴3 ×10 V 1 =18× 10 V
Initial energy stored, −6 −6

1 2 1 −6 2
U i=U 1= C1 V 1= ×6 × 10 ×(150) ⇒ V 1=6 V
2 2
= 6.75 × 10-2 J Energy stored in 3μF capacitor

66
1 2 1 −6 2 energy stored in the first capacitor.
¿ C V 1= × 3× 10 ×(6 V )
2 2 [Roorkee 96] (Ans. 2.5 J, 1:2)
1 −6 36 E 5. Find the total energy stored in the
¿ ×3 ×10 × =18 E
2 3 × 10
−6
capacitors in the network shown
below.
(iii) Total energy drawn = E + 2E + 18E = [CBSE D 04]
21E.
(Ans. 3.6 ×10-5 J)

Problems for Practice


1. For flash pictures, a photographer
uses a capacitor of 30 μF and a
charger that supplies 3 × 10J V.
Find the charge and energy
expended in joule for each flash.
Fig. 2.113
(Ans. 9 × 10-2 C, 135 J)
6. A 10 μF capacitor is charged by a 30
2. Three capacitors of capacitances 10 V d.c. supply and then connected
μF, 20 μF and 30 μF are connected in across an uncharged 50 μF capacitor.
parallel to a 100 V battery as shown in Calculate (i) the final potential
Fig. 2.112. Calculate the energy stored difference across the combination, and
in the capacitors. [ISCE 94] (ii) the initial and final energies. How
(Ans. 0.3 J) will you account for the difference in
energy? [CBSE OD 04]
[Ans. (i) 5V, (ii) ui = 4.5 ×10-3 J, Uf = 0.75 ×
10-3 J]
7. Net capacitance of three identical
capacitors in series is 1 μF. What will
be their net capacitance if connected
in parallel?
Find the ratio of energy stored in the two
Fig. 2.112 configurations if they are both connected
to the same source.
3. (a) How many electrons must be
added to one plate and removed from [CBSE OD 11] (Ans. 9 μF, 1 : 9)
the other so as to store 25.0 J of
8. Figure 2.114 shows a network of five
energy in a 5.0 nF parallel plate
capacitors connected to a 100 V
capacitor ?
supply. Calculate the total charge and
(b) How would you modify this capacitor energy stored in the network.
so that it can store 50.0 J of energy
[CBSE Sample Paper 08]
without changing the charge on its
plates? [CBSE SP 18] (Ans. 4 × 10-4C, 0.02 J)
4. A battery of 10 V is connected to a
capacitor of capacity 0.1 F. The
battery is now removed and this
capacitor is connected to a second
uncharged capacitor. If the charge
distributes equally on these two
capacitors, find the total energy stored
in the two capacitors. Further,
compare this energy with the initial

67
(b) Without changing the charge on the
ε0 A
plates, we can make Chalf. As C=
d
we should either d double the plate
separation d or make the plate area A
half.
4. Initial energy stored in the first
capacitor is
1 2 1 2
U i= C V = ×0.1 ×(10) =5.0 J
9. Two capacitors are in parallel and the 2 2
energy stored is 45 J, when the
When the first capacitor is connected to
combination is raised to potential of
the second uncharged capacitor, the
3000 V. With the same two capacitors
charge distributes equally. This implies
in series, the energy stored is 4.05 J
that the capacitance of second capacitor
for the same potential. What are their
is also C. The voltage across each
individual capacitances?
capacitor is now V/2. The final total energy
(Ans. 9 μF, 1 μF) stored in the two capacitors is
10. Find the ratio of the potential 2 2
1 V 1 V 1 2
differences that must be applied U f = C ( ) + C ( ) = C V =2.5 J
2 2 2 2 4
across the parallel and the series

combination of two capacitors q and q U f 2.5 1
with their capacitances in the ratio 1 : = = =1: 2
U i 5.0 2
2 so that the energy stored in the two
cases, becomes the same. 5. The two 2μF capacitors on the right
side are in series, their equivalent
[CBSE F 10 ; OD 16] (Ans. √ 2 : 3) 2× 2
capacitance = =1 μ F
HINTS 2+2
1. Here C = 30μF = 3×10-5F, V = 3×103V This 1 μF capacitance is in parallel with
the central 1 μF capacitor. Their
Charge, q = CV = 3 × 10-5 × 3 × 103 C = 9
equivalent capacitance = 1 + 1 = 2 μF
×10-2 C
This 2μF capacitance is in series with the
Energy,
2μF capacitor at the bottom. Their
1 2 1 −5 6
U = C V = ×3 ×10 × 9 ×10 =135 J equivalent capacitance
2 2
2× 2
2. C=C 1+C 2 +C3 =10+20+30=60 μF =¿ =1 μ F
2+2
= 60 ×10-6 F Finally, 1 μF capacitance is in parallel with
1 2 1 −6 2 the left out 1 μF capacitor. The equivalent
U = C V = ×60 ×10 ×(100) =0.3 J capacitance is C=1+1=2 μ F=2× 10−6 F
2 2
2 V = 6V

q
3. (a) u=
2C
1 2 1 −6 2 −5
2 −9
q =2 UC=2 ×25 J ×5.0 ×10 C=25× 10
−8
U = C V = ×2 ×10 ×(6) =3.6 × 10 J
2 2
or q = 5 × 10-4C
6. Here q = 10 μF = 10 × 10-6 F, V1 =
If n is the number of electrons added or 30 V,
removed, then q = ne = 5 × 10-4C
q =50μF = 50×10-6 F, V2=0
∴ n=
−4
5× 10 15 (i) Common potential,
−19
=3.125 ×10 electrons.
1.6 × 10

68
C 1 V 1 +C 2 V 2 10× 10−6 ×30+ 0 of equivalent capacitance,
V= = −6
=5 V
C1 +C 2 (10+50)× 10 6×3
C 12= =2 μ F
(ii) Initial electrostatic energy of 10 μF 6+3
capacitor, This combination is in parallel with the fifth
capacitor of 2 μF.
∴ Net capacitance, C = 2 + 2 = 4 μF
1 2
U i= C 1 V 1
2
1 −6 2 −3 Total charge, q = CV = 4 × 10-6 × 100 = 4
¿ ×10 ×10 ×(30) =4.5 ×10 J ×10-4 C
2
Final electrostatic energy of the Total energy stored,
combination, 1 2 1 −6 2
U = C V = × 4 ×10 ×(100) =0.02 J
1 −6 2 −3 2 2
U f = ( 10+50 ) ×10 × ( 5 ) =0.75 × 10 J
2 C1 1
−3 10. Given =
Loss in energy = U i−U f =3.75 ×10 J C2 2
The difference in energy is lost in the form Now u p=u s
of heat and electromagnetic radiation as
P t*
the charge flows from first capacitor to
second capacitor. 1 2 1 2
or C V = C V
C 2 p p 2 s 5
7. Here C 5= =1 μF
3 Vp
2
Cs C1C2
∴C = 3 μF
or = =
V
2
s
C p G1 +C 2
Cp = 3C=9μF C1 C2
1 ¿ 2 (q + q)2
Cs V
2 (C1 +C2)
Us 2 C 1
= = s = =1 :9 C1
Up 1 Cp 9
C V2 C2
2 p 2 2
¿ = =

( ) 2 + 1)
(
2 2
8. The equivalent circuit diagram for the C1 1 9
+1
given network is shown below: C2


Vp
=√ 2 :3
Vs

Fig. 2.115
Two 3μF capacitors in parallel. The
equivalent capacitance,
C1 = 3+ 3 = 6 μF
The lμF capacitor and a 2μF capacitor are
in parallel. Their equivalent capacitance, 2.27 DIELECTRICS AND THEIR
C2 = 1 + 2 = 3 μF POLARIZATION
Then C1and C2 form a series combination 43. What are dielectrics? Explain the

69
difference in the behaviour of a conductor induces dipole moment by stretching or
and a dielectric in the presence an re-orienting the molecules of the dielectric.
external electric field. Distinguish between The induced dipole moment sets up an
polar and non-polar dielectrics. electric field find which opposes E0 but
does not exactly cancel this field. It only
Dielectrics. In insulators, the electrons
reduces it.
remain attached to the individual atoms or
molecules. However, these electrons can Polar and non-polar dielectrics. A
suffer small movements within the atoms dielectric may consist of either polar or
or molecules under the influence of an non-polar molecules. A molecule in which
external electric field. The net effect of the centre of mass of positive charges
these microscopic movements gives rise (protons) does not coincide with the centre
to some important electric properties to of mass of negative charges (electrons) is
such materials. In view of these electrical called a polar molecule.
properties, insulators are called
The dielectrics made of polar
dielectrics.
molecules are called polar dielectrics. The
A dielectric is a substance which does polar molecules have unsym- metrical
not allow the flow of charges through it but shapes. They have permanent dipole
permits them to exert electrostatic forces moments of the order of 10-1 Cm. For
on one another through it. A dielectric is example, a water molecule has a bent
essentially an insulator which can be shape with its two O - H bonds inclined at
polarised through small localised an angle of 105° as shown in Fig. 2.117. It
displacements of its charges. has a very large dipole moment of 6.1 ×
10-30 Cm. Some other polar molecules are
Examples. Glass, wax, water, air,
HC1, NH, CO, CH-.OH etc.
wood, rubber, stone, plastic, etc.
Difference in the behaviour of a
conductor and a dielectric in the presence
of an external electric field. Dielectrics
have negligibly small number of charge
carriers as compared to conductors.
In a conductor, the external field E0
moves the free charge carriers inducing
field Eind in the opposite direction of E0.
The process continues until the two fields
cancel each other and the net electric field Fig. 2.117 Some polar and non-polar
in the conductor becomes zero. molecules.
A molecule in which the centre of
mass of positive charges coincides with
the centre of mass of negative charges is
called a non-polar molecule. The
dielectrics made of non-polar molecules
are called non-polar dielectrics. Non-polar
molecules have symmetrical shapes. They
have normally zero dipole moment.
Examples of non-polar molecules are N2,
O2, CO2, CH4, etc.
44. How does a dielectric develop a
net dipole moment in an external electric
Fig. 2.116 Difference in the behaviour of a
field when it has (i) non-polar molecules
conductor and a dielectric in an
and (ii) polar molecules?
external electric field.
Polarization of a non-polar dielectric in
In a dielectric, the external field E0
an external electric field. In the absence of

70
any electric field, the centres of positive the dipole moments of different molecules
and negative charges of the molecules of tend to align with the field. As a result,
a non-polar dielectric coincide, as shown there is a net dipole moment in the
in Fig. 2.118(a)(z). The dipole moment of direction of the field, as shown in Fig.
each molecule is zero. In the presence of 2.118(b) (ii). The extent of polarisation
an external electric field E0, the centres of depends on relative values of two
positive charges are displaced in the opposing energies:
direction of external field while the centres
1. The potential energy of the dipole in
of negative charges are displaced in the
the external field which tends to align
opposite direction. The displacement of
the dipole with the field.
the charges stops when the force exerted
on them by the external field is balanced 2. Thermal energy of agitation which
by the restoring force due to the internal tends to randomise the alignment of
fields in the molecules. This induces the dipole.
dipole moment in each molecule i.e., each Hence both polar and non-polar
non-polar molecule becomes an induced dielectrics develop a net dipole moment in
dipole. The induced dipole moments of the presence of an external electric field.
different mole- cules add up giving a net This fact is called polarization of the
dipole moment to the dielectric in the dielectric.
direction of the external field, as shown in
Fig. 2.118(a) (ii). The polarization P is defined as the
dipole moment per unit volume and its
magnitude is usually referred to as the
polarization density. The direction of P is
same as that of the external field ⃗
E0
45. Explain why the polarization of
dielectric reduces the electric field inside
the dielectric. Hence define dielectric
constant.
Reduction of electric field by the
Fig. 2.118 [a) Polarization of a non-polar
polarization of a dielectric. Consider a
dielectric in an external electric field.
rectangular dielectric slab placed in a
Polarization of a polar dielectric in an uniform electric field E0 acting parallel to
external electric field. The molecules of a two of its faces, as shown in Fig. 2.119(A).
polar dielectric have permanent dipole Its molecular dipoles
moments. In the absence of any external
electric field, the dipole moments of
different molecules are randomly oriented
due to thermal

Fig. 2.118 (b) Polarization of a polar


dielectric in an external electric field.
agitation in the material, as shown in Fig.
2.118(b)(z). So the total dipole moment is
zero. When an external field is applied,

71
called dielectric constant (K) or relative
permittivity (εr). Thus

E0 ⃗
E0
κ= =

E ⃗ E0− ⃗
Ep
46.Define polarisation density. How is
it related to the induced surface charge
density?
Polarisation density. The induced
dipole moment developed per unit volume
Fig. 2.119 (a) Polarization of a dielectric. of a dielectric zvhen placed in an external
(b) Reduced field in a dielectric, E = E0-Ep. electric field is called polarisation density.
It is denoted by P. Suppose a dielectric
align themselves in the direction of ⃗ E0 . slab of surface area A and thickness d
This results in uniform polarization of the acquires a surface charge density ± σp due
dielectric, i.e., every small volume of the to its polarisation in the electric field and
slab has a dipole moment in the direction its two faces acquire charges ± Qp. Then
of ⃗
E0 . The positive charges of the dipoles Qp
of first vertical column cancel the negative σ p=
A
charges of the dipoles of second column
and so on. Thus the volume charge We can consider the whole dielectric slab
density in the interior of the slab is zero. as a large dipole having dipole moment
However, there is a net uncancelled equal to Qp d. The dipole moment per unit
negative charge on the left face and volume or the polarisation density will be
uncancelled positive charge on the right
dipole moment of dielectric
face of the slab. P=
volume of dielectric
The uncancelled charges are the induced
surface charges due to the external field Qp d Qp
¿ = =σ p

E0 . Since the slab as a whole remains Ad A
electrically neutral, the magnitude of the Thus the polarisation density may be
positive induced surface charge is equal defined as the charge induced per unit
to that of the negative induced surface surface area.
charge.
Obviously, a uniformly polarised
Thus the polarized dielectric is equivalent dielectric with uniform polarisation density
to two charged surfaces with induced P can be replaced by two surface layers
surface charge densities ± σ p . (perpendicular to P) of surface charge
densities ± cp , and zero charge density in
Reduced field inside a dielectric and
the interior.
dielectric constant. In case of a
homogeneous and isotropic dielectric, the 47. Define electric susceptibility.
induced surface charges set up an electric Deduce the relation between dielectric
field ⃗
E p (field due to polarization) inside constant and electric susceptibility.
the dielectric in a direction opposite to that Electric susceptibility. If the field ⃗
E is
of external field ⃗ E0 , thus tending to reduce not large, then the polarisation ⃗ P is
the original field in the dielectric. The proportional to the resultant field ⃗ E
resultant field ⃗ E in the dielectric will be existing in the dielectric, i.e.,
equal to ⃗ E0 −⃗ E p and directed in the

P ∝⃗
E or ⃗
P=ε 0 χ ⃗
E
direction of ⃗E0 .
where x (chi) is a proportionality constant
The ratio of the original field ⃗
E0 and the called electric susceptibility. The
⃗ ⃗
reduced field E0 − E p in the dielectric is multiplicative factor ε0 is used to keep x

72
dimensionless. Clearly, The unit of dielectric strength is same
as that of electric field i.e., Vm-1. But the

P
χ= more common practical unit is kV mm-1.

ε0 E
Table 2.1 Dielectric constants and
Thus the ratio of the polarisation to ε0 dielectric strengths of some common
times the electric field is called the electric dielectrics.
susceptibility of the dielectric. Like P, it
also describes the electrical behaviour of Dielectric Dielectric Dielectric
a dielectric. The dielectrics with constant x strength in kV
constant
are called linear dielectrics. mm-1
Relation between K and x- The net Vacuum 1.00000 00
electric field in a polarised dielectric is

E =⃗
E0− ⃗
Ep Air 1.00054 0.8

σp P Water 81 —

Ep= =
ε0 ε0
Paper 3.5 14
P
But ⃗
E =⃗
E 0− Pyrex 4.5 13
ε0
glass
⃗ P
E =⃗
E 0−
ε0 Mica 5.4 160

ε0 χ ⃗
E Porcelain 6.5 4

E =⃗
E 0−
ε0
2.29 CAPACITANCE OF A PARALLEL
ε0 χ ⃗
E PLATE CAPACITOR WITH A
Or ⃗
E =⃗
E 0− DIELECTRIC SLAB
ε0
49.Deduce the expression for the
[ ⃗P=ε 0 χ ⃗E ] capacitance of a parallel plate capacitor
when a dielectric slab is inserted between
Dividing both sides by ⃗
E , we get its plates. Assume the slab thickness less

E0 than the plate separation.
1= −χ

E Capacitance of a parallel plate
capacitor with a dielectric slab. The
1=κ− χ or κ=1+ χ capacitance of a parallel plate capacitor of
2.28 DIELECTRIC STRENGTH plate area A and plate separation d with
vacuum between its plates is given by
48. What do you mean by dielectric
strength of a dielectric? ε0 A
C 0=
Dielectric strength. When a dielectric is d
placed in a very high electric field, the Suppose initially the charges on the
outer electrons may get detached from capacitor plates are ± Q. Then the uniform
their parent atoms. The dielectric then electric field set up between the capacitor
behaves like a conductor. This plates is
phenomenon is called dielectric
breakdown.
σ Q
E0 = =
ε0 A ε0
The maximum electric field that can
exist in a dielectric without causing the When a dielectric slab of thickness t <
breakdown of its insulating property is d is placed between the plates, the field E0
called dielectric strength of the material. polarises the dielectric. This induces
charge - Qp on the upper surface and

73
ε0 A
C= ⋅ κ=κ C 0
d
Thus the capacitance of a parallel
plate capacitor increases K times when its
entire space is filled with a dielectric
material. C
Fig. 2.120 A dielectric slab placed in a C
Clearly, κ=
parallel plate capacitor. C0
+ Qp on the lower surface of the dielectric. Dielectric constant =
These induced charges set up a field E
inside the dielectric in the opposite Capacitance with dielectric between two plates
direction of ⃗
E0 . The induced field is given Capacitance with vacuum between two plates
by Thus the dielectric constant of a
dielectric material may be defined as the
Ep=
σp P Q
[
= σ p= =P , polarisation density
ε0 ε0 A ] ratio of the capacitance of a capacitor
completely filled with that material to the
capacitance of the same capacitor with
The net field inside the dielectric is
vacuum between its plates.
E0
E=E0−E p= 2.30 CAPACITANCE OF A PARALLEL
κ PLATE CAPACITOR WITH A

[ ]
CONDUCTING SLAB
E0
∵ =κ 50.Deduce the expression for the
E0 −E p
capacitance of a parallel plate capacitor
when a conducting slab is inserted
between its plates. Assume the slab
where K is the dielectric constant of the
thickness less than the plate separation.
slab. So between the capacitor plates, the
field E exists over a distance t and field E0 Capacitance of a parallel plate
exists over the remaining distance (d -1). capacitor with a conducting slab. Consider
Hence the potential difference between a parallel plate capacitor of plate area A
the capacitor plates is and plate separation d. If the space
between the plates is vacuum, its
E0 capacitance is given by
V =E 0 ( d−t )+ Et=E 0 ( d−t )+ t
κ
ε0 A

[ ]
E0 C 0=
∵ =κ d
E Suppose initially the charges on the
capacitor plates are ± Q. Then the uniform
(
¿ E0 d−t +
t
)=
Q
κ ε0 A
d−t +(t
κ ) electric field set up between the capacitor
plates is
The capacitance of the capacitor on σ Q
introduction of dielectric slab becomes E0 = =
ε0 A ε0
Q ε0 A
C= = where σ is the surface charge density.
V t The potential difference between the
d−t+
κ capacitor plates will be
Special Case If the dielectric fills the Qd
entire space between the plates, then t = V 0=E 0 d=
A ε0
d, and we get
When a conducting slab of thickness t
< d is placed between the capacitor

74
plates, free electrons flow inside it so as to energy in the electric field between
reduce the field to zero inside the slab, as their plates.
shown in Fig. 2.121. Charges - Q and + Q
2.32 EFFECT OF DIELECTRIC ON
appear on the upper and lower faces of
VARIOUS PARAMETERS
the slab. Now the electric field exists only
in the vacuum regions between the plates 52.A parallel-plate capacitor is
of the capacitor on the either side of the charged by a battery which is then
slab, i.e., the field exists only in thickness disconnected. A dielectric slab is then
d - t, therefore, potential difference inserted to fill the space between the
between the plates of the capacitor is plates. Explain the changes, if any, that
occur in the values of (i) charge on the
Q
V =E 0 (d−t)= (d−t ) plates, (ii) electric field between the plates,
A ε0 (iii) p.d. between the plates, (To)
capacitance and (v) energy stored in the
capacitor.
Effect of dielectric when the battery is
kept disconnected from the capacitor. Let
Q0, C0, V0, Σ0 and U0 be the charge,
capacitance, potential difference, electric
field and energy stored respectively before
Fig. 2.121 A conducting slab placed in a the dielectric slab is inserted. Then
parallel plate capacitor.
∴Capacitance of the capacitor in the
V0 1 2
Q 0=C 0 V 0 , E 0= , U 0= C 0 V 0
d 2
presence of conducting slab becomes
(i) Charge. The charge on the
Q ε A ε A d d capacitor plates remains QQ because the
C= = 0 = 0 ⋅ or C=( )⋅ C0
V (d−t ) d d−t d−t battery has been disconnected before the
insertion of the dielectric slab.
Clearly, C > C0. Thus the introduction
of a conducting slab of thickness t in a (ii) Electric field. When the dielectric
parallel plate capacitor increases its slab is inserted between the plates, the
d induced surface charge on the dielectric
capacitance by a factor of reduces the field to a new value given by
d−t
E0
2.31 USES OF CAPACITORS E= K
κ
51. Mention some important uses of
capacitors. (iii) Potential difference. The
reduction in the electric field results in the
Uses of capacitors. Capacitors are decrease in potential difference.
very useful circuit elements in any of the
electric and electronic circuits. Some of E0 d V 0
their uses are V =Ed= =
κ κ
1. To produce electric fields of desired (iv) Capacitance. As a result of the
patterns, e.g., for Millikan's decrease in potential difference, the
experiment. capacitance increases K times.
2. In radio circuits for tuning. Q0 Q0 Q0
C= = =κ =κ C 0
3. In power supplies for smoothing the V V 0 /κ V0
rectified current.
iii) Energy stored. The energy stored
4. For producing rotating magnetic fields decreases by a factor of K.
in induction motors.
2
1 2 1 V0 1 1 2 U0
5. In the tank circuit of oscillators. U = C V = (κ C 0)( ) = ⋅ C 0 V 0 =
2 2 κ κ 2 κ
6. They store not only charge, but also

75
53.A parallel plate capacitor is
charged by a battery. When battery
V0 V = V0 (constant)
V=
remains connected, a dielectric slab is κ
inserted between the plates. Explain what
changes, if any, occur in the values of (i) E0 E = E0 (constant)
p.d. between the plates, (ii) electric field E=
κ
between the plates, (iii) capacitance, (iv)
charge on the plates and (v) energy stored C=κ C 0 C = K C0
in the capacitor?
Effect of dielectric when battery U0 U = K U0
remains connected across the capacitor. U=
κ
Let Q0, C0, V0, E0 and U0 be the charge,
capacitance, potential difference, electric For Your Knowledge
field and energy stored respectively,
before the introduction of the dielectric  Capacitance of a parallel plate
slab. Then capacitor with compound
dielectric.
V0 1 2
Q 0=C 0 V 0 , E 0= , U 0= C 0 V 0 A. Series type arrangement. If a capacitor
d 2 is filled with n dielectric slabs of
(i) Potential difference. As the battery thicknesses t1, t2, …. tn, as shown in Fig.
remains connected across the capacitor, 2.122(a), then this arrangement is
so the potential difference remains equivalent to n capacitors connected in
constant at V0 even after the introduction series.
of dielectric slab. With a single dielectric slab, ε∩ A
(ii) Electric field. As the potential ε0 A
difference remains unchanged, so the C=
electric field E0 between the capacitor t
d−t +
plates remains unchanged. k
V V0 Capacitance with n dielectric slabs will be
E= = =E0
d d ε0 A
c=
(iii) Capacitance. The capacitance t1 t2 tn
increases from C0 to C. d−(t 1+ t 2 +…+t n )+( + +…+ )
κ1 κ2 κn
C=κ C 0
But d=t 1 +t 2+t 3 +… .+t n
(iv) Charge. The charge on the
ε0 A

capacitor plates inσeases from Qj'to Q. C=
t1 t2 tn
Q=CV =κ C 0 ⋅V 0=κ Q 0 + +… …+
K1 K2 Kn
(v) Energy stored. The energy stored
in the capacitor increases K times.
1 2 1 2 1 2
U = C V = (κ C 0)V 0=κ ⋅ C0 V 0=κ U 0
2 2 2
Table 2.2 Effect of dielectric on various
parameters.

Battery disconnected Battery kept


from the capacitor connected across
the capacitor

Q = Q0 (constant) Q = K Q0

76
B. Parallel type arrangement. The 2 π ε 0 ×l
arrangement shown in Fig. 2.122(b) C=
b
consists of n capacitors in parallel, having 2303 log 10
plate areas A1, A2, ……, An , and plate
a
separation d. 6. Effect of dielectric with battery
The equivalent capacitance of the parallel disconnected from the capacitor,
arrangement will be V0 E0 U0
Q=Q 0 , V = , E= , C=κ C 0 , U=
C=C 1+C 2 +… .+C n κ κ κ
κ 1 ε 0 A1 κ 2 ε 0 A 2 κnε 0 An 7. Effect of dielectric with battery
¿ + + …+ connected across the capacitor,
d d d
Q=κ Q0 , V =V 0 , E=E 0 , C=κ C0 , U=κ U 0
ε0
C= (κ 1 A 1+ κ2 A 2 +…+κ n A n ) Units Used
d
A Capacitance C is in farad, charge q in
If A1= A 2=… .= A n= , then coulomb, potential difference V in volt,
n area A in m2, thicknesses d and t in metre.
ε0 A Constant Used
C= (κ + k +… .+κ n )
dn 1 2
Permittivity constant,
Examples based on ε 0=8.85 ×10
−12 2
CN m
−1 −2

Capacitors filled with dielectrics and Example 79. In α parallel plate capacitor,
conductors the capacitance increases from 4 μF
Formulae Used to 80 μF, on introducing a dielectric
medium between the plates. What is
1. Capacitance of a parallel plate the dielectric constant of the medium?
capacitor filled with a dielectric of
dielectric constant K, Solution. K=
Capacitance withdielectric
ε 0 κA =
C=κ C 0= Capacitance without dielectric
d 80 μ F
=20
2. Capacitance of a parallel plate 4 μF
capacitor with a dielectric slab of
Example 80. A parallel plate capacitor
thickness t (< d) in between its plates,
with air between the plates has a
ε0 A capacitance of 8 pF. The separation
C= between the plates is now reduced by half
1
d−t (1− ) and the space between them is filled with
κ
a medium of dielectric constant 5.
3. Capacitance of a parallel plate Calculate the value of capacitance of the
capacitor with a conducting slab of capacitor in the second case.
thickness t (<d) in between its plates,
[CBSE OD 06]
ε0 A Solution. Capacitance of the capacitor
C=
d−t with air between its plates,
4. Capacitance of spherical capacitor ε0 A
filled with a dielectric, C 0= =8 pF
d
ab
C=4 π ε 0 κ ⋅ When the capacitor is filled with
b−a dielectric (K = 5) between its plates and
5. Capacitance of a cylindrical capacitor the distance between the plates is
filled with a dielectric, reduced by half, capacitance becomes

77
ε 0 κA ε 0 ×5 × A Capacitance after the dielectric has
C= = =10 C0 =10 ×8=80 pF been put,
d /2 d /2
Example 81. Figure 2.123 shows two C = KC0 = 5 × 20μF = 100μF
identical capacitors, Cj and C2, each ofl μF As the battery has been disconnected,
capacitance connected to a battery of 6 V. charge q0 remains same.
Initially switch 'S' is closed. After some
(i) Energy stored before the dielectric
time 'S' is left open and dielectric slabs of
is put,
dielectric constant K=3 are
2 −6 2
1 q0 1 (2000 ×10 )
U0= ⋅ = J =0.1 J
2 C 0 2 20 ×10−6
(ii) Energy stored after the dielectric input,
2 2
1 q 0 1 ( 2000× 10 )
−6

Fig. 2.123 U= ⋅ = ⋅ J=0.02 J


2 C 2 100 ×10−6
inserted to fill completely the space Example 83. Two parallel plate capacitors,
between the plates of the two capacitors. X and Y, have the same area of plates
How will the (i) charge and (ii) potential and same separation between them. X
difference betiveen the plates of the has air between the plates while Y
capacitors be affected after the slabs are contains a dielectric medium of εr = 4.
inserted? [CBSE D 11]
Solution. With switch S closed, V1 - V2
-6V
∴q 1=q 2=1 μ F × 6 V=6 μ C
When dielectric slabs (K=3) are
Fig. 2.124
inserted, capacitance of each capacitor
becomes 3 μF. (z) Calculate capacitance of each
capacitor if equivalent capacitance of
P.D. across C1 V’1 = 6 V
the combination is 4 μf.
Charge, d1 = 3 μF × 6 V = 18 μC
(ii) Calculate the potential difference
With switch S open, the p.d. on C2 between the plates of X and Y.
attains a new value but charge q2 is still 6
(iii) What is the ratio of electrostatic
μC
energy stored in X and Y?
∴V 2=
' 6 μC
=2 V [CBSE D 04, 09, 16]
3μ F
Solution. (i) Let CX = C. Then CY = εrC
Example 82. A parallel plate capacitor, of =4C
capacitance 20 μF, is connected to a 100
V supply. After sometime the battery is Now X and Y are connected in series.
disconnected, and the space, between the CX CY C .4 C
plates of the capacitor is filled with a C eq= =
dielectric, of dielectric constant 5. C X +C Y C+ 4 C
Calculate the energy stored in the 4
capacitor (i) before (ii) after the dielectric or 4 μ F= C
5
has been put in between its plates.
or C = 5 μF
[CBSE OD 16C]
Hence CX = C = 5 μF and CY = 4 C = 4
Solution. Here C0 =20μF, V0 =100 V, K = 5
× 5 = 20 μF.
Q0 = C0V0 =20μF× 100 V =2000μC
(ii) Total charge, Q = CV, 4μF× 15 V =
60μC

78
Q 60 μ C Suppose the separation between the
V X= = =12 V plates is increased by d' to maintain the
CX 5 μ F
same p.d. after the introduction of the slab
(iii) of thickness t. Then
1 2
C ( 12 ) E0
Energy stored in X 2 4 V 0=E 0 ( d +d −t ) +
'
= = =4 :1 ⋅t
Energy stored in Y 1 2 1 κ
4 C ( 3)
2 E0
E0 ( d + d −t ) +
'
⋅ t=E 0 d
Example 84. An electric field E0 =3 × 10 4 κ

∴ κ=
Vm-1 is established between the plates, t 3.0 mm
0.05 m apart, of a parallel plate capacitor. = =5
t−d 3.0 mm−2.4 mm
'
After removing the charging battery, an
uncharged metal plate of thickness f =0.01 Example 86. The area of parallel plates of
w is inserted between the capacitor plates. an air-filled capacitor is 0.20 m2 and the
Find the p.d. across the capacitor (i) distance between them is 0.01m The p.d.
before, (ii) after the introduction of the across the plates is 3000 V. When a 0.01
plate, m thick dielectric sheet is placed between
(iii) What would be the p.d. if a dielectric the plates, the p.d. decreases to 1000 V.
slab (K = 2) were introduced in place of Determine (i) capacitance of the capacitor
metal plate? before placing the sheet (ii) charge on
[Roorkee 91] each plate (iii) dielectric constant of the
material (iv) capacitance of the capacitor
Solution. (z) The p.d. across the after placing the dielectric (v) permittivity
capacitor plates before metal plate is of the dielectric. Given 8Q = 8.85 × 10-12
inserted, Fm-1.
V0=E0d= 3 × 104 × 0.05 = 1500 V. Solution. (i) Capacitance of air-filled
(ii) As no electric field exists in metal capacitor is
plate, so the p.d. after the introduction of ε 0 A 8.85 ×10−12 ×0.20 −10
metal plate is C 0= = =1.77 × 10 F
d 0.01
V=E0 (d-t) = 3 × 104 × (0.05 -0.01) = 1200
(ii) Charge on each plate,
V.
−10 −7
(iii) When dielectric slab (K = 2) is q=C 0 V 0=1.77 ×10 ×3000=5.31× 10 C
introduced, the p.d. becomes (iii) Dielectric constant of the material
E0 4
3× 10 ×0.01 is
V =E 0 ( d−t )+ t=1200+ =1350 V
κ 2 C q/V V 0 3000
κ= = = = =3
Example 85. A parallel plate capacitor is C 0 q /V 0 V 1000
charged to a certain potential difference.
(iv) Capacitance after the dielectric sheet
When a 3.0 mm thick slab is slipped
is introduced,
between the capacitor plates, then to
−10 −10
maintain the same p.d. between the C=κ C 0=3× 1.77 ×10 =5.31×10 F
plates, the plate separation is to be
increased by 2 A mm Find the dielectric (v) Permittivity of the dielectric is
constant of the slab. ε =κ ε 0=3 × 8.85× 10
−12
=2.65 ×10
−11
Fm
−1

Solution. Let E0 be the electric field


Example 87. The capacitance of a parallel
between the capacitor plates before the
plate capacitor is 50 pF and the distance
introduction of the slab. Then, the p.d.
between the plates is 4 mm It is charged
between the plates is
to 200 V and then the charging battery is
Vo = E0d removed. Now a dielectric slab (K = 4) of
thickness 2 mm is placed.

79
Determine (i) final charge on each plate Solution. Electric field between
(ii) final potential difference between the capacitor plates is given by
plates (iii) final energy in the capacitor and
σ q
(iv) energy loss. E= =
κ ε0 κ ε0 A
Solution. Capacitance of air-filled
capacitor, As the electric field should not exceed
1.9 × 10-7 Vm-1, so the maximum charge
ε0 A that can be stored is
C 0= … (i)
d q = K ε0 AE
Capacitance with dielectric slab of
= 5 × 8.85 × 10-12 × 100 × 10-4 × 1.9 × 107
thickness t (< d) is
= 8.4 × 10-6 C.
ε0 A
C= … (ii) Example 89. A slab of material of
d−t +t /κ
dielectric constant K has the same area as
(i) The charge on capacitor plates, the plates of a parallel plate capacitor but
when 200 V p.d. is applied, becomes has a thickness 3d / 4, where d is the
q = C0 V0 = 50 × 10-12 × 200 = 10-8 C separation of the plates. How is the
capacitance changed when the slab is
Even after the battery is removed, the inserted between the plates? [NCERT;
charge of 10-8 C on the capacitor plates CBSE F 17]
remains the same.
Solution. If V0 is the potential
(ii) On placing the dielectric slab, difference when there is no dielectric, then
suppose the capacitance becomes C and the electric field between the capacitor
potential difference V. Then plates will be
q = C0V0 = CV V0
E0 =
C0 d−t+t /κ d
orV = V = V0
C 0 d After the dielectric is inserted, the
[Using (1) and (2)] electric field in the dielectric reduces to

4−2+2/ 4 E0
¿ × 200=125 V E=
4 κ
(iii) Final energy in the capacitor is Now the potential difference between the
plates will be
1 1 −8 −7
U = qV = ×10 ×125=6.25× 10 J d 3d d E0 3 d
2 2 V =E 0 ⋅ + E ⋅ =E 0 ⋅ + ⋅
4 4 4 κ 4
(iv) Energy loss
1
¿ U 0 −U= q (V 0−V )
2
¿ E0 d ( 14 + 43κ )=V 0
κ+3

1 −8 −7
Thus the potential difference
¿ ×10 ×(200−125)=3.75 ×10 J decreases by a factor of (K + 3)/4K, while
2
the free charge q0 on the plates remains
Example 88. A parallel plate capacitor is same. The capacitance increases to a
formed by two plates, each of area 100 new value given by
cm2, separated by a distance of 1 mm A
q0 4 k q0 4 k
dielectric of dielectric constant 5 and C= = ⋅ = C
dielectric strength 1.9 × 107 Vm-1 is filled V κ +3 V 0 κ+3 0
between the plates. Find the maximum
Example 90
charge that can be stored on the capacitor
without causing any dielectric breakdown. (a) Find the ratio of the capacitances of a
capacitor filled with two dielectrics of

80
same dimensions but of dielectric Ratio of the capacitances in the two
constants K1 and K2, respectively. arrangements is
(b) A capacitor is filled with two dielectrics 2
C ε 0 A (κ 1 +κ 2) d (κ1 +κ 2 ) (κ 1+ κ2 )
of the same dimensions but of '
= ⋅ = .
C 2d 2 ε0 A κ 1 κ 2 4 κ 1 κ2
(b) Here κ1 = 2,κ2 = 3
dielectric constants K1 =2 and K2 =3.
Find the ratio of capacities in two
possible arrangements. [MNREC 85]
∴ = .
2
C (2+3) 25
'
=
C 4 × 2× 3 24
Problems For Practice
1. A parallel-plate capacitor having plate
area 100 cm2 and separation 1.0 mm
(a) holds a charge of 0.12 μC when
connected to a 120 V battery. Find the
dielectric constant of the material filling
the gap. (Ans. 11.3)
2. Find the length of the paper used in a
capacitor of capacitance 2μF, if the
dielectric constant of the paper is 2.5
(b) and its width and thickness are 50 mm
and 0.05 mm, respectively. (Ans.
Fig. 2.125 90 m)
Solution. (a) The two possible 3. A parallel-plate capacitor consists of
arrangements of the two dielectrics are 26 metal strips, each of 3 cm × 4cm,
shown in Figs. 2.125(a) and (b). separated by mica sheets of dielectric
(i) The arrangement (α) can be constant 6 and uniform thickness 0.2
supposed to be a parallel combination of mm. Find the capacitance. (Ans.
two capacitors, each with plate area A/2 7.97 × 10-9 F)
and separation d. Therefore, the total 4. A parallel-plate capacitor of capacity
capacitance is 0.5 μF is to be constructed using
ε 0 ( A /2)κ 1 ε 0( A /2)κ 2 paper sheets of thickness 0.04 mm as
C=C 1+C 2= + dielectric. Find how many circular
d d
metal foils of diameter 0.1 m will have
ε 0 A (κ 1 +κ 2) to be used. Take the dielectric
¿ constant of paper used as 4. (Ans. 73)
2d
(ii) The arrangement (b) can be 5. When a slab of insulating material 4
supposed to be a series combination of mm thick is introduced between the
two capacitors, each with plate area A and plates of a parallel plate capacitor, it is
separation d/2. Therefore, the total found that the distance between the
capacitance C' is given by plates has to be increased by 3.2 mm
to restore the capacitance to the
1 1 1 1 1 original value. Calculate the dielectric
= '+ '= +
C C 1 C 2 ε0 A κ 1 ε0 A κ 2 constant of the material. (Ans. 5)
'

d /2 d /2 6. The two plates of a parallel plate


capacitor are 4 mm apart. A slab of
=
d
(
1 1
+
2 ε0 A κ 1 κ 2 ) dielectric constant 3 and thickness 3
mm is introduced between the plates
with its faces parallel to them. The
or C’ =
d (
2 ε0 A κ 1 κ 2
κ 1 +κ 2 ) distance between the plates is so
adjusted that the capacitance of the
capacitor becomes 2/3rd of its original

81
value. What is the new distance equal thickness, as shown in Fig.

three slabs are to κ1, κ2 and κ3, find


between the plates ? [CBSE OD 2.127. If the dielectric constants of the
08C] (Ans. 8 mm)
7. The distance between the parallel the new capacitance.

(κ1 + κ2 + κ3) ]
plates of a charged capacitor is 5 cm C
and the intensity of electric field is 300 [Ans. C’ =
3
V cm-1. A slab of dielectric constant 5

κ has the same area as the plates of a


and thickness 1 cm is inserted parallel 11. A slab of material of dielectric constant
to the plates. Determine the potential
difference between the plates, before parallel plate capacitor but has
and after the slab is inserted ? (Ans. thickness d/2, where d is the
1500 V, 1260 V) separation between the plates. Find
the expression for the capacitance
8. A parallel plate capacitor with plate when the slab is inserted between the
separation 5 mm is charged by a plates. [CBSE F 10 ; OD 13]
battery. It is found that on introducing
a mica sheet 2 mm thick, while
keeping the battery connections intact, ( Ans. κ2+1κ C )
0

the capacitor draws 25% more energy


from the battery than before. Find the HINTS
dielectric constant of mica. (Ans. q 0.12 μC
2) 1. Capacitance, C = = = 10-9 F
V 120 V
If κ is the dielectric constant, then
9. Figure 2.126 shows a parallel plate
capacitor of plate area A and plate
separation d. Its entire space is filled κ ε 0 A κ ×8.85 ×10−12 ×100 ×10−4
with three different dielectric slabs of C= = −3 =
d 1.0 × 10
same thickness. Find the equivalent
10-9 F
∴ κ = 11.3.
capacitance of the arrangement.

[ Ans. C=
3 ε 0 A κ1 κ 2 κ 3
d ( κ 1 κ 2+ κ 2 κ 3+ κ 3 κ 1 ) ] 3. Arrangement of n metal plates
separated by dielectric acts as a parallel
combination of (n - 1) capacitors.

∴ C=
(n−1)κ ε 0 A
d
−12 −4
25× 6 ×8.85 × 10 ×3 × 4 ×1 0
¿ −3
0.2 ×1 0
−9
¿ 7.97 ×1 0 F .
Fig. 2.126 (n−1)κ ε 0 A
4. As C=
d
∴ 0.5 ×1 0−6 F
−12
¿(n−1)× 4 × 8.85 ×1 0 ×3.14 × ¿¿
3
0.⋅5 × 0.04 ×1 0
or n−1=
4 × 885 ×3.14 × ¿ ¿
or n=73.
Fig. 2.127
10. The space between the plates of a ε0 A
5. Capacitance without dielectric, C=
parallel plate capacitor of capacitance d
C is filled with three dielectric slabs of

82
When dielectric is introduced 8. As the battery connections are intact (V
ε0 A = constant) and the capacitor draws 25%
C= more charge, so the capacitance also
d ’−t 1− ( κ1 ) As the capacitance
increases by 25%. That is
125 5
remains same in both cases, so C= C 0= C 0
100 4
ε0 A ε0 A
= ε0 A
5 ε0 A
( )
d '
d −t 1−
1
κ or
( )
d−i 1−
1
= .
4 d
κ
or d=d −t ( 1− ) or d −d=t ( 1− )
' 1 1 '

or d−t ( 1− )= or t( 1− ) =
κ κ 1 4d 1 d
κ 5 κ 5
But d' – d = 3.2mm, t = 4mm
1 d 5 1
∴ 3.2=4 1− ( )
1 or 1− = = = or κ=2
κ 5t 5×2 2
κ
9. The given arrangement is equivalent to
=0.8 or =0.2 or κ¿ 5.
1 3.2 1 three capacitors connected in series. Each
or 1− =
κ 4 κ such capacitor has plate area A and plate
separation d.
2
∴ C 1=
6. × Capacitance with air ¿ Capacitance κ1 ε 0 A 3 κ1 ε 0 A
3
=
with dielectnc d /3 d
2 ε0 A ε0 A 3 κ 2 ε0 A 3 κ 3 ε0 A
= C 2= and Q=
or 3 d ' t d d
d −t+
κ The equivalent capacity C of the given

( )
2 ' 3 arrangement is given by
or d −3+ =d=4 mm or d ' =¿ 8 mm
3 3
7. P.D. before the dielectric slab is
inserted,
1 1 1 1
= + + =
d 1 1 1
+ +
C C1 C 2 C 3 3 ε 0 A κ 1 κ 2 κ3 ( )
−1 3 ε 0 A κ 1 κ 2 κ3
V 0=E 0 d=300 Vc m ×5 cm ¿ 1500 V. or C= .
d (κ 1 κ 2 +κ 2 κ 3 +κ 3 κ1 )
ε0 A ε0 A
C 0= = farad ε0 A
d 0 . 05 10. Original capacitance, C=
d
or ε 0 A=0.05 C 0
The new arrangement is equivalent to
Capacitance with dielectric slab, three capacitors connected in parallel.
ε0 A 0.05 C 0 Each such capacitor has plate area A/3
C= = and plate separation d. The new
t 0.0 1 capacitance is
d−t + 0.05−0.01+
κ 5 '
C =C1 +C 2+ C3
25C 0
=
¿ κ 1 ε 0 A /3 κ 2 ε 0 A /3 κ 3 ε 0 A /3
21 + +
d d d
For charge to remain constant,
ε0 A
C 0 V 0=CV ¿ (κ + κ +κ ).
3d 1 2 3
25 C0
C 0 × 1500= × V or V =1260 V. ' C
21 or C = (κ + κ +κ ) .
3 1 2 3

83
V0 charge at the pointed end of a conductor
11. Without dielectric, E0 = gets discharged is called corona
d
discharge. The discharge is often
E0 accompanied by a visible glow near the
With dielectric, E=
κ pointed end.

∴ V =E 0 . + E . =E0 .
2.34 COLLECTING ACTION OF A
d d d E0 d
+ ⋅ HOLLOW CONDUCTOR
2 2 2 κ 2
55. A small sphere of radius rand
E0 d (κ+ 1) V 0 (κ +1) charge q is enclosed by a spherical shell
¿ . =
2 κ 2κ of radius R and charge Q. Show that if q is
positive, charge q will necessarily flow
q0 2 κ q0 2κ
or C= = = C from the sphere to the shell (when the two
V V 0 (κ +1) κ +1 0 are connected by a wire) no matter what
the charge Q on the shell is. [NCERT]
2.33 DISCHARGING ACTION OF SHARP
POINTS : CORONA DISCHARGE Collecting action of a hollow sphere.
Consider a small sphere of radius r placed
54. Briefly explain discharging action
inside a large spherical shell of radius R.
of sharp points or corona discharge.
Let the spheres carry charges q and Q,
Discharging action of sharp points : respectively.
Corona discharge. When a spherical
Total potential on the outer sphere,
conductor of radius r carries a charge q,
its surface charge density is VR = Potential due to its own charge Q +
Potential due to the charge q on the inner
q q
σ= = sphere
A 4 π r2
Electric field on the surface is
σ q
E= =
ε0 4 π ε0 r2

Fig. 2.129 Small charged sphere


suspended inside a charged spherical
shell.
Fig. 2.128 Corona discharge.
The pointed end of a conductor is
highly curved and its radius of curvature r
= [
1 Q q
+
4 π ε0 R R ]
very small. If the conductor is given a Potential on the inner sphere due to its
charge q, then the charge density σ at the own charge is
pointed end will be very high.
Consequently, the electric field near the 1 q
V 1= ⋅
pointed end will be very high which may 4 π ε0 r
cause the ionisation or electrical
breakdown of the surrounding air. The As the potential at every point inside a
oppositely charged ions neutralise the charged sphere is the same as that on its
pointed end while the similarly charged surface, so potential on the inner sphere
ions are repelled away. Fresh air due to charge Q on outer sphere is
molecules come near the pointed end and 1 Q
take away its charge, setting up a kind of V 2= ⋅
4 π ε0 R
electric wind. This process by which the

84
∴ Total potential on inner sphere sharply pointed brass combs B1 and B2,
pointing towards the belt. The comb B 1,
V r=
[
1 q Q
+
4 π ε0 r R ] called spray comb is given a positive
potential of 10 kV with respect to the earth
by means of a battery ; while the comb B2,
Hence the potential difference is
called collecting comb, is connected to the
V r −V R =
[
q 1 1

4 π ε0 r R ] spherical shell S.
Working. Due to the high electric field
at the pointed ends of comb B1, the air of
So if q is positive, the potential of the
the neighbourhood gets ionised and its
inner sphere will always be higher than
positive charge repelled or sprayed on to
that of the outer sphere. Now if the two
the belt, which moves up into the shell S.
spheres are connected by a conducting
As it passes close to comb B2, it induces a
wire, the charge q will flow entirely to the
negative charge at the pointed ends of
outer sphere, irrespective of the charge Q
comb B2 and a positive charge on the
already present on the outer sphere. In
shell S. The positive charge spreads
fact this is true for conductors of any
uniformly on the outer surface of the shell
shape.
S. The high electric field at the pointed
2.35 VAN DE GRAAFF GENERATOR* ends of comb B2 ionises the air there and
56. Explain the basic principle, repels the negative charges on to the belt
construction and working of Van de Graaff which neutralise its positive charge. This
generator. process continues. As more and more
positive charge is given to the shell, its
Van de Graaff generator. It is an potential continues to rise. In this way, a
electrostatic generator capable of building high potential of 6 to 8 million volts can be
up high potential differences of the order built upon the sphere.
of 107 volts.
A discharge tube is placed with its
Principle. The working of a Van de upper end inside the hollow sphere and
Graaff generator is based on following two lower end earthed. The ion source is
electrostatic phenomena: placed at the upper end of the tube. The
(i) Discharging action of sharp points high potential on the sphere repels the
(corona discharge) i.e., electric charged particles downward with large
discharge takes place in air or gases acceleration, where they hit the target
readily at the pointed ends of atoms to bring about the nuclear
conductors. disintegration.

(ii) If a charged conductor is brought into


internal contact with a hollow
conductor, all of its charge transfers to
the hollow conductor, howsoever high
the potential of the latter may be.
Construction. A large spherical
conducting shell (of few metres radius) is
supported at a height several metres
above the ground on an insulating column.
A long narrow belt of insulating material,
like rubber or silk, is wound around two
pulleys, P1 at ground level and P2 at the
centre of the shell.
This belt is kept continuously moving
by an electric motor attached to the lower
pulley P1. Near the bottom and the top of
its run, the belt passes close to two

85
Fig. 2.130
Use. The high potential difference set
up in a Van de Graaff generator is used to
accelerate charged particles like protons,
deutrons, α-particles, etc. to high energies
of about 10 MeV, needed for experiments
to probe the small scale structure of
matter.

VERY SHORT ANSWER CONCEPTUAL PROBLEMS

Problem 1. Is the electrostatic [CBSE D 92C]


potential necessarily zero at a point
Solution. Yes. For example, the
where the electric field strength is
potential at any point on the perpendicular
zero ? Give an example to illustrate
bisector of a dipole axis is zero, while
your answer.
electric field is not zero.
[CBSE Sample Paper 90]
Problem 3. The electric potential is
Solution. No. We know that the electric constant in a given region. What can
field is equal to the negative of potential you say about the electric field there ?
gradient: [Haryana 01]
dV dV
E=- Solution. We know that E = -
dr dr
This implies that even if the electric As V is constant, so electric field E is zero.
field at a point is zero, the potential may
Problem 4. In a certain 0.1 m3 of space,
have some non-zero constant value at
electric potential is found to be 5 V
that point.
throughout. What is the electric field in
Examples, (i) Electric field inside a this region ? [ICSE 97]
charged conducting sphere is zero but
dV
potential at any inside point is the same as Solution. We know that E = -
that on the surface of the sphere. dr
(ii) Electric field at the midpoint of the As electric potential is 5 V throughout
line joining two equal and similar charges i.e., constant, so the electric field is zero in
is zero, but potential at this point is twice the given region.
of that due to a single charge. Problem 5. When kept in an electric
Problem 2. Can electrostatic field, does a proton move from lower to
potential at a point be zero, while higher or from higher to lower potential
electric field at that point is not zero ? region ?

86
Solution. Proton is a positively charged
particle. In an electric field, it will move
from higher to lower potential region so as
to reduce its potential energy.
Problem 6. Can there be a potential
difference between two neighbouring
conductors carrying equal positive
charges ? Fig. 2.132
Solution. Yes. The potential of a Solution. The points A and C are at the
conductor depends not only on the net same potential.

charge carried by it, but also on its
geometrical shape and size. So two V B - VA = V B - VC
conductors of different size and shape will Hence, the magnitude of the work
have different potentials even if they carry done in taking charge + q from A to B or
equal charges. from B to C will be same i.e., W1 = W2.
Problem 7. A positive charge + q is Problem 10. A uniform electric field
located at a point. What is the work E exists between two charged plates as
done if a unit positive charge is carried shown in the figure. What would be the
once round this charge along a circle work done in moving a charge 'q' along
of radius r about this point. the closed rectangular path ABCDA ?
Solution. Work done is zero. Force on [CBSE D 01C]
the unit positive charge is along the radius
and direction of motion is perpendicular to
it. W = FS cos 90° = 0.
Problem 8. What would be the work
done if a point charge +q, is taken from
a point A to the point B on the
circumference of a circle with another
point charge +q at the centre ? [CBSE D
Fig. 2.133
01C]
Solution. Zero. Electric field is a
conservative field, so no work is done in
moving a charge q along a closed path in
a uniform electric field.
Problem 11. What is the work done
in moving a 2 microcoulomb point
charge from corner A to comer B of a
Fig. 2.131 square ABCD shown in Fig. 2.134,
when a 10 μC charge exists at the
Solution. The points A and B are at
centre of the square ?
same distance from the charge + q at the
centre, so VA = VB. Hence the work done
in taking another charge from point A to B
will be zero.
Problem 9. In Fig. 2.132, charge +Q is
placed at the centre of a dashed circle.
Work done in taking another charge +q
from A to B is W2 and from B to C is W2.
Which one of the following is correct : Fig. 2.134
W1 > W2, W1 = W2 and W1 < W2 ? [CBSE
SP 18] Solution. As the points A and B are at
the same distance from the charge of 10

87
μC at the centre, so VA = VB. Hence the potential in terms of the base units of
work done in moving a charge of 2 μC SI. [ISCE 02]
from A to B will be zero.
Solution. Electric potential
Problem 12. Two protons A and B
are placed between two parallel plates
Work done ML2 T−2 2 −3 −1
= = =ML T A
having a potential difference V, as Charge AT
shown in Fig. 2.135. Will these protons SI unit of electric potential = kg m2s-3A-1
experience equal or unequal force ?
Problem 16. A metallic sphere is
[CBSE D 98] placed in a uniform electric field. Which
path is followed by lines of force
shown in Fig. 2.137. [IIT 96 ; CBSE F
2010]

Fig. 2.135
Solution. The electric field is uniform in Fig. 2.137
the space between the two plates. Hence
the protons A and B will experience equal Solution. Path 4. This is because lines
force. of force start and end normally at the
surface of a conductor and do not exist
Problem 13. A point charge q is inside it.
placed at O, as shown in Fig. 2.136. Is
VA - VB positive, negative or zero, if q is Problem 17. The work done in
a (i) positive, (ii) negative charge ? carrying a point charge from one point
to another in an electric field does not
[CBSE D 01C, 16 ; OD 06] depend on the path along which it is
taken. Is it true or false ? Give reason.
[IIT 81]
Solution. True. Electric field is a
Fig. 2.136 conservative field, so the work done in
Solution. Clearly, carrying a point charge from one point to
another does not depend on the path
VA – VB = along which it is taken.
1

q

1 q
=
q 1

[
4 π ε 0 OA 4 π ε 0 OB 4 π ε 0 OA OB
1
] Problem 18. No work done is done
in taking a positive charge from one
As OA < OB, so the quantity within point to another inside a positively
brackets is positive. charged metallic shell, while outside
the shell work has to be done in taking
(i) If q is a positive charge, then VA - VB is the charge from one point to the other
positive. towards the shell. Why ?
(ii) If q is a negative charge, then VA - VB is Solution. All points inside the metallic
negative. shell are at the same potential. So no
Problem 14. Does the electric potential work is done in moving any charge inside
increase or decrease along the electric the charged shell. While outside the shell,
line of force ? there is a potential gradient and electric
field. Work has to be done in moving the
Solution. Electric potential decreases
charge against this field.
along the electric line of force.
Problem 19. Can two equipotential
Problem 15 Express the unit of electric
surfaces intersect each other ? Give

88
reason. [CBSE D 09] Problem 24. No work is done in
moving a test charge over an
Solution. No. If two equipotential
equipotential surface. Why ? [Punjab
surfaces intersect, then there would be
01]
two values of electric potential at the point
of intersection, which is not possible. Solution. The potential difference

equipotential surface, ∆V = 0.
between any two points on an
Problem 20. "For any charge

∴ Work done, W = q0 ∆V = 0.
configuration, equipotential surface
through a point is normal to the electric
field". Justify. [CBSE D14 ; OD
16] Problem 25. Does the charge given
to a metallic sphere depend on whether
Solution. If the electric field were not it is hollow or solid ? Give reason for
normal to the equipotential surface, it your answer. [CBSE D 17]
would have a non-zero component along
the surface. To move a test charge Solution. No, because the charge
against this component, work would have resides only on the surface of the
to be done. But no work is needed to conductor. Both hollow and solid spheres
move a test charge on an equipotential will have same capacitance (C = 4πε0R) if
surface. Hence, electric field must be they have the same radii. When charged
normal to the equipotential surface at to same potential, they will have the same
every point. charge.

Problem 21. Why must electrostatic Problem 26. Can a metal sphere of
field be normal to the surface at every radius 1 cm hold a charge of 1 coulomb
point of a charged conductor ? [CBSE ? Justify your answer. [Punjab 95C]
D 12] Solution. Here R = lcm = 10 2 m, q = 1C
Solution. In electrostatic situation, ⃗
E Electric field on the surface of the sphere,
has to ensure that the free charges on the 1 q 1
9
surface do not experience any force or E = ⋅ 2 =9 × 10 × −2 2 = 9 ×
4 π ε0 R (10 )
there are no surface currents. So ⃗ E must
not have any tangential component. 1013 Vm-1

Problem 22 Why should electrostatic The field is much greater than the
field be zero inside a conductor ? dielectric strength of air (3 × 106 Vm-1). It
[CBSE D 12] causes ionisation of the surrounding air
and the charge of the metal sphere leaks
Solution. If any electric field exists into the surrounding air. Thus a metal
inside a conductor, it would produce sphere of 1cm radius cannot hold a
ordered motion of electrons without the charge of 1C.
expense of any external energy. This
violates the law of conservation of energy. Problem 27. What is the justification
of choosing the earth as the zero of
Problem 23 Why is electrostatic potential in practice ?
potential constant throughout the
volume of the conductor and has the Solution. If we consider earth to be a
same value (as inside) on its surface ? conducting sphere surrounded by air, its
[CBSE D 12] capacitance will be equal to 4π ε0 R. Since
the radius R of the earth is very large, so
Solution. Electric field is zero its capacitance is also very large. A
throughout the volume of the conductor. charge q supplied to the earth will

= 0 ⇒ V = constant.
dV increase its potential by amount,
So E = -
dr q
V=
Moreover, the potential just inside the C
conductor has to be equal to that on its
surface.

89
q the ratio of the coulomb to the volt, is
As C is very large, therefore V = →0 doubly large. In other words, the
C
for all finite charges. largeness of the farad goes as the square
of the largeness of the coulomb.
Problem 28. Can we take the
potential of the earth as + 100 V? What Problem 32. A metal plate is
effect would such an assumption have introduced between the plates of a
on the measured values of (a) charged parallel plate capacitor. Sketch
potentials at various positions, and (b) the electric lines of force between the
potential difference between two given plates.
points ?
Solution. All potentials are measured
relative to a reference position, to which
we assign a zero value of potential. Earth
is assigned zero potential value as it is
generally taken as a reference.
However, earth can be assigned a
value of + 100 V. This only increases the
potential values by 100 V. But the Fig. 2.138
potential difference between two points Solution. Inside the metal plate,
would still remain the same. electric field is zero, so no lines of force
Problem 29. Can we give any exist inside the metal plate, as shown in
desired charge to a capacitor ? Fig. 2.138.
Solution. No, as we increase the Problem 33. What is the dielectric
charge on the plates of a capacitor, the constant of a metal ?
potential difference between them also Solution. The dielectric constant of a
increases. A stage is reached when the metal is infinity. The electric field inside a
electric field between the two plates conductor is zero so the dielectric
attains the breakdown value of air. The constant, which is the ratio of applied
surrounding air gets ionised and the electric field to the reduced electric field,
charge begins to leak into air. will be infinite for the metallic conductor.
Problem 30. What happens if the plates Problem 34. Is there any kind of
of a charged capacitor are suddenly material which when placed between
connected by a conducting wire ? the plates of a capacitor reduces its
Solution. The capacitor plates will get capacitance ?
discharged immediately. The energy Solution. No. The dielectric constant of
stored in the capacitor changes into heat a material is always greater than 1.
energy.
and κ > 1 ∴ Cd > Cv
Cd
Problem 31. Why is the farad an As K =
inconveniently large unit of Cv
capacitance ? i.e., the capacitance with dielectric
Solution. We know that between the plates is greater than that
with vacuum between the plates. So there
1F=
1C 1C 2
1V
=
1J
∵1 V=
[1J
1C ] is no such material which when placed
between the plates of a capacitor will
reduce its capacitance.
By daily standards, the SI unit of
charge, the coulomb, is very large and the Problem 35. For a given medium,
SI unit of energy, the joule, is a the dielectric constant is unity. What is
reasonable unit of energy. As a result, the its permittivity ?
SI unit of potential difference, the volt (1V Solution. Permittivity, ε = ε0κ = 8.85 ×
= 1 JC-1) is a very small unit. So the farad,

90
10-12 × 1 charge -Q2 to the negative plate of a
capacitor. What is the charge on the
= 8.85 × 10-12 C2N-1m-2.
capacitor ?
Problem 36. Why do ordinary
Solution. The charge on a capacitor is
capacitors have capacities of the order
equal to the charge on its positive plate.
of microfarads ?
So the charge oil the given capacitor is +
Solution. Farad is a very large unit of Q1.
capacitance. So capacitors having smaller
Problem 41. Is the capacitance C of
capacitances of the order of lμF (= 10-6F)
a capacitor proportional to the charge
are more common.
Q?
Problem 37. Why does the electric
Solution. No. As the charge Q
field inside a dielectric decrease when
increases, the potential difference V also
it is placed in an external electric
increases in the same proportion, so the
field ? [CBSE D 05]
capacitance C = Q/V, remains unaffected
Or i.e., C is independent of Q.
How is the electric field due to a Problem 42. Sketch a graph to show
charged parallel plate capacitor how the charge Q acquired by a
affected when a dielectric slab is capacitor of capacitance C varies with
inserted between the plates fully increase in potential difference
occupying the intervening region ? between its plates.
Solution. As Q ∝ V, the graph between
[CBSE F 10]
Solution. Due to polarisation of the Q and V is a straight line with slope Q/V =
dielectric, an electric field is induced inside C, as shown in Fig. 2.139.
the dielectric in the opposite direction of
external electric field. Thus the net electric
field decreases inside the dielectric.
Problem 38. The introduction of
dielectric slab between the capacitor
plates increases the capacitance.
Why ?

constant κ reduces the electric field from


Solution. A dielectric slab of dielectric Fig. 2.139

E to E / κ. This reduces the potential Problem 43. Sketch a graph to show


difference from V to V / κ. Hence the how the capacitance C of a capacitor
capacitance increases from C to κC. varies with the charge Q given to it.

Problem 39. A thin metal sheet is Solution. As capacitance C is


placed in the middle of a parallel plate independent of charge Q so graph
capacitor. What will be the effect on the between C and Q is a straight line parallel
capacitance ? to the charge axis, as shown in Fig. 2.140.

Solution. No effect. When the metal


sheet is placed in the middle, the new
arrangement is equivalent to a series
combination of two capacitors, each of
plate separation d / 2 and hence
capacitance 2 C.

∴ Cs =
2C × 2 C
= C. Fig. 2.140
2 C+2 C
Problem 44. Two plates are placed
Problem 40. Suppose a charge +Q1
side by side. How many capacitors are
is given to the positive plate and a
formed ?

91
Solution. Three, first between distant will carry greater charge ?
earthed bodies and the first face of the
Solution. The sphere of 13 cm radius
first plate, the second between the two
has greater capacitance than the sphere
plates and the third between the second
of 7 cm radius (C = 4π ε0 R). So the
face of the second plate and the distant
sphere of 13 cm radius will carry more
earthed objects. The capacitances of the
charge, Q = CV. Both spheres attain same
first and third capacitors are negligibly
potential V when connected by a
small than capacitance of the second
conductor.
capacitor.
Q1 Q2
Problem 45. Draw the lines of force In fact, V = = .
between the plates of a charged C1 C 2
parallel plate capacitor.
Problem 48. A spherical shell with
[CBSE Sample Paper 03] radius a and charge Q is expanded to
radius b. What is the work done by the
Solution. The electric lines of force
electrostatic force in this process ?
between the plates of a charged parallel
plate capacitor are shown in Fig. 2.141. Solution. Work done by the
electrostatic force
= Initial stored energy - Final stored
energy
2 2 2 2
1 Q 1Q 1 Q 1 Q
= ⋅ − = ⋅ − ⋅
2 C1 2 C 2 2 4 π ε 0 a 2 4 π ε 0 b

[ ]
2
Q 1 1
= − .
8 π ε0 a b
Problem 49. By what factor does the
capacitance of a metal sphere increase
if its volume is tripled ?
Fig. 2.141
Solution. If V1 and V2 are the initial and
Problem 46. The space between the final volumes, then
plates of a parallel plate capacitor is

( )
1 /3
filled consecutively with two dielectric C2 R 2 V 2
= = =( 3 )1 /3 =1.44
layers 1 and 2 having the thicknesses C1 R 1 V 1
d1 and d2 and the relative permittivities
ε1 and ε2 respectively. The area of each or C2 = 1.44 C1
plate is equal to A. What is the Thus the capacitance increases 1.44
capacitance of the capacitor ? times.
Solution. The effective separation Problem 50. How would you
between the plates is 1/ε times the connect two capacitors across a
geometrical separation. So the battery, in series or parallel, so that
capacitance of the given parallel plate they store greater (i) total charge and
capacitor is (if) total energy ?
ε0 A Solution. Total charge, q = CV
C = d1 d2 . 1
+ Total energy, U = CV 2
ε 1 ε2 2
Problem 47 Two insulated charged As V is constant and Cp > Cs, so the
spheres of radii 7 cm and 13 cm and capacitors must be connected in parallel
having the same charge are connected for storing greater charge and greater
by a conductor and then they are energy.
separated. Which of the two spheres

92
Problem 51. Why should a circuit during the charging and discharging of the
containing capacitors be handled capacitor. This energy appears as heat.
cautiously, even when there is no
Problem 56. n identical capacitors
current ?
are joined in series and the
Solution. Even if there is no current in combination is given a potential
the circuit, a capacitor may have charge. difference V. If these capacitors be
When such a circuit is touched, a disconnected and joined in parallel,
discharge current is produced in the body what potential difference will be
and so the man touching it may receive a obtained across the combination ?
severe shock. This can be avoided by
Solution. In series combination, each
wearing shoes with rubber soles.
capacitor will have a potential difference V
Problem 52. When moulded plastic / n, which will remain same when
parts are removed from metal dies, capacitors are joined in parallel. Hence
they develop a high voltage. Why ? potential difference across the parallel
combination will be V/n
Solution. When the plastic part is
removed, the capacitance of the metal die Problem 57. n identical capacitors
decreases but the charge (produced by are joined in parallel and the
friction) remains unchanged and so the combination is given a potential
voltage increases in accordance with the difference V. If these capacitors be
relation : Q = CV. disconnected and joined in series,
what potential difference will be
Problem 53. What happens to the
obtained across the combination ?
stored energy if after disconnecting the
battery, the plates of the charged Solution. In parallel combination, each
capacitor are drawn apart ? capacitor will have a potential difference
Solution. As C ∝ 1 / d, so when the
V. When the capacitors are disconnected
and joined in series, the potential
plates are drawn apart, the capacitance
differences on n capacitors get added.
decreases. After disconnecting the
Hence the p.d. across the series
battery, the charge on plates remains
combination will be nV.
constant. Hence the energy stored in the
capacitor, U = q2 / 2 C increases. Problem 58. How can the whole
charge of a conductor be transferred to
Problem 54. What happens to the
another insulated conductor ?
energy stored in a capacitor, if the
plates of a charged capacitor are drawn Solution. When the charged conductor
apart, the battery remaining connected is placed in internal contact with the
? hollow insulated conductor, its whole
charge is transferred to the hollow
Solution. When the plates are drawn
conductor.
apart, the capacitance decreases. As the
battery remains connected, the potential Problem 59. Why is it that a man
difference remains constant. sitting in an insulated metal cage does
not receive any shock when it is
1 2 connected to a high voltage supply ?
Hence energy stored, U = CV
2
decreases. Solution. The charge in the cage goes
to its surface. The inside of the cage is
Problem 55. When a capacitor is equipotential. There is no potential
charged and then discharged difference between the man and the cage.
repeatedly, its dielectric gets heated. So the man does not receive any shock.
Why ?
Problem 60. A large hollow metallic
Solution. The energy consumed during sphere A is charged positively to a
the polarisation of a dielectric is not potential of 100 volt and a small sphere
completely recovered during the process B to a potential of 50 volt. Now B is
of depolarization. Some energy is lost

93
placed inside A and they are connected which is lost into the surroundings.
by a wire. In which direction will the
Problem 64. During lightning, it is
charge flow ?
safest to sit inside a car rather than
Solution. The charge will flow from B near a tree. Why ?
to A till no charge is left on B. Inside A, B
Solution. The metallic body of the car
will acquire potential of A and its own
provides an electrostatic shielding from
potential will become 150 V which is
lightning. When we stand near a tree, our
higher than that of A (100 V < VA < VB).
body provides an easy path.
Hence charge starts flowing from B to A.
Problem 65. Is there some way of
Problem 61. Why should the radius
producing high voltage on your body
of the sphere of a Van de Graaff
without getting a shock ?
generator be sufficiently large ?
Solution. Yes. If we stand on an
Solution. Potential on a spherical shell
insulating surface and touch the live wire
is given by
of a high power supply, a high potential is
1 q developed on our body, without causing
V= . any shock.
4 π ε0 R
Larger the radius R of the spherical Problem 66. Two metallic spheres
shell, the more charge it can collect of radii R and 2R are charged so that
without being raised to a high potential. both of these have same surface
charge density σ. If they are connected
Problem 62. Why is Van de Graaff to each other with a conducting wire, in
generator enclosed inside an earth which direction will the charge flow
connected steel tank filled with air and why ? [CBSE OD 16]
under pressure ?
Solution. The potential inside and on
Solution. It prevents the leakage of the surface of a charged metallic sphere
charge due to ionisation. In air under of radius r is given by
pressure, as soon as free ions are formed,
⇒V∝r
2
they recombine to form neutral air 1 q 1 4πr σ
V= = ⋅
molecules. 4 π ε0 r 4 π ε0 r
Problem 63. Why is there loss of Hence, the bigger sphere will be at a
energy when two charged conductors higher potential and the charge will flow
at different potentials are connected by from bigger sphere to smaller sphere
a conducting wire ? when the two spheres are connected by a
Solution. As the charges flow in conducting wire.
connecting wires, some potential energy
of the charges changes into heat energy

SHORT ANSWER CONCEPTUAL PROBLEMS

Problem 1. Figures 2.142(a) and


2.142(b) show the field lines of a single
positive and negative charge
respectively. [NCERT]

Fig. 2.142
(i) Give the signs of the potential
differences, VP - VQ; VB - VA.

94
(ii) Give the sign of the potential in moving a negative charge from B to A is
energy difference of a small positive.
negative charge between the points
(v) As the negative charge moves
Q and P; A and B.
from B to A, it experiences more
(iii) Give the sign of the work done by repulsion, its velocity decreases, and so
the field in moving a small positive its kinetic energy decreases.
charge from Q to P.
Problem 2. Answer carefully :
(iv) Give the sign of the work done by
(i) A comb run through one's dry
an external agency in moving a
hair attracts small bits of paper. Why ?
small negative charge from B to A.
What happens if the hair is wet or if it is
(v) Does the kinetic energy of a small a rainy day ?
negative charge increase or
(ii) Ordinary rubber is an insulator.
decrease in going from B to A ?
But the special rubber tyres of aircrafts
Solution. (i) Potentials at both the are made slightly conducting. Why is
points P and Q are positive. P is nearer to this necessary ?
the source charge than Q. Also, we know
(iii) Vehicles carrying inflammable
that the electrostatic potential at a point is
materials usually have metallic ropes
the point from the charge i.e. V ∝ 1 / r,
inversely proportional to the distance of
touching the ground during motion.
Why ?
therefore
(iv) A bird perches on a bare high
VP > VQ or VP - VQ > 0 power line and nothing happens to the
Potentials at both the points A and B bird. A man standing on the ground
are negative. Point B is farther from the touches the same line and gets a fatal
charge than the point A. So, potential at B shock. Why ? [NCERT]
is less negative than at A.

Solution. (i) Refer answer to Problem
VB > VA or VB – VA > 0 12 on page 1.62.
(ii) P.E. of two point charges, (ii) Refer answer to Problem 13 on page
1.62.
1 q1q2
U= . (iii) Refer answer to Problem 14 on page
4 π ε0 r
1.63.
The P.E. of a positive charge and a (iv) Bird's whole body is at same potential.
negative charge is negative. The potential No charge flows and no shock is
energy of a negative charge will be more produced. The man touching the ground
negative at P than at Q so maintains a potential difference between
(P.E.)Q - (P.E.)P > 0 different parts of his body. A large current
flows, which electrolysis blood and causes
The P.E. of two negative charges is death.
positive. So the P.E. of a negative charge
will be more positive at A than at B, hence Problem 3. Why electric potential in
the field of a negative charge is lower
(P.E.)A - (P.E.)B > 0 at near points and higher at distant
(iii) As VP > VQ, if a small positive points ?
charge is moved by an external agency Solution. For a negative charge the
from Q to P, the work done will be electric potential is
positive. The work done by the electric
field in moving a small positive charge 1 q
from Q to P will be negative. V=- .
4 π ε0 r
(iv) The negative charge will For near points, r is less and V is more
experience less repulsion at B than at A, negative (lower). For distant points, r is
so the work done by the external agency more and V is less negative (higher).

95
Problem 4. The electric field due to
a point charge at a distance r depends

according to inverse square law ∝ ( r1 )


2 .

State how the following quantities


depend upon r :
(i) potential due to a point charge
Fig. 2.143
(ii) potential at a distance r from the
centre of a charged metallic sphere of Solution. No. This is not possible
radius R(r < R). because the work done in carrying a test
charge along a closed path ABCDA, as
Solution. (i) Potential due to a point shown in Fig. 2.143(b), will not be zero.
charge, More work is done along CD, less along
1 q AB, zero along BC and DA. But in an
V= ⋅ i.e., V ∝ 1 . electric field, work done is essentially zero
4π r0 r r as it is a conservative field.
(ii) In case of a charged metallic Problem 7. Figure 2.144 shows lines
sphere, of constant potential in a region in
q which electric field exists. The values
V inside =V surface = =¿ of the potential are indicated. Out of
4 π ε0 R
∴ Potential V does not depend on r.
the points A, B and C, which will be of
greatest electric field strength ? Give
reason. [IIT 84]
Problem 5. Justify that the
electrostatic potential is constant
throughout the volume of a charged
conductor and has the same value on
its surface as inside it.
[CBSE Sample Paper 2011]
Solution. Since electric field inside the
conductor is zero and has no tangential
component on the surface, no work is Fig. 2.144
done in moving a small test charge within
the conductor or on its surface. Solution. Electric field is the rate of fall of
potential i.e.,
This means that, there is no potential
difference between any two points inside dV
E=-
or on the surface of the conductor. Hence dr
For constant dV, E ∝ 1/dr. The
the potential is constant throughout the
volume of the conductor and has the
same value on its surface. stronger the field, the closer the
equipotential surfaces. As the
Problem 6. Is it possible to create equipotential surfaces are closed in the
an electric field in which all the lines of neighbourhood of B, so the field is
force are parallel lines and whose greatest at B.
density increases gradually in a
direction perpendicular to the lines of Problem 8. The equipotential
force, as shown in Fig. 2.143(a) ? surfaces of certain field are shown in
Fig. 2.145. It is given that V1 > V2. Draw
the corresponding lines of force for
this pattern. Also state the region in
which the electric field intensity is
highest.

96
potential more and why ? [CBSE OD
12]

Fig. 2.145

Fig. 2.148
Solution. (i) P.D. does not depend on
the path along which the test charge is
moved. Therefore,
dV V C −V A V C −V A
E=- =- =
dx 2−6 4

Fig. 2.146 or VC - VA = 4E ⇒ VC > VA

Solution. As shown in Fig. 2.146, the (ii) The direction of the electric field is
lines of force are perpendicular to the in the direction of decreasing potential, so
equipotential surfaces and directed from VC > VA.
higher potential to lower potential. The Problem 11. Two uniformly large
electric field intensity is highest in the parallel thin plates having charge
lower left region where the equipotential densities +σ and -σ are kept in the X-Z
surfaces are closest to each other. plane at a distance 'd' apart. Sketch an
Problem 9. Suggest a configuration equipotential surface due to electric
of three point charges separated by field between the plates. If a particle of
finite distances that has zero electric mass m and charge ‘-q’ remains
potential energy. stationary between plates, what is the
magnitude and direction of this field ?
Solution. The configuration of three [CBSE D 11]
charges - q, + q and + q shown in Fig.
2.147 has zero potential energy. Solution. Two parallel plates having
charge densities + σ and -σ kept in the X-
q ⋅q q (−q) (−q )q Z plane are shown in Fig. 2.149. Also, a
U=k +k +k = 0.
plane equipotential surface is shown. The
r 2r 2r
field ⃗E acts in the -Y direction. The
upward electric force on charge -q
balances its weight mg. For the stationary
particle,
mg
qE = mg or E =
q

Fig. 2.147
Problem 10. A test charge ‘q’ is moved
without acceleration from A to C along
the path from A to B and then from B to
C in electric field E as shown in Fig.
2.148. (i) Calculate the potential
difference between A and C. (ii) At
which point (of the two) is the electric

97
connected to the upper plate with a
conducting wire ? [CBSE F 09]

Fig. 2.151
Solution. (i) The arrangement is
equivalent to two capacitors connected in
Fig. 2.149
series. Each such capacitor has plate
Problem 12. Find the P.E. associated separation d / 2 and hence capacitance 2
with a charge ‘q’ if it were present at C. Total capacitance is
the point P with respect to the 'set-up'
2C × 2 C
of two charged spheres, arranged as Cs = =C
shown in Fig. 2.150. Here O is the mid- 2 C+2 C
point of the line O1O2. [CBSE Sample i.e., the capacitance remains unaffected.
Paper 13]
(ii) The capacitance becomes twice
the original capacitance because d
becomes d / 2
Problem 14. The graph [Fig.
2.152(a)] shows the variation of voltage
V across the plates of two capacitors A
and B versus increase of charge Q
stored on them. Which of the
capacitors has higher capacitance ?
Give reason for your answer. [CBSE D
04]

Fig. 2.150


Solution. r1 = O1P = r 2 +(2 a+ b)2


r2 = O2P = r 2 +(a+2 b)2
Potential at point P due to Q1 and Q2 is

V=
[
1 Q1 Q 2
+
4 π ε0 r1 r2 ] Fig. 2.152(a)

P.E. associated with charge q at point P,


U = qV =
q
[ Q1
+
Q2
4 π ε 0 [ r 2+ ( 2 a+b )2 ]1 /2 [ r 2+ ( a+2 b )2 ]1 /2 ]
Problem 13. A sheet of aluminium
Fig. 2.152(b)
foil of negligible thickness is placed
between the plates of a capacitor, as Solution. From Fig. 2.152(b),
shown in Fig. 2.151. What effect has it Q Q
on the capacitance if (i) the foil is CA = and CB =
electrically insulated, and (ii) the foil is VA VB

98
But VA < VB, therefore, CA > CB Thus ε 0 A /2 κ ε 0 A /2
capacitor A has a higher capacitance. C 1= , C 2=
d d

∴C p=C 1+C 2=
Problem 15. The given graph shows ε0 A κ ε 0 A
the variation of charge q versus +
potential difference V for two 2d 2d

= (κ +1)= ( κ+1).
capacitors C1 and C2. The two ε0 A C
capacitors have same plate separation
2d 2
Problem 17. Two identical
capacitors of plate dimensions / ×b and
plate separation d have dielectric slabs
filled in between the space of the plates
as shown in Figs. 2.155(a) and (b).

dielectric constants κ, κ1 and κ2.


Obtain the relation between the

[CBSE OD 13C]
Fig. 2.153
but the plate area of C2 is double than
that of C1. Which of the lines in the
graph correspond to C1 and C2 and why
? [CBSE OD 06, 14C]
q
Solution. As C = and graph A has a
V Fig. 2.155
larger slope
Solution. Capacitance of the capacitor
than B, so the graph A represents a shown in Fig. 2.155(a) is
capacitor of larger capacitance.
κ ε 0 A κ ε 0 (l ×b)
i.e., C ∝ A
ε0 A C 1= =
Also, C = d d
d
Capacitor shown in Fig. 2.155(b) is eqm
As the plate area of C2 is double of valent to a parallel combination of two

( 2l × b) and
that of C1, so C2 has a larger capacitance.
Hence the line A of the graph corresponds capacitors, each of plate area
to C2 and line B corresponds to C1.
plate separation d. Their equivalent
Problem 16. As shown in Fig. 2.154, capacitance is

constant κ is inserted in half portion


a dielectric material of dielectric
C 2=
between the plates of a parallel-plate
capacitor. If its initial capacitance is C,
κ1ε 0 ( 2l ×b) + κ ε ( 2l × b) = (κ + κ )ε (l ×b)
2 0
1 2 0
what is the new capacitance ? d d 2d
As the two capacitors are identical, so
C 1=C2
κ ε 0 (l× b) (κ 1+ κ2 )ε 0 (l ×b)
=
d 2d

∴ κ= .
Fig. 2.154 κ 1+ κ 2
Solution. The new arrangement is 2
equivalent to two capacitors connected in Problem 18. Find the capacitance of
parallel. three parallel plates, each of area A
metre2 and separated by d1 and d2

99
metre. The in-between spaces are filled the series combination does not depend
with dielectrics of relative permittivity on d1 and d2 i.e., it is independent of the
ε1 and ε2. The permittivity of free space position of the central section.
is ε0. [Roorkee 87]
Problem 20. Two charged spherical
Solution. The given system is conductors of radii R1 and R2 when
equivalent to two parallel-plate capacitors connected by a conducting wire
connected in series. Their capacitances acquire charges q1 and q2 respectively.
are Find the ratio of their surface charge
densities in terms of their radii. [CBSE
qε0 A ε2 ε0 A
C1 ¿ and C 2= D 14, 15C]
dη d2
Solution. The charges will flow
If C s is the equivalent capacitance of the between the two spherical conductors till
series combination, then their potentials become equal.
1 1 1 k q1 k q 2 q1 R 1
= + = That is, = or =
C s C 1 C2 R1 R2 q2 R 2

[
1 d 1 d2
+ =
ε0 A ε1 ε2 ε 0 A ] [
1 ε 2 d 1+ ε 1 d 2
ε1 ε2 ] The ratio of the surface charge
densities on the two conductors will be

ε0 A ε1 ε2 ε 0 A q1
C s= = 2 2
σ 1 4 π R 1 q1 R2 R 1 R 2 R2
2
or d 1 d 2 ε 2 d 1+ ε 1 d 2 .
+ = = ⋅ 2= × 2= .
ε1 ε2 σ2 q2 q2 R1 R 2 R 1 R1
Problem 19. Figure 2.156 shows two 4 π R 22
capacitors joined in series, the rigid Problem 21. Two parallel plate
central section of length b being capacitors of capacitances C1 and C2
movable. Prove that the equivalent such that C1 = 2C2 are connected
capacitance of the combination is across a battery of V volts as shown in
independent of the position of the Fig. 2.157. Initially the key (K) is kept
central section. closed to fully charge the capacitors.

dielectric slab of dielectric constant κ


The key is now thrown open and a

is inserted in the two capacitors to


completely fill the gap between the
plates. Find the ratio of (i) the net
capacitance and (ii) the energies stored
in the combination, before and after the
introduction of the dielectric slab.
[CBSE D 14C]
Fig. 2.156
ε0 A ε0 A
Solution. Here C1 = and C2 =
d1 d2
The equivalent capacitance of the series
combination,
Fig. 2.157
ε0 A ε0 A
× Solution. (i) Net capacitance before
C1C2 d1 d2 ε0 A ε0 A the introduction of the dielectric slab,
C= = = =
C1 +C 2 ε 0 A ε 0 A d 1 +d 2 a−b
+ CInitial = C1 + C2
d1 d2
= 2C2 + C2 = 3C2
Clearly, the equivalent capacitance of
Net capacitance after introduction of

100
the dielectric slab, Problem 23. Capacitors P, Q and R
CFinal = κC1 + κC2 = 2κC2 + κC2 = 3κC2
have each a capacity C. A battery can
charge the capacitor P to a potential
Ratio of the net capacitance, difference V. If after charging P, the
battery is disconnected from it and the
C Initial 3C2 1
= = =1: k charged capacitor P is connected in
C Final 3 κ C 2 K following separate instances to Q and
R:
(ii) Energy stored in the combination
before the introduction of dielectric slab, (i) to Q in parallel, and
2 (ii) to R in series, then what will be the
q
U Initial = potential differences between the
3 C2
plates of P in the two instances ?
Energy stored in the combination after
Solution. (i) When P is connected to
the introduction of dielectric slab
Q in parallel, the capacitors P and Q share
2
q charges equally.
∴ P.D. between the plate of P =
U Final =
3 κ C2
U Initial κ q /2 q V
= = .
Ratio of energy stored, = = C 2C 2
κ:1.
U Final 1
(ii) When P is connected to R in series,
charges on both capacitors remain same.
Problem 22. A parallel plate The capacitor P with the same charge will
capacitor is charged to a potential have the same potential difference V
difference ‘V’ by a d.c. source. The across it.
capacitor is then disconnected from
the source. If the distance between the Problem 24. An uncharged
plates is doubled, state with reason capacitor is connected to a battery.
how the following will change : (i) Show that half the energy supplied by
electric field between the plates, (ii) the battery is lost as heat while
capacitance and (iii) energy stored in charging the capacitor.
the capacitor.[CBSE D 01; OD 11,16] Solution. Let capacitance of the
Solution. Here capacitor = C
emf of the battery = V
∴ Charge given to the capacitor, q = CV
ε0 A σ q q
2
C0 = , E 0= = , U0=
d ε0 ε0 A 2 C0
On disconnecting the battery, the Energy supplied by the battery
charge q on the capacitor plates remains = Work done by the battery = qV
unchanged because no transfer of charge
occurs after the disconnection. If the 1
Energy stored in the capacitor = CV2 =
distance d is doubled, then 2
1
q qV
(i) E = =E0 i.e., the electric field 2
ε0 A
∴ Energy lost as heat = qV -
remains unchanged. 1 1
qV = qV
2 2
ε0 A 1
(ii) C = = C0 i.e., the capacitance is Thus, half the energy supplied by the
2d 2
halved. battery is lost as heat while charging the
capacitor.
2 2
q q Problem 25. A capacitor is
(iii) U = = = 2 U0 i.e., the stored
2C C0 connected across a battery. (i) Why
energy is doubled. does each plate receive a charge of

101
exactly the same magnitude ? (ii) Is 1 σ E0
this true even if the plates are of Final field, E = =
κ ε0 κ
different sizes ?
Thus, the electric field decreases.
Solution. (i) This is because of
(ii) Final capacitance, C = κ
conservation of charge. If q1 and q2 are ε0 A κ C 0
the charges taken by the two plates, then =
2d 2
q1 + q2 must be zero because the charge
2
on the battery is simply redistributed and Q
not created or destroyed. Initial energy, U0 =
2C 0
(ii) Yes, the charges will be of equal 2 2
magnitude even if the two plates have Q Q 2 2
Final energy, U = = ⋅ = U0
different sizes. 2C 2 κ C0 κ
Problem 26. Explain briefly the As 1 < 2 < κ, the energy stored increases
process of charging a parallel plate i.e., U > U0.
capacitor when it is connected across Problem 27. Two identical capacitors
a d.c. battery. C1 and C2 are connected to a battery B,
A capacitor of capacitance ‘C' is as shown in Fig. 2.158. A dielectric slab
charged to ‘V’ volts by a battery. After is slipped between the plates of C2, the
some time the battery is disconnected battery remaining connected. What
and the distance between the plates is happens to the charge, the

constant, 1 < κ < 2, is introduced to fill


doubled. Now a slab of dielectric capacitance, the potential difference
and stored energy of each capacitor ?
the space between the plates. How will
the following be affected ?
(a) The electric field between the plates
of the capacitor
(b) The energy stored in the capacitor
Justify your answer by writing the
necessary expressions. [CBSE OD
15] Fig. 2.158
Solution. Electrons start flowing from Solution. On introducing the dielectric
one plate of the capacitor to the positive slab between the plates of capacitor C2, its
terminal of the battery and from the capacitance increases, hence the
negative terminal to the other plate. The capacitance of the whole combination
two plates start acquiring positive and increases. As the battery remains
negative charges respectively. This connected, the p.d. across the
process of charging of the capacitor combination remains constant, and so the
continues until the potential across its charge on the combination will increase.
plates becomes equal to the p.d. across As C1 and C2 are in series, charges on
the terminals of the battery. them will be equal. As charge on C1 has
increased, its capacitance is same and so
Effect of dielectric. When the battery is p.d. across C1 will increase. But V1 + V2=
disconnected, charges on the capacitor constant, so the potential difference
plates remain unchanged. across C2 will decrease.
ε0 A 1
(i) Initial capacitance, C0 = Energy of C1, U1 =
2
C1V 1 will increase.
d 2
σ Energy of C2,
Initial field, E0 =
ε0
( )
2 2
1 V2 1 C2 V 2
U2 =
2
( κ C2 )
κ
=
2 κ
will decrease.

102
However, there will be a net increase dielectric constant κ is now placed
in the energy of the combination because between the plates. What change, if
more charge has been drawn by the any, will take place in : (i) charge on
capacitors from the battery. plates (ii) electric field intensity
Problem 28. A parallel plate between the plates (iii) capacitance of
capacitor is charged by a battery. After the capacitor ?
some time the battery is disconnected Justify your answer in each case.
and a dielectric slab with its thickness [CBSE D 10]
equal to the plate separation is inserted
Solution. As the battery remains
between the plates. What change, in
connected, the potential difference V
any will take place in (i) charge on the
remains constant even after the
plates (ii) electric field intensity
introduction of the dielectric medium.
between the plates (iii) the capacitance
of the capacitor, (iv) potential (i) The capacitance increases from C0
difference between the plates and (v) to C
C = κC0
the energy stored in the capacitor ?
Justify your answer in each case.
(ii) Charge on the capacitor plates
[CBSE D 07, 10 ; OD 09]
increases from Q0 to Q.
Q = CV = κC0V = κQ0
Solution. (i) The charge q0 on the
capacitor plates remains the same
because the battery has been (iii) As the potential difference V
disconnected, before placing the dielectric remains unchanged, the electric field also
slab. remains unchanged.
(ii) The surface charges induced on V
the dielectric slab reduce electric field E= =E0
d
intensity to a new value given by
Problem 30 A parallel plate
E0 capacitor of capacitance C is charged
E=
κ to a potential V by a battery. Without
disconnecting the battery, the distance
(iii) The reduction in the electric field
dielectric medium of κ = 10 is
between the plates is tripled and a
reduces the potential difference
E0 d V 0 introduced between the plates of the
V = Ed = = capacitor. Explain giving reasons, how
κ κ
will the following be affected :

capacitance increases κ times


(iv) Due to the decrease in p.d., the
(i) capacitance of the capacitor
(ii) charge on the capacitor, and
q0 q0 q0
C= = =κ =κ C 0 (iii) energy density of the capacitor.
V V 0 /κ V0
[CBSE OD 17C]
factor of κ
(v) Energy stored decreases by a
Solution. As the battery remains
connected, the potential difference V
U =
remains unchanged.

( )
2
1 2 1 V0 1 1 2 U0
C V = ( κ C0) = ⋅ C 0 V 0= ε0 A
2 2 κ κ 2 κ (i) Original capacitance, C0 =
d
Problem 29. A parallel plate
capacitor, each with plate area A and New capacitance,
separation d, is charged to a potential κ ε 0 A 10 × ε 0 A 10
difference V. The battery used to C= = = C0 .
charge it remains connected. A
d’ 3d 3
dielectric slab of thickness d and

103
10 10 New energy density,
(ii) Q=CV = C0 V = Q 0 .
3 3 u =

( )
2
V V E0 1 2 1 E0 10 1 2 10
(iii) New field, E= = = ε E = κ ε0 = . ε 0 E 0= u 0 .
d’ 3d 3 2 2 3 9 2 9
1 2
Original energy density, u0 = ε 0 E0
2

HOTS
Problems on Higher Order Thinking Skills

Problem 1. The electric field inside n Explain your answer in each case. [CBSE
parallel plate capacitor is E. Find the SP 03] Solution. At any point, we have
amount of work done in moving a charge
dV
q over a closed rectangular loop a b c d a. E=- = Negative slope of V-x graph
dx
[CBSE D 14]
dV
At point A, =0
dx
dV
At point B, <0
dx
dV
Fig. 2.159 At point C, >0
Solution. E ⊥ ab and E ⊥ dc,
dx
Therefore, (i) E is zero at point A
so Wab = 0 and Wcd = 0.
(ii) E is maximum at point B.
Also, Wbc = -Wda
Problem 3. Two identical plane
Total work done in moving charge q over metallic surfaces A and Bare kept
the dosed loop abcda, parallel to each other in air, separated
by a distance of 1 cm, as shown in Fig.
W = Wab + Wbc + Wcd +Wda
2.161.
= 0 – Wda + 0 + Wda = 0.
Problem 2. Figure 2.160 shows the
variation of electrostatic potential V with
distance 'x' for a given charge

Fig. 2.161
Surface A is given a positive potential
of 10 V, and the outer surface of B is
earthed.
(i) What is the magnitude and direction of
Fig. 2.160 the uniform electric field between
points Y and Z ?
distribution. From the points marked A, B
and C, identify the point at which the (ii) What is the work done in moving a
electric field is : charge of 20 μC from point X to point
Y. [CBSE Sample Paper 08]
(i) zero (ii) maximum.

104
dV 10V here, show the variation of electrostatic
Solution. (i) E = - =- potential (V) with 1/r (r being distance
dr 1 cm
of the field point from the point charge)
10V for two point charges q1 and q2.
=- −2 = 1000 Vm-1.
10 m (i) What are the signs of the two
Magnitude of the uniform electric field charges ?
between X and Y = 1000 Vm-1. (ii) Which of the two charges has a
The direction of the electric field is larger magnitude and why ? [CBSE
from plate A to plate B. Sample Paper 08]
(ii) Zero. This is because the points X
and Y are at the same potential.
Problem 4. The electric potential as a
function of distance x is shown in Fig.
2.162. Construct a graph of the electric
field strength E.

Fig. 2.164
Solution. (i) Charge q1 is -ve while charge
q2 is +ve.
1 q q 1
(ii) As V = ⋅ = ⋅
Fig. 2.162
4 π ε0 r 4 π ε0 r

∴ Slope of V vs.
Solution. We know that 1 q
graph =
r 4 π ε0
dV
E=- = Negative slope of V-x graph
dx As the graph for q1 has a slope of
larger magnitude than that for q2, so q1
For 0 < x < 1, has a larger magnitude than q2.
dV Problem 6. The following data was
= +ve constant, so field = - E
dx obtained for the dependence of the
For 1 < x <2, magnitude of electric field, with distance,
from a reference point O, within the
dV charge distribution in the shaded region.
= 0, so field = 0
dx
Field Point A B C A' B' C
For 2 < x < 3,
dV Magnitude of E E E E E E
= -ve constant, so field = + E electric field
dx 8 27 2 16 64
Consequently, we get the E-x graph as
(i) Identify the charge distribution and
shown below:
justify your answer.
(ii) If the potential due to this charge
distribution, has a value V at the point
A, what is its value at the point A' ?
[CBSE Sample Paper 08]

Fig. 2.163 E-x graph.


Problem 5. The two graphs drawn

105
In case of a constant electric field
along Z-direction, the equipotential
surfaces are equidistant planes parallel to
XY-planes, as shown in Fig. 2.200.
Problem 8. An infinite number of
charges, each equal to q, are placed
along the x-axis at x = 1, x = 2, x = 4....
and so on. (i) Find the potential at the
point x = 0 due to this set of charges. (ii)
What will be the potential if in the above
Fig. 2.165 set up the consecutive charges have
Solution. (i) From the given data, we opposite signs ? [IIT]
observe that (a) The magnitude of electric Solution. (i) The potential at x = 0 due
field varies inversely as the cube of the to given set of charges is
distance of the field point along the
horizontal line OC.
(b) The magnitudes of E at points A',
V=
1 q q q q
[
+ + + +…
4 π ε0 1 2 4 8 ]
B’ and C’ on the vertical line OC are just =
half the magnitudes at the corresponding
equidistant points A, B and C
The above properties indicate that
q
4 π ε0[ 1 1 1
1+ + + +… =
2 4 8
q
4 π ε0

1
1−
]
=
q
1 2 π ε0
2
given charge distribution is a short electric
dipole centred at the reference point O.
(ii) As the point A' lies on the
[ ∵ Sum of an infinite G.P. =
b
1−r ]
equatorial line of the dipole, so the electric (ii) When the consecutive charges
potential at point A must be zero. have opposite signs, potential at x = 0 is
Problem 7. Draw 3 equipotential
surfaces corresponding to a field that
uniformly increases in magnitude but
V=
1 q q q q
[
− + − +…
4 π ε0 1 2 4 8 ]
remains constant along Z-direction. How
are these surfaces different from that of a
constant electric field along Z-direction ?
=
q
4 π ε0[ 1 1 1
1− + − + …
2 4 8 ]
[CBSE OD 09] q 1 q
⋅ =
Solution. The equipotential surfaces
corresponding to an electric field that
uniformly increases in magnitude but
= 4 π ε0
1−
2 ( )
−1 6 π ε 0 .

remains in a constant Z-direction are Problem 9. A charge Q is distributed


planes parallel to XY-plane shown in Fig. over two concentric hollow spheres of radii
2.166. But as field increases, such planes r and R, where R > r, such that the surface
get closer. charge densities are equal. Find the
potential at the common centre. [IIT 81]
Solution. Let Q. and QR be the charges
distributed over the smaller and the larger
spheres, respectively.
Then Q = Qr + QR
The surface charge densities will be
QR Qr
σ= 2
= 2
4π R 4πr
Fig. 2.166

106
Qr r 2 Qr +Q R r 2+ R 2 qA = 4π a2 σ, qB = - 4π b2 σ, qC = 4π c2 σ
or = 2 or = 2
QR R QR R The potential at every point inside a
spherical shell is constant and is equal to

( )
2 2 2
Q r +R R that on its surface.
or = 2 or Q R= 2 2 Q
QR R R +r Potential of shell A. Any point on the
surface of shell A lies inside the shells B
( )
2
r and C.
Similarly, Q r = 2 2
Q
R +r
∴ V A= [ ]
1 q A qB qC
Potential due to the charge on the smaller + +
4 π ε0 a b c
sphere is

[ ]
2 2 2
1 4πa σ 4πb σ 4πc σ
Qr
( )
2
1 r = − +
V r= = . Q 4 π ε0 a b c
4 π ε 0 r 4 π ε 0 r R 2+r 2
Q. r σ
¿ or V A = ( a−b+ c ).
2 2
4 π ε 0 (R + r ) ε0
Potential due to the charge on larger Potential of shell B. Any point on shell B
sphere is lies outside the shell A and inside the shell
C.
V R=
∴ V B= [ ]
1 qA qB qC
QR
=
1 R2
. 2 2 Q=
4 π ε0 R 4 π ε 0 R R + r ( Q.R
)
4 π ε 0 ( R2 +r 2 )
+ +
4 π ε0 b b c

[ ]
2 2 2
Total potential at the centre, 1 4πa σ 4πb σ 4πc σ
= − +
4 π ε0 b b c
Q(r + R)
V =V r +V R =
[ ]
. 2
4 π ε 0 (R 2+ r 2) or V B=
σ a
−b+c .
ε0 b
Problem 10. Three concentric metallic
shells A, B and C of radii, a, b and c (a < b Potential of shell C. Any point on shell C
< c) have surface charge densities + σ, - σ lies outside the shells A and B.
and + σ respectively, as shown in Fig.
∴ V C= [ ]
2.167. 1 q A q B qC
+ +
(i) Find potential of three shells A, B, C.
4 π ε0 c c c

[ ]
2 2 2
(ii) If shells A and C are at the same 1 4πa σ 4πb σ 4πc σ
potential, obtain the relation between
= − +
4 π ε0 c c c
radii a, b, c.

[ ]
2 2
[CBSE Sample Paper 03, IIT 90] σ a b
or V C = − +c .
ε0 c c
(ii) If the shells A and C are at the
same potential, then VA = VC

∴ ( )
σ σ a2 b2
(a−b+ c)= − +c
ε0 ε0 c c
(a−b)(a+b)
or a – b =
c
or a + b = c.
Fig. 2.167
Problem 11. In Fig. 2.168, the
Solution, (i) Charges on the three potentials at points A and Bare VA and VB
shells are respectively. Calculate VA - VB for the

107
given arrangement.
[ AB
10 cm
=cos 60°
]
Also dV = 10 - 20 = -10 V

∴E=-
Fig. 2.168 dV −−10 V −1
= =200 Vm
dr 5× 10 m
−2
Solution. The net potential at A due to
the two charges is Angle made by ⃗
E with positive X-axis is

VA =
1
[ q

q
=
1
] .
qy
4 π ε 0 x x+ y 4 π ε 0 x ( x + y )
θ = 180 - 60° = 120°.
Problem 13. The electric potential V at
The net potential at B due to the two any point x, y, z (all in metres) in space is
charges is given by V = 4x2 volts. Calculate the
electric field at the point (1 m, 0, 2 m).
VB=
1
[ q q
− =
−1
].
qy
4 π ε0 x + y x 4 π ε0 x ( x + y )
[AIPMT11]
Solution. Here V = 4x2

∴ V A −V B = .
1 2 qy −∂ V −∂V
. E x= =−8 x , E y = =0 , E z
4 π ε 0 x (x+ y) ∂x ∂y
Problem 12. Figure 2.169 shows some ∂V
equipotential surfaces. What can you say ¿− =0
∂z
about the magnitude and direction of the
electric field ? ⃗
E =E x i+ ^
^ E y ^j+ E z k=−8 x i^
At point (1 m, 0, 2 m),
E = -8 × 1i^ = -8i^ Vm-1.

Problem 14. The electric field outside
a charged long straight wire is given by E
1000
= Vm-1, directed outwards. What is
r
Fig. 2.169 the sign of the charge on the wire ? If two
Solution. As shown in Fig. 2.170, points A and B are situated such that r A
consider two consecutive equipotential =0.2 m and rB =0.4 m, find the value of(V B
surfaces. Electric field is normal to the - VA).
equipotential surfaces and always directed Solution. As the electric field is
from higher potential to lower potential. directed outwards, the charge on the wire
must be positive.
B r=0.4 m
1000
V B−V A =−∫ ⃗
E ⋅⃗
dl=− ∫ dr
A r=0.2 m r
0.4
= -1000 [log e r ] 0.2

= -1000 [loge0.4 - loge 0.2]


0.4
= -1000 loge = -1000 × loge2
Fig. 2.170 0.2
The normal distance between two = -1000 × 0.6931 = -693.1 V.
consecutive equipotential surfaces is Problem 15. There are two particles
dr = AB = 10 cm × cos 60° = 5 cm each of mass m and carrying a charge Q.
Initially one of them is at rest on a smooth-
horizontal plane and the other is projected

108
from a long distance along the plane, Problem 17. Three point charges of
directly towards the first particle with a 1C, 2 C and 3 C are placed at the corners
velocity v. Find the closest distance of of an equilateral triangle of side 1 m.
approach. [Roorkee 83] Calculate the work required to move these
charges to the corners of a smaller
Solution. At the distance of closest
equilateral triangle of sides 0.5 m as
approach r0, both particles acquire a
shown in Fig. 2.172. [Roorkee 97]
common velocity v0. By conservation of
linear momentum,
0 + mv = mv0 + mv0 or v0 = v/2
By conservation of energy,
1 2 1 Q.Q 1 2 1 2
0+ m v = . + m v0+ m v0
2 4 π ε0 r0 2 2

()
2 2
1 2 1 Q v
or mv = . +m Fig. 2.172
2 4 π ε0 r0 2
Solution. Initial P.E. of the system is
or m v =
2
1 1 Q

[ ]
2
. 1 q 1 q 2 q2 q 3 q3 q 1
4 4 π ε0 r 0 U i= + +
2
4 π ε 0 r 12 r 23 r 13
1 4Q
or r 0 = . .
4 π ε0 m v 2
Problem 16. A point charge q moves
¿ 9 ×1 09
[ 1× 2 2 ×3 3 × 1
1
+
1
+
1 ]
from a point P to a point S along the path 9
¿ 9 ×1 0 ×11=9.9 ×1 0 J
10

PQRS in a uniform electric field ⃗ E acting


Final P.E. of the system is
along the positive direction of X-axis. The

[ ]
coordinates of the points P, Q, R and S 1 q1 q2 q1 q2 q3 q1
are(a, b, 0),(2a, 0, 0), (a, - b, 0) and (0, 0, U f= + +
4 π ε 0 r ' 1 2 r ' 1 2 r ' 13
0) respectively. Find the work done by the

[ ]
field in the process. [IIT 92] 1× 2 2 ×3 3 × 1
¿ 9 ×1 09 + +
Solution. As the electric field is a 0.5 0.5 0.5
conservative field, so the work done 9 10
depends only on the initial and final points. ¿ 9 ×1 0 ×22=19.8 ×1 0 J
Work done
10 10
¿ U f −U i =(19.8−9.9)× 1 0 =9.9 ×1 0 J.
Problem 18. Two electric dipoles of
moments p1 and p2 are in a straight line.
Show that the potential energy of each in
1 p1 p 2
the presence of the other is− . 3 ,
2 π ε0 r
Fig. 2.171
where r is the distance between the
Displacement of charge q, dipoles. (Assume r to be much greater
⃗ than the length of the dipole).
PS = (0, 0, 0) -(a, b, 0)
^
= (0−a) i+(0−b) ^
^j+(0−0) k=−a ^
i−b ^j
Force on charge q,
⃗ E = q E i^
F=q⃗ Fig. 2.173
F.⃗
W=⃗ PS = qE i^ .(- ai^ - b ^j ) = - qEa. Solution. Electric field due to the dipole

109
of moment p1 at the other dipole is C = 4πε0a
1 2 p1 When surrounded by an earthed
E= ⋅ 3 sphere of radius b, its capacitance
4 π ε0 r
becomes
The potential energy of the other
' ab
dipole of moment p2 in the electric field E C =4 π ε 0 ⋅
b−a

is
1 2 p 1 −1 p 1 p2
U =−p 2 E=− p2 × ⋅ 3 = C
'
ab b 1 1
4 π ε0 r 2 π ε0 r 3 = = = = =n
C a(b−a) b−a a n−1
Problem 19. Two identical metal plates 1− 1−
b n
are given charges q1 and q2 (< q1)
respectively. If they are now brought close Problem 21. N spherical droplets, each
together to form a parallel plate capacitor of radius r, have been charged to have a
with capacitance C, what will be the potential V each. If all these droplets were
potential difference between the plates ? to coalesce to form a single large drop,
[IIT 99] what would be the potential of this large
drop ?
Solution. Let A be area of each plate.
When the two plates are placed d distance (It is given that the capacitance of a
apart, the capacitance of the parallel plate sphere of radius x equals 4 πε0kx).
capacitor so formed is [CBSE Sample Paper 13]
ε0 A Solution. Potential of each droplet = V
C=
d Capacitance of each droplet, C = 4 πε0kr
If E1 and E2 are the electric fields due Charge on each droplet, q = CV = 4
to the two plates, then the net field πε0krV
between the two plates will be Total (initial) charge on all droplets,
σ1 σ2 q' = Nq = 4πε0kr NV
E = E 1 - E2 = −
2 ε0 2 ε 0 If R is the radius of the large drop, then
q1 / A q 2 / A 1 4 4
= − = (q −q ) π R3 = N × πr3
2 ε0 2 ε0 2 ε 0 A 1 2 3 3
The potential difference between the ∴ R = N1/3r
two plates will be
Capacitance of the large drop,
V = Ed =
C’ = 4πε0kR = 4πε0kN1/3r
1 1
( q1−q2 )× d= (q −q )
2 ε0 A 2 ε 0 A /d 1 2 Potential of the large drop,

q1−q2 ' q
'
4 π ε 0 krNV 2 /3
or V = . V= '
= 1 /3
=N V
2C C 4 π ε0 k N r
Problem 20. Assuming an expression Problem 22.
for the potential of an isolated conductor, (i) Two circular metal plates, each of
show that the capacitance of such a radius 10 cm, are kept parallel to each
sphere will be increased by a factor n if it other at a distance of 1 mm. What kind
is enclosed within an earthed concentric of capacitor do they make ? Mention
sphere, the ratio of the radii of the spheres one application of this capacitor.
being n/(n - 1).
(ii) If the radius of each of the plates is
Solution. The capacitance of an increased by a factor of √2 and their
isolated conducting sphere of radius a is distance of separation decreased to

110
half of its initial value, calculate the plates is decreased by 10%, the energy
ratio of the capacitance in the two stored becomes U’ such that,
cases. '
U d 100 ' 10
(iii) Suggest any one possible method by = = or U = U
U d ' 90 9
which the capacitance in the second
case be increased by n times. Percentage change in the stored
[CBSE Sample Paper 03] energy,

( )
Solution. (i) The two plates form a U −U
'
U
'

parallel plate capacitor. × 100= −1 ×100


U U
Application. Alongwith an inductor, a
capacitor is used in an oscillatory circuit.
(ii) Original capacitance, C =
¿ ( 109 −1) ×100= 1009
2
ε0 A ε0 × π r = 11.11%
=
d d Problem 24. Two identical parallel
plate capacitors A and B are connected to
New capacitance,
a battery of V volts with the switch S
2
'ε 0 × π (r √ 2) 4 ε 0 × π r 2 closed. The switch is now opened and the
C= = =4 C free space between the plates of the
d /2 d
dielectric constant κ. Find the ratio of the
capacitors is filled with a dielectric of
∴ = =1 : 4.
C 1
C 4
' total electrostatic energy stored in both
capacitors before and after the
(iii) The capacitance of a capacitor can introduction of the dielectric.
be increased n times by any of the
following methods : [CBSE OD 17]

(a) By inserting a dielectric of dielectric


constant n between the capacitor
plates. Then C' = nC.

the plates by a factor n, because C ∝ 1


(b) By decreasing the distance d between
Fig. 2.174
/ d.
Solution. When the switch S is closed,

times, because C ∝ A
(c) By increasing the area of the plates n
P.D. across A = P.D. across B = V

(d) By increasing the area of the plates √ n Initial energy stored in both the capacitors
times and simultaneously decreasing is
the distance between the plates by a 1 1
factor of √ n. Ui = U A + U B CV2 + CV2 = CV2
2 2
Problem 23. Keeping the voltage of When the dielectric is inserted, the

κC. The p.d. across A is still V With switch


the charging source constant, what would capacitance of each capacitor becomes
be the percentage change in the energy
stored in a parallel plate capacitor if the S open, the p.d. on B attains a new value
separation between its plates were to be V but charge (q = CV) does not change.
decreased by 10% ? [CBSE S.P. 08]
∴ q = CV = κC × V' or V'=
V
V i.e., U ∝
1 1 ε0 A 2 κ
Solution. U = CV2 =
2 2 d
The final total energy stored in both
1
capacitors is
d
Uf = U'A + U'B
When the separation between the

111
( )
1 1 V
2
κ ε0 A κ ε0 A
= .κC × V2 + .κC × = C 1= and C 2=
2 2 κ d1 d2

( )
2
1 1+κ 2 The energies stored in the capacitor at
CV
2 κ the two separations are
2 2
Ui CV
2
2κ 1 q 1 q
∴ U f 1 1+κ 1+ κ .
= = U 1= and U 2=

( )
2 2 2 C1 2 C2
∴ Work done
2
CV
2 κ
Problem 25. A parallel-plate capacitor
[ ]
2
q 1 1
contains one mica sheet of thickness d1 = = U 2−U 1= −
2 C2 C1
1.0 × 10-3 m and one fibre sheet of

[ ]
thickness d2 =0.5 × 10-3 m The dielectric q d 2−d 1
2
constants of mica and fibre are 8 and 2.5 =
respectively. Fibre breaks down in an 2 κ ε0 A

[∵ q = σA]
electric field of 6.4 × 106 Vm-1. What 2
maximum voltage can be applied in the
σ A
= (d 2−d 1).
capacitor ? 2κ ε 0
Solution. Let σ be the surface density Problem 27. The capacitors C1, C2, having
on the capacitor plates. Electric fields in plates of area A each, are connected in
mica (κ1) of thickness d2 and fibre (κ2) of series, as shown. Compare the
thickness d2 will be capacitance of this combination with the
capacitor C3, again having plates of area
σ σ A each, but 'made up' as shown in Fig.
E1 = and E2 =
κ1ε 0 κ2ε 0 2.175. [CBSE Sample Paper 13]

∴ E 1=
κ2
⋅E
κ1 2
The maximum permissible value for E2
is 6.4 × 106 Vm-1.
∴ Maximum permissible value for E1
2.5
= × 6.4 × 10-6 = 2.0 × 106 Vm-1
8 Fig. 2.175
Maximum voltage that can be applied to Solution. Clearly,
capacitor is
κ1 ε 0 A κ2ε 0 A
V = E1d1 + E2d2 = 2.0 × 106 × 1.0 × 10-3 + C 1= and C2 =
6.4 × 106 × 0.5 × 10-3
d d
The equivalent capacitance of the
= 2.0 × 103 + 3.2 × 103 = 5200 V.
series combination of C1 and C2 is
Problem 26. The insulated plates of a
parallel plate capacitor has a charge
density σ. Show that the work done in
changing the distance from d1 to d2 is
C eq=
C 1+C 2
=
(
C1 C2 ε 0 A κ 1 κ2
d κ1 +κ 2 )
2
Now, capacitor C3 can be regarded as
σ A the series combination of capacitors C’ 1
U= (d 2−d 1 ).
2 ε 0κ and C’2, each of plate area A and
separation d. Again,
Solution. The capacitances at the two
separations are ' κ1 ε 0 A ' κ2 ε 0 A
C 1= and C 2=
d d

112
∴C 3= ( )
' '
C1C2 ε 0 A κ1 κ 2
=
C +C
'
1
'
2
d κ 1+ κ 2
Hence, Ceq = C3.
Problem 28. Three identical parallel
plate (air) capacitors C1, C2, C3 have
capacitances C each. The space between
their plates is now filled with dielectrics as Fig. 2.177
shown. If all the three capacitors, still have
Solution. The given arrangement is
between the dielectric constants κ1, κ2, κ3
equal capacitances, obtain the relation
equivalent to three parallel plate
and κ4. [CBSE Sample Paper 2011]
capacitors connected in series.
Capacitance of first capacitor, (κ = 2)
2 ε 0 A 6 ε0 A
C 1= =
d /3 d
Capacitance of second capacitor,
κ ε 0 A 3 κ ε0 A
C 2= =
d /3 d
Fig. 2.176 where κ = ∞ for copper metal.
Solution. New capacitance of C1 Capacitance of third capacitor,
κ ε0 A ε0 A 3 ε0 A
= C 3= =
d d /3 d
New capacitance of C2 Equivalent capacitance of the series
= Series combination of two capacitors combination is given by
1 1 1 1
= (
ε 0 A 2κ 1 κ2
d κ 1+ κ 2 ) = + +
C C1 C 2 C 3
d d d
New capacitance of C3 = + +
6 ε0 A 3 κ ε0 A 3 ε0 A
= Parallel combination of two capacitors
d d
+0+
( )
ε 0 A κ3 +κ 4 = [κ = ∞ for Cu]
= 6 ε0 A 3 ε0 A
d 2
2 ε0 A
As C1 = C2 = C3, so or C = .
d
2 κ1 κ 2 κ 3 +κ 4 Problem 30. A parallel plate capacitor
κ= =
κ 1 +κ 2 2 consists of three dielectrics as shown in

plate separation d =2 mm If κ1 = 4, κ2 = 6
Fig. 2.178. It has plate area A = 1 cm 2 and
Problem 29. Between the plates of a
and κ3 = 2, find the capacitance of this
plate sits on a dielectric slab of κ = 2, as
parallel-plate capacitor of area A, a copper
arrangement between points A and B.
shown in Fig. 2.177. Find the equivalent
capacitance of this arrangement. Solution. This arrangement consists of
capacitor C1 connected in parallel with the
series arrangement of C2 and C3.

113
Fig. 2.179
Solution. As shown in Fig. 2.180,
suppose the point P is connected to the
Fig. 2.178 positive terminal and point Q to the
negative terminal of a battery. Clearly, we
The equivalent capacitance of C2 and
have two capacitors I and II with their
C3 connected in series is given by
positive plates connected together and
1 1 1 their negative plates connected together.
= + So the two capacitors are in parallel.
C ’ C 2 C3
κ2 ε 0 A/2 3 κ 2 ε 0 A
But C 2= =
d /3 2d
κ3 ε 0 A /2 3 κ 3 ε 0 A
and C 3= =
2 d /3 4d
Fig. 2.180
∴ ∴ Equivalent capacitance,
1 2d 4d
= +
C ’ 3 κ 2 ε0 A 3 κ 3 ε0 A
2 ε0 A

( )
2d 1 2 Cp = C1 + C2 = C + C = 2C = .
¿ + d
3 ε0 A κ 2 κ 3
Problem 32. What is the capacitance
' 3 ε 0 A κ2 κ 3 of arrangement of 4 plates of area A at
or C = distance d in air in Fig. 2.181 ?
2 d (κ 3 +2 κ 2)
[DPMT 87]
The equivalent capacitance between A
and B given by
3 ε0 A κ 2 κ 3 ε0 κ 1 A
C=C ' +C 1= +
2 d (κ 3+ 2κ 2 ) 2d

¿
[
ε 0 A 3 κ2 κ 3
2 d κ 3 +2 κ2 1

] Fig. 2.181
or C=
2d [
ε 0 A 3 κ 2 κ 3 +κ 3 κ1 +2 κ 2 κ 1
κ3 +2 κ 2 ] Solution. As shown in Fig. 2.182,
suppose the point P is connected to the
positive terminal and point Q to the

[ ]
−12 −4
8.85 ×1 0 × 1× 10 3× 6 ×2+2 × 4+2 ×6 ×negative
4 terminal of a battery. Clearly, we
¿ −3
×
2 ×2 ×1 0 2+ 2× 6 have three capacitors I, II and III. Their
−13
positive plates are connected to the same
8.85 ×1 0 92 point P while the negative plates are
¿ ×
4 14 connected to the same point Q. So the
−13 three capacitors are in parallel.
¿ 14.54 ×1 0 F
= 1.454 pF.
Problem 31. What is the capacitance
of arrangement of 4 plates of area A at
distance d in air in Fig. 2.179 ? [CPMT 89]

114
Fig. 2.182 Problem 34. The plate A of a parallel
plate capacitor is connected to a spring of
Equivalent capacitance,
force constant k and can move, while the
3 ε0 A plate B is fixed. The arrangement is held
Cp = C1 + C2 + C3 = 3C = . between two rigid supports as shown in
d
Fig. 2.185. If a charge + q is placed on
Problem 33. A capacitor is made of a plate A and -q on plate B, by how much
flat plate of area A and a second plate does the spring elongate ?
having a stair-like structure, as shown in
Fig. 2.183. The width of each stair is a and
the height is b. Find the capacitance of the
assembly.

Fig. 2.185
Solution. As the two plates carry equal
and opposite charge, they attract each
other with a force F. As a result, the spring
elongates by length l. Then
Fig. 2.183 F = - kl ...(i)
Solution. As shown in Fig. 2.184, the where k is the force constant of the spring.
given arrangement is equivalent to a The capacitance of the parallel plate
parallel combination of three capacitors of capacitor with plate separation x is
capacitances C1, C2 and C3. ε0 A
C=
x
The energy stored in the capacitor is
2 2
1 q q x
U= ⋅ = ⋅
2 C 2 ε0 A
As the electrostatic force is a
conservative force, we can write

( )
2 2
Fig. 2.184 dU −d q x −q
F=- = = …(ii)
dx dx 2 ε 0 A 2 ε 0 A
Here
ε 0 A /3 ε 0 A /3 ε 0 A /3 The negative sign shows that the force
C 1= , C 2= , C 3= , is attractive. From equations (i) and (ii),
d d +b d +2 b

we get
C = C1 + C2 + C3 = q
2

[ ]
ε0 A 1 1 1 - kl = -
+ + 2 ε0 A
3 d d+ b d+2 b
2
q

[ ]
ε 0 A ( d +b ) ( d+ 2b ) +d ( d+ 2 b ) +d ( d +b ) or l = .
= 2 ε 0 kA
3 d ( d +b )( d +2 b )
ε 0 A (3 d 2 +6 bd +2 b2 )
= .
3 d(d +b)(d +2 b)

GUIDELINES TO NCERT EXERCISES

115
2.1. Two charges 5 × 10-8C and -3 × charge on the side of the negative charge.
-8
10 C are located 16 cm apart. At what
2.2. A regidar hexagon of side 10 cm
point on the line joining the two charges is
has a charge of 5 μC at each of its
the electric potential zero ? Take the
vertices. Calculate the potential at the
potential at infinity to be zero.
centre of the hexagon.
Ans. Zero of electric potential for two
Ans. Clearly, distance of each charge
charges. As shown in Fig. 2.186, suppose
from the centre O is
the two charges are placed on X-axis with
the positive charge located at the origin O. r = 10 cm = 0.10 m

Fig. 2.186
Let the potential be zero at the point P
and OP = x. For x < 0 (i.e., to the left of
O), the potentials of the two charges
cannot add up to zero. Clearly, x must be
positive. If x lies between O and A, then Fig. 2.188
V1 + V 2 = 0 Magnitude of each charge is

[ ]
q = 5 μC = 5 × 10-6C
∴ Potential at the centre O is
1 q1 q2
+ =0
4 π ε 0 x 0.16−x

[ ]
9 −6
5 ×10 3 ×10
−8 −8 1 q 6× 9 ×10 ×5 × 10
or 9 ×10 −
9
=0 V = 6. ⋅ = =
x 0.16−x 4 π ε0 r 0.10
2.7 × 106 V.
5 3
or − =0 2.3. Two charges + 2 μC and - 2 μC
x 0.16−x are placed at points A and B, 6 cm apart,
or x = 0.10 m = 10 cm. (i) Identify an ecjuipotential surface of the
system (ii) What is the direction of the
The other possibility is that x may also
electric field at every point on the
lie on OA produced, as shown in Fig.
surface ?
2.187.
Ans. (i) The equipotential surface will
be a plane normal to AB and passing
through its midpoint O, as shown in Fig.
2.189. It has zero potential everywhere.

Fig. 2.187
As V1 + V2 = 0

∴ [ ]
−8 −8
1 5 × 10 3× 10
− =0
4 π ε0 x x−0.16
5 3
or − =0
x x−0.16 Fig. 2.189
or x = 0.40 m = 40 cm. (ii) The direction of electric field is
Thus the electric potential is zero at 10 normal to the plane in the direction AB i.e.,
cm and 40 cm away from the positive from positive to negative charge.

116
2.4. A spherical conductor of V 120
radius 12 cm has a charge of 1.6 × 10-7C or ΔV = = = 40 V.
3 3
distributed uniformly on its surface. What
is the electric field 2.7. Three capacitors of capacitances
2 pF, 3 pF and 4 pF are connected in
(a) inside the sphere parallel, (a) What is the total capacitance
(b) just outside the sphere of the combination ? (b) Determine the
charge on each capacitor if the
(c) at a point 18 cm from the centre of combination is connected to a 100 V
the sphere ? supply.
Ans. Refer to the solution of Example Ans. (a) For the parallel combination,
75 on page 1.60. total capacitance is given by
2.5. A parallel plate capacitor with air C = C1 + C2 + C2 = 2 + 3 + 4 = 9pF.
between the plates has a capacitance of 8
pF (1 pF = 10-12 F). What will be the (b) When the combination is
capacitance if the distance between the connected to 100 V supply, charges on
plates be reduced by half, the space the capacitors will be

dielectric constant, κ = 6 ?
between them is filled with a substance of q1 = C1V = 2 × 10-12 × 100 = 2 × 10-10C
q2 = C2V = 3 × 10-12 × 100 = 3 × 10-10C
Ans. Capacitance of the capacitor with
air between its plates, q2 = C3V = 4 × 10-12 × 100 = 4 × 10-10 C.

ε0 A 2.8. In a parallel plate capacitor with


= = 8 pF air between the plates, each plate has an
d area of 6 × 10-3 m2 and the distance
When the capacitor is filled with between the plates is 3 mm. Calculate the
dielectric (κ = 6) between its plates and capacitance of the capacitor. If the
the distance between the plates is capacitor is connected to a 100 V supply,
reduced by half, capacitance becomes, what is the charge on each plate of the
capacitor ?
ε 0 κA ε 0 ×6 × A ε0 A
C’ = = =12 = 12 × 8 = Ans. Capacitance of capacitor with air
d
'
d /2 d between its plates is
96 pF.
ε 0 A 8.85 ×10−12 ×6 ×10−3
2.6. Three capacitors each of C0 = = −3
capacitance 9 pF are connected in series, d 3 × 10
(a) What is the total capacitance of the = 1.8 × 10-11F =18 pF.
combination ? (b) What is the potential
difference across each capacitor when the Charge,
combination is connected to a 120 V q = C0V = 1.8 × 10-11 × 100 = 1.8 × 10-9 C.
supply ?
2.9. Explain what would happen if in
Ans. (a) If C is the equivalent the capacitor given in Exercise 2.8, a 3
capacitance of the series combination, mm thick mica sheet (of dielectric constant
then = 6) were inserted between the plates, (i)
1 1 1 1 1 1 1 3 1 while the voltage supply remains
= + + = + + = = connected (ii) after the supply was
C C C2 C 3 9 9 9 9 3
disconnected.
or C = 3 pF. [CBSE Sample Paper 98]
(b) As all the capacitors have equal Ans. From the above question, we have
capacitance, so potential drop ΔV would
be same across each capacitor. C0 = 1.8 × 10-11 F = 18pF, q0 = 1.8 ×
-9
10 C
Also, κ = 6
V = ΔV1 + ΔV2 + ΔV3
= ∆V + ΔV + ΔV = 3∆V

117
(i) When the voltage supply remains Final energy stored,
connected, the potential difference
U f=
i.e., 100 V. The capacitance increases κ
between capacitor plates remains same
1 2 1 −12 2
(C +C )V = (600+ 600)×10 ×(100)
times. 2 1 2 2
∴ C = κC0 = 6 × 18 = 108 pF. = 6 × 10-6J
The charge on the capacitor plates will be Electrostatic energy lost,
q = CV = 108 × 10 -12 × 100 = 1.08 × 10-8 ΔU = Ui - Uf = 12 × 10-6 - 6 × 10-6 = 6 × 10-6
C. J
(ii) After the supply is disconnected, 2.12. A charge of 8 mC is located at
the charge on the capacitor plates the origin. Calculate the work done in
remains same i.e., q0 = 18 × 10-9 C taking a small charge of-2 × 10 -9Cfrom a
The capacitance increases κ times.
point P (0, 0, 3 cm) to a point Q (0, 4 cm,

C = κC0 = 108 pF.


0) via a point R(0, 6 an, 9 cm).
Ans. As the work done in taking a
The potential difference between the charge from one point to another is
capacitor plates becomes independent of the path followed,
therefore
V 0 100

[ ]
V= = = 16.6 V. q q
κ 6 W = q 0 [V Q−V P ]=q0 −
4 π ε0 r2 4 π ε0 r1
2.10. A 12 pF capacitor is connected
to a 50 V battery. How much electrostatic
energy is stored in the capacitor ?
Ans. Here C = 12 pF = 12 × 10 -12
F, V = 50
=
[
q0 q 1 1

4 π ε0 r2 r1 ]
V
Energy stored,
U =
1 2 1 −12 2 −8
CV = ×12 ×10 ×(50) =1.5× 10 J
2 2
.
2.11. A 600 pF capacitor is charged by
a 200 V supply. It is then disconnected
from the supply and is connected to
another uncharged 600 pF capacitor. How
much electrostatic energy is lost in the
process ? Fig. 2.190
Ans. Here C1 = 600 pF, V1 = 200 V, Here q = 8 mC = 8 × 10 3C, q0 = - 2 × 10 9

C2 = 600 pF, V2 = 0 C
Common potential, r1 = 3cm = 3 × 10-2 m, r2 = 4cm = 4 × 10-2m
C1 V 1+ C2 V 2 600 ×10−12 ×200+ 0 ∴ W = -2 × 10-9 × 8 × 10-3 × 9 × 109 ×
=
[ ]
V = −12 = 1 1
C 1+ C 2 (600+ 600)×10 −
−2 −2
100 V 4 ×10 3 ×10
Initial energy stored, = 1.2 J.
U i= 2.13. A cube of side b has a charge q
1 2 1 −12 2 −6
at each of its vertices. Determine the
U 1= C V 1= ×600 ×10 ×(200) =12× 10 J potential and electric field due to this
2 2

118
charge array at the centre of the cube. = √ 325 cm - 18 cm = 0.18 m
Ans. Length of longest diagonal of the
cube
= √ b2 +b 2+ b2 = √ 3 b
Distance of each charge (placed at
vertex) from the centre of the cube is

r=
√3 b
2
∴ Potential at the centre of the cube is Fig. 2.191 (b)
V = Electric potential at point P,
1 q 1 2q 4q q1 q2
8⋅ ⋅ =8 × ⋅ = 1
4 π ε0 r 4 π ε 0 √3 b √ 3 π ε 0 b . V P= . +
4 π ε 0 PA PB
Electric fields at the centre due to any

[ ]
−6 −6
pair of charges at the opposite comers will 9 1.5 ×1 0 2.5 × 10 5
¿ 9 ×1 0 + V =2× 1 0
be equal and opposite thus cancelling out 0.18 0.18
in pairs. Hence resultant electric field at V.
the centre will be zero.
Electric field at P due to charge q 1 ,
2.14. Two tiny spheres carrying
charges 1.5 μC and 2.5 μC are located 30 1 q1 9
9 × 10 × 1.5× 10
−6
E1 = . =
cm apart. Find the potential and electric 4 π ε0 P A 2 ¿¿
field (a) at the midpoint of the line joining
the two charges, and (b) at a point 10 cm Electric field at P due to charge q 2 ,
from this midpoint in a plane normal to the
line and passing through the midpoint. 1 q 2 9 ×1 09 ×2.5 ×1 0−6
E2 = . = =
4 π ε0 P A 2 ¿¿
Ans. (a) 6 −1
0.69 ×1 0 V m
∠APO = ∠BOP = θ/2
In right ΔAOP,
Fig. 2.191 (a) θ 15 θ o
tan = =1.5 ⇒ =56.3
Electric potential at the midpoint O, 2 10 2
∴ θ=2× 5.6 3o=112. 6o
V O=
1 q1 q 2
+
4 π ε0 r1 r2 ( ) The resultant field at P will be

[ ] E=√ E 21+ E22 +2 q E2 cos θ


−6 −6
1.5 ×1 0
9 2.5 × 10 5
¿ 9 ×1 0 + V =2.4 ×1 0
0.15 0.15 =√ ¿ ¿ Vm-1
V.
= √ 0 .1764 +0 . 4761−0.2227 ×1 06 V m−1
Electric field at the midpoint O,
[cos112.6 = -0.3843]
EO =E2−E ┐ ¿
1 q2 q 1

(
4 π ε 0 r 22 r 21 ) ¿ √ 0 .4298× 1 0 V m ≃6.6 ×1 0 Vm-1
6 −1 5

Let the field E make angle α with E1. Then,


9
¿ 9 ×1 0 ¿ E2 sinθ
= 4.0 × 105 Vm-1, from charge 2.5 μC to tan α =
E7 + E 2 cos θ
1.5 μC
=
(b) PA = PB = √ 152 +102 cm

119
6 negative charges with total charge zero.
0.69 ×1 0 × 0.9239 0.688
6 6
= =4.3
0.42× 1 0 +0.069 ×1 0 (−0.3843) 0.10
[sin 112.6° = sin 67.4° = 0.9239]
∴ α = tan (4.3) = 76.9°
-1

Let E make angle β with the direction


from B to A Then from right ΔPOC,
Fig. 2.192 Electric field vanishes inside a
θ cavity of any shape.
β+ α - = 90°
2 To overrule this possibility, consider a
θ closed loop PQRSP, such that part PQR
β = 90°- α + = 90° - 76.9°+ 56.3°= 69.4° is inside the cavity along a line of force
2
and the part RSP is inside the conductor.
Hence, the resultant field E makes an Since the field inside a conductor is zero,
angle of 69.4° with the line joining charge this gives a network done by the field (in
2.5μC to 1.5μC. part RSP) in carrying a test charge over a
2.15. A spherical conducting shell of closed loop. But this is not possible for a
inner radius r1 and outer radius r2 has a conservative field like the electrostatic
charge Q. field. Hence there are no lines of force
(i.e., no field), and no charge on the inner
(a) A charge q is placed at the centre of surface of the conductor, whatever be its
the shell. What is the surface charge shape.
density on the inner and outer surfaces of
the shell ? 2.16. (a) Show that the normal
component of electrostatic field has a
(b) Is the electric field inside a cavity (with discontinuity from one side of a charged
no charge) zero even if the shell is not surface to another given by
spherical, but has any irregular shape ?
Explain. σ
(⃗
E2 – ⃗
E 1).n^ ε
0
Ans. (a) The charge q placed at the
centre of the shell induces a charge - q on where n^ is a unit vector normal to the
the inner surface of the shell and charge + surface at a point and σ is the surface
q on its outer surface. charge density at that point. (The direction
∴ Surface charge density on the inner
of n^ is from side 1 to side 2)
surface of the shell Hence show that just outside a
q conductor, the electric field is σn^ / ε0.
charge
= =- 2 (b) Show that the tangential
surface area 4 πr 1
component of electrostatic field is
Surface charge density on the outer continuous from one side of a charged
surface of the shell surface to another.
Q+ q [Hint : For (a), Use Gauss's law. For
= 2. (b), use the fact that work done by
4 πr 2
electrostatic field on a closed loop is zero.]
(b) Even if the shell is not spherical,
the entire charge resides on its outer Ans. (a) Electric field near a plane
surface. The net charge on the inner sheet of charge is given by
surface enclosing the cavity is zero. From σ
Gauss's theorem, electric field vanishes at E=
2 ε0
all points inside the cavity. For a cavity of
arbitrary shape, this is not enough to claim If n^ is a unit vector normal to the sheet
that electric field inside must be zero. The from side 1 to side 2, then electric field on
cavity surface may have positive and side 2

120
σ ' '
where E1 and E2 are the tangential
E 2 = 2 ε n^

0 components of ⃗
E1 and ⃗
E2, respectively.
in the direction of the outward normal to Thus,
the side 2. ' '
E1 = E2 (∵ l ≠ 0)
Similarly, electric field on side 1 is
Hence the tangential component of the
σ electrostatic field is continuous across the
E 1 = - 2 ε n^

surface.
0

charged density λ is surrounded by a


in the direction of the outward normal to 2.17. A long charged cylinder of linear
the side 1.
hollow co-axial conducting cylinder. What
∴ (⃗ ( )
σ −σ σ is the electric field in the space between
E2 - ⃗
E 1).n^ = −
2 ε0 2 ε0
=
ε0 the two cylinders ?

As ⃗ Ans. Refer answer to Q. 35 on page


E 1 and ⃗
E 2 act in opposite directions,
2.30.
there must be discontinuity at the sheet of
charge. Now electric field vanishes inside 2.18. In a hydrogen atom, the electron
a conductor, therefore and proton are bound at a distance of
about 0.53 Å.

E1 = 0
(i) Estimate the potential energy of the
Hence outside the conductor, the electric system in eV, taking the zero of
field is potential energy at infinite separation
σ of the electron from proton.

E= ⃗
E 2 = ε n^
0 (ii) What is the minimum work required to
free the electron, given that its kinetic
(b) Let XY be the charged surface of a
energy in the orbit is half the
dielectric and ⃗
E 1 and ⃗
E 2 be the electric magnitude of potential energy
fields on the two sides of the charged obtained in (i) ?
surface as shown in Fig. 2.193.
(iii) What are the answers to (i) and (ii)
above if the zero of potential energy is
taken at 1.06 A separation ?
Ans. (i) q1 = -1.6 × 10-19 C, q2 = + 1.6 × 10-
19
C,
r = 0.53 Å = 0.53 × 10-10 m
P.E. of the electron-proton system will be
1 q 1 q2
U= ⋅
4 π ε0 r
Fig. 2.193 −19 −19
(−1.6 × 10 )×1.6 × 10
Consider a rectangular loop ABCD
= 9 × 10 × 9
J
0.53 ×10−10
with length 1 and negligibly small breadth.
−19
Line integral along the closed path ABCD 9 ×1.6 ×1.6 × 10
will be = - −19
eV ≃−27.2 eV.
0.53 ×1.6 ×10
∫⃗
E.⃗
dl= ⃗ ⃗ ⃗
E 1 l− E 2 . ⃗l=0 (ii) K.E. of the electron in the orbit
or E1l cos θ1 – E2l cos θ2 = 0 1 1
= P.E. = ×27.2eV = 13.6eV 2 2
2 2
∴ Total energy of the electron
(E1 cos θ1 – E2 cos θ2) l = 0
' '
(E1 −E2)l=0

121
= P.E. + K.E. Fig. 2.194
= (- 27.2 + 13.6) eV = -13.6 eV. Charge on an electron,
As minimum energy of the free q1 = - e = - 1.6 × 10-19 C
electron is zero, so minimum work
Charge on each proton,
required to free the electron
cj2 = q3 = + e = + 1.6 × 10-19 C
= 0 - (-13.6) = 13.6 eV.
If the zero of potential energy is taken
(iii) When the zero of potential energy
at infinity, then potential energy of the
is not taken at infinity, the potential energy
system is
of the system is
U =
U=
q 1 q2 1 1

4 π ε0 r1 r2[ ] U 12 +U 23 +U 13=
[
1 q1 q2 q 2 q3 q 1 q3
4 π ε0 r1
+
r2
+
r3 ]
9 −19 −19
¿ 9 ×1 0 ×(−1.6 ×1 0 )× 1.6 ×1 0

×
[ 1
0.53 × 10
−10

1
1.06 ×1 0
−10
J
]
=
1
[ (−q ) q
4 π ε 0 1 ×10 −10
+
q ⋅q
1.5× 10 −10
+
(−q ) q
1 ×10−10 ]
[ ]
2
e 1
−1+ −1 [q = e]
[ ]
9 −1.9
9× 10 × 1.6 ×1 0 ×1.6 × 10
−19
1 = −10
¿− 1− eV 4 π ε 0 ×10 1.5
−19 −10
1.6 ×1 0 ×053 × 10 2
−19 2

( )
9
9.× 1.6 (1.6 ×10 ) ×9 ×10 −4
¿− eV =−13.6 eV = −10
× J
053× 2 10 3
This indicates that the K.E. of 13.6 eV = -
−19 2 9
of case (i) is used up in increasing the (1.6 ×10 ) ×9 ×10 × 4
P.E. from -27.2 eV to - 13.6 eV as the eV=−19.2eV
electron is carried from 0.53 Å to 1.06 Å
1.6 ×10−19 ×3
position. K.E. in this situation should be [∵ 1 eV = 1.6 × 10-19 J]
zero. As the total energy in this case is
2.20. Two charged conducting spheres
zero, therefore, minimum work required to
of radii a and b are connected to each
free the electron
other by a wire. What is the ratio of
= 0 - (-13.6eV)=13.6 eV. electric fields at the surfaces of the two
2.19. If one of the two electrons of a H 2 spheres ? Use the result obtained to
molecule is removed, we get a hydrogen explain why charge density on the sharp
molecular ion (H2+). In the ground state of and pointed ends of a conductor is higher
a H2 ion, the two protons are separated by than that on its flatter portions.
roughly 1.5 Å, and the electron is roughly Ans. The charges will flow between
1 Å from each proton. Determine the the two spheres till their potentials
potential energy of the system. Specify become equal. Then the charges on the
your choice of the zero of potential two spheres would be
energy.
Q1 C 1 V C1
Ans. The system of charges is shown = =
Q2 C 2 V C2
in Fig. 2.194.
C1 a
But =
C2 b


Q1 a
=
Q2 b
The ratio of the electric fields at the
surface of the two spheres will be

122
1 Q1
⋅ 2
E1 4 π ε 0 a Q 1 b 2 a b2 b
= = ⋅ = ⋅ =
E2 1 Q 2 Q 2 a2 b a2 a

4 π ε0 b 2
E1 σ 1
Also, =
E2 σ 2


σ1 b
=
σ2 a
Thus the surface charge densities are
inversely proportional to the radii of the
spheres. Since the flat portion may be
considered as a spherical surface of large
radius and a pointed portion as that of
small radius, that is why, the surface
charge density on the sharp and pointed
ends of a conductor is much higher than
that on its flatter portion. Fig. 2.195,

2.21. Two charges-q and + q are When the point P lies closer to charge
located at points (0, 0, -a) and (0, 0, α) - q, as shown in Fig. 2.195(b), it can be
respectively. easily seen that

(i) What is the electrostatic potential at the 1 p


V=- ⋅ 2 2
points (0, 0, z) and (x, y, 0) ? 4 π ε 0 z −a
(ii) Obtain the dependence of potential on Again, any point (x, y, 0) lies in XY-
the distance r of a point from the origin plane which is perpendicular bisector of Z-
when r / a >> 1 axis. Such a point will be at equal
distances from the charges - q and + q.
(iii) How much work is done in moving a
Hence potential at point (x, y, 0) will be
small test charge from the point (5, 0,
zero.
0) to (-7,0,0) along the x-axis ?
(ii) If the distance of point P from the
Does the answer change if the path of
origin O is r, then from the results of part
the test charge between the same points
(i), we get
is not along the x-axis ?
Ans. (i) When the point P lies closer to
1 p
V=± ⋅ 2 2 [Put z = r]
the charge + q as shown in Fig. 2.195(A), 4 π ε 0 z −a
the potential at this point P will be If r >> a, we can neglect a 2 compared to
V = r2, so
1
[
q q
− =
]
1 q
[

q
4 π ε 0 r 1 r 2 4 π ε 0 z−a z−(−a ) ] V=±
1 p
⋅ 2
4 π ε0 r
q

2a ∴ For r >> a, the dependence of potential
=
4 π ε 0 z 2−a2 V on r is 1/ r2 type.
1 p (iii) (5, 0, 0) and (- 7, 0, 0) are the
or V = ⋅ [∵ p = q × 2a] points on the X-axis i.e., these points lie
4 π ε 0 z 2−a2
on the perpendicular bisector of the
dipole. Each point is at the same distance
from the two charges. Hence electric
potential at each of these points is zero.

123
Work done in moving the test charge him each of which can withstand a
q0 from the point (5, 0, 0) to (-7, 0, 0) is potential difference of not more than 400
V. Suggest a possible arrangement that
W = q(V1 - V2) = q(0 - 0) = 0.
requires a minimum number of capacitors.
No, the answer will not change if the
Ans. Let this arrangement require n
path of the test charge between the same
capacitors of 1 μF each in series and m
two points is not along X-axis. This is
such series combinations to be connected
because the work done by the
in parallel.
electrostatic field between two points is
independent of the path connecting the P.D. across each capacitor of a series
two points. combination
2.22. Figure 2.196 below shows a 1000 1000
charge array known as an electric = = 400 or n = = 2.5
n 400
quadrupole. For a point on the axis of the
quadrupole, obtain the dependence of But number of capacitors cannot be a
fraction,
∴n=3
potential on r for r >> a. Contrast your
result with that due to an electric dipole
and an electric monopole (i.e. a single
charge). Equivalent capacitance of the combination
is
1
. m = 2 or m = 2n = 6
n
Fig. 2.196 ∴ Total number of capacitors required
Ans. Potential at point P is = 3 × 6 = 18

[ ]
1 q q q q So six series combinations, each of
V= − − + three 1 μF capacitors, should be
4 π ε 0 r−a r r r + a
connected in parallel as shown in Fig.
= 2.197.
1
4 π ε0
⋅q
[
r ( r +a )−2 ( r−a )( r + a ) +r ( r−a )
r ( r −a ) ( r +a ) ]
[ ]
2 2 2 2
1 r +ar −2r +2 a + r −ar
= ⋅q
4 π ε0 r ( r 2−a 2)
2
1 2qa 1 Q
= ⋅ = ⋅
4 π ε 0 r (r 2−a 2) 4 π ε 0 r (r 2−a 2)
where Q = 2q a2 is the quadrupole
moment of the given charge distribution.
As r >> a, so we can write
1 Q
V= ⋅
4 π ε0 r3
Hence for large r, quadrupole potential
varies as 1 / r3, whereas dipole potential
varies as 1 / r2 and monopole potential
Fig. 2.197
varies as 1 / r.
2.24. What is the area of the plates of
2.23. An electrical technician requires
a 2 F parallel plate capacitor ? Given that
a capacitance of 2 μf in a circuit across a
the separation between the plates is 0.5
potential difference of 1 kV. A large
cm
number of 1 μP capacitors are available to

124
Ans. Here C = 2F, d = 0.5 cm = 5 × 10-3 m P.D. between points A and B
ε0 A = V - V4 = (300 - 200) V = 100 V
∴ V1 = 100 V
As C =
d

∴A=
−3
Cd 2 ×5 ×10 q1 = ClV1= 100 × 10-12 × 100 = 10-8 C
= m2
ε 0 8.85 ×10−12 Also the P.D. across the series
≃ 1130 ×10 m =1130 km .
6 2 2 combination of C2 and C3

2.25. Obtain the equivalent capacitance of = 100 V


the network shown in Fig. 2.198. For a Now since C2 = C3

∴ V 2 = V3 =
300 V supply, determine the charge and
voltage across each capacitor.
100
= 50 V
[CBSE OD 08] 2
and q2 = q3 = 200 × 10-12 × 50 = 10-8 C.
2.26. The plates of a parallel plate
capacitor have an area of 90 cm2 each
and are separated by 2.5 mm The
capacitor is charged by connecting it to a
400 V supply.
(i) How much energy is stored by the
capacitor ?
(ii) View this energy stored in the
Fig. 2.198
electrostatic field between the plates
Ans. As C2 and C3 are in series, their and obtain the energy per unit volume
equivalent capacitance u. Hence arrive at a relation between u
and the magnitude of electric field E
C 2 C 3 200 ×200
= = =100 pF between the plates.
C2 +C 3 200+200
Ans. (i) Here A = 90 cm 2 = 90 × 10-4 m2
Series combination of C2 and C3 is in = 9 × 10-3m2
parallel with C1, their equivalent
d = 2.5 mm = 2.5 × 10-3m,
capacitance
ε0 = 8.85 × 10-12Fm-1,
= 100 pF+ 100 pF = 200 pF
V = 400 V
The combination of C1, C2 and C3 is in
series with C4, equivalent capacitance of Capacitance of the parallel plate capacitor
the network is
200× 100 200 ε 0 A 8.85 ×10−12 ×9 ×10−3
= pF= pF C= = F
200+100 3 d 2.5 × 10
−3

Total charge on the network is = 31.86 × 10-12 F = 31.86 pF.


200 Electrostatic energy stored by the
q = cV = × 10-12 × 300 = 2 × 10-8 C
3 capacitor,
This must be equal to charge on C4 1 1
and also to the sum of the charges on the U= CV2 = × 31.86 × 10-12 × (400)2 J
2 2
combination of C1, C2 and C3.
∴ q4 = q = 2 × 10 C
= 25488 × 10-8 J = 2.55 × 10-6 J.
-8
(ii) Energy stored per unit volume or
q4 2× 10
−8
energy density of the capacitor is
V4 = = V=200 V
C4 100 ×10−12

125
−6 2.28. Show that the force on each
U 2.55 ×1 0
u= = Jm −3 plate of a parallel plate capacitor has a
Ad 9 ×1 0−3 × 2.5× 10−3
1
¿ 0.113 Jm −3. magnitude equal to qE, where q is the
2
The relation between u and E can be charge on the capacitor, and E is the
arrived at as follows: magnitude of electric field between the
1
1 plates. Explain the origin of the factor .
CV
2 2
( )
2 2
U 2 1ε A V 1 V
u= = = 0 . = ε0 Ans. Let A be the plate area and σ, the
Ad Ad 2 d Ad 2 d surface charge density of the capacitor.
1 Then
or u= ε 0 E2 .
2 q = σA
2.27. A 4 μF capacitor is charged by a 200 σ
V supply. It is then disconnected from the E=
ε0
supply and is connected to another
uncharged 2 μF capacitor. How much Suppose we increase the separation
electrostatic energy of the first capacitor is of the capacitor plates by small distance
lost in the form of heat and ∆x against the force F. Then work done by
electromagnetic radiation 7 [CBSE OD 05] the external agency = F. ∆x
Ans. Initial electrostatic energy of the 4 If u be the energy stored per unit
μ F capacitor is volume or the energy density of the
1 1 capacitor, then increase in potential
2 −6
U i= CV = × 4 × 10 × ¿ energy of the capacitor
2 2
= u × increase in volume = u. A. ∆x
∴ F.∆x = u.A.∆x
−2
¿ 8 ×1 0 J
Charge on 4 μF capacitor
1 1
−6
¿ CV =4 ×1 0 ×200=8 ×1 0 C
−4 or F = uA = ε0 E2 . A = (ε0E) A E
2 2
When the 4 μF and 2μF capacitors are 1 1
connected togetherJ both attain a = .σA.E = qE
2 2
common potential V. Thus
−4 1
Total charge 8 ×1 0 C 400 The physical origin of the factor in
V= = −6
= V 2
Tota l capacitance (4+2)×1 0 F 3 the force formula lies in the fact that just
Final electrostatic energy of the inside the capacitor, field is E, and outside
combination, it is zero. So the average value E / 2
contributes to the force.

( )
2
1 −6 400
U f = × ( 4+ 2 ) × 10 × J 2.29. A spherical capacitor consists of
2 3 two concentric spherical conductors, held
16 −2
in position by suitable insulating supports
¿ ×1 0 J (Fig. 2.199). Show that the capacitance of
3
a spherical capacitor is given by
= 5.33 × 10-2 J
4 π ε0 r1 r2
Electrostatic energy of the first C=
r 1−r 2
capacitor lost in the form of heat and
electromagnetic radiation is where r1 and r2 are the radii of outer and
inner spheres, respectively.
∆U = Ui - Uf = (8 - 5.33) × 10-2 J
= 2.67 × 10-2 J.

126
When an earthed conductor is placed
near a charged conductor, the
capacitance of the latter increases. The
two conductors form a capacitor. But the
capacitance of an isolated conductor is
always small.
2.31. Answer carefully :
(i) Two large conducting spheres carrying
charges Q1 and Q2 are brought close
Fig. 2.199 to each other. Is the magnitude of
Ans. Refer answer to Q. 34 on page electrostatic force between them
2.30. Q1 Q2
exactly given by 2, where r is
2.30. A spherical capacitor has an 4 π ε0 r
inner sphere of radius 12 an and an outer the distance between their centres ?
sphere of radius 13 cm. The outer sphere (ii) If Coulomb's law involved 1/r3
is earthed and the inner sphere is given a dependence (instead of 1/r2), would
charge of 2.5 μC. The space between the Gauss' law be still true ?
co-centric spheres is filled with a liquid of
dielectric constant 32. (a) Determine the (iii) A small test charge is released at rest
capacitance of the capacitor, (b) What is at a point in an electrostatic field
the potential of the inner sphere ? (c) configuration. Will it travel along the
Compare the capacitance of this capacitor line of force passing through that
with that of an isolated sphere of radius 12 point ?
cm. Explain why the latter is much (iv) What is the work done by the field of a
smaller. nucleus in a complete circular orbit of
Ans. Here a = 12 cm = 12 × 10-2m, the electron ? What if the orbit is
elliptical ?
b = 13 cm = 13 × 10-2cm,
(v) We know that electric field is
q = 2.5 μC = 2.5 × 10-6C, κ = 32 discontinuous across the surface of a
(a) Capacitance of the spherical capacitor charged conductor. Is electric potential
is also discontinuous there ?
ab (vi) What meaning would you give to the
C = 4 π ε0 κ ⋅ capacity of a single conductor ?
b−a
−2 −2 (vii)Guess a possible reason why water
32 12 ×10 ×13 ×10
= 9
⋅ −2 F has a much greater dielectric constant
9 ×10 (13−12)× 10 (= 80) than say, mica (= 6).
32× 12×13 Ans. (i) No. When the two spheres are
= × 10-11 F = 5.5 × 10-9 F.
9 brought close to each other, their charge
distributions do not remain uniform and
(b) Potential of the inner sphere is they will not act as point charges.
−6
q 2.5 ×10 (ii) No. Gauss's law will not hold if
V= = V = 0.45 × 103 V = 4.5
C 5.5 ×10−9 Coulomb's law involved 1/r3 or any
×102 other power of r (except 2). In that
case the electric flux will depend upon
(c) Capacitance of the isolated sphere of r also.
radius 12 cm is
−2
(iii) Not necessarily. The small test charge
12× 10 will move along the line of force only if
C = 4πε0R = 9
F = 1.3 × 10-11 F.
9 ×10 it is a straight line. The line of force
gives the direction of acceleration, and
not that of velocity.

127
(iv) Zero. But when the orbit is elliptical, What minimum area of the plates is
work is done in moving the electron required to have a capacitance of 50 pF ?
from one point to the other. However, [CBSE OD 05]
net work done over a complete cycle is
Ans. Maximum permissible voltage
zero, (z;) No, potential is everywhere
constant as it is a scalar quantity. = 1 kV = 103 V
(vi) A single conductor is a capacitor with Maximum permissible electric field
one plate at infinity. It also possesses = 10% of 107 Vm-1 = 106 Vm-1
∴ Minimum separation d required between
capacitance.
(vii)Because of its bent shape and the
the plates is given by
presence of two highly polar O - H
3
bonds, a water molecule possesses a V V 10 −3
large permanent dipole moment about E= or d = = 6 =10 m
d E 10
0.6 × 10-29 Cm. Hence water has a
large dielectric constant. Capacitance of a parallel plate capacitor is
2.32. A cylindrical capacitor has κ ε0 A
two co-axial cylinders of length 15 cm and C=
d
radii 1.5 cm and 1.4 cm The outer cylinder
∴A=
−12 −3
is earthed and the inner cylinder is given a Cd 50 ×10 ×10 2
= m
charge of 3.5 μC. Determine the κ ε 0 3 × 8.85× 10−12
= 18.8 × 10-4m2 ≃ 19 cm2.
capacitance of the system and the
potential of the inner cylinder. Neglect end
effects (i.e., bending of field lines at the 2.34. Describe schematically the
ends). equipotential surfaces corresponding to
Ans. Here L = 15 cm = 0.15 m, q = 3.5 (i) a constant electric field in the Z-
μC = 3.5 × 10-6 C, a = 1.4 cm = 0.014 m, b direction.
=1.5 cm =0.015 m
(ii) a field that uniformly increases in
Capacitance of a cylindrical capacitor magnitude but remains in a constant
is given by (say, Z) directions.
2 π ε0 L L (iii) a single positive charge at the origin.
=
C= b 1 b (iv) a uniform grid consisting of long
ln 2 2.303 log
a 4 π ε0 a equally spaced parallel charged wires
0.15 in a plane.
= 0.015 F Ans. (i) For a constant electric field in
2× 9 ×109 ×2.303 log Z-direction, equipotential surfaces will be
0.014
planes parallel to XY-planes, as shown in
−9
0.15 ×10 Fig. 2.200.
= F
18× 2.303 ×0.03
= 0.1206 × 10-9 F = 1.2 × 10-10 F
Potential,
−6
q 3.5 ×10
V= = V = 2.9 ×104 V.
C 1.2× 10−10
2.33. A parallel plate capacitor is to be
designed with a voltage rating 1 kV, using
a material of dielectric constant 3 and
dielectric strength about 107 Vm-1. For
safety, we would like the field never to
exceed say 10% of the dielectric strength. Fig. 2.200

128
(ii) In this case also, the equipotential (i) The top of the atmosphere is at
surfaces will be planes parallel to XY- about 400 kV with respect to the surface
plane. However, as field increases, such of the earth, corresponding to an electric
planes will get closer. field that decreases with altitude. Near the
surface of the earth, the field is about 100
(iii) For a single positive charge at the
Vm-1. Why do then we not get an electric
origin, the equipotential surfaces will be
shock as we step out of our house into the
concentric spheres having origin as their
open ? (Assume the house to be a steel
common centre, as shown in Fig. 2.25.
cage so there is no field inside.)
The separation between the equipotentials
differing by a constant potential increases (ii) A man fixes outside his house one
with increase in distance from the origin. evening a two metre high insulating slab
carrying on its top a large aluminium sheet
(iv) Near the grid the equipotential
of area 1 m2. Will he get an electric shock
surfaces will have varying shapes. At far
if he touches the metal sheet next morning
off distances, the equipotential surfaces
?
will be planes parallel to the grid.
(iii) The discharging current in the
2.35. In a Van de Graaff type
atmosphere due to the small conductivity
generator, a spherical metal shell is to be
of air is known to be 1800 A on an
a 1.5 × 106 V electrode. The dielectric
average over the globe. Why then does
strength of the gas surrounding the
the atmosphere not discharge itself
electrode is 5 × 107 Vm-1. What is the
completely in due course and become
minimum radius of the spherical shell
electrically neutral ? In other zoords, what
required ? [CBSE OD 08]
keeps the atmosphere charged ?
Ans. Maximum permissible potential, V =
(iv) What are the forms of energy into
1.5 × 106 V
which the electric energy of the
For safety, the maximum permissible atmosphere is dissipated during a
electric field is E = 10% of dielectric lightning ?
strength
Ans. (i) Normally the equipotential
= 10% of 5 × 107 Vm-1 = 5 × 106 Vm-1 surfaces are parallel to the surface of the
Now for a spherical shell, earth as shown in Fig. 2.201. Now our
body is a good conductor. So as we step
1 q out into the open, the original equipotential
V= ⋅
4 π ε0 r surfaces of open air get modified, but
keeping our head and the ground at the
1 q V same potential and we do not get any
E= ⋅ 2=
4 π ε0 r r electric shock.
∴ Minimum radius required is
6
V 1.5 × 10 V
r = = = 3 × 10-1 m = 30
E 5 ×10 6 Vm−1
cm.
2.36. A small sphere of radius r 1 and
charge q1 is enclosed by a spherical shell
of radius r2 and charge q2. Show that if q1
is positive, charge will necessarily flow
from the sphere to the shell (when the two
are connected by a wire) no matter what Fig. 2.201
the charge q2 on the shell is.
(ii) Yes. The aluminium sheet and the
Ans. Refer answer to Q. 55 on page ground form a capacitor with insulating
2.67. slab as dielectric. The discharging current
2.37. Answer the following : in the atmosphere will charge the

129
capacitor steadily and raise its voltage. an equilibrium with the discharge of the
Next morning, if the man touches the atmosphere in ordinary weather
metal sheet, he will receive shock to the conditions.
extent depending upon the capacitance of
(iv) The electrical energy is lost as (i)
the capacitor formed.
light energy involved in lightning (ii) heat
(iii) The atmosphere is continuously and sound energy in the accompanying
being charged by thunder storms and thunder.
lightning bolts all over globe and maintains

Text Based Exercises


TYPE A : VERY SHORT ANSWER QUESTIONS (1 mark each)

1. Define electric potential. Is it a scalar 13. How much is the electric potential of a
or a vector quantity ? [Punjab 01 ; charge at a point at infinity ?
CBSE OD 06]
14. What is the nature of symmetry of the
2. Define the unit of electric potential. potential of a point charge ?
[Punjab 02]
15. What are the points at which electric
3. Write down the relation between potential of a dipole has maximum
electric field and electric potential at a value ?
point.
16. What are the points at which electric
4. Name the physical quantity whose SI, potential of a dipole has a minimum
unit is JC-1. Is it a scalar or a vector value ?
quantity ? [CBSE OD 2010]
17. What is the nature of symmetry of a
5. Write the SI unit of potential gradient. dipole potential ?
6. Define electric potential difference 18. What is electrostatic potential energy ?
between two points. Is it scalar or Where does it reside ?
vector ? [Punjab 01]
19. What is the value of the angle between
7. What do you mean by a potential the vectors ⃗p and ⃗ E for which the
difference of 1 volt ? potential energy of an electric dipole of
8. Write the dimensional formula of dipole moment ⃗p, kept in an external
potential difference. electric field ⃗
E , has maximum value ?
[CBSE SP 15]
9. 5 J of work is done in moving a
positive charge of 0.5 C between two 20. Write an expression for potential at
points. What is the potential difference point P(r⃗ ) due to two charges q 1 and
between these two points ? [ISCE 95] q2 located at positions ⃗ r 1 and ⃗ r2
10. A charge of 2 C moves between two respectively.
points maintained at a potential 21. Define electron volt. How is it related
difference of 1 volt. What is the energy to joule ?
acquired by the charge ? [CBSE D
10C] 22. How many electron volts make up one
joule ? [Himachal 93]
11. In a conductor, a point P is at a higher
potential than another point Q. In 23. Will there be any effect on the
which direction do the electrons potential at a point if the medium
move ? around this point is changed ?

12. Give two examples of conservative 24. What work must be done in carrying
forces. [Himachal 93] an α-particle across a potential
difference of 1 volt ?

130
25. What is an equipotential surface ? 41. Write the dimensions of capacitance.
Give an example. [Punjab 2000, 02 ;
42. What is the net charge on a charged
CBSE D 03]
capacitor ?
26. Why are electric field lines
43. On what factors does the capacitance
perpendicular at a point on an
of a capacitor depend ?
equipotential surface of a conductor ?
[CBSE OD 15C] 44. Write two applications of capacitors in
electrical circuits.
27. Can you say that the earth is an
equipotential surface ? 45. In what form is the energy stored in a
charged capacitor ?
28. What is the geometrical shape of
equipotential surfaces due to a single 46. What is the basic purpose of using a
isolated charge ? [CBSE D13] capacitor ?
29. What is the shape of the equipotential 47. Write different expressions for the
surfaces for a uniform electric field ? energy stored in a capacitor.
[Punjab 01] 48. Write down the expression for the
30. How much work is done in moving a capacitance of a spherical capacitor.
500 μC charge between two points on 49. The difference between the radii of the
an equipotential surface ? [CBSE D two spheres of a spherical capacitor is
02] increased. Will the capacitance
31. A charge of + 1C is placed at the increase or decrease ? [Punjab
centre of a spherical shell of radius 10 2000]
cm. What will be the work done in 50. What is a dielectric ?
moving a charge of +1 μC on its
surface through a distance of 5 cm ? 51. Define dielectric constant in terms of
the capacitance of a capacitor.
32. What is the optical analogue of an [CBSE D 06]
equipotential surface ?
52. Write down the relation between
33. The middle point of a conductor is dielectric constant and electric
earthed and its ends are maintained at susceptibility.
a potential difference of 220 V. What is
the potential at the ends and at the 53. Write a relation for polarisation ⃗
P of a
middle point ? dielectric material in the presence of
34. Define capacitance of a conductor. an external electric field ⃗
E . [CBSE OD
15]
35. Can there be a potential difference
between two conductors of same 54. Define dielectric strength of a medium.
volume carrying equal positive What is its value for vacuum ?
charges ? 55. Where is the knowledge of dielectric
36. The capacitance of a conductor is 1 strength helpful ?
farad. What do you mean by this 56. What is the effect of temperature on
statement ? dielectric constant ?
37. What is a capacitor ? [Punjab 96C] 57. An air capacitor is given a charge of 2
38. Write the physical quantity that has its μC raising its potential to 200 V. If on
unit coulomb volt-1. Is it a vector or inserting a dielectric medium, its
scalar quantity ? [CBSE D 93C, 98] potential falls to 50 V, what is the
dielectric constant of the medium ?
39. Define capacitance. Give its SI unit.
[CBSE D 93C ; ISCE 98]
40. Define SI unit of capacitance.
[CBSE OD 94]

131
58. An uncharged insulated conductor A is Fig. 2.202
brought near a charged insulated
68. Define the term 'potential energy' of
conductor B. What happens to the
charge ‘q’ at a distance ‘r’ in an
charge and potential of B ? [CBSE OD
external electric field. [CBSE OD 09]
01C]
69. What is the work done in moving a test
59. For a given potential difference, does
charge q through a distance of 1 cm
a capacitor store more or less charge
along the equatorial axis of an electric
with a dielectric than it does without a
dipole ? [CBSE OD 09]
dielectric ?
70. What is the electrostatic potential due
60. Can we place a parallel plate capacitor
to an electric dipole at an equatorial
of 1 F capacity in our house ?
point ? [CBSE OD 09]
61. What is the basic difference between a
71. A metal plate is introduced between
capacitor and an electric cell ?
the plates of a charged parallel plate
62. Two capacitors of capacitances C1 and capacitor. What is its effect on the
C2 are connected in parallel. A charge capacitance of the capacitor ? [CBSE
q is given to the combination. What will F 09]
be the potential difference across each
72. A hollow metal sphere of radius 5 cm
capacitor ?
is charged such that the potential on
63. What is the order of capacitances its surface is 10 V. What is the
used in a radio receiver ? potential at the centre of the sphere ?
[CBSE OD 11]
64. Is there any conductor which can take
unlimited charge ? 73. In which orientation, a dipole placed in
a uniform electric field is in (i) stable,
65. A parallel plate capacitor with air
(ii) unstable equilibrium ? [CBSE OD
between the plates has a capacitance
08 ; D 10]
of 8 pF. What will be the capacitance if
the distance between the plates be 74. Write the expression for the work done
reduced by half and the space on an electric dipole of dipole moment

substance of dielectric constant κ =


between them is filled with a ⃗p in turning it from its position of stable
equilibrium to a position of unstable
6 ? [CBSE D 05] equilibrium in a uniform electric field ⃗ E.
66. A 500 μC charge is at the centre of a [CBSE D13C]
square of side 10 cm. Find the work 75. Two charges 2 μC and -2 μC are
done in moving a charge of 10 μC placed at points A and B, 5 cm apart.
between two diagonally opposite Depict an equipotential surface of the
points on the square. [CBSE D 08] system. [CBSE D13C]
67. The graph of Fig. 2.202, shows the 76. What is the equivalent capacitance, C,
variation of the total energy (E) stored of the five capacitors, connected as
in a capacitor against the value of the shown in Fig. 2.203 ? [CBSE Sample
capacitance (Q itself. Which of the two Paper 2011]
the charge on the capacitor or the
potential used to charge it is kept
constant for this graph ? [CBSE SP
08]

Fig. 2.203
77. A charge ‘q’ is moved from a point A
above a dipole of dipole moment' p’ to

132
a point B below the dipole in equatorial [CBSE ODBC, 16]
plane without acceleration. Find the
79. A point charge Q is placed at point O
work done in the process.[CBSE OD
as shown in the figure 2.205. The
16]
potential difference VA - VB positive. Is
the charge Q negative or positive ?
[CBSE F 16]

Fig. 2.205
80. A point charge Q is placed at point 'O'
Fig. 2.204 as shown in Fig. 2.205. Is the potential
78. What is the amount of work done in at point A, i.e., VA, greater, smaller or
moving a charge Q around a circular equal to potential, VB, at point B, when
arc of radius r at the centre of which Q is (i) positive, and (ii) negative
another point charge q is located ? charge ? [CBSE F 17]

Answers

1. Refer to point 3 of Glimpses on page 14. The potential of a point charge is


2.113. spherically symmetric.
2. Refer to point 4 of Glimpses on page 15. At axial points, the electric potential of
2.113. a dipole has a maximum positive or
negative value.
−dV
3. E= . 16. At equatorial points, the electric
dr
potential of a dipole is zero.
4. Electric potential or potential
difference. It is a scalar quantity. 17. The dipole potential is cylindrically
symmetric.
5. SI unit of potential gradient = Vm-1.
18. The electrostatic potential energy of a
6. Refer to point 1 of Glimpses on page system of charges may be defined as
2.113. the work required to be done to bring
7. Refer to point 2 of Glimpses on page the various charges to their respective
2.113. positions from infinity.
8. Potential difference 19. P.E. = - pEcos θ. Clearly, P.E. is
maximum when cos θ = -1 or θ = 180°.
=

[ q1 q2
]
2 −2 2 −2
work done ML T ML T 2 −3 −1 1
= = =[ML T A ] 20. v(r⃗ ) = +
charge C AT 4 π ε 0 |r⃗ −⃗
r 1| |⃗r −⃗r 2|
W 5J 21. Electron volt is the potential energy
9. V = = = 10 V.
q 0.5 C gained or lost by an electron in moving
10. Energy acquired by the charge = qV = through a potential difference of one
2 C × 1 V = 2J. volt.

11. From Q to P. 1 electron volt = 1eV = 1.6 × 10-19 J

12. (i) Electrostatic force, (ii) Gravitational 22. 1J = 6.25 × 1018 eV.
force. 23. Yes. If the dielectric constant of the
13. Zero. medium is increased, the electric
potential will decrease.

133
24. W = q∆V = 2e ∆V defined as the charge required to be
supplied to either of the conductors so
= 3.2 × 10-19 C × 1V = 3.2 × 10-19 J.
as to increase the potential difference
25. Any surface which has same electric between them by unit amount.
potential at every point is called an
40. The SI unit of capacitance is farad (F).
equipotential surface. The surface of a
A capacitor has a capacitance of 1 F if
charged conductor is an equipotential
1 coulomb of charge is transferred
surface.
from its one plate to another on
26. If it were not so, the presence of a applying a potential difference of 1 volt
component of the field along the across the two plates.
surface would destroy its equipotential 2
nature. 1C 1C 1C2 1(As)
41. As 1 F = = = =
1 V 1 J /C 1 J 1 Nm
27. Yes. Earth is a conductor, so its
∴ [Capacitance] =
2 2
surface is equipotential. A T
−2 = [M-1L-2T4A2].
28. For a point charge, the equipotential ML L
surfaces are concentric spherical 42. Zero, because the two plates have
shells with their centre at the point equal and opposite charges.
charge.
43. The capacitance of a capacitor
29. For a uniform electric field, the depends on the geometry of the
equipotential surfaces are parallel plates, distance between them and the
planes perpendicular to the direction of nature of the dielectric medium
the electric field. between them.
30. Zero. W = q ∆V = 500 μC × 0 = 0. 44. (i) Capacitors are used in radio circuits
31. Zero. This is because the surface of for tuning purposes.
the spherical shell will be an (ii) Capacitors are used in power supplies
equipotential surface. for smoothening the rectified current.
32. Wavefront. 45. In a charged capacitor, energy is
33. The potential at the middle point of the stored in the form of electrostatic
conductor is zero and that at the ends potential energy in the electric field
+ 110 V and - 110 V, so that the p.d. at between its plates.
ends = 110 - (- 110) = 220 V. 46. To store charge and electric energy.
34. The capacitance of a conductor may
1 1 Q2 1
be defined as the charge required to 47. U = CV2 = = QC.
raise its potential by unit amount. 2 2 V 2
35. Yes. Two conductors of same volume ab
48. C = 4πε0 . , where a and b are
but of different shapes will have b−a
different capacitances. the radii of the inner and outer spheres
36. A conductor is said to have a respectively.
capacitance of 1 farad, if 1 coulomb of 49. The capacitance will increase.
charge increases its electric potential
50. A dielectric is essentially an insulator
through 1 volt.
which allows electric induction to take
37. A capacitor is a device to store electric place through it but does not permit
charge. It consists of two conducting the flow of charges through it.
plates separated by an insulating
medium.
38. Capacitance has its unit coulomb volt-
1
. It is a scalar quantity.
39. The capacitance of a capacitor may be

134
51. The ratio of the capacitance (Cd) of the capacitor.
capacitor completely filled with the
Net capacitance, C = C1 + C2
∴ P.D. across each capacitor, V =
dielectric material to the capacitance
(Cv) of the same capacitor with
vacuum between its plates is called q q
dielectric constant.
= .
C C1 +C 2
Cd 63. In the power supply, it is 1 - 10 μF and
κ=
Cv for tuning purposes, it is 100 μF.
52. κ=1+ χ , where κ is dielectric constant 64. Yes, the earth because of its large
and % is electric susceptibility. capacitance can take unlimited
charge.
P = ε0 χ e ⃗
53. ⃗ E.
65. With air between the capacitor plates,
54. The maximum value of electric field
ε0 A
that can exist inside a dielectric C 0= =8 pF
without causing its electrical d
breakdown is called its dielectric With dielectric between the capacitor
strength. The dielectric strength for plates,
vacuum is infinity.
ε0 A
55. The knowledge of dielectric strength C=κ =2κ C0 = 2 × 6 × 8 = 96pF.
helps in designing a capacitor by d /2
determining the maximum potential 66. The work done in moving a charge of
that can be applied across the 10 μC between two diagonally
capacitor without causing its electrical opposite points on the square will be
breakdown. zero because these two points will be
56. The value of dielectric constant equipotential.
decreases with the increase of 1 1 Q2
temperature. 67. Energy stored, E = CV2 =
2 2 C

When Q is constant, E ∝
V vacuum 200
57. κ= = =4 1
V dielectric 50 , and we get a
C
58. The charge on the conductor B graph of the type given in the question.
remains unchanged but its potential Hence the charge Q on the capacitor is
gets lowered. kept constant.
59. A capacitor with a dielectric has a 68. The potential energy of a charge q is
higher capacitance and hence stores the work done in bringing charge q
more charge. from infinity to the position r⃗ in the
60. No. If d = 1cm = 10 -2 m, then area of external electric field. U (r⃗ ) = qV(r⃗ )
such a capacitor would be 69. As potential at any point on the
Cd 1 F ×10 m
−2
9 2
equatorial axis of an electric dipole is
A= = =10 m zero, so
ε 0 8.85 ×10−12 C2 N−1 m −2
W = q∆V = q(0 - 0) = 0.
This is a plate about 30 km in length and
breadth. 70. Zero.
61. A capacitor provides electrical energy 71. The introduction of a metal sheet of
stored in it. A cell provides electrical thickness t in a parallel plate capacitor
energy by converting chemical energy increases its capacitance by a factor
into electrical energy. d
, where d is the plate separation
62. In parallel combination, potential d−t
difference is same across each of the capacitor.

135
72. Potential at the centre=Potential at the been shorted.
surface = 10 V.
77. No work is done. Potential at
73. (i) When the dipole moment ⃗p is equatorial points of the dipole is zero.
parallel to the the electric field ⃗ E (θ = W = qVAB = q(0 - 0) = 0.
0°), the dipole is in stable equilibrium.
78. Zero, because all points of the circular
(ii) When the dipole moment ⃗p is arc will be at the same potential.
antiparallel to the electric field ⃗
E (θ =
180°), the dipole is in unstable
equilibrium.
79. VA - VB =
Q
[ 1

4 π ε 0 OA OB
1
]
74. W = pE(cos θ1 - cos θ2) = pE(cos 0°-
cos 180°) VA - VB = 0 and [ 1

OA OB
1
]
> 0,
= pE(1 + 1) = 2pE.
so the charge Q must be positive.
75. See Fig. 2.26 on page 2.15.
80. (i)VA > VB (ii) VA < VB.
76. C = C3, because the combinations of
C1 and C2 as well as C4 and C5 have
TYPE B : SHORT ANSWER QUESTIONS (2 or 3 marks each)

1. Distinguish between electric potential at a point due to a continuous charge


and potential energy and write the distribution.
relation between them.
10. Show that the electric field at any point
2. Define electric potential. Derive an is equal to the negative of the potential
expression for the electric potential at gradient at that point.
a distance r from a charge q.
11. Describe how can we determine the
3. Draw a plot showing the variation of (i) electric potential at a point from the
electric field (E) and (ii) electric knowledge of electric field.
potential (V) with distance r due to a
12. Two closely spaced equipotential
point charge Q. [CBSE D 12]
V + δV, (where δV is the change in V),
surfaces A and B with potentials V and

are kept δl distance apart as shown in


4. Derive the expression for the electric
potential at any point P, at distance r
from the centre of an electric dipole, the figure. Deduce the relation
making angle α, with its axis. [CBSE between the electric field and the
OD 17C] potential gradient between them. Write
5. Derive an expression for the electric the two important conclusions
potential at a point along the axis line concerning the relation between the
of the dipole. [CBSE D 2000, 08, 17 ; electric field and electric potentials.
OD 01C, 02, 13C] [CBSE D 14C]

6. Show mathematically that the electric


potential at any equatorial point of an
electric dipole is zero. [CBSE OD 01]
7. Give three differences between the
nature of electric potentials of a single
point charge and an electric dipole. Fig. 2.206
8. Obtain an expression for the electric 13. Show that the amount of work done in
potential at a point due to group of N moving a test charge over an
point charges. equipotential surface is zero.
9. Obtain an expression for the potential 14. Show that the direction of the electric

136
field is normal to the equipotential 22. An electric dipole is held in a uniform
surface at every point. electric field ⃗
E . (a) Show that the net
15. Show that the equipotential surfaces force acting on it is zero.
are closed together in the regions of (b) The dipole is aligned with its dipole
strong field and far apart in the region moment ⃗p parallel to the field ⃗
E.
of weak field.
Find : (i) the work done in turning the
16. Sketch equipotential surfaces for dipole till its dipole moment points in the
(i) a positive point charge. [CBSE D 2000] direction opposite to ⃗ E . (ii) the orientation
of the dipole for which the torque acting on
(ii) a negative point charge. [CBSE D 01]
it becomes maximum. [CBSE OD 12, 14C]
(iii) two equal and opposite charges
23. Using Gauss's law, show that electric
separated by a small distance.
field inside a conductor is zero.
(iv) two equal and positive charges [CBSE D 2000]
separated by a small distance. [CBSE OD
24. Just outside a conductor electric field
15]
is perpendicular to the surface. Give
(v) a uniform electric field. [CBSE D 2000, reason.
01]
25. Show that the excess charge on a
17. Define an equipotential surface. Draw conductor resides only on its surface.
equipotential surfaces : (a) in the case
26. Show that the electric field at the
of a single point charge, and (b) in a
surface of a charged conductor is
constant electric field in Z-direction.
Why the equipotential surfaces about
σ
given by ⃗
E = n^ , where σ is the
a single charge are not equidistant ? ε0
(c) Can electric field exist tangential to surface charge density and n^ is a unit
an equipotential surface ? Give vector normal to the surface in the
reason. [CBSE OD 16] outward direction.
18. Draw the equipotential surfaces due to Or [CBSE OD 10]
an electric dipole. Locate the points
Derive an expression for the electric field
where the potential due to the dipole is
at the surface of a charged conductor.
zero. [CBSE OD 13 ; D 17]
[CBSE OD 09]
19. Two point charges q1 and q2 are
27. Show that electric field is zero in the
located at ⃗
r 1 and ⃗
r 2 respectively in an cavity of hollow charged conductor.
external electric field ⃗ E . Obtain the
28. What is electrostatic shielding ?
expression for the total work done in
Mention its two applications.
assembling this configuration. [CBSE
D 14C] 29. Define electrical capacitance of a
conductor. On what factors does it
20. (a) Write two properties by which
depend ?
electric potential is related to the
electric field. 30. Show that the capacitance of a
spherical conductor is proportional to
(b) Two point charges q1 and q2,
its radius. Hence justify that farad is a
separated by a distance of r12 are kept
large unit of capacitance. [Himachal
in an external electric field. Derive an
96]
expression for the potential energy of
the system of two charges in the field. 31. An isolated conductor cannot have a
[CBSE OD 15 ; D 17C] large capacitance. Why ?
21. Derive an expression for the potential 32. Why does the capacitance of a
energy of an electric dipole of dipole conductor increase, when an earth
moment ⃗p in an electric field ⃗ E. connected conductor is brought near it
[Himachal 02 ; CBSE D 08] ? Briefly explain.

137
33. What is a capacitor ? Explain its 43. What is meant by dielectric
principle. [Punjab 2000, 02, 03] polarisation ? Hence establish the
relation : κ=1+ χ [Haryana 01]
34. Derive an expression for the

constants κ1 and κ2 are filled in


capacitance of a parallel plate 44. Two dielectric slabs of dielectric
capacitor. [CBSE D 05C, 14 ; OD 03]
35. Distinguish between polar and non- between the two plates, each of area
polar dielectrics. Give one example of A, of the parallel plate capacitor as
each. shown in Fig. 2.207. Find the net
capacitance of the capacitor. [CBSE
36. Three capacitors of capacitancesC1, OD 05]
C2 and C3 are connected in series.
Find their equivalent capacitance.
[CBSE D 16C]
37. Three capacitors of capacitancesC1,
C2 and C3 are connected in parallel.
Find their equivalent capacitance.
[CBSE D 92, 94 ; Himachal 99] Fig. 2.207
38. Deduce the expression for the 45. A parallel plate capacitor of
electrostatic energy stored in a capacitance C is charged to a potential
capacitor of capacitance ‘C’ and V. It is then connected to another
having charge 'Q'. How will the (i) uncharged capacitor having the same
energy stored and (ii) the electric field capacitance. Find out the ratio of the
inside the capacitor be affected when energy stored in the combined system
it is completely filled with a dielectric to that stored initially in the single
material of dielectric constant ‘κ’? capacitor. [CBSE OD 14]
[CBSE D 14 ; OD 08, 12] 46. Explain, using suitable diagrams, the
39. If two charged conductors are touched difference in the behaviour of a (i)
mutually and then separated, prove conductor and (ii) dielectric in the
that the charges on them will be presence of external electric field.
divided in the ratio of their Define the term polarization of a
capacitances. dielectric and write its relation with
susceptibility. [CBSE D 15]
40. Two capacitors with capacity C1 and
C2 are charged to potential V1 and V2 47. A capacitor is charged with a battery
respectively and then connected in and then its plate separation is
parallel. Calculate the common increased without disconnecting the
potential across the combination, the battery. What will be the change in
charge on each capacitor, the (a) charge stored in the capacitor ?
electrostatic energy stored in the
system and the change in the (b) energy stored in the capacitor ?
electrostatic energy from its initial (c) potential difference across the plates
value. [CBSE Sample Paper 08] of the capacitor ?
41. Explain why the polarization of a (d) electric field between the plates of the
dielectric reduces the electric field capacitor ? [CBSE Sample Paper
inside the dielectric. Hence define 2011]
dielectric constant. [CBSE D 99]
48. The charges q1 = 3 μF, q2 = 4 μF and
42. Define 'dielectric constant' of a q3 = - 7 μF are placed on the
medium. Briefly explain why the circumference of a circle of radius 1.0
capacitance of a parallel plate m, as shown in Fig. 2.208. What is the
capacitor increases, on introducing a value of charge q4 placed on the same
dielectric medium between the plates. circle if the potential at centre, VC = 0 ?
[CBSE OD 06C] [ISCE 03]

138
Fig. 2.208
49. Two thin concentric shells of radii r 1
and r2(r2 > r1) have charges q1 and q2.
Write the expression for the potential
at the surface of inner and outer
shells. [CBSE OD 13 C]
50. A charge Q is distributed uniformly
over a metallic sphere of radius R.
Obtain the expressions for the electric
field (E) and electric potential (V) at a
point 0 < x < K Show on a plot the
variation of E and V with x for 0 < x <
2R [CBSE F 17]

charge density + λ1 is surrounded by a


51. A long charged cylinder of linear

hollow coaxial conducting cylinder of


linear charge density -λ2. Use Gauss's
law to obtain expression for the
electric field at a point (i) in the space
between the cylinders, and (ii) outside
the larger cylinder. [CBSE F 17]

139
Answers

1. Refer to points 3 and 11 of Glimpses. equatorial points of the dipole.


2. Refer answer to Q. 2 on page 2.2. 19. Refer answer to Q. 20 on page 2.17.
3. See Fig. 2.3 on page 2.2. 20. (a) Refer answer to Q. 10 on page
2.12.
4. Refer answer to Q. 5 on page 2.3.
(b) Refer answer to Q. 20 on page 2.17.
5. Refer answer to Q. 3 on page 2.2.
21. Refer answer to Q. 22 on page 2.18.
6. Refer answer to Q. 4 on page 2.3.
22. (a) Refer answer to Q. 40 on page
7. Refer answer to Q. 6 on page 2.3.
1.41 of chapter 1.
8. Refer answer to Q. 7 on page 2.4.
(b) (i) W =
9. Refer answer to Q. 8 on page 2.4. id π π

Refer answer to Q. 10 on page 2.11. ∫ τdθ=∫ pEsinθ= pE[−cosθ ]π0 =−2 pE


0 0
11. Refer answer to Q. 11 on page 2.12.
(ii) As τ = pEsin θ, so τ is maximum when

charge through distance δl,


12. Work done in moving a unit positive θ = 90°.

E × δl = VA - VB = V-(V + δV) = -δV ∴ E = -


23. Refer answer to Q. 25 on page 2.24.

δV 24. Refer answer to Q. 25 on page 2.24.


δl 25. Refer answer to Q. 25 on page 2.24.
For conclusions, refer answer to Q. 10 on 26. Refer answer to Q. 25 on page 2.24.
page 2.12. 27. Refer answer to Q. 25 on page 2.24.
13. Refer answer to Q. 14 on page 2.14. 28. Refer answer to Q. 26 on page 2.25.
14. Refer answer to Q. 14 on page 2.14. 29. Refer answer to Q. 27 on page 2.26.
15. Refer answer to Q. 14 on page 2.14. 30. Refer answer to Q. 29 on page 2.26.
16. Refer answer to Q. 15 on page 2.15. 31. Refer answer to Q. 30 on page 2.28.
17. (a) See Fig. 2.25 on page 2.15. 32. Refer answer to Q. 31 on page 2.28.
(b) See Fig. 2.200 on page 2.101. 33. Refer answer to Q. 31 on page 2.28.
dV dV 34. Refer answer to Q. 33 on page 2.29.
As E = - or dr = -
dr E
35. Refer answer to Q. 43 on page 2.56.
∴ For constant dV, dr ∝ ∝ r2
1
36. Refer answer to Q. 36 on page 2.33.
E
37. Refer answer to Q. 37 on page 2.33.
Hence the spacing between the
equipotential surfaces will increase with 38. Refer answer to Q. 38 page 2.48.
the increase in distance from the point When the capacitor is completely filled
charge. with a dielectric material and for constant
(c) No. If the field lines are tangential, charge Q,
work will be done in moving a test charge C = κq and V = V0/κ
on the surface. But no work is needed in
moving a test charge on an equipotential (i)
2
surface. 1 2 1 V0 1 1 2 U0
U = C V = (κ C 0)( ) = ⋅ C 0 V 0 =
18. See Fig. 2.26 on page 2.15. The 2 2 κ κ 2 κ
electric potential is zero at the

140
E0 (a) q = CV decreases due to the decrease
(ii) E= in the value of C.
κ
39. Refer answer to Q. 41 on page 2.49. 1
(b) U = CV2 decreases due to the
2
40. Refer answer to Q. 42 on page 2.49. decrease in the value of C.
41. Refer answer to Q. 45 on page 2.57. (c) V remains unchanged because the
42. Refer to point 35 of Glimpses on page battery remains connected.
2.116. (d) E = V / d decreases due to the
43. Refer answer to Q. 47 on page 2.58. increase in the value of d.
44. The given arrangement is equivalent 48. As VC = 0


to a parallel combination of two
capacitors each with area A / 2 A / 2
and plate separation d. Hence the net
1
[
q1 q 2 q3 q 4
+ + + =0
4 π ε0 r r r r ]
capacitance of the resulting capacitor
or q1 + q2 + q3 + q4 = 0
is
or 3 + 4 - 7 + q4 = 0 or q4 = 0.
C = C1 + C2
49. Potential at the surface of inner shell,
ε 0 ( A /2) κ 1 ε 0 ( A /2)κ 2 ε 0 A (κ 1+ κ2 )
= + = . V1 = Potential due to its own charge q 1 +
d d d
Potential due to charge q2 on outer shell
45. Initial energy stored in the single
capacitor
1 1q
2
=
1
(
q1 q2
+
4 π ε0 r1 r2 )
= C V 2=
2 2C Potential at the surface of outer shell
Capacitance of the combined (parallel) V2 = Potential due to charge q1 on inner
system shell + Potential due to charge q2 on outer
shell
= C + C = 2C
As the total charge q remains the same,
so
=
1
(
q1 q2
+
4 π ε0 r2 r2 )
Final energy stored in the combined 50. By Gauss's theorem,
2
1 q
system = q
2 2C ΦE = ∫ ⃗
E ⋅ d ⃗S =
ε0

S
Final energy 1
= =1 :2 For 0 < x < R, q = 0
∴E=0
Initial energy 2
46. Refer answer to Q. 43 on page 2.56.
Polarisation of a dielectric. The induced dV
But E = -
dipole moment set up per unit volume of a dr
dielectric when placed in an external ∴ V = constant
electric field is called polarisation. For
linear isotropic dielectrics, 1 Q
= ⋅
⃗ 4 π ε0 R
P=ε 0 χ e ⃗
E where χ e is the electric
susceptibility of the dielectric medium.
ε0 A
47. C= . When d is increased, C
d
decreases.

141
∴⃗
λ1
E 1=
2 π ε0 r1

Fig. 2.209

Fig. 2.211
(ii) In the region outside the larger
Fig. 2.210
cylinder,
q
51. By Gauss's law, ΦE = ∮ ⃗
E ⋅d ⃗S = ( λ1−λ 2)l
ε0 ϕ E=∮ ⃗
E2 ⋅d ⃗S =
S2 ε0
(i) In the space between the two cylinders,
field is due to charge on inner cylinder ( λ1−λ 2)l
only. or E2 ×2 π r 2 l=
ε0

∴ϕ E=∮ ⃗ or E1 ×2 π r 1 l= ∴⃗
λ1 l λ1 l (λ1− λ2)
E1 ⋅d ⃗S = E 2= r^
S1 ε0 ε0 2 πε r 2 2

TYPE C : LONG ANSWER QUESTIONS (5 marks each)

1. Find the expression for the electric 3. Define the term electric dipole
field intensity and the electric potential, moment. Derive an expression for the
due to a dipole at a point on the total work done in rotating the dipole
equatorial line. Would the electric field through an angle θ in uniform electric
be necessarily zero at a point where fieId ⃗
E . [CBSE OD 93, 95 ; D 96C]
the electric potential is zero ? Give an
example to illustrate your answer. 4. Derive an expression for the potential
[CBSE Sample Paper 2011] energy of an electric dipole placed in a
uniform electric field.
2. (a) Obtain the expression for the
potential due to a point charge. Hence discuss the conditions of its
stable and unstable equilibrium.
(b) Use the above expression to show that
the potential, due to an electric dipole 5. (a) Explain why, for any charge
(length 2a), varies as the 'inverse configuration, the equipotential surface
square' of the distance r of the 'field through a point is normal to the electric
point' from the centre of the dipole for r field at that point.
>> a. [CBSE D 16C]

142
Draw a sketch of equipotential 03]
surfaces due to a single charge (-q),
9. (a) Derive the expression for the energy
depicting the electric field lines due to
stored in a parallel plate capacitor. Hence
the charge.
obtain the expression for the energy
(b) Obtain an expression for the work density of the electric field.
done to dissociate the system of three
(b) A fully charged parallel plate capacitor
charges placed at the vertices of an
is connected across an uncharged
equilateral triangle of side 'a' as shown
identical capacitor. Show that the energy
here. [CBSE OD 16]
stored in the combination is less than that
stored initially in the single capacitor.
[CBSE OD 15]
10. Define the terms (i) capacitance of a
capacitor (ii) dielectric strength of a
dielectric. When a dielectric is inserted
between the plates of a charged
parallel plate capacitor, fully occupying
Fig. 2.212 the intervening region, how does the
6. (a) Define the capacitance of a polarization of the dielectric medium
capacitor. Obtain the expression for the affect the net electric field ? For linear
capacitance of a parallel plate capacitor in dielectrics, show that the introduction

κ,
vacuum in terms of plate area A and of a dielectric increases its
separation d between the plates. capacitance by a factor
characteristic of the dielectric. [CBSE

κ has the same area as the plates of a


(b) A slab of material of dielectric constant D 08C]
11. Find the expression for the
parallel plate capacitor but has a thickness
capacitance of a parallel plate
3d capacitor of area A and plate
. Find the ratio of the capacitance with
4 separation d if (i) a dielectric slab of
dielectric inside it to its capacitance thickness t and (ii) a metallic slab of
without the dielectric. [CBSE F 17] thickness t, where (t < d) are
7. (i) Distinguish, with the help of a introduced one by one between the
suitable diagram, the difference in the plates of the capacitor. In which case
behaviour of a conductor and a dielectric would the capacitance be more and
placed in an external electric field. How why ? [CBSE Sample Paper 2011]
does polarised dielectric modify the 12. What is a dielectric ? A dielectric slab
original external field ? [CBSE OD 16] of thickness t is kept between the
(ii) If two similar large plates, each of area plates of a parallel plate capacitor
A having surface charge densities + σ and separated by distance d. Derive the
–σ are separated by a distance d in air, expression for the capacitance of the
find the expressions for (a) field at points capacitor for t << d. [Himachal 02 ;
between the two plates and on outer side CBSE D 93 ; OD 01C]
of the plates. Specify the direction of the 13. (a) A small sphere, of radius ‘a’,
field in each case (b) the potential carrying a positive charge q, is placed
difference between the plates (c) the concentrically inside a larger hollow
capacitance of the capacitor so formed. conducting shell of radius b(b > a).
8. Obtain the expression for the This outer shell has a charge Q on it.
capacitance of a parallel plate capacitor. Show that if these spheres are
Three capacitors of capacitances C1, C2 connected by a conducting wire,
and C3 are connected (i) in series, (ii) in charge will always flow from the inner
parallel. Show that the energy stored in sphere to the outer sphere,
the series combination is the same as that irrespective of the magnitude of the
in the parallel combination. [CBSE OD two charges. [CBSE F 15]

143
(b) Name the machine which makes use the maximum potential difference which
of this principle. Draw a simple labelled this machine can built up ? [CBSE D 09C ;
line diagram of this machine. What OD 14]
'practical difficulty' puts on upper limit on

Answers

[ ]
1. For derivation of electric field intensity 1 9× (−4 q ) q ×2 q (−4 q ) ×2 q
at equatorial point of a dipole, refer + +
4 π ε0 a a a
answer to Q. 38 on page 1.40 of
chapter 1. 1 2 2 2 +10 q
2
=- [−4 q +2 q −8 q ]= .
1 p 4 π ε0 4 π ε0 a
E equa =
4 π ε 0 (r +a2 )3/ 2
2
6. (a) Refer answer to Q. 33 on page 2.29.
For derivation of electric potential at an (b) Refer to the solution of Example 89 on
equatorial point of a dipole, refer answer page 2.64.
to Q. 4 on page 2.3 of chapter 2. Vequa = 0
7. (i) See Fig. 2.116 on page 2.56.
No, the electric field may not be
necessarily zero at a point where the In a conductor, the induced field Eind is
electric potential is zero. For example, the exactly equal and opposite to the external
electric potential at an equatorial point of a field E0. The net field inside the conductor
dipole is zero, while electric field is not is zero.
zero. In a dielectric, the induced field E ind
2. (a) Refer answer to Q. 2 on page 2.2. opposes E0 but does not exactly cancel it.
It only reduces it.
(b) Refer answer to Q. 3 on page 2.2.
σ σ σ
1 p (ii) (a) E inside =E1 + E2= + =
We get, V = ⋅ 2 2 2 ε0 2 ε 0 ε0
4 π ε 0 r −a

For r >> a, V =
1
⋅ 2 ⇒V∝ 2
4 π ε0 r
p
r
1 E outside =± ( E 2−E1 ) =±
( εσ − εσ )=0
0 0

3. Refer answer to Q. 22 on page 2.18.


4. Refer answer to Q. 22 on page 2.18.
5. (a) Refer answer to Q. 14 on page
2.14. The equipotential surfaces and
the field lines of a point charge -q are
shown in the figure.
σd q σA ε0 A
(b) Work required to dissociate the system (b)V = Ed = (c)C = = = .
ε0 V σd /ε 0 d
of three charges
8. Refer answer to Q. 33 on page 2.29
and Q. 39 on page 2.48.
9. (a) Refer answer to Q. 38 and Q. 40
on page 2.48.
(b) Refer to the solution of Example 75 on
Fig. 2.213 page 2.52.
= - P.E. of the system 10. Refer answer to Q. 45 on page 2.57
and Q. 49 on page 2.59.
= -

144
11. For derivation, refer answers to Q. 49 13. (a) Refer answer to Q. 55 on page
on page 2.59 and Q. 50 on page 2.60. 2.67
ε0 A (b) A Van-de-Graaff generator works on
C dielectric = this principle. See Fig. 2.130. The
t
d−t+ potential on the outer surface of its
κ metallic shell cannot exceed the break-

[For metal, κ = oo]


ε0 A down field of air (≃ 3 × 106 Vm-1) because
C metal = then the charges begin to leak into air.
d −t
This puts the limit on the potential
Clearly, Cmetal > Cdielectric difference which the machine can built up.
12. Refer answer to Q. 49 on page 2.59.

TYPE D : VALUE BASED QUESTIONS (4 marks each)

1. Immediately after school hour, as (a) What are the values being displayed
Bimla with her friends came out, they by Pankaj ?
noticed that there was a sudden
(b) What may be the reason behind the
thunderstorm accompanied by the
electric shock received by Pankaj ?
lightning. They could not find any
suitable place for shelter. Dr. Kapoor
who was passing thereby in his car
noticed these children and offered
them to come in his car. Fie even took
care to drop them to the locality where
they were staying. Bimla's parents,
who were waiting, saw this and
expressed their gratitude to Dr.
Kapoor. [CBSE OD 15C]
(a) What values did Dr. Kapoor and
Bimla's parents display ?
(b) Why is it considered safe to be inside
a car especially during lightning and
thunderstorm ?
(c) Define the term 'dielectric strength'.
What does this term signify ?
2. One evening, Pankaj outside his
house fixed a two metre high
insulating slab and attached a large
aluminium sheet of area lm2 over its
top. To his surprise, next morning
when he incidently touched the
aluminium sheet, he received an
electric shock. He got afraid. He
narrated the incident to his Physics
teacher in the school who explained
him the reason behind it.
Answer the following questions based on
the above information :

145
Answers

1. (a) Dr. Kapoor displayed helpfulness, maximum electric field upto which the
empathy and scientific temper. dielectric can safely play its role.
Bimla's parents displayed gratefulness 2. (a) Keen observer and curiosity.
and indebtedness.
(b) The aluminium sheet and the ground
(b) It is safer to sit inside a car during a form a capacitor alongwith the insulating
thunder-storm because the metallic body slab. The discharging current of the
of the car becomes an electrostatic atmosphere charges the capacitor steadily
shielding from lightning. and raises its voltage. So, when Pankaj
touches the aluminium sheet, he receives
(c) The maximum electric field that a
an electric shock.
dielectric medium can withstand without
break-down of its insulating property is
called its dielectric strength. It signifies the

COMPETITION SECTION
Electrostatic Potential and Capacitance
GLIMPSES
Work done
Electric potential =
1. Potential difference. The potential Charge
difference between two points is W
defined as the amount of work done in or V =
q
bringing a unit positive charge from
one point to another against the Electric potential is a scalar quantity.
electrostatic forces. 4. SI unit of electric potential is volt. The
Work done electric potential at a point in an
Potential difference = electric field is said to be -1 volt if one
Charge
joule of work has to be done in moving
W AB a positive charge of 1 coulomb from
or V AB=V B−V A=
q infinity to that point against the
electrostatic forces.
Potential difference is a scalar
quantity. 5. Electric potential due to a point
charge. The electric potential of a point
2. SI unit of potential difference is volt charge q at distance r from it is given
(V). The potential difference between by
two points in an electric field is said to
1 q
be 1 volt if 1 joule of work has to be V= . i.e., V ∝ 1
done in moving a positive charge of 1 4 π ε0 r r
coulomb from one point to the other
against the electrostatic forces. It is spherically symmetric.

1 V= 1 JC-1 = 1 Nm C-1 6. Electric potential due to a dipole.


Electric potential at a point having position
3. Electric potential. It is defined as the vector r⃗ , due to a dipole of moment ⃗p at
amount of work done in bringing a unit the origin is given by
positive charge from infinity to the
observation point against the 1 ⃗p . ⃗r 1 p cos θ
V= . 3 =
electrostatic forces. 4 π ε0 r 4 π ε0 r2

146
At points on the axial line of the dipole ( (iii) Equipotential surfaces are close
0 0
θ=0 or 18 0 ), together in the regions of strong field
and farther apart in the regions of
1 p weak field.
V a xia l=± .
4 π ε0 r2
(iv) No two equipotential surfaces can
At points on the equatorial line of the intersect each other.
dipole (θ=9 0o ) , 11. Electric potential energy. The electric
V equa=0. potential energy of a system of point
charges is defined as the amount of
7. Electric potential due to a group of N work done in assembling the charges
point charges. If r 1 , r 2 , r 3… r N are the at their locations by bringing them in,
distances of N point charges from the from infinity.
observation point , then P.E. of a charge = Charge × Electric

[ ]
potential at the given point
1 q 1 q2 q 3 qN
V= + + + … .+ It is measured in joule (J) or electron
4 π ε0 r1 r 2 r3 rN
volt (eV).
N
1 qi 1eV = 1.6 × 10-19 J
¿ ∑
4 π ε 0 i=1 r i 12. Potential energy of a system of two
8. Determination of electric field from point charges. If two point charges q1
electric potential. The rate of change of and q2 are separated by distance r12,
potential with distance is called potential then their potential energy is
gradient. Electric field at any point is equal q1q2
1
to the negative of the potential gradient at U= ⋅
that point 4 π ε 0 r 12
dV 13. Potential energy of a system of three
E=- point charges. It is given by
dr
∴ SI unit of electric field = Vm-1
The direction of ⃗
E is in the direction of
U=
[
1 q 1 q 2 q 2 q 3 q3 q 1
4 π ε 0 r 12
+
r 23
+
r 31 ]
steepest decrease of potential. 14. Potential energy of N point charges. It
9. Determination of electric potential from is given by
electric field. The electric potential at a 1 qi q j
point having position vector r⃗ is given U=
4 π ε0
∑ r ij
All pairs
by
r N N
1 1 qi q j
V = - ∫⃗
E .⃗
dr = ⋅ ∑
2 4 π ε 0 i=1
∑ r ij
∞ j=1

i≠j
10. Equipotential surface. Any surface that
has same electric potential at every
point on it is called an equipotential
15. Potential energy of a dipole in a
surface. The surface of a charged
uniform electric field It is equal to the
conductor is an equipotential surface.
amount of work done in turning the
Some of the important properties of
dipole from orientation θ1 to θ2 in the
equipotential surface are as follows :
field E.
(i) No work is done in moving a test
U = -pE (cos θ2 - cos θ1)
charge over an equipotential surface.
If initially the dipole is perpendicular to the
(ii) Electric field is always normal to the
field E, θ1 =90° and θ2 = θ (say), then
equipotential surface at every point.
U = - pE cos θ = -⃗p.⃗
E

147
When θ = 0°, U = - pE i.e., the potential amount.
energy of the dipole is minimum. The
Charge
dipole is in stable equilibrium. Capacitance =
Potential
When θ = 90°, U = 0
q
When θ = 180°, U = +pE or C =
V
i.e., the potential energy of the dipole is
20. Capacitance of a spherical conductor
maximum. The dipole is in unstable
It is proportional to the radius R of the
equilibrium.
spherical conductor.
16. Conductors and insulators.
C = 4πε0 R
Conductors are the substances which
allow large scale physical movement 21. Capacitor. It is an arrangement of two
of electric charges through them when conductors separated by an insulating
an external electric field is applied. medium that is used to store electric
They contain a large number of free charge and electric energy.
electrons. Insulators are the 22. Capacitance of a capacitor. The
substances which do not allow capacitance of a capacitor is the
physical movement of electric charges charge required to be supplied to one
through them when an external electric of its conductors so as to increase the
field is applied. They contain a potential difference between two
negligibly small number of free charge conductors by unit amount.
carriers.
q
17. Electrostatic properties of a conductor. C=
When placed in an electrostatic field, a V
conductor shows the following 23. Farad. It is the SI unit of capacitance.
properties : The capacitance of a capacitor is 1
(i) Net electrostatic field is zero in the farad (F) if 1 coulomb of charge is
interior of a conductor. transferred from its one plate to
another on applying a potential
(ii) Just outside the surface of a difference of 1 volt across the two
conductor, electric field is normal to plates. •
the surface.
1coulomb 1C
(iii) The net charge in the interior of a 1 farad = or 1 F =
1 volt 1V
conductor is zero and any excess
charge resides on its surface. 1 mF = 10-3 F, 1 μ F = 10-6 F, 1 pF =
10-12 F.
(iv) Potential is constant within and on the
surface of a conductor. 24. Parallel plate capacitor. It consists of
two large parallel conducting plates,
(v) Electric field at the surface of a
each of area A, and separated by a
charged conductor is proportional to
small distance d. Its capacitance is
the surface charge density.
ε0 A
(vi) Electric field is zero in the cavity of a C=
hollow charged conductor. d
18. Electrostatic shielding. The 25. Spherical capacitor. It consists of two
phenomenon of making a region free concentric spherical conducting shells
from any electric field is called of inner and outer radii a and b.
electrostatic shielding. It is based on 4 π ε 0 ab
the fact that electric field vanishes C=
inside the cavity of a hollow conductor. b−a
19. Capacitance of a conductor. It is the 26. Cylindrical capacitor. It consists of two
charge required to increase the coaxial conducting cylinders of inner
potential of a conductor by unit and outer radii a and b and of common

148
length l. C 1 V 1+ C 2 V 2+ C 3 V 3+ …
= .
l l C1 +C 2+C 3 +… .
C=2 π ε 0 =2 π ε 0
b b 32. Loss of energy on sharing charges. If
log e 2303 log 10
a a two conductors of capacitances C1 and
27. Capacitors in series. The equivalent C2 at potentials V1 and V2 respectively
capacitance Cs of number of are connected together, a loss of
capacitors connected in series is given energy takes place which is given by
by 1 C1C2 2
ΔU = (V −V 2) .
1 1 1 1 2 C1 +C 2 1
= + + +…
C s C 1 C2 C 3 33. Dielectric. A dielectric is a substance
In a series combination of capacitors, which does not allow the flow of
the charge on each capacitor is same charges through it but permits them to
but the potential difference across any exert electrostatic forces on one
capacitor is inversely proportional to its another. It is essentially an insulator
capacitance. which can be polarised through small
localised displacements of its charges.
28. Capacitors in parallel. The equivalent
capacitance of a number of capacitors 34. Polar and non-polar dielectrics. The
connected in parallel is given by dielectrics made of polar molecules
(such as HCl, NH3, H2O, CH3OH, etc.)
Cp = C1 + C2 + C3 +... are called polar dielectrics. In a polar
In a parallel combination of capacitors, molecule, the centre of mass of
the potential difference across each positive charges does not coincide
capacitor is same but the charge on with the centre of mass of negative
each capacitor is proportional to its charges.
capacitance. The dielectrics made of non-polar
29. Energy stored in a capacitor. The molecules are called non-polar
energy stored in a capacitor of dielectrics. In a non-polar molecule,
capacitance C and charge q with the centre of mass of positive charges
voltage V is coincides with the centre of mass of
2
negative charges e.g., H2, O2, CO2,
1 2 1 Q 1 CH4, etc.
U = C V = ⋅ = QV
2 2 C 2 35. Polarisation of dielectric. If the medium
30. Energy density. The electrical energy between the plates of a capacitor is
stored per unit volume or energy filled with a dielectric, the electric field
density in a region with electric field E due to the charged plates induces a
is net dipole moment in the dielectric.
This effect is called polarisation which
1 induces a field in the opposite
u= ε0 E 2
2 direction. The net electric field inside
31. Common potential. If a number of the dielectric and hence the potential
conductors of capacitances C1, C2, C3, difference between the plates are
…, at potentials V1, V2, V3, …, having reduced. Consequently, the
charges q1, q2, q3, .... respectively are capacitance C increases from its value
placed in contact, their common C0 when there is vacuum.
potential V is given by C = Κ C0 .
V = 36. Dielectric constant. It is the ratio of the
Total charge q 1 +q2 +q 3 +… . capacitance (C) of the capacitor with
=
Total capacitance C1 +C 2 +C3 +… . the dielectric as the medium to its
capacitance (C0) when conductors are
in vacuum.

149
Κ
C between its plates. For t < d,
=
C0
( d−td ) ( )
ε0 A d
C= = C .
It is also equal to the ratio of the d d−t 0
applied electric field (E0) to the
reduced value of electric field (E) on 40. Capacitance of a spherical capacitor
inserting the dielectric slab between filled with a dielectric.
the plates of the capacitor. ab
C = 4 π ε0 κ .
E0 E0 b−a
κ= = .
E E0−E ' 41. Capacitance of a cylindrical capacitor
filled with a dielectric
Here E' is the field set up due to
polarisation of the dielectric in the 2 π ε 0 κl
C=
opposite direction of E0. b.
2.303 log 10
37. Capacitance of a parallel plate a
capacitor filled with a dielectric. 42. Van de Graaff generator. It is an
ε 0 κA electrostatic generator capable of
C = κ C 0= . building up high potential differences
d of the order of 107 volts. It is based on
38. Capacitance of a parallel plate the principle that when a charged
capacitor with a dielectric slab conductor is brought into internal
between its plates. If t is the thickness contact with a hollow conductor, it
of the dielectric slab and t < d, then transfers whole of its charge to the
hollow conductor, howsoever high the
ε0 A potential of the latter may be. Also, it
C=
( 1κ ) .
d−t 1−
uses discharging action of sharp
points. It is used for accelerating
charged particles.
39. Capacitance of a parallel plate
capacitor with conducting slab
JEE Advance

Multiple Choice Questions with one


correct answer
1. Consider a thin spherical shell of
radius R with its centre at the origin,
carrying uniform positive surface charge
density. The variation of the magnitude of
the electric field |⃗E (r)| and the electric
potential V(r) with the distance r from the
centre, is best represented by which graph
? [IIT 12] (b)
(a)

150
(c) 5. Three charges Q, + q and + q are
placed at the vertices of a right angled
isosceles triangle as shown. The net
electrostatic energy of the configuration is
zero, if Q is equal to

(d)
−q −2 q
(a) (b)
1+ √2 2+ √2
(c) -2q (d)+q [IIT 2000; IPUEE 13]
q
6. Consider a system of three charges ,
3
q 2q
and - placed at points A, Band C,
3 3
respectively, as shown in the figure. Take
O to be the centre of the circle of radius R
2. A uniform electric field pointing in
and angle CAB = 60°.
positive X-direction exists in a region. Let
A be the origin, B be the point on the X-
axis at x = +1 cm and C be the point on
the Y-axis at y = +1 cm. Then the potential
at points A, B and C satisfy.
(a) VA < VB (b) VA > VB.
(c)VA < VC (d) VA > VC [IIT 2001]
3. The electric potential V at any point x, y,
z (all the coordinates are in metres) in q
(a) The electric field at point O is 2,
space is given by V = 4x volt. The electric 8 π ε0 R
field at the point (1m, 0, 2m) in volt metre -1 directed along the negative x-axis.
is
(b) The potential energy of the system is
(a) 8 along negative X-axis. zero.
(b) 8 along positive X-axis. (c) The magnitude of the force between
2
(c) 16 along negative X-axis. q
the charges at C and B is 2.
(d) 16 along positive X-axis. [IIT 1989] 54 π ε 0 R
4. Two equal charges are fixed at x = -a q
and x = +a on the X-axis. Another point (d) The potential at point O is .
12 π ε 0 R
charge Q is placed at the origin. The [IIT 2009]
change in the electrical potential energy of
Q, when it is displaced by a small distance 7. A metal foil of negligible thickness is
x along the X-axis, is approximately introduced between two plates of a
proportional to capacitor at the centre. The capacitance
of capacitor will be
(a) x (b) x2
(a) same (b) double
3
1
(c) x (d) [IIT 2002] (c) half (d) K times [IIT 82 ; AIEEE
x
03]

151
8. A solid sphere and a hollow sphere same capacitance C. One of them is
of equal diameters are raised to the same charged to potential V1 and the other to V2.
potential. Then, The negative ends of the capacitors are
connected, the decrease in energy of the
(a) hollow sphere has more charge.
combined system is
(b) both have equal charge.
1 2 2 1 2 2
(c) only hollow sphere has charge. (a) C (V 1−V 2 ) (b) C (V 1+ V 2)
4 4
(d) solid sphere has more charge. 1 2 1 2
(IIT 1984] (c) C ( V 1−V 2 ) (d) C ( V 1−V 2 )
4 4
9. Two capacitors of capacitances C1 [IIT 2002]
and C2 are connected in parallel. If a
13. A parallel plate capacitor of area A,
charge q is given to the assembly, the
plate separation d and capacitance C is
charge gets shared. The ratio of the
materials having dielectric constants κ1,
filled with three different dielectric
charge on the capacitor C1 to the charge
κ2
that on C2 is
C1 C2
(a) (b)
C2 C1
1
(c) C1C2 (d) [IIT
C1 C2
1990]
10. Two identical metal plates are and κ3 as shown. If a single dielectric
given positive charges q1 and q2(<q1) material is to be used to have the same

dielectric constant κ is given by


respectively. If they are now brought close capacitance C in this capacitor, then its
together to form a parallel plate capacitor
with capacitance C, the potential
1 1 1 1 1 1 1
difference between them is (a) = + + (b) = +
κ κ 1 κ 2 2 κ3 κ κ 1 +κ 2 2κ 3
(q 1+ q2 ) (q 1+ q2 )
(a) (b) κ1 κ 2
2C C (c) κ= +2 κ3 (c)
κ 1+ κ 2
(q 1−q 2) (q 1−q 2) κ1 κ 3 κ 2 κ3
(c) (d) [IIT 1999]
C 2C κ= +
κ 1+ κ3 κ2 +κ 3
11. In the given circuit, a charge of
+80 μC is given to the upper plate of the 4 [IIT 2000]
μF capacitor. Then in the steady state, the 14. Consider the situation shown in the
charge on the upper plate of the 3 μF figure. The capacitor A has charge q on it,
capacitor is whereas B is uncharged.

The charge appearing on the capacitor B


a long time after the switch S is closed, is
(a) +32 μC (b)+40μC (a) zero (b) q / 2
(c) + 48 μC (d) +80 μC [IIT 2012] (c) q (d) 2q [IIT 2002]
12. Two identical capacitors have the 15. For the circuit shown in figure,

152
which of the following statements is true ? more than one correct answer
18. A charge + q is fixed at each of the
points x = x0, x0 = 3X0; x = 5x0, ... to ∞ on
the X-axis and a charge -q is fixed at each
of the points x = 2 x0, x = 4 x0, x = 6 x0, ...
to ∞. Here x0 is a positive constant. Take
(a) With S1 closed, V1 = 15 V, = 20V. the electric potential at a point due to a
charge Q at a distance r from it to be
(b) With S3 closed, V1 = V2 = 25 V.
1 Q
(c) With S1 and S2 closed, V1 = V2 = 0.
. .
4 π ε0 r
(d) with S1 and S3 closed, V1 = 30 V, V2 = Then, the potential at the origin due to
20V. the above system of the charges is
[IIT 1999] (a) 0 (b) ∞
16. A parallel plate capacitor C with plates 9 q ln 2
a liquid of dielectric constant κ = 2. The
of unit area and separation d is filled with (c) (d) [IIT
8 π ε 0 x 0 ln 2 4 π ε0 x 0
d 1998]
level of liquid is initially. Suppose the
3 19. A solid conducting sphere having a
liquid level decreases at a constant speed charge Q is surrounded by an uncharged
v, the time constant as a function of time t concentric conducting hollow spherical
is shell. Let the potential difference between
the surfaces of the solid sphere and that
of the outer surface of the hollow shell be
V. If the shell is now given a charge -3 Q,
the new potential difference between the
same two surfaces is
(a) V (b)2V
(c) 4 V (d) - 2 V [IIT 1989]
6 ε0 R (15 d +9 vt )ε 0 R 20. An ellipsoidal cavity is carved
(a) (b) 2 2 2
5 d+ 3 vt 2 d −3 dvt−9 v t within a perfect conductor as shown in
figure. A positive charge q is placed at the
6 ε0 R (15 d−9 vt )ε 0 R centre of the cavity. The points A and B
(c) (d) 2 2 2
5 d−3 vt 2 d +3 dvt−9 v t are on the cavity surface.
[IIT 2008] Then,
17. A 2 μF capacitor is charged as
shown in the figure. The percentage of its
stored energy dissipated after the switch S
is turned to position 2 is

(a) electric field near A in the cavity


= electric field near B in the cavity
(b) charge density at A- charge density at
B
(c) potential at A = potential at B
(a) 0% (b) 20%
(d) total electric field flux through the
(c) 75% (d) 80%
surface of the cavity is q / ε0 . [IIT 1999]
[IIT 2011]
21. A spherically symmetric charge
Multiple Choice Questions with one or system is centred at origin. Given that

153
Electric potential, [IIT 2011]
24. Six point charges are kept at the
vertices of a regular hexagon of side Land
centre O, as shown in the figure. Given
1 q
that K = , which of the following
4 π ε 0 L2
statement(s) is (are) correct ?
Q
V= ,r ≤ R0
4 π ε 0 R0
Q
V= , r > R0
4 π ε0 r
(a) Within r = 2R0, total enclosed net
charge is Q.
(b) Electric field is discontinuous at r = R0.
(c) Charge is only present at r = R0.
(d) Electrostatic energy is zero for r < R0. (a) The electric field at O is 6 K along OD.
[IIT 2006] (b) The potential at O is zero.
22. Two identical thin rings, each of (c) The potential at all points on the line
radius R metre are coaxially placed at PR is same.
distance R metre apart. If Q1 and Q2
coulomb are respectively the charges (d) The potential at all points on the line
uniformly spread on two rings, the work ST is same.
done in moving a charge q from the centre [IIT 12]
of one ring to that of the other is
25. Two non-conducting spheres of radii
q(Q 1−Q2)( √2−1) R1 and R2 and carrying uniform volume
(a) zero (b) charge densities +ρ and -ρ
4 √2 π ε0 R
respectively, are placed such that they
q √ 2(Q1 +Q2) partially overlap, as shown in the
(c) (d) figure. At all points in the overlapping
4 π ε0 R
region,
q(Q 1+Q2 )( √ 2+1)
4 √2 π ε 0 R
23. Which of the following statement(s)
is/are correct ?
(a) if the electric field due to a point
charge varies as r2/5 instead of r2, then
the Gauss law will still be valid (a) the electrostatic field is zero
(b) the Gauss law can be used to (b) the electrostatic potential is constant
calculate the field distribution around
(c) the electrostatic field is constant in
an electric dipole
magnitude
(c) if the electric field between two point
(d) the electrostatic field has same
charges is zero somewhere, then the
direction
sign of the two charges is the same
[JEE Adv. 13]
(d) the work done by the external force in
moving a unit positive charge from 26. In the circuit shown in the figure,
point A at potential VA to point B at the there are two parallel plate capacitors
potential VB is (VB - VA). each of capacitance C. The switch S1 is

154
pressed first to fully charge the capacitor whose negative plate is at x = 0 and
C1. positive plate is at x = 3d. The slab is
equidistant from the plates. The capacitor
is given some charge. As x goes from 0 to
3d,
(a) the magnitude of the electric field
remains the same.
(b) the direction of the electric field
and then released. The switch S2 is then remains the same.
pressed to charge the capacitor C2. After (c) the electric potential increases
some time, S2 is released and then S3 is continuously.
pressed. After some time,
(d) the electric potential increases at first,
(a) The charge on the upper plate of C 1 is then decreases and again increases.
2 CV0
The magnitude of electric field ⃗
E in the
(b) The charge on the upper plates of C1 is annular region of a charged cylindrical
CV0 capacitor
(c) The charge on the upper plates of C 2 is (a) is same throughout.
0
(b) is higher near the outer cylinder than
(d) The charge on the upper plate of C 2 is near the inner cylinder.
-CV0.
1
[JEE Adv. 13] (c) varies as , where r is the distance
r
27. A parallel plate capacitor is from the axis.
charged and the charging battery is then
disconnected. If the plates of the capacitor 1
(d) varies as 2 , where r is the distance
are moved farther apart by means of r
insulating handles, from the axis. [IIT 1996]
(a) the charge on the capacitor A parallel plate capacitor has a
increases. dielectric slab of dielectric constant K
between its plates that covers 1/3 of the
(b) the voltage across the plates
area of its plates, as shown in the figure.
increases.
The total capacitance of the capacitor is C
(c) the capacitance increases. while that of the portion with dielectric in
(d) the electrostatic energy stored in between is C1. When the capacitor is
the capacitor increases. [IIT1987] charged, the plate area covered by the
dielectric gets charge Q1 and the rest of
28. A parallel plate air capacitor is the area gets charge Q2. The electric field
connected to a battery. The quantities in the dielectric is E1 and that in the other
charge, voltage, electric field and energy portion is E2. Choose the correct
associated with capacitor are given by Q0, option/options, ignoring edge effects.
V0, E0 and U0 respectively. A dielectric
slab is now introduced to fill the space
between the plates with battery still in
connection. The corresponding quantities
now given by Q, V, E and U are related to
the previous ones as
(a) Q > Q0 (b) V > V0
E1 E1 1
(c) E > E0 (d) U > U0 [IIT 1985] (a) =1 (b) =
E2 E2 K
29. A dielectric slab of thickness d is
inserted in a parallel plate capacitor,

155
Q1 3 C 2+ K Instructions. This question contains
(c) = (d) = statement -1 (assertion) and statement - 2
Q2 K C1 K
(reason). Of these statements mark
[JEE Adv. 14] correct choice if
32. A parallel plate capacitor having (a) statements - 1 and 2 are true and
plates of area S and plate separation d, statement - 2 is a correct explanation
has capacitance C1 in air. When two for statement - 1.
dielectrics of different relative permittivities
(b) statements - 1 and 2 are true and
(ε1 = 2 and ε2 = 4) are introduced between
statement - 2 is not a correct
the two plates as shown in the figure, the
explanation for statement – 1
C2
capacitance becomes C2. The ratio is (c) statement - 1 is true, statement - 2 is
C1
false
(d) statement - 1 is false, statement - 2 is
true.
33. Statement -1: For practical
purposes, the earth is used as a reference
at zero potential in electrical circuits.
Statement - 2 : The electrical potential
of a sphere of radius R with charge Q
uniformly distributed on the surface is
Q
(a) 6/5 (b) 5/3 given by . [IIT 08]
4 π ε0 R
(c) 7/5 (d) 7/3 [JEE Adv. 15]
Reasoning Type

Answers and Explanations

1. (d)

2. (b) Electric potential decreases in


the direction of the electric field, so V A >
VB. Dotted lines represent equipotential
surfaces, so VA = VC.

3. (a) Given V = 4x2 volt

156
i.e., electric potential changes only along o
AC=2 R cos 6 0 =R
X-axis.
BC=2 R sin 6 0 o=√ 3 R
∴ E1 = -
dV −d 2
= ( 4 x ) = -8x
[ ]
2 2 2
dx dx 1 q 2q 2q
U= − − ≠0
Electric field at point (1m, 0, 2m) is 4 π ε 0 12 R 9 √ 3 R 9 R
Ex = -8 × 1 = -8 Vm-1. [Put x = 1 m] Option (b) is wrong.
4. (b) Electric force, between the charges at B
and C,
1 q 2q
F= ×
4 π ε0 3 3 ¿ ¿
Option (c) is correct.
1 2Qq
U i= ⋅ Potential at point O,
4 π ε0 a

U f=
Qq
[
1
+
1
4 π ε 0 a+ x a−x ] V=
1
[ q
+
q

2q
4 π ε0 3 R 3 R 3 R ]
Option (d) is wrong.
1 2Qq x 2
ΔU =u f −U i= . [For x <¿ a] 7. The arrangement is equivalent to
4 π ε0 a
3

∴ ΔU ∝ x 2 .
two capacitors connected in series. Each
such capacitor has plate separation d / 2
and hence capacitance 2 C. Total
5. (b) The net electrostatic energy, capacitance is

U= +
[
1 q.q Q.q Q.q
+
4 π ε0 a √ 2 a a
=0
] CS =
2C × 2 C
2 C+2 C
=C

or q √ 2+Q+ √ Q=0 i.e., the capacitance remains same.

−q √ 2 −2 q 8. (b) For a spherical conductor,


C = 4πε0R i.e., C ∝ R
or Q= = .
√ 2+ 1 2+ √ 2
6. (c) Electric field at O , As both the spheres have equal radii,
they have the same capacitance. When
2q 1 charged to the same potential, they will
E= ×
3 4 π ε 0 R2 have the same charge.
q 9. (a) For the two capacitors
¿ 2 connected in parallel,
6 π ε0 R
V1 = V 2
Option (a) is wrong.
C1 C2
or =
q1 q2
q1 C 1
or = .
q2 C 2
ε0 A
10. (d) Capacitance, C =
d
Potential energy of system, Electric field between the plates,

U=
1 q q
[
× ×
1 q −2 q
+ x
4 π ε0 3 3 2 R 3 3
×
1
BC( )
+
3
× ×
3 AC ( )
−2q E q= E1 -1E2
]

157
σ1 σ2 1 1 ∴
= − = (σ −σ )= (q −q ) κ ε0 A ε 0 A /2 ε 0 A /2
2 ε0 2 ε 0 2 ε0 1 2 2 ε0 A 1 2 = +
d d d d d d d d d
P.D. difference between the plates, d− − + + d− − + +
2 2 2 κ 1 2 κ3 2 2 2 κ 2 2 κ3
V = Ed
κ ε0 A ε0 A ε0 A
d q1−q2 = +
= ( q1−q2 )= . d d d d d
2 ε0 A 2C + +
κ 1 κ3 κ 2 κ 3
11. (c) Net charge on the isolated system
κ1 κ 3 κ 2 κ3
= 0. The capacitors of 2μF and 3μF are in κ= +
parallel. κ 1+ κ3 κ2 +κ 3
14. (a) When switch S is closed, there
is no transfer of charge from capacitor A
to capacitor B because the charge -q of A
is held by its charge +q.
15. (b) With the closing of switch S3,
the potential drops across C1 and C2
would become identical and equal to the
average of V1 and V2
30V + 20V
P.D. across each of them must be same. i.e., = 25 V.
2
80−x x 16. (a) At any instant t, level of the liquid =
=
2 3 d
∴ x = +48μC.
- vt
3
12. (a) ∆U = Ui - Uf The arrangement can be regarded as
a series combination of two capacitances

( )
2
1 2 2 1 V 1 +V 2 with A = unit area.
= (V 1+V 2)− (2C )
2 2 2
1 2
= C (V 1−V 2 ) .
4
13. (d) The given arrangement can be ε0

represented as follows : C 1=
2d
+ vt
3
2 ε0
C 2=
and d
−vt
3
2d d
+vt −v t
For each portion, we use 1 1 1 3 3
= + = +
ε0 A Ceq C 1 C2 ε0 2 ε0
C= t1 t2 6 ε0
d−t 1−t 2+ + or C eq =
κ1 κ2 3 vt+5 d

Time constant, τ¿ R C eq ¿
As C = C1 + C2 6 ε0 R
.
3 v t+5 d

158
1 2 1 2 2
17. (d) U i= C 1 V = ×2 ×V =V
2 2
2 2
1 C1V 1
U f= = .¿¿
2 C1 +C 2 2
2 2
V 4V
Loss of energy ¿ V 2− =
5 5
2
4V
% loss 5 4
¿ 2 ×100= × 100=80 %
V 5
18. (d) Potential at the origin will be

V=
1 q
[−
q
+
q

q
4 π ε0 x0 2 x 0 3 x 0 4 x 0
+…
]
(a) Option (a) is correct because the entire
=
q
4 π ε0 x 0 [
1 1 1
1− + − +…
2 3 4 ] charge Q lies within a sphere of radius r =
2 R0.

=
q
4 π ε0 x 0
ln 2. (b) For r < R0, |E| = |dVdr |
= 0,

| dr | 4 π ε r
19. (a) The potential at any point dV Q
inside the shell will be equal to that on its For r < R0, |E| = = 2

∴ Electric field is discontinuous at r = R0.


0
surface. When the shell is given a charge
-3Q, the potential on its surface changes
by same amount as that inside. Hence the
potential difference V between the The option (b) is correct.
surfaces of solid sphere and hollow (c) As V changes continuously for r > R0,
sphere remains unchanged. the option (c) is wrong.
20. (c) and (d) Under electrostatic (d) For r < R0, E = 0. Hence there is no
condition, all points lying on the conductor electrostatic energy for r < R0. The
are at the same potential. Therefore, option (d) is correct.
potential at A = potential at B. Hence
option (c) is correct. From Gauss's 22. (b) W = q(VA - VB)
theorem, total flux through the surface of
9
the cavity = .
ε0
21. (a), (b) and (d) The given V-r graph
is for a charged solid or hollow metallic
sphere.
VA =
[
1 Q1 Q2
+
4 π ε 0 R √2 R ]
VB =
[
1 Q 2 Q1
+
4 π ε 0 R √2 R ]
q(Q 1−Q2)( √ 2−1)
∴W= .
4 √2 π ε0 R
23. (c), (d) Gauss law is applicable
only to that electric field which obeys

159
inverse square law of distance. So option 4 3
(a) is incorrect.
πr ρ
E × 4πr = 3
2

symmetric, so the integral ∮ ⃗


As the field of an electric dipole is not ε0
E . d ⃗S cannot
be evaluated. Option (b) is incorrect.
In case of similar charges, null point
lies between the two charges. Option (c) is
correct.
Wext = q(VB - VA) .So option (d) is


correct for a unit positive charge. ρ
E= r
24. (a), (b), (c) 3 ε0

∴ ⃗
Net electric field at O due to the ρ ⃗ −p⃗
charges at B and E, EP ¿ ⃗ E¿
E +⃗ C P+
3 ε0 1
C P
3 ε0 2
1 q 1 q
E1 = + = 2 K, along O⃗
E ρ ⃗ ⃗ ρ ⃗
4 π ε 0 L 4 π ε 0 L2
2
¿ ( C1 P−C 2 P)= C C
3 ε0 3 ε0 1

[⃗
C 1 C +⃗ C1 P ]
C2 P=⃗
⇒ E is constant in magnitude and
direction.
Moreover, electrostatic potential is not
constant because E is a non-zero field.
26. (b), (d)
When S1 is pressed and released,
Similarly, net electric field at O due to
the charges at C and F,
E2 = 2K, along ⃗
OC
Net field at O due to the charges at A
and D,
E3 =
1 2q 1 2q When S2 is pressed and released,
⋅ + ⋅ =4 K , along ⃗
OD.
4 π ε 0 L 2 4 π ε 0 L2
Hence, total electric field at O,
E = E1 cos60°+ E2 COS60°+ E3
= 2 × 2Kcos60°+ 4K = 6K, along ⃗
OD
Potential at O,
When S3 is pressed,
V= [
1 2q q q 2q q q
+ − − − + =0
4 π ε0 L L L L L L ]
Due to the symmetry of the distribution
of opposite charges about the line PR, the
potential at all points on the line PR is
zero or same.
25. (c), (d) 27. (b) and (d)
For r < R,

160
Hence the direction of the field remains
the same from +ve plate to -ve plate.
The electric potential increases from -
ve plate to +ve plate. Due to the weaker
electric field, the increase in potential is
Charge on plate = q Charge on plate = q less inside the dielectric slab. The
ε0 A ε0 A variations of electric field E and potential V
C 0= C= ⇒ C <C 0 with distance x are shown below :
d0 d
q q
V 0= V= ⇒ V >V 0
C0 C
1 1
U0= q V 0 U = qV ⇒ U >u0
2 2

∴ Options (b) and (d) are correct.


28. (a) and (b)

30. (c) In the annular region of a


charged cylindrical capacitor,
λ
E=
P.D. = V0 P.D. = V0 2 π ε0 r

i.e., E ∝
Capacitance = C0 Capacitance = K C0 1
r
where λ is the charge per unit length of
Q0 = C0V0 Q = KC0V0
1 2 1 2
U0= C0 V 0 U = K C0V 0 the capacitor.
2 2
31. (a), (d)
V0 V0
E0 = E= As both parts have the same potential
d d
difference,
∴ Options (a) and (d) are correct.

V E1
E1 = E 2 = =1
29. (b) and (c) The situation is shown in d E2
∴ Option (a) is correct and option (b) is
the figure.
incorrect.
K ε 0 A /3
Now, C 1=
d
ε 0 2 A /3
C2 =
d

∴ C=C 1+C 2=
(K + 2)ε 0 A
Due to polarisation of the dielectric, a field 3d
Ei is induced in the opposite direction of
⇒ ∴ Option (d) is correct.
original field ⃗
E 0. But net field ⃗
E net = ⃗
E0 - ⃗
Ei C K+ 2
=
is in the direction of ⃗
E 0. C1 K

161
Again, Q 1=C 1 V =
K ε0 A S
V 4ε0
3d 2 4 ε0 S
C '= = =4 C1 '
d d
2 ε0 A
Q2=C 2 V = 2
3d
S
∴ = ⇒ Option (c) is incorrect.
Q1 K 2ε0
2 ε0 S
Q2 2 C '= = =C 1
d d

∴ C 2=
32. (d) '
2C 1 × 4 C 1
CC ''
+C = +C 1
C +C
'
2 C 1+ 4 C 1
4 7
¿ C1 +C 1= C 1
3 3
C2 7
Hence, = .
C1 3
33. (b) As radius of the earth is very
large, so the potential on the earth can be
S taken equal to zero.
2 ε0
ε0 S 2 2ε0 S
C 1= , C= = =2 C 1
d d d
2

JEE Main

1. Two points P and Q are maintained 3. Two spherical conductors A and B


at the potentials of 10 V and -4 V of radii 1 mm and 2 mm are separated by
respectively. The work done in moving a distance of 5 cm and are uniformly
100 electrons from P and Q is charged. If the spheres are connected by
a conducting wire, then in equilibrium
(a) -19× 10-17J (b) 9.60 × 10-17 J
condition, the ratio of the magnitudes of
(c) -2.24 × 10-16 J (d) 2.24× 10-16 J the electric fields at the surfaces of
[AIEEE 2009] spheres A and B is

2. Thin spherical conducting shell of (a) 2:1 (b) 1:4


radius R has a charge q. Another charge (c) 4 : 1 (d) 1 : 2 [AIEEE
Q is placed at the centre of the shell. The 2006]
electrostatic potential at a point P at a
4. A charged particle q is shot towards
distance R / 2 from the centre of the shell
another charged particle Q which is fixed,
is
with a speed v. It approaches upto a
2Q closest distance r and then returns.
(a) (b)
4 π ε0 R
2Q 2q

4 π ε0 R 4 π ε 0 R
If q was given a speed 2v, the closest
2Q q (q +Q)2 distance of approach would be
(c) + (d)
4 π ε0 R 4 π ε0 R 4 π ε0 R
(a) r (b) 2r
[AIEEE 2003]

162
r r 20
(c) (d) [AIEEE 2004] V(x) = volts
2 4 ( x −4 )
2

5. Two thin wire rings each having a The electric field E at x = 4μm is given by
radius R are placed at a distance d apart
with their axes coinciding. The charges on 10
(a) volt / μm and in the +ve x-direction.
the two rings are + q and -q. The potential 9
difference between the centres of the two
5
rings is (b) volt / μm and in the -ve x-direction.
3
(a)
q R
4 π ε0 d 2
(b)
q
( 1

1
2 π ε 0 R √ R 2+ d 2 ) (c)
5
3
volt / μm and in the +ve x-direction.

(c) zero (c)


q
( 1 1
− 2 2
4 π ε0 R √ R + d ) (d)
10
9
volt / μm and in the -ve x-direction.

[AIEEE 2005] [AIEEE 2007]

6. Charges are placed on the vertices of a 9. The electrostatic potential inside a


square as shown. Let ⃗ E be the electric charged spherical ball is given by ϕ = ar2 +
field and V the potential at b, where r is the distance from the centre ;
a, b are constants. Then the charge
density inside the ball is
(a) -24 πaε0r (b) -6aε0r
(c) -24πaε0 (d) -6qε0 [AIEEE
2011]
10. Assume that an electric field ⃗E = 30x2i^
exits in space. Then the potential
difference VA - V0, where V0 is the potential
the centre. If the charges on A and B are at the origin and VA the potential at x = 2
intercharged with those on D and C m is
respectively, then
(a) 120 V (b) -120 V
(a) ⃗
E changes, V remains unchanged.
(c) -80 V (d) 80 V [JEE Main
(b) ⃗
E remains unchanged, V changes. 14]
(c) both ⃗
E and V change. 11. An electric field ⃗E = (25i^ + 30 ^j )NC-1
exists in a region of space. If the potential
(d) ⃗
E and V remain unchanged. [AIEEE
at the origin is taken to be zero, then the
2007]
potential at x = 2 m, y = 2 m is
7. An electric charge 10-3μ C is placed at
(a) -110 J (b) -140 J
the origin (0, 0) of X-Y co-ordinate system.
Two points A and B are situated at ( √ 2, (c) -120 J (d) -130 J [JEE Main
√ 2) and (2,0) respectively. The potential 15]
difference between the points A and B will 12. A charge Q is uniformly distributed
be over a long rod AB of length L as shown in
(a) 4.5 volt (b) 9 volt the figure. The electric

(c) zero (d) 2 volt [AIEEE 2007]


8. The potential at a point x (measured in
μ m) due to some charges situated on the potential at the point O lying at distance L
X-axis is given by from the end A is

163
Q 3Q 18. If there are n capacitors in parallel
(a) (b) connected to V volt source, then the
8 π ε0 L 4 π ε0 L
energy stored is equal to
Q Qln 2
(c) (d) 1
4 π ε 0 Lln 2 4 π ε0 L (a) CV (b) nCV2
2
[JEE Main 13]
1
13. A uniformly charged solid sphere of (c) CV2 (d) CV2
2n
radius R has potential V0 (measured with
respect to ∞) on its surface. For this [AIEEE 2002]
sphere the equipotential surfaces with 19. A parallel plate capacitor is made by
3V 0 5 V 0 3 V 0 V0 stacking n equally spaced plates
Potentials , , and have
connected alternatively. If the capacitance
2 4 4 4
radii R1, R2, R3 and R4 respectively. Then between any two adjacent plates is C,
then the resultant capacitance is
(a) R1 = 0 and R2 > (R4 – R3)
(a) (n - 1) C (b) (n + 1) C
(b) R1 ≠ 0 and (R2 - R1) > (R4 - R3)
(c)C (d) nC [AIEEE2005]
(c) R1 = 0 and R2 < (R4 - R3)
20. A fully charged capacitor has a
(d) 2R < R4 [JEE Main 15] capacitance C. It is discharged through a
14. Capacitance (in F) of a spherical small coil of resistance wire embedded in
conductor of radius 1 m is a thermally insulated block of specific heat

of the block is raised by ∆T, then the


capacity s and mass m. If the temperature
(a) 1.1 × 10-10 (b) 10-6
(c) 9 × 10-9 (d) 10-3 [AIEEE 2002] potential difference V across the capacitor
is
15. The work done in placing a charge of


8 × 10-18 coulomb on a capacitor of 2 mCΔT mCΔT
capacity 100 microfarad is : (a) (b)
s s
(a) 16 × 10-32 joule (b) 3.1 × 10-26 joule
(c) 4 × 10-10 joule (d) 32 × 10-32 joule
16. A parallel plate capacitor is made of
(c)
msΔT
C
(d)
√ 2 msΔT
C
[AIEEE 2005]
two circular plates separated by a
distance of 5 mm and with a dielectric of 21. A battery is used to charge a
dielectric constant 2.2 between them. parallel plate capacitor till the potential
difference between the plates becomes
When the electric field in the dielectric is 3 equal to the electromotive force of the
× 104Vm-1, the charge density of the battery. The ratio of the energy stored in
positive plate will be close to the capacitor and the work done by the
(a) 6 × 10-7C/m2 (b) 3 × 10-7C/m2 battery will be
(c) 3 × 104C/m2 (b) 6 × 104C/m2 (a) 1/2 (b) 1
[JEE Main 1.4] (c) 2 (d) 1/4 [AIEEE 2007]

dielectric of dielectric constant κ between


17. Two capacitors C1 and C2 are charged 22. A parallel plate condenser with a
to 120 V and 200 V respectively. It is
found that by connecting them together the plates has a capacity C and is charged
the potential on each one can be made to a potential of V volts. The dielectric ??
zero. Then is slowly removed ?? between are plates
and then reinserted. The net work done by
(a) 5C1 = 3C2 (b) 3C1 = 5C2
the system in this process is
(c) 3C1 = 5C2 (d) 9C1 = 4C2
(a) zero (b) 1/2 (κ-1)CV2
[JEE Main 13]
(c) CV2(κ -1)4κ (d) (κ -1)CV2 [AIEEE

164
2007]
23. A parallel plate capacitor with air
between the plates has a capacitance of 9
pF. The separation between its plates is
now filled with two dielectrics. One of the
dielectrics has dielectric constant Kj =3

has dielectric constant κ2 =6 and


and thickness d / 3 while the other one
(c)
thickness 2d /3. Capacitance of the
capacitor is now
(a) 20.25 pF (b) 1.8 pF
(c) 45 pF (d) 40.5 pF [AIEEE 2008]
24. If the capacitance of a
nanocapacitor is measured in terms of a
unit ‘u’ made by combining the electronic (d)
charge ‘e’, Bohr radius ‘a0’, Planck's
constant ‘h’ and speed of light ‘c’, then
2 2
e h e c
(a) u = (b) u =
e a0 h a0
hc 2
e a0
(c) u = 2 (d) u =
e a0 hc
[JEE Main 15] 26. This question contains statement -
1 and statement - 2. Of the four choices
25. In the given circuit, charge Q2 on the given after the statements, choose the
2Μf capacitor changes as C is varied from one that best describes the two
1 μF to 3 μF. Q2 as a function of C is given statements.
properly (figures are drawn schematically
and are not to scale) by [JEE Main 15] Statement 1 : For a charged particle
moving from point P to point Q, the net
work done by an electrostatic field on the
particle is independent of the path
connecting point P to point Q.
Statement 2 : The net work done by a
conservative force on the object moving
along a closed loop is zero.
(a) (a) Statement - 1 is true, Statement - 2 is
false
(b) Statement - 1 is true, Statement - 2 is
true ; Statement - 2 is the correct
explanation of Statement — 1
(c) Statement - 1 is true, Statement - 2 is
true ; Statement -- 2 is not the correct
explanation of Statement -1
(b)
(d) Statement - 1 is false, Statement - 2 is
true
[AIEEE 2009]
27. This question has statement-1 and
statement-2. Of the four choices given

165
after the statements, choose the one that (c) Statement-1 is false Statement-2 is
best describes the two statements. true.
An insulating solid sphere of radius R (d) Statement-1 is true, Statement-2 is
has a uniformly positive charge density p. true, Statement-2 is the correct
As a result of this uniform charge explanation of Statement-1.
distribution, there is a finite value of [AIEEE 12]
electric potential at the centre of the
28. In the figure, is shown a system of
sphere, at the surface of the sphere and
four capacitors connected across a 10 V
also at a point outside the sphere. The
battery. Charge that will flow from switch S
electric potential at infinity is zero.
when it is closed is
Statement 1. When a charge ‘q’ is taken
from the centre of the surface of the
sphere, its potential energy changes by

.
3 ε0
Statement 2. The electric field at a
distance r(r < R) from the centre of the
ρr
sphere is .
3 ε0 (a) 5 μC from b to a
(a) Statement-1 is true, Statement-2 is (b) 20 μC from a to b
true ; Statement-2 is not the correct (c) zero
explanation of statement-1.
(d) 5 μC from a to b
(b) Statement-1 is true Statement-2 is
false. [JEE Main 15]

Answers and Explanations

1. (d) Q = 100 e 1 q 2Q
V =V 1+V 2= . + .
= -100 × 1.6 × 10 -19
= -1.6 × 10 -17
C 4 π ε0 R 4 π ε0 R
∆V = VQ - VP 3. (a) When the two conductors are
connected by a wire, they share charges
= -4 – 10 = -14 V
∴ W = Q∆V
till their potentials become equal. That is
V 1=V 2
= 14 × 1.6 × 10-17 = 2.24 × 10-16 J.
1 q1 1 q2 q1 r 1
2. (c) Potential at point P due to the . = . or =
4 π ε0 r1 4 π ε0 r2 q2 r 2
charge q on the spherical shell is
1 q 1 q1
V 1= . Now, E1= . 2
4 π ε0 R 4 π ε0 r1

Potential at point P due to the charge Q at 1 q2


the centre of shell is and E2= . 2
4 π ε0 r2
1 Q 2Q

V 2= . = 2 2
E 1 q1 r 2 r 1 r 2 r 2 2
4 π ε 0 R /2 4 π ε 0 R = . = . = = =2: 1
E2 q2 r 21 r 2 r 21 r 1 1
Net potential at point P is

166
4. (d) When the charge q is shot with 7. (c) The situation is shown in the figure.
a speed v.
At the distance of closest approach r,
Initial K.E. of charge q = P.E. of charges q
and Q
1 Qq
mv2 = k
2 r
When the charge q is shot with a speed
2v. r 1=OA =√ ¿ ¿
Let r’ be the distance of closest approach. r 2=OB =2−0=2
Then
1 q
Qq V A= .
1 4 π ε0 r1
m(2v)2 = k '
2 r −3 −6
1 1 0 ×1 0
Qq Qq r ¿
or 4 × k = k ' or r’ = . 4 π ε0 2
r r 4 −3 −6
1 q 1 10 ×10
5. The two charged rings are shown in the V B= . = .
4 π ε0 r2 4 π ε0 2
∴ V A −V B =0.
figure.

20
8. (a) Given V (x )= 2 volts
x −4
dV
E=
dx
V A=
1
[q −q
+ 2 2
4 π ε0 R √ R + d ] ¿ 20 ¿
At x=4 μm,
V B=
1 −q
[ +
q
4 π ε 0 R √ R2 +d 2 ] E=
40 ×4
¿¿
∴ ¿
10
V/μm, in the +ve x‐direction

[ ]
1 q q q q 9
V A −V B = − 2 2+ − 2 2
4 π ε 0 R √ R +d R √ R + d 9. (d) Potential, ϕ=a r 2 +b

[ ]
q 1 1 −dϕ
¿ − 2 2 Electric field, E= =−2ar
2 π ε 0 R √ R +d dl
Using Gauss’ law,
6. A unit positive charge placed at the
centre will be repelled by charges at A and 4 3
ρ× πr
B, and attracted by charges at C and D. 3
The net field will be in the downward −2 ar × 4 π r 2=
ε0
∴ ρ=−6 a ε 0
direction when the charges are
interchanged, the direction of the field is
reversed. Electric potential, being scalar,
remains zero whatever be the position of E.⃗
10. (c) dV =−⃗ dx


the charges. VA

∴ ⃗
2

E changes while V remains ∫ d V =−∫ 3 0 x 2 dx


V0 0
unchanged.

167
or V A −V 0=−¿ 1 −10
C=4 π ε 0 r = 9
×1=1.1× 1 0 F .
11. (a) Potential at x = 2 m, y = 2 m is 9 ×1 0
2
V = - 1 q
x=2 y=2 2 2
15. (d) W = .
2 C
∫ E x dx− ∫ E y dy=−∫ 25 dx−∫ 30 dy 1
0 0 0 0
¿ ׿ ¿
2 2 2
= −[25 x ]0−[30 y ]0=−50−60 = -110 V.
σ
12. (d) 16. (a) E =
κ ε0
∴ σ = κε0 E = 2.2 × 8.85 × 10-12 × 3 × 104
= 6 × 10-7C/m2.
2L 2L 17. (c) For potential to be made zero, after
1 dq 1 λdx
V =∫ = ∫ connection
L 4 π ε0 x 4 π ε 0 L x
λ 2L λ 2 L Q ln 2
¿ ¿In x ¿L = . ln = .
4 π ε0 4 π ε0 L 4 π ε0 L
13. (c), (d) On the surface (r =R) ,
kQ
V 0=
R 120 C1 = 200C2
Inside the sphere (r < R) , ⇒ 3C1 = 5C2.
kQ 2 2
V = 3 (3 R −r ) 18. (b) For parallel combination,
2R
∴ At the centre (r =0),
V = constant
∴ U = n × energy stored in a single
3 kQ 3 capacitor
V= = V ⇒ R 1=0
2 R 2 0
1 1
5 V 0 5 kQ =π× CV2 = nCV2.
kQ 2 2 2 2
At r ¿ R2 ,V = 3
(3 R −R 2 )= =
2R 4 4 R 19. (a) When the alternate plates of


2 capacitor consisting of n plates are
5 R R
2
=3− ⇒ R 2= connected, it gives rise to a parallel
2 R √2 combination of (n - 1) capacitors, each of
capacitance C.
∴ Resultant capacitance = (n - 1) C.
kQ
Outside the sphere (r > R), V =
γ
3V 0 kQ V 0 R 4 20. (d) Energy released by the capacitor
At r =R 3 , = = ⇒ R 3= R
4 R3 R3 3 = Heat gained by the block


V 0 kQ V 0 R 1 2 msΔT .
At r =R 4 , = = ⇒ R4 =4 R >2 R or CV2 = ms∆T or V =
4 R4 R4 2 C
4 8 21. (a) Let V be the emf of the battery.
R4 −R3=4 R− R= R> R 2 Work done by the battery,
3 3
Hence, options (c) and (d) are correct. W = QV = CV2

14. (a) For a spherical conductor, Energy stored in the capacitor,


1
U= CV2
2

168
1 a = 2, b = 1, c = -1, d = -1
∴U 2 1.
2
CV
2
= = e a0
W CV
2
2 u= .
hc
22. (a) The potential energy stored in
3C
the charged capacitor with dielectric 25. (b) Ceq =
between its plates is 3+C

U i=
q
2 κC
2
Total charge, Q = ( 3+C
3C

When the dielectric is removed, the Charge on 2 μF capacitor,
stored energy increases to the value given 2 2 3 Cε 2 Cε 2ε
by Q= × = =
Q2 = 2+ 1 3 3+C 3+C 3
2 1+
q C
U f=
2C dQ2 2
d Q2
When the dielectric is again inserted, Clearly, > 0 and 2
<0
dC dC
⇒ Q2 - C curve has a decreasing slope.
the energy again decreases from Uf to Ui.
Hence net work done = 0.
Hence, option (b) is correct.
23. (d) Original capacitance,
26. (b) Work done by conservative
ε0 A
C= =9 pF force does not depend on the path.
d Electrostatic force is a conservative force.
With dielectric, 27. (c) Electric field at r < R is
κ ε0 A q 1 4 3
C= ϕE = E× 4πr2 = = × πr ρ
d ε0 ε0 3

∴E= ⇒ Statement 2 is true.


3r
3 ε0
When charge q taken from the centre
to the surface of the sphere, the work
done is

∴ C 1=
3 ε0 A R R R

d /3
6 ε0 A
=9 C
W = ∫ qEdr=∫ q
0 0
ρr
3 ε0
dr=
( )

∫ rdr
3 ε0 0

[ ]
C2 = =9 C qρ r 2
R
qp R2
2 d /3 = =
3 ε0 2 0 6 ε0
Now, C1 and C2 form a series combination.

∴ C Final = ⇒ Statement 1 is false.


2
C 1 C 2 9 C × 9C qρ R
= ΔU = W =
C 1 +C 2 9C +9 C 6 ε0

9 9 28. (a) Before switch S is closed


= × C= ×9
2 2 2× 3 6
C1 = = μF
= 40.5 pF. 2+3 5
24. (d) [u] = [e]a[a0]b[h]c[c]d
[M-1L-2T+4A+2] = [A1T1]a[L]b[ML2T-1]c[LT-1]d
[M-1L-2T+4A+2] = [McLb+2c+dTa-c-dAa]

169
3× 2 6
C2 = = μF C1 and C2 are in series,
3+5 5
5× 5
6 C= = 2.5 μF
C1 = C 2 = μF 5+5
5
Charges on combinations C1 and C2 will
C1 and C2 are in parallel. Charge on
be same.
each (C1, C2) will remain same.
q = CV
q = C1V = C2V
= 2.5 μF × 10 V = 25 μC
6
= μF × 10 V = 12μC These charges split on individual
5
capacitors in proportion to their individual
After switch S is closed capacitances as shown.
5 μC charge will flow from b to a.

IPU CET and other Engineering Entrance Exams

1. The potential to which a conductor kinetic energy of proton ?


is raised, depends on
1
(a) the amount of charge (a) 1 keV (b)
1840 keV
(b) geometry and size of the conductor (c) 1840 keV (d) 1 kV [DCE 2001]
(c) both (a) and (b) 4. Three charges 1 μC, 2 μC, 3 μC are
(d) only on (a) [IPUEE 2006] kept at vertices of an equilateral triangle of
side 1 m. If they are brought nearer, so
2. What is the electric potential at the that they now form an equilateral triangle
centre of the square ? of side 0.5 m, then work done is
(a) 11 J (b) 1.1 J
(c) 0.01 J (d) 0.11 J [DCE 2007]
5. Two charges q1 and q2 are placed 30
cm apart, as shown in the figure. A third
charge q3 is moved along
(a) zero
kq kq
(b) (c)
a √2 a
2

(d) none of the these [DCE 1997, 2003]


3. Mass of a proton is 1840 times that
of an electron. It is accelerated through a
potential difference of 1 kV, what is the

170
minimum when the angle between its axis
and field is
the arc of a circle of radius 40 cm from C
to D. The change in the potential energy π
q3 (a) zero (b) π (c) (d) 2π [DCE 1998]
2
of the system is k , where k is
4 π ε0 12. A dipole is placed parallel to the
(a) 8q2 (b) 8q1 electric field. If W is the work done in
rotating the dipole by 60°; then work done
(c) 6q2 (d) 6ql [DCE 06, CBSE 05] in rotating it by 180° is
6. What is angle between electric field (a) 2 W (b)3W
and equipotential surface ?
W
(a) 90° always (b) 0° always (c) 4W (d) [DCE 2005]
2
(c) 0° to 90° (d) 0° to 180° [DCE
13. A hollow conducting sphere is
2006]
placed in an electric field produced by a
7. Electric potential at any point is V = -5x point charge placed at P as
+ 3y + √ 15z, then the magnitude of the
electric field is
(a) 3√ 2 (b) 4√ 2
(c) 5√ 2 (d) 7 [IPUEE 10]
8. A hollow charged metal sphere shown in the figure. Let VA, VB, VC be the
has a radius r. If the potential difference potentials at points A, B and C
between its surface and a point at respectively.
distance 3r from the centre is V, then the
electric intensity at distance 3r from the Then
centre is (a) VC > VB (b) VB > VC
(a) V/6r (b) V/4r (c) VA > VB (d) VA = VC [IPUEE
(c) V/3r (d) V/2r [IPUEE 13] 2005]
9. The insulation property of air 14. In the case of a charged metallic
breaks down at E = 3 × 106 V/m. The sphere, potential (V) changes with respect
maximum charge that can be given to a to distance (r) from the centre as [DCE
sphere of diameter 5 m is approximately 2003]
(in coulomb) (a)
-2 -3
(a) 2 × 10 (b) 2 × 10
(c) 2 × 10-4 (d) 2 × 10-5 [IPUEE 10]
10. A positive point charge +q is
placed at the origin.
There is an electric field E(x)= E0

( )
2
x x
2 +3 2 that accelerates the point (b)
d d
charge along the x-axis. Determine the
energy of the charge when it reaches the
position x = 2d.
(a) 6qdE0 (b) 12q
(c) 12 qdE0 (d) 24qdE0 [IPUEE 15]
11. A dipole is placed in a uniform (c)
electric field, its potential energy will be

171
(a) 2V (b) 4 V
(c) 1V (d) 6 V [DCE 2003]
20. The equivalent capacitance is

(d)

(a) 15 μF (b) 20 μF
(c) 25 μF (d) 30 μF [DCE 2000]
21. Equivalent capacitance between A
15. n small metal drops of same size and B is
are charged to V volt each. If they
coalesce to form a single large drop, then
its potential will be
V
(a) (b) Vn
n
(c) Vn1/3 (d) Vn2/3 [IPUEE 2005]
16. A spherical drop of mercury having
a potential of 2.5 V is obtained as a result
of merging 125 droplets. The potential of a
constituent droplet would be (a) 8 μF
(a) 1.0 V (b) 0.5 V (b) 6 μF
(c) 0.2 V (d)0.1V [IPUEE (c) 268 μF
2005J
10
17. 8 drops of Hg are combined to (d) [DCE 2001]
38 μF
form a bigger single drop. The
capacitance of a single small drop and 22. A parallel plate air capacitor is
that of the single big drop will be in the charged and then isolated. When a
ratio of dielectric material is inserted between the
plates of the capacitor, then which of the
(a) 1:2 (b) 1:8 following does not change ?
(c) 8 : 1 (d) none of these [DCE (a) Electric field between the plates
1998]
(b) Potential difference across the plates
18. A parallel plate capacitor C has a
charge Q. The actual charges on its plates (c) Charge on the plates
are (d) Energy stored in the capacitor [IPUEE
(a) Q/Q (b) Q/2, Q/2 2005]

(c)Q,-Q (d) Q/2,-Q/2 [IPUEE 23. An air filled parallel plate


2015] condenser has a capacity of 2 pF. The
separation of the plates is doubled and the
19. Three capacitors of capacitances 1 interspace between the plates is filled with
μF, 2 μF and 3 μF are connected in series wax. If the capacity is increased to 6 pF,
and a p.d. of 11 V is applied across the the dielectric constant of wax is
combination. Then, the p.d. across the
plates of 1 μF capacitor is (a) 2 (b) 3

172
(c) 4 (d) 6 [IPUEE 2006] A ε 0 (κ1 +κ 2) (κ 1 +κ 2)
(a) (b) A ε 0
24. A parallel plate air capacitor has a 2 2 2d
capacitance C. When it is half filled with a
dielectric of dielectric constant 5, the
percentage increase in the capacitance
(c) (
A ε 0 κ 1 κ2
2 d κ1 +κ 2 ) (d) (
A ε 0 κ1 κ 2
d K 1+ κ 2 )
will be
[DCE 1999, AIIMS 01]
(a) 400% (b) 66.6%
28. A capacitor of capacitance 1 μF is
(c) 33.3% (d) 200% [IPUEE filled with two dielectrics of dielectric
2007] constants 4 and 6. What is the new
25. Two capacitors of capacitance C capacitance ?

filled with dielectric substance κ, what is


are connected in series. If one of them is

the effective capacitance ?


κC
(a) (b) C(κ + 1)
(1+ K )
2κC
(c) (d) none of these [DCE 2007]
1+ K (a) 10 μF (b) 5 μF
26. A parallel plate capacitor of plate area (c) 4 μF (d) 7 μF [DCE 2006]
A and separation d is filled with dielectric
29. A 10 microfarad capacitor is
as shown in the
charged to 500 V and then its plates are
joined together through a resistance of 10
ohm. The heat produced in the resistance
is
(a) 500 J (b) 250 J
(c) 125 J (d) 1.25 J [IPUEE 2005,08]

figure. The dielectric constants are κ1 and


30. If there are n capacitors in parallel
κ2. Net capacitance is
connected to V volt source, then the
energy stored is equal to

(a)
ε0 A
d
(κ 1+ κ2 ) (b)
d (
ε 0 A κ1 +κ 2
κ 1 κ2 ) (a) cv (b)
1
2
nCV2

1
(c)
d (
2 ε0 A κ 1 κ 2
κ 1 +κ 2 ) (d) (c) CV2 (d)
2n
CV2

[DCE 2002]
d (
2 ε 0 A κ 1 +κ 2
κ1κ2 ) 31. A capacitor is charged by
connecting a battery across its plates. It
[DCE 2002] stores energy U. Now the battery is
disconnected and another identical
27. What is the equivalent capacitance
capacitor is connected across it, then the
of the arrangement given in the figure, if A
energy stored by both capacitors of the
is area ?
system will be
U
(a) U (b)
2
3
(c) 2U (d) U [DCE 2000]
2
32. A capacitor of capacitance C has

173
charge Q and stored energy is W. If the (c) remain the same (d) become zero
charge is increased to 2 Q, the stored [DCE 2009]
energy will be
35. A parallel plate capacitor is
W w charged. If the plates are pulled apart,
(a) (b)
4 2 (a) the capacitance increases
(c) 2 W (d) 4W [IPUEE 08] (b) the potential difference increases
33. If the charge of 10 μC and -2 μC are (c) the total charge increases
given to two plates of a capacitor which
are connected across a battery of 12 V, (d) the charge and potential difference
find the capacitance of the capacitor. remain the same. [DCE 2009]

(a) 0.33 μF (b) 0.5 μF 36. The capacity of a spherical


condenser is 1 μF. If the spacing between
(c) 0.41 μF (d) 0.66 μF [IPUEE 08] the two spheres is 1 mm, the radius of the
34. An air capacitor is charged with an outer sphere is
amount of charge q and dipped into an oil (a) 3 cm (b) 6 cm
tank. If the oil is pumped out, the electric
field between the plates will (c) 3 m (d) 6 m [IPUEE 14]

(a) increase (b) decrease


Answers and Explanations

Q d W =uf −U i=(198−99)×1 0 =99 ×1 0 J


−3 −3
1. (c) V = = Q.
C ε0 A
¿ 0.099 J ≈ 0.01 J.
Thus the potential on a conductor
depends on the amount of charge, 5. (a) ΔU =U f −U i
geometry and size of the conductor. or

2. (a) V =
4 π ε0 [
1 + q−q+q−q
√2 a
= 0. ]
q3
4 π ε0
k=
1
[
q1 q 2 q2 q3 q1 q3
+ +
4 π ε 0 0 . 3 0 . 1 0.4
− ] + [
1 q 1 q 2 q2 . q3
4 π ε0 0 . 3 0 . 5
+

3. (a) K.E. of the proton = Work done on


the proton
or q 3 k=q 2 q 3 [ 1

1
0 .1 0.5 ]
= 8 q2 q3

= q∆V = e × 1kV = 1keV. or k =8 q 2 .


4. (c) Initial P.E. of the three charges, 6. (a) Electric field is always perpendicular
1 q 1 q 2 + q2 q3 + q1 q3 to the equipotential surface at any point.
U j= .
4 π ε0 r 7. (d) V =−5 x +3 y + √ 15 z

[ ]
9 1× 2× 1 0
−12
+ 2× 3× 1 0
−12
+ 1× 3× 10
−12 ⃗
E =−(−5 i+ ^ √ 15 k^ )
^ s i+
¿ 9 ×1 0
1 ¿ 5 î−3 ^j−√ 15 k^
−3
¿ 99 ×1 0 J |⃗E|=√ 52 +¿ ¿
Final P.E. of the three charges,
1 q q
− (=V )
[ ]
−12 −12 8.
−12(a) Given
9 1 ×2 ×1 0 +2× 3 ×1 0 +1× 3 ×1 0 4 π ε0 r 3 r
U f =9 × 10
05
1 2q
−3 or ⋅ =V
99 ×1 0 −3 4 π ε0 3 r
¿ =198 ×1 0 J
0.5

174
1 q 3V 1 q
or . = V=
4 π ε0 r 2 4 π ε0 R

∴ E=
1 q At outside points,
.
4 π ε0 ¿ ¿ 1 q 1
V= i.e., V ∝ .
= ⋅ = .
3V 1 V 4 π ε0 r r
2 9r 6r Hence option (b) is correct.
1 q 9 q 15. (d) Volume of the big drop
9. (a) E= . 2 =9 ×1 0 × 2
4 π ε0 r r = n × Volume of a small drop
2
E i 6 4 3 4 3
or q= 9
=3 ×1 0 × ¿ ¿ π R =n × π r
q ×1 0 3 3
q should be less than 2.08 ×1 0−3 C ∴ R=n1/ 3r
So option (b) is correct. Potential of a small drop,

( )
χ x
2 1 q
10. (c) E(x )=E 0 2 +3 2 V=
d 4 π ε0 r
d
Potential of the big drop,
Potential at x=2 d will be
1 nq 1 nq 2/ 3

( )
2d
χ x
2 V '= = =n V .
V =E 0∫ 2 + 3 2 dx 4 π ε0 R 4 π ε 0 n r
1/ 3

0 d d
16. (d) Potential of big drop

([ ] [ ])
2 2d 3 2d
2 x 3 x ¿n
2 /3
× Potential of a small drop
¿ E0 + 2
d 2 0 d 3 0
2.5=¿

[ ]
2 3
4 d 8d or V =0.1V .
¿ E0 + 2 =12 d E 0
d d 4 3 4 3
17. (a) π R =8 × π r
Energy of the point charge at x=2 d is 3 3
U =qV =12 qd E0 . or R=2r

11. (a) P.E. of a dipole is mimmum when C( sma ll drop) 4 π ε 0 r r 1


= = = =1 :2
its axis (or dipole moment) is parallel to C (big drop) 4 π ε0 R 2 r 2
the electric field.
18. (c) The Q charge on a capacitor
o
U =−pE cos 0 =−pE . indicates that the charges on its plates are
+Q and -Q.
12. (c)
1 o o 19. (d)
W 1= pE (cos 0 −cos 6 0 )= pE=W
2
o o
W 2 =pE (cos 0 −cos 18 0 )=2 pE=4 W .
13. (d) Potential is same at every point of
the conducting sphere.
∴ V A =V B =V C .
14. (b) Potential at any point inside the
sphere is constant and is equal to that on V = V1 + V 2 + V 3
the surface. q q q
V= + + =11V
C 1 C2 C 3

175
( )
1 1 1 When the capacitor is half-filled with a
q + + =11q dielectric, it becomes a series combination
1 2 3
of two capacitances C1 and C2.
6
¿ 11× =6 μC
11
q 6 μC
V 1= = =6 V.
c1 1 μ F
20. (b) Three 15 μF capacitors on the left
are connected in series. Their equivalent
capacitance C is given by
ε0 A ε0 A
1 1 1 1 1 C 1= =2 =2C 0
= + + = or C' = 5μF d /2 d
C ’ 15 15 15 5
ε0 A ε0 A
Now C' is in parallel with fourth 15 μF C 2=κ =5 × 2 =10 C 0
capacitor. d /2 d

∴ Ceq = C' + 15 = 5 + 15 = 20μF.


Equivalent capacitance,

21. (a) The two sets of 4 μF and 4 μF


C 1 C 2 2 C0 ×10 C 0 5 C 0
C= = =
form series combinations with equivalent C1 +C 2 2C 0 +10 C0 3
capacitance,
Increase in capacitance
4×4
C' = = 2 μF 5 C0
4+ 4 −C0
3 200
¿ × 100= =666 % .
C0 3
25. (a) We now have two capacitors of
capacitances C and κC connected in
senes.
C×κC κ C
C eq= = .
The equivalent circuit then takes the C+κ C 1+κ
form as shown. 26. (c) The arrangement is equivalent to
Ceq = 2 + 4 + 2 = 8 μF. series combination of two capacitors of
plate area A and separation d/2.
22. (c) When the battery is
disconnected, the charge on the capacitor 1 1 1 1 1
= + = +
plates remains the same. C C1 C 2 ε 0 A κ 1 ε 0 A κ 2
23. (d) For air filled capacitor, d /2 d /2

C 0=
ε0 A
d
=2 p F ¿
d
(1 1
+
2 ε0 A κ 1 κ 2 )
( )
For wax filled capacitor. 2 ε0 A κ 1 κ 2
or C=
ε0 A κ d κ 1 +κ 2
C=κ = C =6 p F
2d 2 0
27. (b) The arrangement is equivalent to a
2 ×6 2 ×6 parallel combination of two capacitors,
or κ= = =6.
C0 2 each with plate area A/2 and separation d.

24. (b) Original capacitance, C=C 1+C 2


ε0 A ( A /2)κ1 ε 0 ( A /2)κ 2
C 0= ¿ +
d d d

176
ε0 A ' q
2
q 1
2
¿ (κ + κ ) . U= = = U
2d 1 2 2 Ceq 2 ×2 C 2
28. (b) From the above problem, Q
2
32. (d) W =
ε0 A 2C
C= (κ + κ )
2d 1 2 '
W =¿¿
C0 33. Charge on the capacitor,
¿ (κ + κ )
2 1 2 q 1−q 2 10−(−2)
1 Q= = =6 μC
¿ (4+ 6)=5 μ F . 2 2
2 −6
Q 6 × 10
29. (d) Heat produced¿Energy stored in C= = F
V 12
capacitor
= 0.5μF.
1 2 1 −6
¿ C V = × 10× 1 0 ׿ 34. (a) If K is the dielectric constant of oil,
2 2
then
= 1.25J
E0
30. (b) Energy is additive for a parallel E=
combination of capacitors. κ
∴ U = n × Energy stored in one capacitor. When oil is pumped out,

1 1 E0 =κ E
=n× CV2 = nCV2.
2 2 35. (b) V =Ed
31. (b) Initial energy stored in one As E remains the same, so V increases as
capacitor, distance increases.
2
q 4 π ε 0 ab
U= 36. C=
2C b−a
When the battery is disconnected, −6
C=1 μ F=1 0 F ,b−a=1 mm=1 0 m
−3

charge q remains the same i.e., q =


∴ 10 =
constant. The capacitors form a parallel −6 ab
9 −3
combination. 9 × 10 × 10
Ceq = C + C = 2C or ab=9
Final energy stored by the As b−a is small, b ≃a
∴ b=√ 9=3 m .
combination.

AIIMS

1. The work done in carrying a charge Q⋅ q


Q once round a circle of radius r with (c) zero (d)
2r
charge q at the centre of the circle is
[AIIMS 97, IPUEE 06]
1 Q Q⋅ q
(a) ⋅ (b) 2. A tin nucleus has charge + 50 e. If
4 π ε0 r 4 π ε0 r
the proton is at 10-12 m from the nucleus,
then the potential at this position is
(charge on proton = 1.6 × 10-19 C)

177
(a) 1.44× 104 V (b) 7.2 × 104 V (c) zero (d) 2 volt [AIIMS 2010]
(c) 7.2 × 108 V (d) 14.4 × 108 V 9. In figure, a particle having mass m = 5 g
and charge q’ = 2 × 10-9 C starts from rest
[AIIMS 1995]
at point a and
3. The voltage of clouds is 4× 106 V
with respect to ground. In a lightning strike
lasting 0.1 s, a charge of 4 C is delivered
to the ground. The power of the lightning
strike is moves in a straight line to point b. What is
its speed v at point b ?
(a) 160 MW (b) 80 MW
(a) 2.65 cms-1 (b) 3.65 cms-1
(c) 20 MW (d) 500 MW
(c) 4.65 cms-1 (d) 5.65 cms-1 [AIIMS 2010]
[AIIMS 2006]
10. There are four point charges +q, -q, +q
4. It is possible to have a positively
and –q placed at the corners A, B, C, and
charged body at
D respectively of a square of side a. The
(a) zero potential (b) negative potential 1
potential energy of the system is
(c) positive potential 4 π ε0
times
(d) all of these [AIIMS 1995]
5. Electric potential of earth is taken
to be zero, because earth is a good
(a) insulator (b) conductor
(c) semiconductor (d) dielectric
[AIIMS 1998]
2 2
6. Equipotential surfaces associated q ( q (
(a) −4+ √ 2 ) (b) −4+ √ 2 )
with an electric field, which is increasing in a 2a
magnitude along the X-direction, are
−4 √ 2 q
2 2
4q
(a) planes parallel to YZ-plane. (c) (d)
a a
(b) planes parallel to XY-plane. [AIIMS 13]
(c) planes parallel to XZ-plane. 11. In which of the states shown in the
(d) coaxial cylinders of increasing radii figure, is potential energy of a electric
around the X-axis. [AIIMS 2004] dipole maximum ?
7. If an electron is brought towards [AIIMS 13]
another electron, the electric potential (a)
energy of the system
(a) increases (b) decreases
(c) becomes zero (d) remains the
same (b)
[AIIMS 1995]
8. An electric charge 10-3 μ C is
placed at the origin (0, 0) of (x - y) co-
ordinate system. Two points A and B B
are situated at (√ 2, √ 2) and (2, 0)
respectively. The potential difference (c)
between the points A and B will be
(a) 4.5 volt (b) 9 volt

178
[AIIMS 11]
16. In the capacitor of capacitance C,
charge Q and energy W is stored. If
charge is increased upto 2 Q, the energy
stored will be
(d) W W
(a) (b)
4 2
(c) 2W (d) 4W [AIIMS 12]
17. Five capacitors, each of
12. A charged spherical conductor of
capacitance value C are connected as
radius a and q, is surrounded by another
shown in the figure. The ratio of
charged concentric sphere of radius b(b >
a). The potential difference between
conductors is V. When, the spherical
conductor of radius b is discharged
completely, then the potential difference
between conductor will be
Va
(a) V (b)
b
q1 q2 capacitance between P and R ; and the
(c) − (c) none of these capacitance between P and Q is
4 π ε0 a 4 π ε0 b
(a) 3:1 (b)5: 2
[AIIMS 2012]
(c) 2:3 (d) 1 : 1 [AIIMS 2006]
13. A spherical drop of capacitance 1
μF is broken into eight drops of equal 18. Given a number of capacitors
radius. Then, the capacitance of each labelled as 8μF =250 V. Find the minimum
small drop is number of capacitors needed to get an
arrangement equivalent to 16 μF -1,000 V.
1 1
(a) μF (b) μF (a) 4 (b) 16
2 4
(c) 32 (d) 64 [AIIMS 2000]
1
(c) μF (d) 8 μF 19. Two spherical conductors A and B
8
of radii a and b (b > a) are placed
[AIIMS 12] concentrically in air. The two are
14. When a capacitor is connected to
a battery,
(a) a current flows in the circuit for some
time, then decreases to zero.
(b) no current flows in the circuit at all.
(c) an alternating current flows in the
circuit.
connected by a copper wire as shown in
(d) none of the above [AIIMS 1997] the figure. The equivalent capacitance of
15. Two capacitors of 10μF and 20 μF the system is 4πε∩αb
are connected in series with a 30 V 4 π ε 0 ab
battery. The charges on the capacitors will (a) (b) 4πε0(a +
be respectively b−a
b)
(a) 100 μC, 100 μC (b) 200 μC, 100 μC
(c) 4πε0b (d) 4πε0a
(c) 200 μC, 200 μC (d) 100 μC, 200 μC

179
[AIIMS 13] (C1 +C 2) V
(a) (b)
20. Three plates of common surface C1 +C 2+C 3
area A are connected as shown in the C1V
figure.
C1 +C 2+C 3
C2V C3V
(c) (d)
C1 +C 2+C 3 C1 +C 2+C 3
[AIIMS 2010]
The effective capacitance will be
24. What is the energy stored in the
3 ε0 A ε0 A
(a) (b) capacitor between terminals a and b of the
d d network shown in the figure ?
2 ε0 A (Capacitance of each capacitance C = 1
3 ε0 A
(c) (d) μF)
d 2 d
[AIIMS 13]
21. A gang capacitor is formed by
interlocking a number of plates as shown
in the figure. The distance

(a) 12.5 μj (b) zero


(c) 25 μj (d) 50 μj
[AIIMS 2009]:
25. A conducting sphere of radius R
between the consecutive plates is 0.885 carrying charge Q lies inside an
cm and the overlapping area of the plates uncharged conducting shell of radius 2R.
is 5cm2. The capacity of the unit is If they are joined by a metal wire, the
(a) 1.06 pF (b) 4 pF amount of heat that will be produced is
2 2
(c) 6.36 pF (d) 12.72 pF 1 Q 1 Q
(a) . (b) .
22. A 40 F capacitor in a defibrillator is 4 π ε0 4 R 4 π ε0 2 R
charged to 3000 V. The energy stored in 2 2
the capacitor is sent through the patient 1 Q 2 Q
(c) . (d) .
during a pulse of duration 2 ms. The 4 π ε0 R 4 π ε0 3 R
power delivered to the patient is [AIIMS 2009]
(a) 45 kW (b) 90 kW Assertions and Reasons
(c) 180 kW (d) 360 kW [AIIMS 2004] Directions : In the following questions
23. What would be the voltage across C3 ? (26-31), a statement of assertion (A) is
followed by a statement of reason (R).
Mark the correct choice as :
(a) If both assertion and reason are true
and reason is the correct explanation
of assertion
(b) If both assertion and reason are true
but reason is not the correct
explanation of assertion

180
(c) If assertion is true but reason is false point. [AIIMS 12]
(d) If both assertion and reason are false. 29. Assertion. Dielectric polarisation
means formation of positive and negative
26. Assertion. Electric potential of
charges inside the dielectric.
earth is taken zero.
Reason. Free electrons are formed in
Reason. No electric field exists on
this process.
earth surface.
[AIIMS 2014]
[AIIMS 2009]
30. Assertion. When charges are
27. Assertion. Work done in moving a
shared between any two bodies, no
charge between any two points in a
charge is really lost and some loss of
uniform electric field is independent of the
energy does occurs.
path followed by the charge, between
these points. Reason. Some energy disappears in
the form of heat, sparking etc.
Reason. Electrostatic forces are not
[AIIMS 2014]
conservative. [AIIMS 13]
31. Assertion. A spherical
28. Assertion. A metallic shield in the
equipotential surface is not possible for a
form of a hollow shell may be built to block
point charge.
an electric field.
Reason. A spherical equipotential
Reason. In a hollow spherical shield,
surface is not possible inside a spherical
the electric field inside it is zero at every
capacitor. [AIIMS 15]

Answers and Explanations

1. (c) The potential at every point of will be the equipotential surfaces.


the circle will be same.
7. (a) The work done against the
W = q∆V = q × 0 = 0. force of repulsion in moving the two
9 charges closer increases the potential
1 q 9× 10 ×50 e energy of the system.
2. (b) V = ⋅ = −12
4 π ε0 r 10 8. (c)
9 −19
9 ×10 ×50 ×1.6 × 10 4
= −12
=7.2× 10 V .
10
6
W qV 4 × 4 ×10
3. (a) P = = = W = 160
t t 0.1
MW.
4. (d) A positively charged body can
have +ve, -ve or zero potential.
5. (b) Earth is a conducting sphere of
√ 2
OA = ( √ 2−0) +( √ 2−0) = 2
2

large capacitance. √
OB = (2−0)2+(0−0)2 = 2

V=
q ∴ OA = OB
C
Hence, VA = VB or VA - VB = 0.
As C is very large, so V → 0 for all

[ ]
−9 −9
finite charges. 3 ×10 −3 ×10
9. (c) Va = 9 × 109 +
6. (a) Planes perpendicular to X-
0.01 0.02
direction i.e., planes parallel to YZ-plane = 1350 V

181
Vb = For a series combination,

[ ]
−9 −9 q 1=q 2=q=100 μC
9 3 ×10 −3 ×10
9 ×10 + =−1350 V
0.02 0.01 Q
2
16. (d) W =
Va - Vb = 2700 V 2C
'
1 2 W =¿¿
m v =q (V a−V b)
2 1 1 1 1 3
17. (c) = + + =

√ √
2 q (V a−V b) −9 C PT S R C C C C
2 ×2 ×1 0 × 2700
v= = −3
m 5 × 10 1 1 1 2
= + =
−2 −1 −1 C PQR C C C
¿ 4.65 × 10 m s =4.65 cm s .
10. (a)
C C 5C
C PR=C PTSR +C PQR= + =
3 2 6
U=
4 π ε0 [
1 q (−q ) (−q ) ×q q × (−q ) (−q )( q ) q × q (−q ) (−q )
a
+
a
+
a
+
a
+ 1+ 1 1 1 1 4
aC√ 2 =aC√+2 C + C + C = C
PTSRQ
]
[ ]
2
1 q 1 1 C 5C
¿ . −1−1−1−1+ + C PQ =C+ =
4 π ε0 a √ √2
2 4 4
2
1 q C PR 5 C 4 2
¿ . (−4+ √ 2) . = × = =2 :3.
4 π ε0 a C PQ 6 5C 3
11. (a) P . E . of a dipole is maximum when 18. (c) We can connect 4 capacitors in
⃗p is antiparallel to ⃗
E. series across 1000 V. Its capacitance will
be 2 μF. And 8 such combinations in
U =−pE cos180° ¿+ pE=¿ maximum + ve parallel will give 16 μF capacitance.
∴ Total number of capacitors = 4 × 8 = 32.
value.
12. (a) The p . d . between the two spheres
depends on the electric field present in the 19. (c) The charge given to the inner
space between the two spheres. The field sphere will pass on to the outer sphere.
in this space due to outer sphere is zero. The capacitance of the outer sphere and
Thus the p . d . between the two spheres hence the system will be 4πε0b.
does not depend on the charge on the 20. (c) The system is equivalent to
outer sphere. parallel combination of two capacitors
4 3 4 3 between A and B.
13. (a) π R =8 × π r ⇒ R=2r
3 3
For large drop, C=4 π ε 0 R=4 π ε 0 × 2r
For each small drop, C '=4 π ε 0 r


'
C 1 C ' = 1 C= 1 × 1 μ F= 1 μ F .
= or 2ε0 A
C 2 2 2 2 C eq=C 1 +C 2=
d
14. (a) A current flows in the circuit during
the time the capacitor is charged. After the 21. (b) The system is equivalent to a
capacitor gets fully charged, the current parallel combination of eight capacitors.

∴ C=
stops flowing. 8 ε0 A
10 ×20 20 d
15. (a) C eq= = µF
10+20 3 −12 −4
8 ×8.854 . ×1 0 ×5 ×1 0
20 ¿ −2
Γ
q=C eq V = μ F × 30V =100 μC 0 . 885× 10
3

182
−12
¿ 4 ×1 0 F=4 p F . ¿ 12.5 μJ .
22. (b) 25. (a) Capacitance of first capacitor,
1 2
C 1=4 π ε 0 R
CV
W 2 1
P= = = ×(40 ×1 0−6) ׿ ¿ Capacitance of second capacitor,
t t 2
4 C 2=4 π ε 0 (2 R)
¿ 9 ×1 0 W =90 kW .
2 2
23. (a) Parallel combination of C 1 and C 2 Q 1 Q
Initial energy, U i= = .
is in series with C 3. So 2 C1 4 π ε 0 2 R
2 2
(C1 +C 2)C3 Q 1 Q
C eq= Final energy, U f = = .
C 1+C 2 +C3 2C 2 4 π ε 0 4 R

Charge on C 3, Heat produced

Q=C eq V ¿ U i−U f

[ ]
2 2
(C 1+ C2 )C 3 V 1 Q 1 1 Q
¿ . ¿ 1− = .
C 1+ C2 +C 3 4 π ε0 2 R 2 4 π ε0 4 R
26. (c) Electric potential of the earth is
Q (C 1 +C 2)V taken zero because its capacitance C is
Voltage across C 3= = .
C 3 C1 +C 2+C 3 very large and so,
24. (a) The equivalent network is a q
balanced Wheatstone's bridge. V= → 0 for all finite charges.
C
27. (c) Electrostatic forces are
conservative. Work done in moving a
charge in a uniform electric field is path
independent. Assertion is true but reason
is false.
28. (a) Both assertion and reason are
true and reason is the correct explanation
of the assertion.
29. (d) Polarisation induces +ve and -
C ×C C ×C ve charges on the two opposite faces of
C eq = + =C
C+ C C +C the dielectric. No free electrons are
formed in the process. Both assertion and
Charge on capacitor between term⋅nalsa
reason are false.
and b
30. (a) Charge is always conserved
Q CV but some energy is lost in the form of
¿ =
2 2 heat. Both assertion and reason are true.
Energy stored in this capacitor 31. (d) Both assertion and reason are
¿¿¿ false.
−6
¿ 1 ×1 0 ׿ ¿

AIPMT

1. Some charge is being given to a (a) is maximum at surface.


conductor. Then, its potential

183
(b) is maximum at centre.
(c) remains the same throughout the
conductor.
(d) is maximum somewhere between
surface and centre. [CBSE 2002]
2. In bringing an electron towards
another electron, the electrostatic potential
energy of the system
(a) increases (b) decreases
(a) ( 4 π1ε ⋅ qQa ) √2 a ( 4−1π ε qQa ) √2 a
0
2 (b)
0
2

(c) remains unchanged (d) becomes zero


[CBSE 99]
(c) ( 4 π1ε qQa ) √a2
0
2 (d) zero

3. Four point charges -Q, -q, 2q and 2 Q [CBSE 2005]


are placed, one at each comer of the
7. Charges +q and -q are placed at points
square. The relation between Q and q for
A and B respectively which are a distance
which the potential at the centre of the
2 L apart, C is the
square is zero is
1
(a) Q = -q (b) Q = -
q
1
(c) Q = q (d) Q = [AIPMT Pre 12]
q
4. A conducting sphere of radius R is midpoint between A and B. The work done
given a charge Q. The electric potential in moving a charge + Q along the
and the electric field at the centre of the semicircle CRD is
sphere respectively are
qQ qQ
Q Q (a) (b)
(a) and 2 π ε0 L 6 π ε0 L
4 π ε0 R 4 π ε0 R
2
−qQ qQ
Q (c) (d)
6 π ε0 L 4 π ε0 L
(b) both zero (c) zero and 2
4 π ε0 R
Q [CBSE 2007]
(d) 2 and zero [AIPMT 14]
4 π ε0 R 8. Four electric charges + q, +q, -q and -q
5. Three concentric spherical shells are placed at the comers of a square of
have radii a, b and c(a < b < c) and have side 2 L (see figure). The
surface charge densities σ, -σ and σ
respectively. If VA, VB and VC denote the
potentials of the three shells, then for c = a
+ b, we have
(a) VC = VB ≠ VA (b) Vc ≠ VB ≠ VA
(c) VC = VB = VA (d) VC = VA ≠ VB
electric potential at point A midway
[CBSE 2009] between the two charges + q and + q is
6. A point charge +q is placed at the origin
O as shown in the figure. Work done in
taking another point charge -Q from the
(a)
1 2q
4 π ε0 L
(1+ √ 5) (b)
1 2q
4 π ε0 L
1+
1
√5 ( )
point A(0, a) to another point B(a, 0) along
the straight path AB is (c)
1 2q
4 π ε0 L
1−
(1
√5 )
(d) zero [CBSE Pre

184
2011] E =î z 3 + ^j xyz+ k^ z 2
(b) ⃗
9. Three charges, each + q, are placed at E =î (2 xy−z 3 )+ ^j x u2 + k^ 3 z 2 x
(c) ⃗
the comers of an isosceles triangle ABC of
sides BC and AC, each (d) ⃗ ^ xy + z 3 )+ ^j x 2 + k^ 3 x z 2
E =i(2
[CBSE PMT 2009]
13. In a region, the potential is
represented by V(x, y, z) = 6x - 8xy - 8y +
6yz, where V is in volts and x, y, z are in
metres. The electric force experienced by
a charge of 2 coulomb situated at point (1,
1, 1) is

equal to 2a. D and E are the mid points of (a) 24 N (b) 4√ 35N
BC and CA The work done in taking a (c) 6√ 5 N (d) 30 N [AIPMT 14]
charge Q from D to E is
14. If potential (in volts) in a region is
3 qQ expressed as V(x, y, z) = 6xy – y + 2yz,
(a) zero (b)
4 π ε0 a the electric field (in N/C) at point (1, 1, 0)
is
3 qQ qQ
(c) (d) [CBSE Final (a) -(6i^ + 9 ^j + k^ ) (b) - (3i^ + 5 ^j + 3k^ )
8 π ε0 a 4 π ε0 a
2011] (c) -(6i^ + 5 ^j + 2k^ ) (d) -(2i^ + 3 ^j + k^ )
10. A, B and C are three points in a
[AIPMT Re 15]
uniform electric field.
15. Identical charges (-q) are placed at
each corner of a cube of side b. Then, the
electrostatic potential energy of charge
(+q) placed at the centre of the cube will
be
The electric potential is
4 √2 q 8 √2 q
2 2

(a) maximum at A (b) maximum at B (a) - (b)


π ε0 π ε0 b
(c) maximum at C
4 q2 8 √2 q
2

(d) same at all the three points A, B and (c) - (d)


√3 π ε 0 b 4 π ε0 b
C.
[CBSE 2002]
[NEET 13]
16. An electric dipole of moment 'p' is
11. The mean free path of electrons in
placed in an electric field of intensity ‘E’.
a metal is 4 × 10-8 m. The electric field
The dipole acquires a position such that
which can give on an average 2 eV
the axis of the dipole makes an angle θ
energy to an electron in the metal will be
with the direction of the field. Assuming
in units of V/m
that the potential energy of the dipole to
(a) 5 × 1011 (b) 8 × 10-11 be zero when θ = 90°, the torque and the
potential energy of the dipole will
(c) 5 × 107 (d) 8 × 107
respectively be
[CBSE PMT 2009]
(a) pE sin θ, - pE cos θ (b) pE sin θ, - 2 pE
12. The electric potential at a point (x, cos θ
y, z) is given by V = -x 2y – xz3 + 4. The
(c) pE sin θ, 2 γE cos θ (d) pE cos θ, - pE
electric field ⃗
E at that point is cos θ
E =î 2 xy + ^j(x 2 + y 2 )+ k^ ( 3 xz− y 2 )
(a) ⃗ [AIPMT Pre 12]

185
17. Two metallic spheres of radii 1 cm V are joined in series. The capacitance
and 3 cm are given charges of -1 × 10 -2 C and breakdown voltage of the combination
and 5 × 10-2 C, respectively. If these are will be
connected by a conducting wire, the final
V C
charge on the bigger sphere is (a) 3C, (b) 3V
3 3
(a) 2 × 10-2C (b) 3 × 10-2C
C V
(c) 4 × 10-2C (d) 1 × 10-2C [AIPMT 12] (c) 3.C, 3 V (d) , [CBSE PMT 2009]
3 3
18. A parallel plate air capacitor has
22. What is the effective capacitance
capacity 'C', distance of separation
between points X and Y ?
between plates is 'd' and potential
difference ‘V’ is applied between the
plates. Force of attraction between the
plates of the parallel plate air capacitor is
2 2 2 2
C V C V
(a) 2 (b)
2d 2d
2 2
CV CV
(c) (d)
2d d (a) 24 μF (b) 18 μF
[AIPMT Re 15] (c) 12 μF (d) 6 μF [CBSE 1999]
19. Three capacitors each of 23. Four capacitors of 25 μF each are
capacitance 4 μF are to be connected in connected as shown in figure. If the d.c.
such a way that the effective capacitance voltmeter reads 200 V, charge on each
is 6 μF. This can be done by connecting plate of the capacitor is
(a) all of them in series.
(b) all of them in parallel.
(c) two in parallel and one in series.
(d) two in series and one in parallel.
20. A network of four capacitors of
capacitances equal to C1 = C, C2 = 2C, C3
= 3C and C4 = 4C are (a) 2 × 10-3 C (b) 5×10-3C
(c) 2 × 10-2 C (d) 5 × 10-2 C [CBSE 1994]
24. A parallel plate capacitor with oil

oil, κ = 2) has a capacitance C. If the oil is


between the plates (dielectric constant of

removed, then capacitance of the


capacitor becomes
(a) √ 2 C (b) 2 C
connected to a battery as shown in the
C C
figure. The ratio of the charges on C2 and (b) (d) [CBSE 1999]
C4 is √2 2
4 3 25. A parallel plate air capacitor is
(a) (b) charged to a potential difference of V volt.
7 22
After disconnecting the charging battery,
7 22 the distance between the plates of the
(c) (d) [CBSE 2005]
4 3 capacitor is increased using an insulating
handle. As a result, the potential
21. Three capacitors each of difference between the plates
capacitance C and of breakdown voltage

186
(a) decreases (b) increases 1
(a) ε0E2 (b) E2Ad/ε0
(c) does not change (e) becomes zero 2
[CBSE 200b] 1
(c) ε0E2Ad (d) ε0EAd [AIPMT
2
26. A capacitor of capacitance C1 is
12]
charged upto potential V and then
connected in parallel to an uncharged 31. A 4 μF capacitor is charged to 400
capacitor of capacitance C2. The final V. If its plates are joined through a
potential difference across each capacitor resistance of 2kΩ, then heat produced in
will be the resistance is
C2V C1V (a) 0.16 J (b) 0.32 J
(a) (b)
C1 +C 2 C1 +C 2 (c) 0.64 J (d) 1.28 J [CBSE 1994, 95]

( ) ( )
C2 C2 32. A capacitor is charged by
(c) 1+ V (d) 1− V [CBSE 2002] connecting a battery across its plates. It
C1 C1 stores energy 17. Now the battery is
27. The energy stored in a capacitor of disconnected and another identical
capacitance C and potential V is given by capacitor is connected across it. Then the
energy stored by both capacitors of the
1 2 1 system will be
(a) C V (b) CV2
2 2
U
1 1 (a) U (b)
(c) CV (d) C2 V2 [CBSE 1996]
2
2 2
3U U
1 (c) (d)
28. The dimension of ε0E2, where ε0 2 4
2
33. Two condensers, one of capacity
is permittivity of free space and E is
C and the other of capacitor C / 2 are
electric field, is
connected to a V volt battery, as shown.
(a) ML2T-2 (b) ML-1 T-2 The work done in charging fully both the
condensers is
(c) ML2T-1 (d) MLT-1
[CBSE Pre 2010]
29. In a parallel plate capacitor, the
distance between the plates is d and
potential difference across plates is V.
Energy stored per unit volume between
the plates of capacitor is 1 1
(a) CV2 (b) CV2
2 2
4 2
Q 1 V
(a) (c) 3
2V
2
2 ε0 d 2 (c) CV2 (d) 2 CV2
4
2 2
1 V 1 V [CBSE 2007]
(b) ε0 (d) ε0
2 d2 2 d
34. A series combination of n1 capacitors,
[CBSE 2001] each of value C1, is charged by a source
of potential difference 4 V. When another
30. A parallel plate capacitor has a
parallel combination of n2 capacitors, each
uniform electric field E in the space
of value C2, is charged by a source of
between the plates. If the distance
potential difference V, it has the same
between the plates is d and area of each
(total) energy stored in it, as the first
plate is A, the energy stored in the
combination has. The -value of C2, in
capacitor is :
terms of C1, is then

187
2C 1 n2 (c)
(a) (b)16 C
n1 n2 n1 1
n2 16 C1
(c) 2 C (d) [CBSE Pre 2010]
n1 1 n1 n2

dielectric constants κ1 and κ2(κ1 < κ2) are


35. Two thin dielectric slabs of

inserted between plates of a parallel plate


capacitor, as shown in the figure. The (d)
variation of electric field E between the
plates with distance d as measured from
plate P is correctly shown by

[AIPMT 14]
36. A parallel plate air capacitor of
(a) capacitance C is connected to a cell of

dielectric slab of dielectric constant κ,


emf V and then disconnected from it. A

which can just fill the air gap of the


capacitor, is now inserted in it. Which of
the following is incorrect ?

decreases κ times.
(a) The energy stored in the capacitor

(b) (b) The change in energy stored is


1
2
1
( )
C V 2 −1
κ
(c) The charge on the capacitor is not
conserved.

plates decreases κ times.


(d) The potential difference between the
[AIPMT 15]

Answers and Explanations

1. (c) The potential at all points inside


a conductor is same as that on its surface.
2. (a) Work has to be done against
the force of repulsion in bringing an
electron towards another electron. This
increases the electrostatic potential
⇒ -Q-q + 2c] + 2Q= 0
energy of the system.

⇒ Q+q=0
3. (a) Potential at the centre O is

⇒ Q = -q.
V=k [ −Q
+
−q
+
2q
+
2Q
a / √2 a/ √ 2 a / √2 a/ √ 2 ]
=0

188
4. (d) At the centre of the conducting
sphere,
Q
V E−V D=
q
4 π ε0 [( 1
+
1
+
EA EB EC
1
− )(
1
+
1
+
DA D B DC
1
)]
[( )]
V= and E = 0.
4 π ε0 R
5. (d) From the solution of Problem
¿
q
4 π ε0
1
+
1
+
EA EB EC
1

1
+ )(
1
+
EB EA EC
1
=0

10 on page 2.83, we get


σ W = Q(VE - VD) = 0.
V A= (a−b+c )
ε0 10. (b) Electric field acts in the
direction of decreasing potential.
( ) ∴ VB > VC > VA.
2
σ a
V B= −b+ c
ε0 b
11. (c)
σ a2−b 2
V C= [
ε0 c
+ c]=
σ ( a−b )( a+ b )
ε0 c
+c
[ ] V
E= =
2
d 4 ×1 0−8
8 7
=0.5 ×1 0 =5 ×1 0 V m
−1

σ
¿ (a−b+ c) [ ∵ a+b=c ]
ε0
∴ V A =V C ≠V B .
12. (d) ⃗
E=
∂r
=
∂x
i−
∂y[
−∂V −∂V ^ ∂V ^ ∂ V ^
j−
∂z
k
]
¿ [ ( 2 x y + z ) i+ x ^j+3 x z k^ ]
3 ^ 2 2
1 q
6. (d) VA = VB = ⋅
4 π ε0 a 13. (b) V =6 x−8 xy−8 y +6 yz
W = -Q( VB - VA) = 0. At the point (1, 1, 1), we have
7. (c) −∂ V
E x= =−(6−8 y )=2
∂x
−∂V
E y= =−(−8 x−8+ 6 z)=10
∂y
−∂ V
E z= =−6 y =−6
∂z
E=√ E 2x + E2y + E 2z =√ 4+ 100+36= √ 140
V C=
1 q −q
+ =
[1 q q
− =0
4 π ε 0 AC BC 4 π ε 0 L L ] [ ] ¿ 2 √ 35 N C
−1

V D=
1 q −q
+
[ =
1 q q
− =
]
4 π ε 0 AD BD 4 π ε 0 3 L L 6 π ε 0 L [
−q F=qE=2 ×2 √ 35=4 √ 35 N.

14. (c) ⃗
E=
]
−∂V ^ ∂V ^ ∂ V ^
i− j− k
Work done in moving the charge +Q along ∂x ∂y ∂z
the semicircle CRD is
E =−¿ (6y¿ î −(6 x−1+2 z) ^j−(2 y ) k^

W =+Q(V D−V C )=+Q
−q
6 π ε0L
−0 =
−qQ
(
6 π ε0 L
. ) At the point ¿1, 1, 0 ¿ ,
^ ^j+2 k^ )N C−1 .
E =−6 î‐5 ^j−2 k^ ¿−(6 i+5

8. (c) V =
q 1 1
+ −
1
[

1
4 π ε 0 L L √5 L √5 L ] 15. (c) Half diagonal of the tube,

r =√
b 2+b 2+ b2 √ 3 b
¿
2q
4 π ε0 L [
1−
1
√5 ] 2
=
2
P.E. of the charge +q at the centre due to
9. (a)
eight charges (each = -q) at the corners of
the cube is

189
1 q ×(−q) 6 CV
U =8 × q 1=q 2=q 3=C123 × V =
4 π ε0 r 11

1 q
2 Also, the charge on the capacitor C 4 ,
¿−8 ×
4 π ε 0 √ 3 b/2 q 4=C 4 V =4 CV


2
4q q 2 6 CV /11 3
¿− . = = .
√3 π ε 0 b q4 4 CV 22
16. (a) τ = pE sin θ 21. (b) V eff =V +V +V =3 V
U =−pE cosθ 1 1 1 1 C
= + + ⇒ Ceff = .
17. (b) q tota l=4 ×1 0 C
−2 Ceff C C C 3

Charge on a sphere ∝Capacitance ∝ 22. (d) The network is a balanced


Radius Wheatstone bridge because

∴ q sma≪¿ :q =1 :3 ¿
big

3 −2 −2
q big = × 4 × 1 0 C=3× 10 C .
4
2
q q
18. (c) P=qE=q × =
2 ε0 A 2 ε 0 A
ε A C AB C AC 6
But q=CV and C= 0 = =
d C BD CCD 6

⇒ ε 0 A=Cd 20 μF capacitor is ineffective. The


simplified network is
∴ F=
2 2 2
C V CV
= .
2Cd 2d
19. (d) The series combination of two
capacitors should be connected in parallel
with the third capacitor. Then
C1 C2 4×4
C eff = +C 3= + 4=2+4=6 μ F .
C 1+C 2 4+ 4
20. (b) The series combination of C 1 , C 2 These two capacitances are in parallel.
and C 3 is in parallel with C 4. The ∴ C = 3 + 3 = 6 μF.
equivalent capacitance C 123 of the series
23. (b) Charge on each plate of the
combination is given by
capacitor,
1 1 1 1 1 1 1 11
= + + = + + = Q = CV = 25 × 10-6 F × 200 V = 5 × 10-3 C.
C123 C 1 C2 C 3 C 2 C 3 C 6 C
∴ C vac = med = .
C med C C
6C 24. (d) K=
or C 123 ¿ C vac K 2
11
25. (b) As the battery is disconnected,
Let q 1 , q 2 , q 3 and q 4 be the charges on the q=¿constant.
respective capacitors. Since in a series
combination, charge on all the capacitors ' ε0 A
New capacitance, C =
is same, d’
But d ' >d , so C ' <C

190
' q q
2
q 1
2
Now V = '
U= = = U.
C’ 2 Ceq 2 ×2 C 2
As capacitance decreases, so potential 33. (c) The equivalent capacitance of the
difference increases. two condensers connected in parallel is
26. (b) The common potential difference ' C 3C
across the parallel combination of two C =C+ =
2 2
capacitors,
The work done in charging both the
C 1 V 1+ C 2 V 2
V=
' capacitors is stored as potential energy.
C 1+ C 2
∴ W =U= C V = ( )
1 1 3C 2 3
But V 1=V , V 2=0 ∴ V =
' 2
C1 V V = CV2.
' 2 2 2 4
C1 +C 2
1 2 1
34. (d) C pV = Cs ¿
1 2 2 2
27. (b) U = C V .
2
1 2 1 C1
or n2 C 2 V = ¿
28. (b)
1
[ ]
ε E2 = ¿ energy density
2 0
2
16 C 1
2 n1

2
ML T
−2 or C 2= .
¿
−1
=[M L T ].
−2 n1 n2
3
L
Q
29. (b) Energy stored per unit volume in a 35. (a) In a region of vacuum, E=
ε0 A
capacitor,
In a region where dielectric is present,

( )
2 2
1 2 1 V 1 V Q
u= ε 0 E = ε 0 = ε0 . 2 . E=
2 2 d 2 d κ ε0 A

⇒ E∝
30. (c) 1
1 2
U= C V =
2
1 ε0 A 2 2 1
2 d ( ) 2
E d = ε 0 E Ad .
2
κ
As κ 1 <κ 2, decrease in E is greater for κ 2
31. (b) Heat produced in the 2 kΩ resistor than for κ 1 .
= Energy stored in the charged capacitor 36. (c) Charge on the capacitor plates
1 2 1 −6 remains constant, Q = CV
¿ C V = × 4 × 10 × ¿ J.
2 2 ' E V
V= d=
32. (b) Initial energy stored in the κ κ
capacitor, Q Q
C '= = =κ C
V V /κ
2 '
1 2 q
U= C V =
2 2C
( )
2
' 1 '2 1 V 11 2 U
When the battery is disconnected, charge
U = CV = κ C× = CV =
2 2 κ κ2 κ
q = constant. Another capacitor connected
∴ U −U =U ( 1κ −1)= 12 C V ( 1κ −1).
across the first capacitor is in parallel with ' 2
it. So,
C eq=C +C=2 C Hence, only option (c) is incorrect.
Final energy stored by the system of two
capacitors,

VMMC PMT and other Medical Entrance Exams

191
1. How much work is required to (c) 0.76 × 1012 C/kg (d) none of these.
carry a 6 μC charge from the negative to [DPMT 03]
the positive terminal of a 9 V battery ?
8. The angle between the electric
(a) 54× 10-3 J (b) 54 × 10-9 J lines of force and an equipotential surface
(c) 54× 10-6 J (d) 54 × 10-12 J (a) 45° (b) 90°
[DPMT 1995] (c) 0° (d) 180° [VMMC
2013]
2. The potential at a point, due to a
positive charge of 100 μC at a distance of 9. If we carry a charge once around
9 m, is an equipotential path, then work done by
the charge is
(a) 104 V (b) 106 V
(a) infinity (b) positive
(c) 105 V (d) 107 V [DMPT
1998] (c) negative (d) zero
3. The potential at the centre of the [DPMT 1993]
sphere, if hollow metallic sphere of radius
10. Two points charges of +10 μC and
10 cm is charged such that potential of its
-10 μC are placed at points A and B. If P
surface is 70 V, is
and Q are the two points lying on the
(a) 100 V (b) 35 V perpendicular bisector of the line AB, then
work done in taking a charge of 5 μC from
(c) 70 V (d) 7 V [DPMT 1993]
P to Q will be equal to
4. The potential on the hollow sphere
(a) distance between P and Q
of radius 1 m is 1000 V, then potential at
1 (b) zero (c) distance between A & B
m from the centre of the sphere is
4 (d) either (a) and (c) [DPMT
(a) 1000 V (b) 500 V 1993]
(c) 250 V (d) Zero V [VMMC 09] 11. In a certain charge distribution, all
points having zero potential can be joined
5. The electric potential at the surface by a circle S. Points inside S have positive
of an atomic nucleus (Z = 50) of radius 9.0 potential and points outside S have
× 10-13 cm is negative potential. A positive charge,
(a) 9 × 105volt (b) 8 × 106volt which is free to move, is placed inside S.
(c) 80 volt (d) 9 volt [VMMC (a) It will remain in equilibrium
2005] (b) It can move inside S, but it cannot
6. At a point A, there is an electric cross S.
field of 500 V/m and potential difference of (c) It must cross S at some time.
3000 V. The distance between the point
charge and A is (d) It may move, but will ultimately
return to its starting point. [DPMT
(a) 6 m (b) 36 m 2004]
(b) 12 m (d) 144 m [DPMT 1992, 95] 12. The velocity v acquired by an
7. An electron initially at rest, is electron starting from rest and moving
accelerated through a potential difference through potential difference V is shown by
of 200 volt, so that it acquires a velocity which of the following graphs ?
8.4 × 106m / s. The value of e/ m of (a)
electron will be
(a) 1.76 × 1011 C/kg (b) 2.76 × 1012
C/kg

192
the common potential after sharing of
charges becomes V, then the dielectric
constant of the dielectric must be
V0 V
(a) (b)
V V0
(b)
(V 0 −V ) ( V 0−V )
(c) (d) [VMMC 10]
V V0
17. If the circumference of a sphere is
2 m, then capacitance of sphere in water
would be
(a) 2700 pF (b) 2760 pF
(c)
(c) 2780 pF (d) 2800 pF [VMMC 09]
18. What is the potential difference
across 2μF capacitor in the circuit
shown ?

(d)

(a) 12 V (b) 4 V
(c) 6 V (d) 18 V [VMMC 12]

[DPMT 1999 ; VMMC 2004] 19. 0.2 F capacitor is charged to 600 V


by a battery, on removing the battery, it is
13. Capacitors are used in electrical connected with another parallel plate
circuits where appliances need more condenser of 1 F. The potential decreases
(a) current (b) voltage to

(c) watt (d) resistance [VMMC 09] (a) 100 V (b) 120 V

14. The earth has volume V and (c) 300 V (d) 600 V [VMMC 10]
surface area A, then capacitance would 20. The ratio of charge to potential of a
be body is known as
A V (a) capacitance (b) inductance
(a) 4πε0 (b) 4πε0
V A (c) conductance (d) resistance
V A [DPMT 1998]
(c) 12πε0 (d) 12πε0 [VMMC 09]
A V
21. If P.D. across a capacitor is
15. Three capacitors of 2.0, 3.0 and changed from 15 V to 30 V, work done is
6.0 μF are connected in series to a 10 V W. What will be the work done when P.D.
source. The charge on the 3.0 μF is changed from 30 V to 60 V ?
(a) 5μC (b) lOμC (a) W (b) 4 W
(c) 12 μC (d) 15 μC [VMMC 11] (c) 3 W (d) 2 W [VMMC
16. An uncharged capacitor with a 2002]
solid dielectric is connected to a similar air 22. Capacity of a parallel plate
capacitor charged to a potential of V0. If condenser is 10 μF when the distance

193
between its plates is 8 cm. If the distance 27. In the circuit shown in the figure,
between the plates is reduced to 4 cm, its the potential difference across the 4.5 μF
capacity will be capacitor is
(a) 10 μF (b) 20 μF
(c) 15 μF (d) 40 μF [DPMT
2001]
23. The equivalent capacity of two
capacitors in series is 3 μF and in parallel
is 16 μF. Their individual capacities are
(a) 12, 4 (b) 8, 8
(a) 8/3 volt (b) 4 volt
(c) 10, 16 (d) 12, 2 [DPMT
(c) 6 volt (d) 8 volt [VMMC
2002]
2005]
24. Four capacitors are connected in a
28. The capacity of a condenser is 4 ×
circuit as shown in figure. The effective
10-6 farad and its potential is 100 volts.
capacitance in between P and Q will be
The energy released on discharging it fully
will be
(a) 0.04 J (b) 0.02 J
(c) 0.025 J (d) 0.05 J
[VMMC1995]
29. A 4 μF capacitor is charged to 400
(a) 10 μF (b) 5 μF V. If its plates are joined through a
(c) 2 μF (d) 7.5 μF [DPMT resistance, then heat produced in the
2003] resistance is
25. Equivalent capacitance of the (a) 0.16 J (b) 0.64 J
given combination of five capacitors is (c) 0.32 J (d) 1.28 J [VMMC 08]
30. If the potential of a capacitor
having capacity 8 μF is increased from 10
V to 20 V, then increase in its energy will
be
(a) 4× 10-4 J (b) 12 × 10-4 J
(c) 4 × 10-6 J (d) 12 × 10-6 J [DPMT
1996]
(a) 4 μF (b) 10 μF
31. A capacitor of 20 μF is charged up
(c) 8 μF (d) 120 μF to 500 V is connected in parallel with
[AIIMS 02 ; VMMC 03] another capacitor of 10 μF which is
charged up to 200 V. The common
26. For circuit the equivalent
potential is (a) 500 V (b) 300 V
capacitance between P and Q is
(c) 400 V (d) 200 V [DPMT 1995]
32. Two insulated metallic spheres of
3 μF and 5 μF capacitances are charged
to 300 V and 500 V respectively. The
energy loss, when they are connected by
(a) 6C (b) 4C a wire, is
(a) 0.012 J (b) 0.0375 J
3C 6C
(c) (d) [VMMC 2006] (c) 0.0218 J (d) 3.75 J [DPMT 1996]
2 11

194
33. When a dielectric material is value. The dielectric constant of the plate
introduced between the plates of a will be
charged condenser, then electric field
21 2t
between the plates (a) (b)
2 d+t 2 d−t
(a) decreases
t t
(b) remains constant (b) (d) [VMMC 2002]
d +t d−t
(c) increases
38. A parallel plate capacitor has
(d) first (c) then (a) capacitance C. If it is equally filled with
parallel layers of materials of dielectric
34. If the distance between the plates
constants κ1 and κ2, its capacity becomes
of parallel plate capacitor is halved and
C1. The ratio of C1 to C is
the dielectric constant is doubled, then its
capacity will κ1 κ 2
(a) κ1 + κ2 (b)
(a) increase by 16 times (b) increase κ 1+κ 2
by 4 times
κ 1+κ 2 2κ 1 κ 2
(c) increase by 2 times (b) (d) [VMMC 2003]
κ1 κ 2 κ 1+ κ 2
(d) remain the same
κ1 and κ2 are filled in the gap of parallel
39. Two dielectrics of dielectric constants
[DPMT 2001, VMMC 2004]
35. A parallel plate condenser with oil plate capacitor as shown in the figure. The
capacitor has plate each of area A and
oil κ = 2) has a capacitance C. If the oil is
between the plates (dielectric constant of
separation d. The capacitance of the
removed, then capacitance of the capacitor is
capacitor becomes
C
(a)√ 2C (b)
√2
C
(c)2C (d) [DPMT 1997]
2
ε 0 A (κ 1 +κ 2)
36. A copper plate of thickness b is (a) (b)
placed inside a parallel plate capacitor of 2d
plate distance d and area A as shown in
figure. The capacitance of capacitor is (
ε 0 A κ1 +κ 2
2 d κ 1 κ2 )
(c)
d κ1 +κ 2(
ε 0 A κ 1 κ2
) (d)

(
2 ε0 A κ 1 κ 2
d κ 1 +κ 2 )
A ε0 A ε0 [VMMC 2007]
(a) (b)
d b 40. Across each of two capacitors of
capacitance 1 μF and 4 μF, a potential
A ε0
(b) (d) ∞ difference of 10 V is applied. Then positive
d−b plate of one is connected to the negative
[DPMT 2002] plate of the other, and negative plate of
one is connected to the positive plate of
37. If a dielectric plate of thickness t is the other. After contact,
placed between the plates of a parallel
plate capacitor of plate distance d, the (a) charge on each is zero
capacitance becomes half of the original

195
(b) charge on each is same but non-zero (a) 990 (b) 900
(c) charge on each is different but non- (c) 9090 (d) 909 [DPMT 2011]
zero
46. The plates in a parallel plate
(d) none of these [DPMI 20051 capacitor are separated by a distance d
with air as the medium between the
41. A parallel plate capacitor of
plates. In order to increase the capacity by
capacitance C is connected to a battery
66% a dielectric slab of dielectric constant
and is charged to a potential difference V.
5 is introduced between the plates. What
Another capacitor of capacitance 2 C is
is the thickness of the dielectric slab ?
similarly charged to a potential difference
2 V. The charging battery is then d d
disconnected and the capacitors are (a) (b)
4 2
connected in parallel to each other in such
a way that the positive terminal of one is 5d
(b) (d) d [VMMC 2014]
connected to the negative terminal of the 8
other. The final energy of the configuration 47. Given two hollow spherical
is conductors A and B as shown in the
3 figure.
(a) zero (b) CV2
2
25 9
(c) CV2 (d) CV2
6 2
[DPMT 2006]
42. Top of the stratosphere has an
electric field E (in units of V/m) nearly
Points M and N can be connected by
equal to
conductor wire.
(a) 0 (b) 10
Initially (before the connection of
(c) 100 (d) 1000 [DPMT points M and N) conductor B is neutral,
2009] while charge appeared on conductor A is
43. The surface charge density (in q. After connecting the spheres, the
C/m2) of the earth is about potential of sphere B

(a) 10-9 (b) -109 q q


(a) (b)
8 π ε0 ( r + R ) 4 π ε0 R
(c) 109 (d) -10-9 [DPMT 2009]
44. Which of the following is not true ? q
(b) (d) zero
4 π ε0 ( r + R )
(a) For a point charge, the electrostatic
potential varies as 1 / r [VMMC 15]
(b) For a dipole, the potential depends on
the position vector and dipole moment
vector
(c) The electric dipole potential varies as
1/ r at large distance
(d) For a point charge, the electrostatic
field varies as 1/r2 [DPMT 2011]
45. How many 1 μF capacitors must
be connected in parallel to store a charge
of 1 C with a potential of 110 V across the
capacitors ?

196
Answers and Explanations

∴ v 2 ∝V
1. (c) W = Qv = 6 × 10-6C × 9V = 54 × 10- 1 2
6
J.
m v =eV
2
1 q 100× 100
−6
Thus the graph between v and V must be
2. (c) V = = 9× 109×
4 π ε0 r 9 a parabola. Only option (b) is correct.
= 105V. 13. (a) Capacitor is a device to store
3. The potential at any point inside the charge. It is used in appliances where
charged hollow metallic sphere is same as more current is needed.
that on its surface. 4 3
14. (c) V = π R and A=4 π R2
4. (a) Same reason as in above problem. 3


5. (b) Electric potential at the surface of an V R 3V
atomic nucleus, = ⇒ R=
A 3 A
−19
1 Ze 50 × 1.6× 10
( 3VA )=
V =
9
⋅ =9 × 10 × 12 π ε 0 V
4 π ε0 r 9× 10
−15 C=4 π ε 0 R=4 π ε 0 .
A
= 8× 106 V.
1 1 1 1 6 1
1 q 1 q 15. (b) = + + = =
6. (a) E= . 2 and V = . Ceq 2 3 6 6 1
4 π ε0 r 4 π ε0 r
C eq =1 μ F
∴ r= =
V 3000 V
=6 m. Charge on each capacitor is
E 500 V /m
7. (a) K.E. gained by an electron when ∴ q=CV =1 μ F ×10 V =10 μC
accelerated through a potential difference 16. (c) Common potential,
of V volts is
C 1 V 1 +C 2 V 2 0+C V 0 CV 0
1 2 V= = =
eV = m v C1 +C 2 κ C+C C ( 1+κ )
2

∴ =
2 V0
e v ¿
=¿ ¿ 1+ κ
m 2V
∴ κ=⋅
8. (b) An electric line of force is V0 V 0−V
−1=
perpendicular to the equipotential surface V V
at every point. 17. (d) 2 πR = 2 m
9. (d) For an equipotential surface, ΔV = 0 C0 = 4πε0R = 4ε0
W = qΔV = q × 0 = 0. Capacitance in water,
C = κC0 = 80 × 4 × 8.85 × 10-12F = 2800
10. (b) Potential at any point on the
perpendicular bisector of dipole is zero.
∴ ΔV =V Q −V P=0
pF.
18. (c) εnet =16 – 6 = 10 V
W =qΔV =5 μC ×0=0. 2 ×3 6
C eq= = μF
11. (c) A positive charge tends to move 2+ 3 5
from a region of higher potential to that of 6
lower potential. So the positive charge will q=C eq ε ne t= ×10=12 μC
cross the circle S at some time. 5
12. (b) K.E. gained by the electron, P.D. across 2 μF capacitor,

197
q 12 μC 25. (a) The equivalent circuit is a
V 1= = =6 V. balanced Wheatstone bridge.
C1 2 μ F
19. (a) By conservation of charge,
0.2 ×600=(0.2+ 1)V

∴ V=
0.2 ×600
=100 V.
1.2
Q
20. (a) By definition of capacitance, C =
V 4 μF 4 μF

21. (b) W = C V ⇒ W ∝V 2
=
1 2 4 μF 4 μF
2 The 8 μF capacitor is ineffective.

∴ ( )(
2

) ( )
2 2
W2 ΔV 2 60−30 30 4×4
= = = =4 C ABC = =2 μ F
W1 ΔV 1 30−15 15 4 +4

or W 2 =4 W 1=4 W .
4×4
C ADC = =2 μ F
4 +4
ε0 A 1
22. (b) C= i.e., C ∝ CAC = CABC + CADC = 2 + 2 = 4ΜF.
d d
26.(d)
C2 d 1 8
= = =2
C1 d 2 4
C 2=2C 1=2 ×10 μ F=20 μ F .
23. (a) C p=C 1+C 2=16 μ F
The equivalent circuit is
C1 C2
C s= =3 μ F
C1 +C 2
or C1C2 = 3(C1 + C2) = 3 × 16 = 48 μF
C1(16 - C1) = 48
On solving, C1 = 12 μF, C2 = 4 μF.
24. (b) The equivalent circuit is
1 1 1 1 11
= + + =
Ceq 3 C 2C C 6C
6C
or C eq= .
11
27. (d) Effective capacitance of 3 μF
and 6 μF capacitors connected in parallel,
C' = 2 μF + 2 μF = 4 μF
C' = 3 + 6 = 9 μF
C' and C1 are in series with effective
capacitance, Now C’ and 4.5 μF capacitors are in
series.

∴ Ceq =
'
'' C ×C 1 4 ×12 48 9 ×4.5
C = '
= = =3 μ F = 3 μF
C +C1 4+ 12 16 9+ 4.5
Effective capacitance between P and Q, Charge through the circuit,
C = C’’ + C2 = 3 + 2 = 5 μE q = Ceq V = 3 × 12 = 36 μC

198
P.D. across 4.5 μF capacitor, 36. (c) Electric field inside the copper
plate is zero. It exists only in the region of
q 36 μ C
V= = =8 volt thickness d - b.
C 4.5 μ F
∴ C=
ε0 A
1 1
28. (b) U = CV2 = × 4 × 10-6 × (100)2 d−b
2 2
ε0 A
= 2 × 10-2 J = 0.02 J 37. (c) Without dielectric, C 0=
d
29. (c) Heat produced = Energy stored in
capacitor With dielectric,
1 ε0 A C 0 1 ε0 A
= × 4 × 10-6 × (400)2 C= = =
2 t 2 2 d
d−t +
1 κ

∴ 2 d=d−t+
= CV2 J = 0.32 J
2 t
1 κ
C( V 2−V 1 )
2 2
30. (b) ∆U = U2 – U1 =
2 t t
or d +t= or κ=
1 κ d +t
= × 8 × 10-6(202 - 102)
2 ε0 A
38. (d) For original capacitor, C=
= 4 × 10-6 × 300 J = 12 × 10-4 J. d
C 1 V 1+ C 2 V 2 The new arrangement is equivalent to a
31. (c) V = series combination of two capacitors, each
C 1+ C 2 with plate area A and separation d/2.
20 μ F × 500 V + 10 μ F × 200 V The new capacitance C 1 is given by
¿
20 μ F+ 10 μ F
1 1 1 1 1
12000 = + = +
¿ V =400 V . C1 C ’ C ' ' ε 0 A κ1 ε 0 A κ 2
30
d /2 d /2
1 C1 C 2
32. (b) Energy loss ¿

−6
.
2 C 1+ C 2
−6
¿
¿
d
(1 1
+
2 ε0 A κ 1 κ 2 )
1 3× 10 ×5 ×1 0
( )
¿ ⋅ −6
¿ 2 ε 0 A κ1κ2
2 (3+5) 1 0 or C 1=
d κ 1+ κ 2
−6
¿ 15 ×1 0 ׿ ¿ J .

C1 2κ 1 κ2
33. (a) Due to polarisation of the dielectric, = .
an electric field is induced in the opposite C κ 1+ κ 2
direction of the applied field. The net field 39. (a) Refer to the solution of
between the capacitor plates decreases. Example 90(a)(i) on page 2.64.
ε0 A κ 40. (c) Charge on each capacitor
34. (b) C=κ C 0=κ i.e., C ∝
d d becomes zero when two capacitors of

∴C∝
equal capacitances are charged and then
' 2κ ' 4κ connected to opposite terminals. But
or C ∝
d /2 d capacitances of the two capacitors are
' given to be different.
C
or =4 or C ' =4 C 41. (b) Here Q1 = CV, Q2 = 2C × 2V =
C
4CV
C C
35. (d) C0 = = As the two capacitors are connected
κ 2
with opposite polarity, the common

199
potential is q
V B= .
Q2−Q1 4 CV −CV 4 π ε0 R
V’ = = =V
C 1+ C 2 C +2C
Equivalent capacitance, C' = C + 2C = 3C
Final energy of the configuration is
1 ' '2 1 2 3 2
U' = C V = ×3 C ×V = C V .
2 2 2
42. (c) At the top of the stratosphere, E =
100 Vm-1.
43. (d) Surface charge density of the earth
= -10-9 Cm-2.
44. (c) The electric dipole potential at
large distance,
1 pcosθ 1
V= ⋅ 2 i.e., V ∝ 2
4 π ε0 r r
45. (c) Q = Q1 + Q2 + Q3 + … Qn = nC × V
Q 1C
n= =
CV 1 μ F × 110 V
6
10
¿ =9090.
110
ε0 A
46. (b) C 0=
d
With a dielectric slab of κ=5 ,
ε0 A
C=
t
d−t +
5

∴ C0
t
d−t +
5
=
C d
4t
d−
or 100 5
=
166 d
4t
166 × =66 d
5
66 d × 5 d
t= ≃ .
166 ×4 2
47. (b) After the connection, the charge on
inner sphere transfers to outer sphere.
Then the outer sphere is

200

You might also like

pFad - Phonifier reborn

Pfad - The Proxy pFad of © 2024 Garber Painting. All rights reserved.

Note: This service is not intended for secure transactions such as banking, social media, email, or purchasing. Use at your own risk. We assume no liability whatsoever for broken pages.


Alternative Proxies:

Alternative Proxy

pFad Proxy

pFad v3 Proxy

pFad v4 Proxy